Está en la página 1de 527

1

MAT023
A PUNTES DE CLASES

Departamento Matemática UTFSM Santiago


Parte I

Ecuaciones diferenciales ordinarias

5
Índice general

Índice general 1

I Ecuaciones diferenciales ordinarias 5

1. Ecuaciones diferenciales ordinarias de primer orden 6


1.1. Definiciones . . . . . . . . . . . . . . . . . . . . . . . . . . . . . . . . . . . 6
1.2. Modelos simples . . . . . . . . . . . . . . . . . . . . . . . . . . . . . . . . . 11
1.3. Métodos Elementales de Resolución . . . . . . . . . . . . . . . . . . . . . . 19
1.3.1. Integración directa . . . . . . . . . . . . . . . . . . . . . . . . . . . 19
1.3.2. Ecuaciones de variable separable . . . . . . . . . . . . . . . . . . . . 20
1.3.3. Ecuación lineal de primer orden . . . . . . . . . . . . . . . . . . . . 24
1.3.4. Ecuación de Bernoulli . . . . . . . . . . . . . . . . . . . . . . . . . 28
1.3.5. Ecuación de Ricatti . . . . . . . . . . . . . . . . . . . . . . . . . . . 30
1.3.6. Ecuaciones homogéneas . . . . . . . . . . . . . . . . . . . . . . . . 34
1.3.7. Otros cambios de variables . . . . . . . . . . . . . . . . . . . . . . . 37
1.4. Modelos simples: Segunda parte . . . . . . . . . . . . . . . . . . . . . . . . 42
1.5. Análisis cualitativo . . . . . . . . . . . . . . . . . . . . . . . . . . . . . . . 51
1.5.1. Métodos cualitativos . . . . . . . . . . . . . . . . . . . . . . . . . . 52
1.5.2. Ecuaciones diferenciales autónomas . . . . . . . . . . . . . . . . . . 55
1.5.3. Equilibrio y estabilidad . . . . . . . . . . . . . . . . . . . . . . . . . 59
1.6. Ejercicios del capı́tulo . . . . . . . . . . . . . . . . . . . . . . . . . . . . . . 67

2. Ecuaciones diferenciales lineales de orden superior 76


2.1. Elementos de transformaciones lineales . . . . . . . . . . . . . . . . . . . . 76
2.1.1. Definiciones . . . . . . . . . . . . . . . . . . . . . . . . . . . . . . . 76
2.1.2. Núcleo e imagen . . . . . . . . . . . . . . . . . . . . . . . . . . . . 85
2.1.3. Isomorfismo . . . . . . . . . . . . . . . . . . . . . . . . . . . . . . . 92
2.1.4. Matriz asociada a una transformación lineal . . . . . . . . . . . . . 93
2.1.5. Cálculo con coordenadas . . . . . . . . . . . . . . . . . . . . . . . . 101
2.1.6. Ejercicios resueltos de Transformaciones lineales . . . . . . . . . . . 105

1
Apuntes Mat023 (versión preliminar actualizada 23-05-2014)

2.2. Ecuaciones diferenciales lineales . . . . . . . . . . . . . . . . . . . . . . . . 142


2.2.1. Definiciones . . . . . . . . . . . . . . . . . . . . . . . . . . . . . . . 142
2.3. Teorema de Existencia y Unicidad . . . . . . . . . . . . . . . . . . . . . . . 148
2.4. El wronskiano . . . . . . . . . . . . . . . . . . . . . . . . . . . . . . . . . . 152
2.5. Ecuaciones diferenciales a coeficientes constantes . . . . . . . . . . . . . . . 158
2.5.1. Definiciones . . . . . . . . . . . . . . . . . . . . . . . . . . . . . . . 158
2.5.2. La ecuación de orden 2 . . . . . . . . . . . . . . . . . . . . . . . . . 158
2.5.3. La ecuación de orden superior . . . . . . . . . . . . . . . . . . . . . 163
2.6. Método de variación de parámetros . . . . . . . . . . . . . . . . . . . . . . 167
2.7. Método del anulador . . . . . . . . . . . . . . . . . . . . . . . . . . . . . . 174
2.8. Movimiento vibratorio . . . . . . . . . . . . . . . . . . . . . . . . . . . . . 184
2.9. Ejercicios del capı́tulo . . . . . . . . . . . . . . . . . . . . . . . . . . . . . . 190

3. Sistemas de ecuaciones diferenciales 196


3.1. Definiciones . . . . . . . . . . . . . . . . . . . . . . . . . . . . . . . . . . . 196
3.2. Ecuación con coeficientes constantes . . . . . . . . . . . . . . . . . . . . . . 200
3.2.1. Matriz A diagonalizable . . . . . . . . . . . . . . . . . . . . . . . . 201
3.3. Variación de parámetros en sistemas . . . . . . . . . . . . . . . . . . . . . 206
3.4. Análisis cualitativo de sistemas . . . . . . . . . . . . . . . . . . . . . . . . 213
3.4.1. Valores propios reales y distintos (no nulos) . . . . . . . . . . . . . 214
3.4.2. Valores propios complejos . . . . . . . . . . . . . . . . . . . . . . . 218
3.4.3. Valores propios repetidos (no nulos) . . . . . . . . . . . . . . . . . . 222
3.5. Ejercicios del capı́tulo . . . . . . . . . . . . . . . . . . . . . . . . . . . . . . 226

4. Transformación integral de Laplace 231


4.1. Definiciones y teoremas fundamentales . . . . . . . . . . . . . . . . . . . . 231
4.2. Cálculo de transformadas . . . . . . . . . . . . . . . . . . . . . . . . . . . . 235
4.3. Primer Teorema de la Traslación . . . . . . . . . . . . . . . . . . . . . . . 238
4.4. Transformada de la derivada . . . . . . . . . . . . . . . . . . . . . . . . . . 239
4.5. Funciones escalonadas y Segundo Teorema de la Traslación . . . . . . . . . 246
4.6. La Transformada de integrales de convolución . . . . . . . . . . . . . . . . 250
4.7. Ejercicios del capı́tulo . . . . . . . . . . . . . . . . . . . . . . . . . . . . . . 256

2
Apuntes Mat023 (versión preliminar actualizada 23-05-2014)

5. Series de Fourier 263


5.1. Definiciones . . . . . . . . . . . . . . . . . . . . . . . . . . . . . . . . . . . 263
5.1.1. El espacio SC [a, b] . . . . . . . . . . . . . . . . . . . . . . . . . . . 263
5.1.2. Teorema de la mejor aproximación . . . . . . . . . . . . . . . . . . 269
5.2. Convergencia Puntual de series de Fourier . . . . . . . . . . . . . . . . . . 279
5.3. Series de Fourier de senos y cosenos . . . . . . . . . . . . . . . . . . . . . . 284
5.4. Derivación e integración de Series de Fourier . . . . . . . . . . . . . . . . . 294
5.5. Ejercicios del capı́tulo . . . . . . . . . . . . . . . . . . . . . . . . . . . . . . 295

II Cálculo diferencial en varias variables 301

6. Elementos de topologı́a de Rn 302


6.1. El espacio euclidiano Rn . . . . . . . . . . . . . . . . . . . . . . . . . . . . 302
6.2. Producto interno y norma . . . . . . . . . . . . . . . . . . . . . . . . . . . 302
6.3. Elementos de topologı́a de Rn . . . . . . . . . . . . . . . . . . . . . . . . . 306
6.4. Ejercicios del capı́tulo . . . . . . . . . . . . . . . . . . . . . . . . . . . . . . 310

7. Funciones de varias variables 314


7.1. Definiciones básicas . . . . . . . . . . . . . . . . . . . . . . . . . . . . . . . 314
7.2. Gráficos, conjuntos de nivel y trazas . . . . . . . . . . . . . . . . . . . . . . 316
7.3. Ejercicios del capı́tulo . . . . . . . . . . . . . . . . . . . . . . . . . . . . . . 321

8. Lı́mites y continuidad 323


8.1. Definiciones . . . . . . . . . . . . . . . . . . . . . . . . . . . . . . . . . . . 323
8.2. Cálculo de lı́mites . . . . . . . . . . . . . . . . . . . . . . . . . . . . . . . . 329
8.2.1. Álgebra de lı́mites . . . . . . . . . . . . . . . . . . . . . . . . . . . . 330
8.2.2. Desigualdades y Teorema del Sandwich . . . . . . . . . . . . . . . . 331
8.3. Continuidad . . . . . . . . . . . . . . . . . . . . . . . . . . . . . . . . . . . 332
8.4. Álgebra de funciones continuas . . . . . . . . . . . . . . . . . . . . . . . . 334
8.5. Continuidad de funciones vectoriales . . . . . . . . . . . . . . . . . . . . . 336
8.6. Ejercicios del capı́tulo . . . . . . . . . . . . . . . . . . . . . . . . . . . . . . 338

3
Apuntes Mat023 (versión preliminar actualizada 23-05-2014)

9. Diferenciación en varias variables 341


9.1. Derivadas parciales . . . . . . . . . . . . . . . . . . . . . . . . . . . . . . . 341
9.2. Interpretación de la derivada parcial . . . . . . . . . . . . . . . . . . . . . 348
9.3. Diferenciabilidad . . . . . . . . . . . . . . . . . . . . . . . . . . . . . . . . 355
9.4. Derivadas de orden superior y funciones de clase C n . . . . . . . . . . . . . 369
9.5. Gradiente y matriz jacobiana . . . . . . . . . . . . . . . . . . . . . . . . . 373
9.6. La regla de la cadena . . . . . . . . . . . . . . . . . . . . . . . . . . . . . . 377
9.7. Gradiente y planos tangentes . . . . . . . . . . . . . . . . . . . . . . . . . 388
9.8. Derivada direccional y direcciones de crecimiento máximo . . . . . . . . . . 391
9.9. Ejercicios del capı́tulo . . . . . . . . . . . . . . . . . . . . . . . . . . . . . . 396

10.Máximos y mı́nimos 409


10.1. Extremos locales . . . . . . . . . . . . . . . . . . . . . . . . . . . . . . . . 409
10.2. Máximos y mı́nimos en compactos y/o con restricciones . . . . . . . . . . . 425
10.3. Extremos restringidos Multiplicadores de Lagrange . . . . . . . . . . . . . 428
10.3.1. Criterio de la segunda derivada para extremos condicionados . . . . 434
10.4. Ejercicios del capı́tulo . . . . . . . . . . . . . . . . . . . . . . . . . . . . . . 439

11.Funciones implı́citas e inversas 444


11.1. El teorema de la función implı́cita . . . . . . . . . . . . . . . . . . . . . . . 444
11.2. El teorema de la función inversa . . . . . . . . . . . . . . . . . . . . . . . . 460
11.3. Ejercicios del capı́tulo . . . . . . . . . . . . . . . . . . . . . . . . . . . . . . 470

III Evaluaciones de años anteriores 473

12.Controles 474

13.Certámenes 489

Bibliografı́a 524

4
Parte I

Ecuaciones diferenciales ordinarias

5
Capı́tulo 1 : Ecuaciones diferenciales ordinarias de primer orden

Definiciones

En general, entenderemos el modelamiento matemático como el proceso de establecer


un modelo matemático (es decir, un sistema expresado en términos de variables, funciones,
ecuaciones, etc.) que represente una situación —principalmente de naturaleza fı́sica—, su
resolución matemática, y finalmente la interpretación de los resultados en los términos
fı́sicos originales.
Como muchos conceptos de la naturaleza, tales como velocidad, aceleración, las reaccio-
nes quı́micas, los cambios de temperatura observados en un cuerpo, etc. se expresan como
razones de cambio tiene pleno sentido el uso de derivadas de funciones adecuadas. En este
tipo de situaciones, un modelo matemático es frecuentemente una ecuación que contiene
una o más derivadas de una función desconocida. Tal modelo matemático es llamado una
ecuación diferencial [1].

Ejemplo 1.1.1. Son ejemplos de ecuaciones diferenciales las siguientes expresiones ma-
temáticas:

d dy

1. dx
(1 − x2 ) dx + n (n + 1) y = 0 (Ecuación de Legendre)

d y 2 dy
2. x2 dx 2 2
2 + x dx + (x − α ) y = 0 (Ecuación de Bessel)

d y 3 2
2d y dy
3. x3 dx 3 + 4x dx2 + x dx + 5y = 0 (Ecuación de Euler)

4. y 00 + ky = A sin(ωo x), A, ωo ∈ R (Problemas de resortes)

5. y 00 − xy = 0 (Ecuación de Airy)

Definición 1.1.1. Una ecuación diferencial se dice de orden n si n corresponde al mayor


orden de derivada de la variable dependiente y presente en la ecuación.

d y 2
3
Ejemplo 1.1.2. La ecuación dx 2 + (5x) y = y 6 es una ecuación diferencial de segundo
 3 4
d y dy
grado. La ecuación dx 3 = dx + 5y es una ecuación diferencial de grado tres.

6
Apuntes Mat023 (versión preliminar actualizada 23-05-2014)

Comenzaremos el estudio de las ecuaciones diferenciales con las ecuaciones diferenciales


de primer orden.

Definición 1.1.2. Sea f : U ⊆ R2 → R una función de dos variables. Una ecuación


diferencial de primer orden es una ecuación de la forma:

y 0 = f (x, y) (1.1)

La variable x en este caso se conoce como variable independiente. Si la variable inde-


pendiente es el tiempo t, frecuentemente una ecuación diferencial se anota como:

y 0 = f (t, y) o bien ẏ = f (t, y)

Observación 1.1.1. Una ecuación diferencial (ordinaria) de orden n es una ecuación de


la forma:
f x, y, y 0 , y 00 , . . . , y (n) = 0


para una cierta función f .

Ejemplo 1.1.3. Son ecuaciones diferenciales de primer orden:

1. y 0 = f (x) , f función integrable.

2. y 0 + y = cos x

3. x−3 y 0 − y 2 = 0, x>0

Definición 1.1.3. Sea I ⊆ R un intervalo abierto del tipo ]a, b[, o posiblemente intervalos
abiertos infinitos del tipo ]−∞, b[ , ]a, +∞[, o bien ]−∞, +∞[. Una función ϕ : I ⊆ R → R
se dice solución de la ecuación diferencial (1.1) en el intervalo I si:

1. t, ϕ (t) ∈ U, ∀t ∈ I

2. ϕ0 (t) = f t, ϕ (t) ,

∀t ∈ I

1
Ejemplo 1.1.4. La función ϕ : ]1, +∞[ → R, t → ϕ (t) = t−1
es solución de la EDO

dy
= −y 2
dt

7
Apuntes Mat023 (versión preliminar actualizada 23-05-2014)

pues, para t ∈ ]1, +∞[


 
d 1 1
= −
dt t−1 (t − 1)2
 2
1
= −
t−1

Ejemplo 1.1.5. Verifique que la función ϕ (x) = sin x definida en R es solución de la


ecuación diferencial de segundo orden:

y 00 + y = 0

dϕ d2 ϕ
Solución. Derivando la función ϕ obtenemos: dx
= cos x y dx2
= − sin x se sigue

d2 ϕ (x)
+ ϕ (x) = 0
dx2
para todo x ∈ R.

Observación 1.1.2. Note que al escribir:

ϕC (x) = sin x + C

con C una constante cualquiera, ϕC también es solución de la ecuación diferencial.

Ejemplo 1.1.6. Consideremos la ecuación:

xy 0 − x2 − y = 0, x>0

Se sabe que toda función de la forma:

y = x2 + Cx, C∈R (1.2)

es solución de la ecuación diferencial. En efecto, derivando la ecuación (1.2), tenemos:

y 0 = 2x + C

Reemplazando en la ecuación diferencial, se tiene que:

x (2x + C) − x2 − x2 + Cx = 0


para todo x > 0.

8
Apuntes Mat023 (versión preliminar actualizada 23-05-2014)

Ejemplo 1.1.7. Hallar una ecuación diferencial para la familia de parábolas:

y = C1 (x − C2 )2

donde C1 y C2 constantes arbitrarias.

Solución. Despejamos C1
y
= C1
(x − C2 )2
luego  
d y
=0
dx (x − C2 )2
esto es
y 0 (x − C2 )2 − y2 (x − C2 )
=0
(x − C2 )4
se sigue
y 0 (x − C2 )2 − y2 (x − C2 ) = 0

luego
y 0 (x − C2 ) = 2y

de donde obtenemos
2y
x − C2 =
y0
derivando  0
2y
1=
y0
se sigue
1 (y 0 )2 − yy 00
=
2 (y 0 )2
luego
(y 0 )2
= yy 00
2

9
Apuntes Mat023 (versión preliminar actualizada 23-05-2014)

Ejercicios de la sección

1. Establezca el orden de la ecuación diferencial dada:


 7
00 0 (4) dy
(6) 00
(a) (1 − x) y − 4xy + 3y = tan x (b) y − 2xy y =
s dx
4  2
d3 y 2

dy dy dy
(c) x 3− + ex y = 0 (d) = 1 +
dx dx dx2 dx
2. Comprobar que las siguientes funciones satisfacen las ecuaciones dadas y dar un
intervalo en el cual esto se cumpla:
(a) y = A sin (x + B) ; y 00 + y = 0 (b) y = ex − e−x ; y 00 − y = 0
(c) y = tan (x) ; y 0 = 1 + y 2 (d) y 0 = 25 + y 2 ; y = 5 tan 5x

3. Demostrar que la ecuación dada define implı́citamente una solución de la ecuación


dada
y3 dy −2xy
(a) x2 y − =1 dx
= x2 −y 2
para x 6= y
3
(b) x3 + 3xy 2 = 1 2xyy 0 + x2 + y 2 = 0 para x ∈ ]0, 1[

4. Muestre que la familia de funciones


Z x
−x2 2 2
y=e et dt + Ce−x
0

son soluciones de la ecuación


dy
+ 2xy = 1
dx
5. Muestre que la función definida por tramos

 −x2 si x < 0
ϕ (x) =
 x2 si x ≥ 0

es una solución de la ecuación diferencial xy 0 − 2y = 0 en R.

6. Determine α ∈ R para que la función y = xα sea solución de 2x2 y 00 − y = 0. Si


encuentra más de un valor, muestre que cualquier combinación lineal de esas dos
funciones resulta ser una solución del problema.

7. Encontrar valores de m para los cuales la función es solución de la ecuación dada:

a) y (x) = emx donde y 000 − 3y 00 − 4y 0 + 12y = 0

10
Apuntes Mat023 (versión preliminar actualizada 23-05-2014)

b) y (x) = xm donde x2 y 00 + 2xy 0 − 6y = 0

8. Encontrar una E.D.O. de la forma y 00 + A (x) y 0 + B (x) y = 0 que tenga entre sus
soluciones las funciones y1 (x) = ex y y2 (x) = xex .

9. Juan, Leo y Roberto están tomando café y un joven del paralelo 19 les pregunta por
la solución de la ecuación diferencial

dy y+1
=
dt t+1

después de un rato, Juan dice y (t) = t, Leo y (t) = 2t + 1 y Roberto y (t) = t2 − 2


¿Quién esta en lo correcto?

10. Construir una ecuación diferencial de la forma

dy
= 2y − t + g (y)
dt

que tenga la función y (t) = e2t como solución.


dy
11. Construir una ecuación diferencial de la forma = f (t, y) que tenga por solución
2
dt
y (t) = et donde f (t, y) dependa explı́citamente de t y y.

12. Hallar una ecuación diferencial para la familia de curvas:

y = C1 + (x − C2 )2

donde C1 y C2 constantes arbitrarias.

13. Construya una ecuación diferencial que no tenga ninguna solución real.

Modelos simples

Estudiaremos algunos ejemplos elementales de modelamiento matemático:

Problema 1.2.1 ([2]). Desde una cierta altura se ha arrojado un cuerpo de masa m.
Determinar la ley según la cual varı́a la velocidad de caı́da v, si sobre el cuerpo, además

11
Apuntes Mat023 (versión preliminar actualizada 23-05-2014)

de la fuerza de gravedad, actúa la fuerza de resistencia del aire que es proporcional a la


velocidad v.

Solución. Sea m la masa del cuerpo en caı́da libre. En virtud de la Segunda Ley de
Newton:
X
Fi = ma
i
P
donde i Fi representa la suma (vectorial) de fuerzas aplicadas al cuerpo y a representa la
aceleración del cuerpo, se obtiene:
dv
m = mg − kv (1.3)
dt
donde g es la aceleración de gravedad y k es la constante (positiva) de proporcionalidad.
Resolver esta ecuación diferencial significa encontrar una función v = f (t) que satisfaga
idénticamente la ecuación diferencial dada. Existen una infinidad de funciones de este tipo
(esto será probado más adelante). Es fácil comprobar que toda función del tipo:
k mg
v (t) = Ce− m t + (1.4)
k
satisface la ecuación (1.3), cualquiera que sea la constante C. Pero, ¿cuál de estas funciones
dará la dependencia buscada entre v y t? Para encontrar dicha relación, se debe utilizar un
condición adicional. Esta condición adicional se llama condición inicial. Supongamos que

12
Apuntes Mat023 (versión preliminar actualizada 23-05-2014)

en el momento inicial del experimento arrojamos el cuerpo con una velocidad inicial v0
conocida. Ası́, la función v = f (t) que deseamos encontrar debe satisfacer la condición:

f (0) = v0

Es decir, v = v0 en t = 0. Ası́, reemplazando en (1.4), se obtiene:


mg
C = v0 −
k
Por lo tanto, la ley según la cual varı́a la velocidad de caı́da está dada por la ecuación:
 mg  − k t mg
v (t) = v0 − e m +
k k
Observación 1.2.1. Una cuestión de interés es el comportamiento asintótico de la solución.
Es decir, el comportamiento de la solución para t suficientemente grande. Más precisamente,
nos interesa:
lı́m v (t)
t→+∞

En particular, para el ejemplo:


n mg  − k t mg o
lı́m v (t) = lı́m v0 − e m +
t→+∞ t→+∞ k k
mg
=
k
La interpretación del resultado anterior, es que cuando el tiempo t es suficientemente
grande, la velocidad final del cuerpo no depende de la velocidad inicial.

Observación 1.2.2. Observar que desde la ecuación (1.3), si suponemos k = 0, es decir,


suponemos que la resistencia del aire es tan pequeña que puede ser despreciada, se obtiene:

v (t) = v0 + gt

Definición 1.2.1. Se llamará solución general de la ecuación diferencial de primer


orden:
y 0 = f (x, y)

a la función:
y = ϕ (x, C)

que depende de una constante arbitraria C y satisface las condiciones siguientes:

13
Apuntes Mat023 (versión preliminar actualizada 23-05-2014)

1. satisface la ecuación diferencial para cualquier valor de la constante C.

2. cualquiera que sea la condición inicial y = y0 para x = x0 , es decir, y (x0 ) = y0


se puede encontrar un valor C = C0 tal que la función y = ϕ (x, C0 ) satisfaga la
condición inicial dada.

Problema 1.2.2 (Espejo parabólico). Hallar la forma que debe tener un espejo convexo,
simétrico respecto de un eje, llamado eje focal, de tal modo que si un haz de luz es apuntado
hacia el espejo, paralelo al eje focal, se refleje directamente en un punto F fijo (llamado
foco) del eje focal.

Solución. Obtendremos tal espejo mediante la rotación de una curva en el plano. Sea
C : y = f (x) tal curva. Consideremos un sistema de coordenadas de tal modo que el
origen del sistema coincida con el foco F de la curva. En particular, el eje focal de la curva
coincide con el eje de las abscisas de tal sistema de referencia. Considere un punto P (x, y)
en la curva C y sea T la recta tangente a C en el punto P (x, y). Denotemos por Q el punto
de intersección de T con el eje focal (o de las abscisas del sistema de referencia). Si L es
una recta que representa el haz de luz paralelo al eje focal y que incide en P , por la ley de
Snell, el ángulo de incidencia ]T P L y ángulo de reflexión ]QP F coinciden. Es decir, se
tiene que:
]T P L = ]QP F = α

Entonces, del triángulo 4QP F se obtiene:


y
tan 2α = (1.5)
x
14
Apuntes Mat023 (versión preliminar actualizada 23-05-2014)

por ser F el origen del sistema de coordenadas. Como T es tangente a la curva C, se tiene
que ]P QF es también α. Luego:
tan α = y 0 (1.6)

Reemplazando las ecuaciones (1.5) y (1.6), en la identidad trigonométrica siguiente:

2 tan α
tan 2α =
1 − tan2 α

se obtiene la ecuación diferencial de primer orden:

y 2y 0
= (1.7)
x 1 − (y 0 )2

Problema 1.2.3 (Braquistócrona). [3] Hallar la forma que debe tener un alambre de
modo que una argolla que se desliza por él, sin roce, bajo la acción de la gravedad de un
punto A a un punto B de menor altura en el mismo plano y no exactamente bajo el punto
A, lo haga en el menor tiempo posible.

Solución. Este problema fue planteado en 1696 por Jean Bernoulli a la comunidad cientı́fi-
ca de su época. La solución que él mismo encontró (independientemente también lo hicieron
Leibnitz, L’Hôpital, Newton) usa una versión generalizada de la Ley de Snell de la óptica.
Comenzaremos modelando primeramente esta situación: la versión generalizada de la Ley
de Snell. A modo de ejercicio lo haremos utilizando las herramientas del cálculo diferencial,
en particular minimización. Considere, entonces, el siguiente problema:

15
Apuntes Mat023 (versión preliminar actualizada 23-05-2014)

Supongamos que un atleta, situado en la orilla oriental de un rı́o de ancho a debe atravesarlo
nadando a velocidad constante v1 hasta un cierto punto C en la orilla occidental. Luego de
esto, debe correr a velocidad constante v2 por sobre la arena de la ribera de ancho b del rı́o
hasta la meta en el punto B. Se supone que v1 < v2 . Se desea encontrar el punto C en la orilla
occidental de tal modo que el tiempo empleado por el atleta desde el punto A hasta la meta
en B sea el menor posible.

Consideremos las rectas paralelas (y verticales) l1 , l2 y l3 . Supongamos que la recta l1


representa la orilla oriental del rı́o, l2 representa la ribera del rı́o y l3 la lı́nea de meta. Por
la condiciones del problema tenemos:

1. A ∈ l1

2. C ∈ l2

3. B ∈ l3

4. a es la distancia entre l1 y l2

5. b es la distancia entre l2 y l3

Supongamos, además, que c y x son las distancias verticales de A a B, y de A a C,


respectivamente. Entonces, el tiempo total de la carrera está dado por:


q
x 2 + a2 (c − x)2 + b2
T (x) = +
v1 v2

Derivando e igualando a 0, se obtiene la coordenada x0 de tiempo mı́nimo. Es decir, x0


debe cumplir con:
x0 c − x0
p = q
v1 x20 + a2 v2 (c − x0 )2 + b2
Si introducimos los ángulos de incidencia α y β respecto de la recta normal a l2 para este
valor de x0 se obtiene:
sin α sin β
=
v1 v2
que es la conocida Ley de Snell.

16
Apuntes Mat023 (versión preliminar actualizada 23-05-2014)

Ahora bien, si consideramos otro segmento (es decir, otra recta vertical l4 ) por el cual
el atleta se deba desplazar a velocidad constante v3 , se obtendrá:

sin α1 sin α2 sin α3


= = = constante
v1 v2 v3

De manera análoga, la argolla de masa m que cae bajo la acción de la gravedad g tiene
una velocidad v que va en aumento de acuerdo a la distancia vertical recorrida y. Ası́,
igualando energı́as potencial y cinéticas se obtiene la ecuación:

1 2
mv = mgy
2

de donde:
p
v= 2gy (1.8)

Por esta razón, Bernoulli conjeturó la Ley de Snell generalizada siguiente:

sin α
= λ, λ constante (1.9)
v

donde α es el ángulo que instantáneamente forma la tangente a la curva en la posición de


la argolla y la vertical. Finalmente, notando que:
π 
sin α = cos −α
2
1
= q
π

1 + tan2 2
−α
1
= q
1 + (y 0 )2

Utilizando, entonces, las fórmulas (1.8) y (1.9) se obtiene la ecuación:


p 1
2gy = q
λ 1 + (y 0 )2

O bien:
 
2
y 1 + (y 0 ) = k 2
1
donde k = 2gλ2
.

17
Apuntes Mat023 (versión preliminar actualizada 23-05-2014)

Ejercicios de la sección

1. Determine la ecuación que debe cumplir la familia de curvas que forman un ángulo
de 45 grados en la intersección con la familia de curvas y (x + c) = 1.

2. Determine la ecuación de la familia de curvas ortogonales a la familia y 2 = cx3 .

3. En este problema se analiza la caı́da de una gota de agua. Supongamos que al


caer esta se evapora y mantiene su forma esférica, la rapidez con que se evapora es
proporcional al área con una constante de proporcionalidad ζ < 0 y no se considera
la resistencia del aire. Designemos por ρ la densidad del agua, r0 el radio de la gota
cuando t = 0 y la dirección positiva es hacia abajo.

a) Muestre que el radio de la gota r (t) disminuye de acuerdo a la ley


 
ζ
r (t) = t + r0
ρ

b) Obtener la ecuación diferencial que satisface la velocidad v (t) de la gota en su


caı́da libre. Determine la velocidad si la gota cae del reposo.

4. En la figura suponga que el eje y y la recta x = 1 corresponden a las orillas oeste


y este de un rı́o de 1 [km] de ancho. El rı́o fluye hacia el norte con una velocidad
vr donde kvr k = vr [km/h]. Un hombre entra al rı́o en el punto (1, 0) en la costa
este y nada en dirección a la ribera contraria a una velocidad constante de kvs k = vs
[km/h]. El hombre quiere llegar al punto (0, 0) de modo que nada de forma tal que
su vector velocidad vs siempre apunta a (0, 0). Muestre que la trayectoria que sigue
el nadador satisface la ecuación
p
dy vs y − vr x2 + y 2
=
dx vs x

18
Apuntes Mat023 (versión preliminar actualizada 23-05-2014)

Métodos Elementales de Resolución

Recordaremos primeramente un teorema esencial para resolver ecuaciones diferenciales


elementales: el Teorema Fundamental del Cálculo.

Teorema 1.3.1. (Teorema Fundamental del Cálculo)


Sea f : [a, b] → R función integrable en [a, b]. Dado x0 ∈ [a, b] e y0 ∈ R se tiene que la
función F : [a, b] → R dada por

Zx
F (x) = f (t)dt + y0 con x ∈ [a, b],
x0

es continua en [a, b] y F (x0 ) = y0 . Además, si f es continua en [a, b], entonces la función


F es derivable en [a, b] y satisface la ecuación:

F 0 (x) = f (x)

Observación 1.3.1. Recordemos que en este contexto la función F es conocida como una
primitiva de f .

Integración directa

Definición 1.3.1. Una ecuación diferencial ordinaria de primer orden, digamos y 0 = f (x, y),
se dice que es resoluble por integración directa si existe una función g integrable sobre
un intervalo abierto I ⊆ R tal que f (x, y) = g(x)

19
Apuntes Mat023 (versión preliminar actualizada 23-05-2014)

Observación 1.3.2. En particular, una ecuación diferencial de integración directa es de


la forma:
y 0 = g(x) (1.10)

para todo x ∈ I.
Ası́, si g : D ⊆ R → R es una función integrable, entonces integrando en ambos lados de
la igualdad 1.3.2 y usando el Teorema Fundamental del Cálculo, obtenemos que la solución
general de la ecuación diferencial es de la forma:

φ(x) = F (x) + C,

donde F es una primitiva de g y C ∈ R es una constante.

Ejemplo 1.3.1. Una partı́cula se mueve a lo largo de una lı́nea recta de manera que su
velocidad en el instante t es 2 sin t. Si f (t) indica su posición en el tiempo t medio a partir
del punto de partida, se tiene que f 0 (t) = 2 sin t. Por la observación anterior, se concluye
que:
f (t) = −2 cos t + C

Note que para fijar la función posición se necesita algún otro dato. En particular, si se
conoce el valor de f en algún instante en particular, entonces se puede determinar C. Por
ejemplo, si f (0) = 0, entonces C = 2 y la función posición es f (t) = −2 cos t + 2.

Ejemplo 1.3.2. Determine la solución del problema


dy 2
= e−x
dx
y (0) = 5
2
Solución. Se puede demostrar que e−x dx no es una función elemental, sin embargo,
R

podemos expresar la solución por


Z x
2
y (x) = e−t dt + 5
0

Ecuaciones de variable separable

Definición 1.3.2. Una ecuación diferencial de primer orden y 0 = f (x, y) se dice una
ecuación de variable separable si:

f (x, y) = Q (x) R (y) (1.11)

20
Apuntes Mat023 (versión preliminar actualizada 23-05-2014)

para Q y R funciones continuas sobre algún intervalo abierto I ⊆ R.

Observación 1.3.3. [5] Si R (y) 6= 0 se puede dividir por R (y) y escribir la ecuación
(1.11) en la forma:
A (y) y 0 = Q (x) (1.12)

donde A (y) = 1/R (y). Recordemos, además, que y representa una cierta función descono-
cida y = Y (x), luego la ecuación (1.12), se expresa como:

A (Y (x)) Y 0 (x) = Q (x)

Integrando la ecuación anterior, se sigue que:


Z Z
0
A (Y (x)) Y (x) dx = Q (x) dx + C

Haciendo la sustitución y = Y (x) en la integral de la izquierda, se tiene que dy = Y 0 (x) dx.


Por consiguiente, se obtiene:
Z Z
A (y) dy = Q (x) dx + C

Ahora bien, si G es una primitiva de A y H es una primitiva de Q, la ecuación anterior se


puede escribir como:
G (y) = H (x) + C (1.13)

Recı́procamente, si y es una función que satisface la ecuación (1.13), entonces:

G0 (y) y 0 = H 0 (x)

es decir, tenemos A (y) y 0 = Q (x). Por lo tanto, la ecuación (1.13) da una representación
implı́cita de una familia de soluciones a un parámetro.

Observación 1.3.4. En particular, el formalismo anterior se reduce considerablemente si


se considera la notación de Leibnitz para las derivadas. Es decir, escribimos y 0 = dy/dx y
separamos las variables, con lo cual la ecuación (1.12) se escribe directamente como:

A (y) dy = Q (x) dx

y la ecuación anterior tiene sentido al ser dy la diferencial de la función desconocida y.


Formalmente, podemos integrar la ecuación anterior, obteniendo:
Z Z
A (y) dy = Q (x) dx + C

pero esto sólo es formal, pues las variables de integración son distintas.

21
Apuntes Mat023 (versión preliminar actualizada 23-05-2014)

Ejemplo 1.3.3. Resolver la ecuación


dy
= ey−x
dx
Solución. Se trata de una E.D.O. de variables separadas pues
dy
= ey−x = e−x ey
dx
se sigue
e−y(x) y 0 (x) = e−x

integrando respecto a x Z Z
−y(x) 0
e y (x) dx = e−x dx

se sigue
−e−y(x) = −e−x + C

despejando
e−y(x) = e−x + K

luego
y (x) = − ln e−x + K


es una familia de soluciones.

Ejemplo 1.3.4. Resolver la ecuación


dy
= x2 y + x2
dx
Solución. Note que x2 y+x2 = x2 (y + 1) luego se trata de una EDO de variables separables
dy
= x2 (y + 1)
dx
se sigue Z Z
dy
= x2 dx
y+1
integrando
x3
ln |y + 1| = +C
3

x3
y + 1 = Ke 3

22
Apuntes Mat023 (versión preliminar actualizada 23-05-2014)

ası́
x3
y (x) = Ke 3 − 1

Ejemplo 1.3.5. Diremos que una ecuación diferencial es una ecuación diferencial lineal
de primer orden homogénea si es de la forma:

y 0 + P (x) y = 0 (1.14)

si P es una función continua sobre su dominio abierto.

Solución. Separando las variables e integrando:


Z Z
dy
= − P (x) dx + K
y
con K una constante de integración. Luego, la ecuación anterior implica que:
Z
ln |y| = − P (x) dx + K

Ası́:
R
y = ±eK e− P (x)dx

Por consiguiente:
R
y = C e− P (x)dx

Observación 1.3.5. Note que en el procedimiento anterior, se ha dividido por y, por


tanto, debe explicarse que toda solución de (1.14) se puede expresar mediante la fórmula
R
y = C e− P (x)dx
. Sea y una solución de (1.14) y considere la función g definida por:
R
P (x)dx
g (x) = y e

Luego:
R R
g 0 (x) = y 0 e P (x)dx
+ P (x) ye P (x)dx

R
= e P (x)dx
(y 0 + P (x) y)
= 0

como x pertenece a un intervalo abierto, se obtiene que g (x) = C, por el Teorema del Valor
Medio. Por tanto:
R
y = C e− P (x)dx

23
Apuntes Mat023 (versión preliminar actualizada 23-05-2014)

Observación 1.3.6. El razonamiento anterior da origen a un método de resolución de


ecuaciones diferenciales del tipo:

y 0 + P (x) y = Q (x)

donde P y Q son funciones continuas en un intervalo abierto I ⊆ R. La ecuación diferencial


anterior se llama ecuación diferencial lineal de primer orden.

Ecuación lineal de primer orden

Definición 1.3.3. Una ecuación diferencial de primer orden se dice lineal si es de la forma:

y 0 + P (x) y = Q (x) (1.15)

para dos funciones P y Q continuas en un intervalo abierto I ⊆ R.

Definición 1.3.4. Un factor integrante o un factor de integración para la ecuación


(1.15) es una expresión de la forma:
R
P (x)dx
µ (x) = e (1.16)

Observación 1.3.7. Multipliquemos la ecuación diferencial (1.15) por el factor integrante


en (1.16), luego:
R R
e P (x)dx
{y 0 + P (x) y} = e P (x)dx
Q (x)

Note que la ecuación anterior, puede escribirse como:

d n R P (x)dx o R
ye = e P (x)dx Q (x)
dx

Integrando respecto de x, obtenemos:


R
Z R
P (x)dx P (x)dx
ye = e Q (x) dx + C

Finalmente: Z 
R R
− P (x)dx P (x)dx
y=e e Q (x) dx + C

24
Apuntes Mat023 (versión preliminar actualizada 23-05-2014)

Teorema 1.3.2 (Fórmula de Leibnitz). Sean P y Q dos funciones continuas sobre un


intervalo abierto I ⊆ R. La solución general de la ecuación diferencial lineal:

y 0 + P (x) y = Q (x)

está dada por la fórmula:


R
Z R

− P (x)dx P (x)dx
y (x) = e e Q (x) dx + C

R
P (x)dx
Observación 1.3.8. Si µ (x) = e es el factor integrante, la fórmula anterior queda:
Z 
1
y (x) = µ (x) Q (x) dx + C
µ (x)

Ejemplo 1.3.6. Resolver la ecuación lineal de primer orden:

xy 0 + (1 − x) y = e2x

Solución. Supongamos que x 6= 0. La ecuación anterior queda como:

e2x
 
0 1
y + −1 y =
x x

Utilizando las notaciones del teorema anterior, tenemos que P (x) = 1/x−1 y Q (x) = e2x /x.
Calculamos, primeramente, el factor integrante:
1
µ (x) = e ( x −1)dx
R

= |x| e−x

Luego:
Z 
1
y (x) = µ (x) Q (x) dx + C
µ (x)
ex 2x
Z 
−x e
= |x| e dx + C
|x| x

Si x > 0, se tiene que |x| = x y la solución es:

ex
Z 
x
y = e dx + C
x
e2x ex
= +C
x x

25
Apuntes Mat023 (versión preliminar actualizada 23-05-2014)

Por otro lado, si x < 0, se tiene que |x| = −x y la solución es:

ex
 Z 
x
y = − e dx + C
−x
e2x ex
= −C
x x
Ahora bien, como C es una constante arbitraria, si x 6= 0 se puede escribir:
e2x ex
y= +C
x x
Ejemplo 1.3.7. Deternine una función f : D ⊆ R → R tal que
Z t
f (t) = t + et
e−u f (u) du + tf (t)
0

Solución. Notemos que f (0) = 0 y


Z t
−t
(f (t) − t − tf (t)) e = e−u f (u) du
0

usando el teorema fundamental del cálculo

(f 0 (t) − 1 − f (t) − tf 0 (t)) e−t − (f (t) − t − tf (t)) e−t = e−t f (t)

se sigue
f 0 (t) − 1 − f (t) − tf 0 (t) − f (t) + t + tf (t) = f (t)

ası́
(1 − t) f 0 (t) − (3 − t) f (t) = 1 − t

es una EDO de primer orden lineal


df 3−t
− f =1
dt 1−t
el factor de integración es

− 3−t
R
dt
µ (t) = e 1−t = e2 ln(t−1)−t
= (t − 1)2 e−t

se sigue
d
(t − 1)2 e−t f = (t − 1)2 e−t

dt
26
Apuntes Mat023 (versión preliminar actualizada 23-05-2014)

integrando Z
2 −t
(t − 1) e f = (t − 1)2 e−t dt

es decir
(t − 1)2 e−t f = −e−t t2 + 1 + C


de donde
−e−t (t2 + 1) + C
f (t) =
(t − 1)2 e−t
como f (0) = 0 se sigue
0 = −1 + C ⇒ C = 1

finalmente
−e−t (t2 + 1) + 1
f (t) =
(t − 1)2 e−t
Ejemplo 1.3.8. Resolver la ecuación

dy
− 2xy = x
dx
2
R
−2xdx
Solución. Es una ecuación lineal, el factor integrante es µ (x) = e = e−x , multipli-
cando la ecuación por µ (x) se tiene

2 dy 2 2
e−x − 2xe−x y = xe−x
dx

luego
d  −x2  2
e y = xe−x
dx
integrando
Z
−x2 2
e y = xe−x dx
1 2
= − e−x + C
2

ası́
1 2
y (x) = − + Cex
2

27
Apuntes Mat023 (versión preliminar actualizada 23-05-2014)

Ecuación de Bernoulli

Observación 1.3.9. Numerosas aplicaciones pueden ser modeladas con ecuaciones di-
ferenciales ordinarias que no son lineales, sin embargo, mediante cambios de variables
adecuados y algo de manipulación algebraica, estas ecuaciones pueden ser transformadas
en ecuaciones diferenciales lineales. Un caso importante es la llamada ecuación de Bernoulli.

Definición 1.3.5. Sean P (x) y Q (x) funciones continuas sobre un intervalo abierto I ⊆ R.
Una ecuación diferencial de la forma:

y 0 + P (x) y = Q (x) y α , α 6= 1 (1.17)

se llama ecuación de Bernoulli.

Observación 1.3.10. Multipliquemos la ecuación de Bernoulli en (1.17) por y −α , de donde


obtenemos:
y −α y 0 + P (x) y 1−α = Q (x) (1.18)

Sea z = y 1−α . Luego, por la regla de la cadena, obtenemos:

z 0 = (1 − α) y −α y 0

Reemplazando en la ecuación (1.18), se obtiene:


1
z 0 + P (x) z = Q (x)
1−α
O bien:
z 0 + (1 − a) P (x) z = (1 − α) Q (x) (1.19)

que es una ecuación diferencial lineal de primer orden. Es importante notar que una vez
resuelta la ecuación (1.19) se debe volver a la variable original y = y (x).

Ejemplo 1.3.9. Hallar la solución de la ecuación diferencial:


dy
+ xy = x3 y 3
dx
Solución. Dividiendo todos los términos por y 3 , tenemos:

y −3 y 0 + xy −2 = x3 (1.20)

28
Apuntes Mat023 (versión preliminar actualizada 23-05-2014)

Consideremos, tal como en la observación anterior, el cambio de variables z = y −2 . Luego:


dz dy
= −2y −3
dx dx
Reemplazando la ecuación anterior en (1.20), se obtiene la ecuación diferencial lineal:
dz
− 2xz = −2x3
dx
cuya solución es:
2
z = 1 + x2 + Cex

Por consiguiente, la solución general de la ecuación diferencial dada es:


1
y=√
1+ x2 + Cex2
con C una constante cualquiera.

Ejemplo 1.3.10 (Ecuación logı́stica). Resuelva la siguiente ecuación de Bernoulli, conocida


como ecuación logı́stica:
dN
= αN (β − N ) (1.21)
dt
con α, β > 0.

Solución. Note que podemos escribir la ecuación (1.21) como:


dN
= αβN − αN 2
dt
Sean A = αβ y B = α, entonces:
dN
− AN = −BN 2
dt
Dividiendo la ecuación anterior por N 2 , obtenemos:
dN
N −2 − AN −1 = −B (1.22)
dt
Sea z = N −1 , luego:
dz dN
= −N −2
dt dt
Reemplazando en la ecuación (1.22), obtenemos:
dz
− − Az = −B
dt
29
Apuntes Mat023 (versión preliminar actualizada 23-05-2014)

Es decir:
dz
+ Az = B
dt
cuya solución exacta es:
B
z = Ce−At +
A
1
Pero z = N
, luego:
1 1
N= = −At
z Ce + B/A
Finalmente, como A = αβ y B = α, obtenemos:

β
N (t) =
1 + Ce−αβt

Ecuación de Ricatti

Definición 1.3.6. Una ecuación de Ricatti es una ecuación diferencial de la forma:

y 0 + P (x) y + Q (x) y 2 = R (x) (1.23)

donde P, Q y R son funciones continuas sobre un intervalo I ⊆ R.

Observación 1.3.11. Existe un método para obtener una familia de soluciones de una
ecuación de Ricatti si se conoce una solución particular u = u (x). Supongamos esto y
consideremos el cambio de variables:

1
y =u+ (1.24)
v

con v = v (x) la nueva variable. Derivando la ecuación anterior, obtenemos:

1 0
y 0 = u0 − v (1.25)
v2

Reemplazando las ecuaciones (1.24) y (1.25) en la ecuación de Ricatti (1.23), se sigue que:
     2
0 1 0 1 1
u − 2 v + P (x) u + + Q (x) u + = R (x)
v v v

Reordenando los términos de la ecuación anterior, tenemos que:


v0
 
 0 2 1 u 1
u + P (x) u + Q (x) u − 2 + P (x) + Q (x) 2 + 2 = R (x)
v v v v

30
Apuntes Mat023 (versión preliminar actualizada 23-05-2014)

Ahora bien, como u es una solución particular de la ecuación (1.23), obtenemos:

v0
 
1 u 1
− 2 + P (x) + Q (x) 2 + 2 = 0
v v v v

Amplificando la ecuación anterior por −v 2 , finalmente se obtiene la ecuación:

v 0 − P (x) v − Q (x) u (x) v − Q (x) = 0

Es decir:
v 0 − {P (x) + u (x) Q (x)} v = Q (x)

que es una ecuación lineal de primer orden.

Ejemplo 1.3.11. Resuelva la ecuación de Ricatti:


2
y0 = y2 − (1.26)
x2
1
Solución. Notamos primeramente que u = x
es una solución particular de la ecuación.
En efecto:
2 1 2
u2 − = −
x2 x2 x2
1
= − 2
x
0
= u

1
Ahora bien, sea y = x
+ v1 , luego:

dy 1 1 dv
=− 2 − 2
dx x v dx
1
Reemplazando la ecuación anterior y el cambio de variables y = x
+ v1 en la ecuación (1.26),
obtenemos:  2
1 1 dv 1 1 2
− 2− 2 = + −
x v dx x v x2
Simplificando y agrupando términos semejantes, se tiene:
1 dv 2 1
− 2
= + 2
v dx vx v
Amplificando por −v 2 , finalmente se obtiene:
2
v 0 + v = −1
x
31
Apuntes Mat023 (versión preliminar actualizada 23-05-2014)

Como se puede observar, la ecuación anterior es una ecuación lineal de primer orden cuya
solución exacta es:
C x
v= −
x2 3
1
Sin embargo, recordemos que y = x
+ v1 . Luego, la solución obtenida es:

1 1
y= + C x
x x2
− 3

O bien:
1 3x2
y= +
x C − x3
Ejemplo 1.3.12. Obtenga una solución del problema

dy 1 1
= 2 y2 − y + 1
dx x x
y (1) = 3

Indicación:Primero buscar una solución dela ecuación de la forma y = ax + b.

Solución. Usamos la indicación para buscar una solución (la ecuación es de Ricatti)
entonces
x2 a = (ax + b)2 − x (ax + b) + x2

se sigue
ax2 = a2 − a + 1 x2 + (2ab − b) x + b2


ası́
b=0

y
a = a2 + 1 − a

que tiene solución a = 1, se sigue que una solución es

y=x

(la cual no cumple y (1) = 3) hacemos el cambio de variables

1
y= +x
u

32
Apuntes Mat023 (versión preliminar actualizada 23-05-2014)

entonces  2  
1 du 1 1 1 1
− 2 +1= 2 +x − +x +1
u dx x u x u
ası́
1 du 1 1
− 2
+1= + 2 2 +1
u dx ux u x
eliminando
1 du 1 1
− 2
= + 2 2
u dx ux u x
luego
du u 1
=− − 2
dx x x
es lineal y tiene solución
C 1
u= − ln x
x x
1
ası́, como y = u
+ x se sigue
1
y= C
+x
x
− x1 ln x
pero y (1) = 3 ası́
1
3= +1
C
1
luego C = 2
reemplazando
1
y= 1 +x
2x
− x1 ln x
es la solución del problema.

Ejemplo 1.3.13. Resolver la ecuación

y 0 − xy 2 + (2x − 1) y = x − 1

si se sabe que tiene una solución constante.

Solución. La solución constante es y = 1 (verificar) entonces podemos hacer el cambio


1
y= +1
z
dy dz
= −z −2
dx dx
reemplazamos
 2 
−2 dz 1 1
−z −x +1 + (2x − 1) +1 −x+1 = 0
dx z z
dz 1
−z −2 − 2 (x + z) = 0
dx z
33
Apuntes Mat023 (versión preliminar actualizada 23-05-2014)

ası́
dz
+ (x + z) = 0
dx
la cual es lineal, la solución es z = Ae−x − x + 1 ası́
1
y= +1
Ae−x − x + 1

Ecuaciones homogéneas

Definición 1.3.7. Una función f : U ⊆ R2 → R se dice homogénea de grado n si:

f (tx, ty) = tn f (x, y)

para todo t ∈ R tal que (tx, ty) ∈ U .

Observación 1.3.12. Si M y N son funciones homogéneas de grado n, entonces la


ecuación:
M (x, y) dx + N (x, y) dy = 0 (1.27)

se escribe como:
M 1, xy

dy M (x, y) y
=− =− = F
N 1, xy

dx N (x, y) x
donde F : R → R. Luego, la ecuación (1.27) puede escribirse como:
y
0
y =F
x
Definición 1.3.8. La ecuación de primer orden:

y 0 = f (x, y)

se llama homogénea, si la función f (x, y) es homogénea de grado 0.

Observación 1.3.13. En vista de la observación anterior, toda ecuación homogénea puede


escribirse en la forma:
y
0
y =F (1.28)
x
haciendo t = x1 . Para resolver esta ecuación se considera el cambio de variables z = xy ; o
bien, y = zx. Luego, al derivar respecto de x tenemos que:
dy dz
=z+x
dx dx
34
Apuntes Mat023 (versión preliminar actualizada 23-05-2014)

Reemplazando la ecuación anterior en la ecuación (1.28), se obtiene:

dz
z+x = F (z)
dx

la cual es una ecuación de variable separable. Finalmente, podemos escribir:


Z Z
dz dx
= +C
F (z) − z x

con C una constante arbitraria.

Ejemplo 1.3.14. Resuelva la ecuación diferencial:

x+y
y0 =
x−y

Solución. Se trata de una ecuación con función homogénea de grado cero pues
y
x+y 1+ y
y0 = = x
y =F
x−y 1− x
x
y
ponemos u = x
luego u + xu0 = y 0 reemplazando

1+u
u + xu0 =
1−u

luego
 
du 1 1+u
= −u
dx x 1−u
1 u2 + 1
 
=
x 1−u

esta ecuación es de variables separadas, se sigue

1−u
Z Z
dx
2
du =
1+u x

luego
1
ln 1 + u2 = ln |x| + C

arctan u −
2
volvemos a la variable
y   y 2 
1
arctan − ln 1 + = ln |x| + C
x 2 x

35
Apuntes Mat023 (versión preliminar actualizada 23-05-2014)

Ejemplo 1.3.15. Resuelva el P.V.I.

dy 1/2
x = y + x2 − y 2
dx
y (1) = 0

Solución. Note que


dy y
  y 2 1/2
= + 1−
dx x x
es homogénea, hacemos el cambio

y du dy
u= ⇒u+x =
x dx dx

luego
du √
u+x = u + 1 − u2
dx
se sigue √
du 1 − u2
=
dx x
resolvemos esta ecuación de variables separables
Z Z
du dx
√ =
1−u 2 x
arcsin u = ln |x| + C

ası́
u = sin (ln |x| + C)

luego
y = x sin (ln |x| + C)

evaluando
0 = sin (ln |1| + C) = sin C

se sigue C = kπ con k ∈ Z, ası́

y = x sin (ln |x| + kπ) con k ∈ Z

36
Apuntes Mat023 (versión preliminar actualizada 23-05-2014)

Otros cambios de variables

Observación 1.3.14. La regla de la cadena nos permite cambiar variables en las ecuaciones
diferenciales para llevarlas a ecuaciones diferenciales que se resuelven mediante los métodos
elementales. Consideremos los siguientes ejemplos:

Ejemplo 1.3.16. Recordemos la ecuación (1.7) obtenida en el problema del espejo pa-
rabólico. Es decir, la ecuación:
y 2y 0
=
x 1 − (y 0 )2
Utilizando un cambio de variables adecuado, resolveremos esta ecuación diferencial. En
primer lugar, despejando y 0 de la ecuación, se obtiene:
2
y (y 0 ) + 2xy 0 − y = 0

Por la fórmula de la ecuación de segundo grado, encontramos que:


p
−x ± x2 + y 2
y0 =
y
p
Por la simetrı́a de la curva y = f (x) y el hecho que |x| < x2 + y 2 , se obtiene que y 0 > 0.
Luego: p
−x + x2 + y 2
y0 = (1.29)
y
Consideremos, ahora, el cambio de variables z = x2 + y 2 . Entonces:
dz dy
= 2x + 2y
dx dx
Reemplazando la ecuación anterior en la ecuación (1.29), se obtiene:
p !
dz −x + x2 + y 2
= 2x + 2y
dx y
p
= 2x − 2x + 2 x2 + y 2

Es decir, obtenemos la ecuación de variable separable:


dz √
=2 z
dx
Ası́: Z Z
dz
√ = dx + C
2 z

37
Apuntes Mat023 (versión preliminar actualizada 23-05-2014)

implica que:

z =x+C

Por tanto, elevando al cuadrado y como z = x2 + y 2 , se obtiene finalmente:

x2 + y 2 = x2 + 2Cx + C 2

O bien:
y 2 = 2Cx + C 2

con C ∈ R.

Ejemplo 1.3.17. Resuelva la ecuación diferencial:


dy
= (x + y + 1)2 − 2
dx
utilizando para ello un cambio de variables adecuado.

Solución. Haciendo z = x + y + 1, tenemos z 0 = 1 + y 0 , de modo que:

z0 − 1 = z2 − 2

o bien:
z0 = z2 − 1

Separando variables, obtenemos:


Z Z
dz
2
= dx + C
z −1
Integrando se tiene que:
1
{ln (z − 1) − ln (z + 1)} = x + C
2
o:
z−1
ln = 2x + C
z+1
Luego:
z−1
= e2x+C
z+1
Es decir, obtenemos:
z − 1 = Ce2x (z + 1)

Despejando z y recordando que z = x + y + 1, se obtiene finalmente:


1 + Ce2x
y= −x−1
1 − Ce2x

38
Apuntes Mat023 (versión preliminar actualizada 23-05-2014)

Ejemplo 1.3.18. Para un valor de n ∈ N adecuado, el cambio de variables u = xn y


transforma la ecuación diferencial
dy p
x2 + 2xy = 1 − y 2 x4 x2
dx
en una ecuación de variables separables. Determine tal valor de n y resolver la ecuación.

Solución. Note que


y = x−n u

luego
dy du
= −nx−n−1 u + x−n
dx dx
reemplazando
 
−n du
 q
2
x −nx−n−1
u+x + 2x x−n u = 1 − (x−n u)2 x4 x2
dx
ordenando
du  p
−nx−n+1 u + x2−n + 2 x−n+1 u = 1 − (x4−2n u2 )x2
dx
se sigue
du  p
+ (2 − n) x−n+1 u = 1 − (x4−2n u2 )x2
x2−n
dx
tomando n = 2 obtenemos
du √
= 1 − u 2 x2
dx
que es de variables separables, se sigue
Z Z
du
√ = x2 dx
1 − u2
ası́
x3
arcsin u = +C
3
 
x3
luego u = sin 3
+ C y finalmente
 
x3
sin 3
+C
y=
x2
Ejemplo 1.3.19. Resolver la ecuación
2
yy 00 = y 0 y 2 + (y 0 )

haciendo el cambio u = y 0 .

39
Apuntes Mat023 (versión preliminar actualizada 23-05-2014)

Solución. Si u = y 0 entonces
du dy
y 00 =
dy dx
du
= u
dy
reemplazando
du
yu = uy 2 + (u)2
dy
du
y = y2 + u
dy
esta ecuación es lineal
du 1
− u=y
dy y
que tiene solución
u = y 2 + cy

luego
dy
= y 2 + cy
dx
es de variables separadas, tiene solución
c
y (x) =
Ae−cx −1
Obs.: Esta técnica reduce el orden de la E.D.O. si es de la forma F (y, y 0 , y 00 ) es decir,
no depende de la variable x.

Observación 1.3.15. Como muestran los ejemplos anteriores, en general, se puede efectuar
cualquier cambio de variables o sustitución que se desee al intentar resolver una ecuación
diferencial. Sin embargo, cualquier no asegura el éxito en facilitar la resolución de la
ecuación diferencial. Por tanto, debe buscarse el cambio de variables adecuado, [4].

Ejercicios de la sección

1. Resuelva las siguientes ecuaciones mediante integración directa


 dy dy
(a) x2 + 4 =4 (b) = x ln x
dx dx
1 1 dy
(c) y0 = 2
(d) e−x = sin x
arctan x (1 + x ) dx

40
Apuntes Mat023 (versión preliminar actualizada 23-05-2014)

2. Resuelva las siguientes ecuaciones de variables separables:


(a) yy 0 − x = xy 2 (b) y 0 = ex+y
(c) y ln y + xy 0 = 0 (d) (1 + ex ) y 0 = ey
(e) y 0 = 1 + x + y + xy (f) y 0 = x 2 y 2 + x2
3. Verifique que las siguientes ecuaciones son homogéneas y resuélvalas:
dy dy p
(a) 3x − y + (2y − x) = 0 (b) x = y + y 2 − x2
dx dx
2 2 dy 2 2
 dy
(d) 3x2 − y 2

(c) 4x + xy − 3y + −5x + 2xy + y = 0 = 2xy
dx dx
dy x+y dy
(e) = (f) x2 y = x3 + y 3
dx x−y dx
4. Resuelva las siguientes ecuaciones lineales de primer orden:
(a) xy 0 + y = xex (b) y 0 + y cos (x) = sin (x) cos (x)
(c) y 0 + 2y = x2 + 2x (d) y 0 + 2y = sin x
1 1
1 + x2 y 0 + xy = 1 (f) y 0 + y = 2

(e)
x x
5. Resolver las siguientes ecuaciones de Bernoulli:
1.- y 0 − 4y = 2ex y 1/2 3.- xy 0 − 2y = 4x3 y 1/2
2.- y 0 − y + y 2 (x2 + x + 1) = 0 4.- xy 0 + y = y 2 x2 ln x
6. Las ecuaciones en este ejercicios pueden ser transformadas en lineales mediante un
cambio de variables adecuado, descubra tal cambio y resuelva la ecuación:

a) y 0 + x tan y = x2 sec y
b) 2xyy 0 + (1 + x) y 2 = ex

7. Sean A, B, C, D, E, F constantes. Muestre que utilizando un cambio de coordenadas


conveniente podemos transformar una ecuación del tipo
 
dx Ax + Bt + C
=f
dt Dx + Et + F
en una ecuación de variables separables o en una homogénea. Ind.: x = X+h, y = Y +k
donde h, k son constantes por determinar.

8. Resolver el P.V.I.
dx 2x + t − 1
=
dt x + 2t + 1
x (0) = 1

41
Apuntes Mat023 (versión preliminar actualizada 23-05-2014)

9. Resuelva la ecuación de Ricatti:

y 0 − 2x3 + 1 y = −x2 y 2 − x4 − x + 1


sabiendo que u (x) = x es una solución particular de la ecuación.

10. Si u (x) = x2 es una solución particular de la ecuación:

y 0 + 3 − 2x2 sin x y = −y 2 sin x + 2x + 3x2 − x4 sin x




Hallar una familia infinita de soluciones.

Modelos simples: Segunda parte

Observación 1.4.1. Ahora consideraremos algunos ejemplos más: el problema de mezclas,


o en términos más generales, el problema de análisis de compartimientos, [4], el problema
de las curvas de persecución y un problema geométrico.

Observación 1.4.2 (Análisis de compartimientos, [4]). Un proceso fı́sico o biológico com-


plejo puede ser dividido algunas veces en varios estados distintos. El proceso total puede
describirse por la interacción entre los estados individuales. Cada estado se llama comparti-
miento (lo podemos considerar como un tanque) y se supone además que el contenido de
cada compartimiento está mezclado homogeneamente. En cada compartimiento se transfiere
material que es inmediatamente incorporado al siguiente en el sistema.
Considere un sistema formado por un solo compartimiento, suponga que un material es
introducido en tal compartimiento a una razón e (t), el cual se incorpora a una cantidad
x (t) de material existente al interior del compartimiento, y luego se extrae material (que
puede pasar a otro compartimiento) a una razón de s (t). Por consiguiente, la variación de
material al interior del compartimiento está dada entonces por la ecuación diferencial:
dx
= e (t) − s (t)
dt
Consideremos algunos ejemplos:

42
Apuntes Mat023 (versión preliminar actualizada 23-05-2014)

Ejemplo 1.4.1. Considere un tanque que contiene 100 litros de agua, en el cual se han
disuelto 50 kilogramos de sal. Suponga que 2 litros de salmuera cada uno con 1 kilogramo
de sal disuelta, entran por minuto al tanque, y la mezcla que se mantiene homogénea
revolviéndola a gran velocidad, sale del tanque a razón de 2 litros por minuto. Hallar la
cantidad de sal al interior del tanque en el tiempo t.

Solución. Sea x (t) el número de kilogramos de sal disueltos en el tanque en t minutos.


Notamos que las unidades ayudan bastante en la extracción de información. En efecto,
dx/dt está en [kg/ mı́n] y entonces, e (t) y s (t) deben esta en las mismas unidades. Ası́:
    
kg lt kg
e (t) = 2 =2
lt mı́n mı́n

y      
lt x (t) kg x (t) kg
s (t) = 2 · =
mı́n 100 lt 50 mı́n
Por tanto, la ecuación diferencial queda:

dx x (t)
=2−
dt 50

la cual es una ecuación lineal de primer orden. Ası́, por la fórmula de Leibnitz, tenemos
que:
 Z 
−t/50 t/50
x (t) = e 2 e dt + C

= 100 + Ce−t/50

43
Apuntes Mat023 (versión preliminar actualizada 23-05-2014)

pero sabemos que en t = 0, x (0) = 50. Ası́, 50 = 100 + C. Por tanto:

x (t) = 100 − 50e−t/50

Observación 1.4.3 (Curvas de persecución, [4]). Utilizando la propiedad geométrica de


que la pendiente de la recta tangente a una curva y en un punto dado de la curva es y 0 , se
pueden construir ecuaciones diferenciales que permiten estudiar la trayectoria que describe
un depredador tras su presa.

Consideremos algunos ejemplos:

Ejemplo 1.4.2. Un esquiador acuático P localizado en el punto (a, 0), con a > 0, es
halado por un bote de motor Q localizado en el origen y que viaja hacia arriba a lo largo
del eje Y . Hallar la trayectoria del esquiador si éste se dirige en todo momento hacı́a el
bote. La trayectoria se denomina tractriz.
←→
Solución. Observemos que la recta que une P y Q, digamos P Q es tangente al camino
recorrido por P . Por tanto, su pendiente está dada por:

dy a2 − x 2
=− (1.30)
dx x
puesto que la longitud del segmento P Q es a. La ecuación diferencial (1.30) es de integración
directa, luego: Z √ 2
a − x2
y=− dx + C
x
Ası́1 : √

 
a+ a2 − x 2
y = a ln − a2 − x 2 + C
x
Z √ a + √a2 ± x2

1 a2 ± x2 p
dx = a2 ± x2 − a ln +C

x x

44
Apuntes Mat023 (versión preliminar actualizada 23-05-2014)

Como y = 0 cuando x = a, vemos que C = 0, de modo que la ecuación de la trayectoria es:




 
a + a2 − x 2
y = a ln − a2 − x 2
x

Ejemplo 1.4.3. Suponga que un halcón P situado en el punto (a, 0) descubre una paloma
Q en el origen, la cual vuela a lo largo del eje Y a una velocidad v. El halcón emprende el
vuelo inmediatamente hacia la paloma a una velocidad w. ¿Cuál será el camino seguido
por el halcón en su vuelo?

Solución. Sea t = 0 el instante en que el halcón comienza a volar hacia la paloma. Después
de t segundos la paloma estará en el punto Q = (0, vt) y el halcón en P (x, y). Como la
←→
recta T = P Q es otra vez tangente a la trayectoria, encontramos que su pendiente mT = y 0 .
Luego, se obtiene la ecuación diferencial:

y − vt
y0 = (1.31)
x

Debemos ahora eliminar t de la ecuación anterior, pues y 0 = dy/dx. Para ello debemos
calcular la longitud del camino recorrido por el halcón. Si ds representa un elemento
diferencial de longitud del arco formado por la trayectoria, tenemos que:
Z a
wt = ds
x
q
pero ds = 1 + (y 0 )2 dx. Ası́, la fórmula anterior queda como:
Z a
1
q
t= 1 + (y 0 )2 dx
w x

Despejando t de la ecuación (1.31) e igualando con la ecuación anterior, obtenemos:

y − xy 0 1 a
Z q
= 1 + (y 0 )2 dx
v w x

Derivando:
y 0 − (y 0 + xy 00 ) 1
q
= 1 + (y 0 )2
v w
Ordenando la ecuación anterior, se tiene que:

v
q
00
xy = 1 + (y 0 )2 (1.32)
w

45
Apuntes Mat023 (versión preliminar actualizada 23-05-2014)

Note que la ecuación anterior es una ecuación diferencial de segundo orden. Sin embargo,
mediante el cambio de variables:
u = y0

la ecuación (1.32) queda como:


v√
xu0 = 1 + u2
w
que es una ecuación de primer orden de variable separable. Entonces, separando variables
e integrando, obtenemos: Z Z
du v dx
√ = +C
1+u 2 w x
Luego:
 √  v
2
ln u + 1 + u = ln x + K
w
pero u = y 0 = 0 cuando x = a, se sigue que K = (v/w) ln a. Tomando exponenciales a
ambos lados de la ecuación anterior se tiene:
√  x v/w
u+ 1 + u2 =
a
que, después de algunas operaciones algebraicas, nos da:
 
dy 1  x v/w  x −v/w
= −
dx 2 a a
Suponiendo que w > v, se obtiene finalmente que:
( )
a (x/a)1+v/w (x/a)1−v/w
y= − +C
2 1 + v/w 1 − v/w

1 3

Ejemplo 1.4.4. Determine la curva que pasa por ,
2 2
y corta a cada miembro de la
familia x2 + y 2 = c2 con c ∈ R+ formando un ángulo de 45o .

Solución. El ángulo entre dos curvas es dado por el ángulo entre sus rectas tangentes
luego
m1 − m2
tan θ =
1 + m1 m2
note que la pendiente de las rectas tangentes a la familia de curvas es
−x
2x + 2yy 0 = 0 ⇒ m1 =
y

46
Apuntes Mat023 (versión preliminar actualizada 23-05-2014)

ası́ −x
π 
y
− y0
tan =
4 1 − xy y 0
esto es −x
y
− y0
1=
1 − xy y 0
se sigue
x −x
1 − y0 = − y0
y y

x x 0
1+ = y − y0
y y
luego
dy 1 + xy y
+1
= x = x y
dx y
−1 1− x
y
que es una ecuación homogenea , hacemos el cambio u = x
de donde
du u+1
u+x =
dx 1−u
luego
du u+1
x = −u
dx 1−u

u2 + 1
 
du 1
=
dx 1−u x
resolvemos
1−u
Z Z
1
du = dx
1 + u2 x
se sigue
1
ln 1 + u2 = ln |x| + C

arctan u −
2
volvemos a la variable y,
y 1
ln x2 + y 2 = C

arctan −
x 2
y determinamos la constante con el punto 21 , 23

 
1 5
arctan (3) − ln =C
2 2
ası́ la curva es  
y 1 1 5
2 2

arctan − ln x + y = arctan 3 − ln
x 2 2 2
.

47
Apuntes Mat023 (versión preliminar actualizada 23-05-2014)

Ejercicios de la sección

1 3

1. Determinar la curva que pasa por ,
2 2
y corta a cada miembro de la familia de
curvas x2 + y 2 = c2 formando un ángulo de 30o .

2. Encuentre la curva que pertenece a la familia de trayectorias ortogonales de la familia


de curvas x + y = cey que pasa por (0, 5).

3. Suponga que un halcón situado en (a, 0) descubre una paloma en el origen, la


cual vuela a lo largo del eje Y a una velocidad v; El halcón emprende el vuelo
inmediatamente hacia la paloma con una velocidad de w. ¿Cual es el camino seguido
por el halcón en su vuelo persecutorio?.

4. Un destructor está en medio de una niebla muy densa que se levanta por un momento
y deja ver un submarino enemigo en la superficie a cuatro kilómetros de distancia.
Suponga:

a) que el submarino se sumerge inmediatamente y avanza a toda máquina en una


dirección desconocida.

b) que el destructor viaja tres kilómetros en lı́nea recta hacia el submarino.

¿Qué trayectoria deberı́a seguir el destructor para estar seguro que pasará directamente
sobre el submarino, si su velocidad v es tres veces la del submarino?

5. Suponga que el eje Y y la recta x = b forman las orillas de un rı́o cuya corriente tiene
una velocidad v en la dirección negativa del eje Y . Un hombre esta en el origen y
su perro esta en el punto (b, 0). Cuando el hombre llama al perro, este se lanza al
rı́o y nada hacia el hombre a una velocidad constante w (con w > v). ¿Cuál es la
trayectoria seguida por el perro?.

6. Cuatro caracoles situados en las esquinas de un cuadrado [0, a] × [0, a] comienzan a


moverse con la misma velocidad, dirigiéndose cada uno hacia el caracol situado a su
derecha. ¿Qué distancia recorrerán los caracoles al encontrarse?

48
Apuntes Mat023 (versión preliminar actualizada 23-05-2014)

7. Hallar la ecuación de todas las curvas que tienen la propiedad de que el punto de
tangencia es punto medio del segmento tangente entre los ejes coordenados.

8. Un cuerpo se calienta a 110o C y se expone al aire libre a una temperatura de 100 C.


Si al cabo de una hora su temperatura es de 60o C. ¿Cuánto tiempo adicional debe
transcurrir para que se enfrı́e a 30o C?

9. Una persona de un pueblo de 1000 habitantes regresó con gripe. Si se supone que la
gripe se propaga con una rapidez directamente proporcional al número de agripados
como también al número de no agripados. Determinar el número de agripados cinco
dı́as después, si se observa que el número de agripados el primer dı́a es 100.

10. Un colorante sólido disuelto en un lı́quido no volátil, entra a un tanque a una velocidad
v1 galones de solución/minuto y con una concentración de c1 libras de colorante/galón
de solución. La solución bien homogeneizada sale del tanque a una velocidad de v2
galones de solución/min. y entra a un segundo tanque del cual sale posteriormente a
una velocidad de v3 galones de solución/min.

Inicialmente el primer tanque tenı́a P1 libras de colorante disueltas en Q1 galones


de solución y el segundo tanque P2 libras de colorante disueltas en Q2 galones de
solución. Encontrar dos ecuaciones que determinen las libras de colorante presentes
en cada tanque en cualquier tiempo t.

11. Un teatro de dimensiones 10 × 30 × 50m3 , contiene al salir el público 0,1 % por


volumen de CO2 . Se sopla aire fresco a razón de 500 m3 por minuto y el sistema
de aire acondicionado lo extrae a la misma velocidad. Si el aire atmosférico tiene

49
Apuntes Mat023 (versión preliminar actualizada 23-05-2014)

un contenido de CO2 del 0,04 % por volumen y el lı́mite saludable es de 0,05 % por
volumen. ¿ En que tiempo podrá entrar el público?.

12. Un tanque contiene inicialmente agua pura. Salmuera que contiene 2 libras de sal/gal.
entra al tanque a una velocidad de 4 gal./min. Asumiendo la mezcla uniforme, la
salmuera sale a una velocidad de 3 gal./min. Si la concentración alcanza el 90 % de
su valor máximo en 30 minutos, calcular los galones de agua que habı́an inicialmente
en el tanque.

13. Un tanque de una cierta forma geométrica está inicialmente lleno de agua hasta una
altura H. El tanque tiene un orificio en el fondo cuya área es A pie2 . Se abre el orificio
dQ
y el lı́quido cae libremente. La razón volumétrica de salida dt
es proporcional a la
velocidad de salida y al área del orificio, es decir,

dQ
= −kAv
dt

aplicando la ecuación de energı́a 12 mv 2 = mgh se obtiene v = 2gh donde g = 32
pie/seg2 .

La constante k depende de la forma del orificio:

a) Si el orificio es de forma rectangular, la constante k = 0, 8.

b) Si el orificio es de forma triangular, la constante 0, 65 ≤ k ≤ 0, 75.

c) Si el orificio es de forma circular, la constante k = 0, 6.

Con estos datos:

a) Un tanque semiesférico tiene un radio de 1 pie; el tanque está inicialmente lleno


de agua y en el fondo tiene un orificio de 1 pulg. de diámetro. Calcular el tiempo
de vaciado.

b) Modelar el caso: Cilindro circular de altura H0 pies y radio r pies, dispuesto en


forma vertical y con un orificio circular de diámetro ρ (pulgadas), suponga que
esta lleno de agua y calcule el tiempo de vaciado.

50
Apuntes Mat023 (versión preliminar actualizada 23-05-2014)

14. Un torpedo se desplaza a una velocidad de 60 millas/hora en el momento de agotarse


el combustible; si el agua se opone al movimiento con una fuerza proporcional a su
velocidad y si en una milla de recorrido reduce su velocidad a 30 millas/hora. ¿A que
distancia se detendrá?

15. Una bala se introduce en una tabla de h = 10 cm. de espesor con una velocidad v0 =
200 m/seg, traspasándola con v1 = 80 m/seg. Suponiendo que la resistencia de la
tabla al movimiento de la bala es proporcional al cuadrado de la velocidad. Hallar el
tiempo que demora la bala en atravesar la tabla.

16. Una cadena de 4 pies de longitud tiene 1 pie de longitud colgando del borde de una
mesa. Despreciando el rozamiento, hallar el tiempo que tarda la cadena en deslizarse
fuera de la mesa.

Análisis cualitativo

Es equivocado pensar que el objetivo principal del estudio de las ecuaciones diferenciales
consiste en encontrar artificios de cálculo que permitan resolverlas. Anteriormente presenta-
mos una selección de técnicas que permiten resolver algunas ecuaciones diferenciales. Como
en toda selección la lista no es completa. Existen tratados en donde se elaboran tablas de
soluciones de manera análoga a las tablas de antiderivadas.

La pericia para resolver ecuaciones diferenciales va perdiendo poco a poco importancia


con la llegada de los computadores y el diseño de software especializado para computación
simbólica. La tendencia actual es dejar al computador este tipo de tareas de cálculo. Un
programa como Mathematica puede resolver mediante instrucciones sencillas casi todas las
ecuaciones diferenciales tratadas en este curso.

Sin quitarle importancia a este tipo de programas debe quedar claro que ni el más
refinado de los software ni el más ingenioso de los matemáticos puede resolver en términos
de funciones elementales todas las ecuaciones diferenciales, ni siquiera las más importantes
de ellas. El problema más que de habilidad es de principio. En casos tan simples como
dx 1
=− +t
dt x
51
Apuntes Mat023 (versión preliminar actualizada 23-05-2014)

se desconocen soluciones clásicas. La búsqueda de recetas para resolver todas las ecuaciones
diferenciales en términos de funciones elementales es una búsqueda sin esperanzas. Ante este
hecho se presentan algunas alternativas: Los métodos cualitativos, los métodos numéricos,
y los métodos de aproximación. No es parte de los objetivos de estas notas un estudio
detallado al respecto. Se quiere sin embargo ilustrar los métodos cualitativos.

Métodos cualitativos

En muchos problemas, más que cálculos cuantitativos puntuales, lo que interesa es el


comportamiento cualitativo de las soluciones en términos de las condiciones iniciales o de
valores de los parámetros. Saber que una solución es creciente, que es cóncava o que tiene
un lı́mite en el infinito puede ser de ayuda en el entendimiento de un modelo. Ocurre, que
bajo ciertas circunstancias, podemos obtener tal información sin resolver explı́citamente la
ecuación diferencial. Analizaremos primero el siguiente modelo:

El modelo de Verhulst

Resumiremos los principales resultados concernientes al modelo de Verhulst para la


dinámica de poblaciones
dx
= x (a − bx) (1.33)
dt
donde a, b > 0, esta ecuación tiene dos soluciones constantes x1 (t) = 0 y x2 (t) = ab . Estas
soluciones dividen al plano xt en 3 regiones de poblaciones
n a o n ao
R1 = (t, x) : < x , R2 = (t, x) : 0 < x < , R3 = {(t, x) : x < 0}
b b

tales que el gráfico de cualquier solución no constante x = x(t) de (1.33) permanece


confinado en una y sólo una de estas regiones. Más aún, podemos determinar cuándo es
creciente, y cuándo es decreciente la solución x = x(t) de (1.33) a partir de la condición
inicial x(t0 ) = x0 .

52
Apuntes Mat023 (versión preliminar actualizada 23-05-2014)

Z Z
dx
= dt
x (a − bx)
Z  
1 1 b
+ = t+C
ax a a − bx
 
1 1
ln |x| − ln |a − bx| = t+C
a a

x
ln = at + C
a − bx
x
se sigue a−bx
= Keat ,
Kaeat a
x (t) = =
at
Kbe + 1 b + Ce−at
 
a 1 a
1. Si x0 < 0 entonces x0 = b+Ce−at0
, entonces C = − e−at 0
b− x0
, se sigue

ax0
x (t) =
bx0 + (a − bx0 ) e−a(t−t0 )

note que ax0 < 0 y

bx0 + (a − bx0 ) e−a(t−t0 ) = 0



(a − bx0 ) e−a(t−t0 ) = −bx0

−bx0
e−a(t−t0 ) = >0
(a − bx0 )

ası́
1 −bx0
T = − ln + t0 > t0
a (a − bx0 )
i h
−bx0
el intervalo de definición de esta solución es −∞, − a1 ln (a−bx0)
+ t0 ≡ ]−∞, T [ en

este intervalo

d ax0 a2 x0 e−a(t−t0 ) (a − bx0 )


= <0
dt bx0 + (a − bx0 ) e−a(t−t0 ) (bx0 + aeat0 −at − bx0 eat0 −at )2

la función es estrictamente decreciente y

ax0
lı́m− = −∞
t→T bx0 + (a − bx0 ) e−a(t−t0 )

53
Apuntes Mat023 (versión preliminar actualizada 23-05-2014)

Diremos que esta solución explota en un tiempo finito. Note que esta solución no
representará una solución asociada al problema de poblaciones pues siempre es
negativa.

a ax0
2. Si 0 < x0 < b
entonces x (t) = bx0 +(a−bx0 )e−a(t−t0 )
esta bien definida en todo R pues el
denominador no se anula

0 < bx0 + (a − bx0 ) e−a(t−t0 )

además
ax0 a
0 < x (t) = −a(t−t )
<
bx0 + (a − bx0 ) e 0 b
y
d ax0 a2 x0 e−a(t−t0 ) (a − bx0 )
= >0
dt bx0 + (a − bx0 ) e−a(t−t0 ) (bx0 + aeat0 −at − bx0 eat0 −at )2
la función es estrictamente creciente y

ax0 a
lı́m =
t→+∞ bx0 + (a − bx0 ) e−a(t−t0 ) b

además
ax0
lı́m =0
t→−∞ bx0 + (a − bx0 ) e−a(t−t0 )

i h
a ax0 −bx0
3. Si x0 > b
entonces x (t) = bx0 +(a−bx0 )e−a(t−t0 )
esta bien definida en − a1 ln (a−bx0)
+ t0 , +∞ ≡
]T, +∞[ donde T < t0 además

ax0 a
lı́m −a(t−t )
=
t→+∞ bx0 + (a − bx0 ) e 0 b

y
d ax0 a2 x0 e−a(t−t0 ) (a − bx0 )
= <0
dt bx0 + (a − bx0 ) e−a(t−t0 ) (bx0 + aeat0 −at − bx0 eat0 −at )2
la función es estrictamente decreciente, x (t) > ab .

54
Apuntes Mat023 (versión preliminar actualizada 23-05-2014)

En el gráfico se muestra el comportamiento de las soluciones

Veremos ahora que es posible analizar los comportamientos de las soluciones sin la
necesidad de resolver la ecuación.

Ecuaciones diferenciales autónomas

Definición 1.5.1. Diremos que una ecuación diferencial de primer orden es autónoma si
se puede expresar en la forma
dx
= f (x) (1.34)
dt
donde f : Ω → R es una función definida en el intervalo abierto Ω.

2
Las ecuaciones dx
dt
= x (1 − x2 ); dx
dt
= x (1 − x); dx
dt
= e−x sin x son ejemplos de
dx
ecuaciones autónomas, mientras la ecuación dt
= et x + t no lo es.

Teorema 1.5.1. Sean f : Ω → R una función de clase C 1 (Ω), x0 ∈ Ω y t0 ∈ R entonces


existe una única solución del P.V.I.

dx
= f (x)
dt
x (t0 ) = x0

x : I → R de clase C 1 (I) definida en un intervalo abierto que contiene a t0 .

55
Apuntes Mat023 (versión preliminar actualizada 23-05-2014)

Definición 1.5.2. Llamaremos intervalo maximal de definición al mayor intervalo abierto


donde esta definida la solución del P.V.I. anterior.

Ejemplo 1.5.1. Determine el intervalo maximal de la solución del P.V.I.


dx
= x2
dt
x (0) = 2

Solución. Aplicando la técnica de separación de variables


Z 0 Z
x (t)
dt = 1dt
x (t)2

−1
= t+C
x (t)

−1
x (t) =
t+C
−1
usando la condición inicial 2 = C
entonces c = −1/2 se sigue
−1
x (t) =
t − 12

el mayor intervalo que contiene a t = 0 en el cual esta función esta definida es I = −∞, 12
 

el cual corresponde al intervalo maximal.

Observación 1.5.1. Note que las ecuaciones autónomas son de variables separadas.

Definición 1.5.3. Las soluciones constantes x (t) = c, t ∈ R de la ecuación (1.34) son


llamadas soluciones de equilibrio.

Note que si x (t) = c es una solución de equilibrio


dx (t)
0= = f (x (t)) = f (c)
dt
en otras palabras las soluciones de equilibrio corresponden a las raı́ces de la ecuación
f (x) = 0.

56
Apuntes Mat023 (versión preliminar actualizada 23-05-2014)

Ejemplo 1.5.2. Determine las soluciones de equilibrio de la ecuación


dx
= sin x + sin2 x
dt
Solución. Las soluciones de equilibrio corresponden a las raı́ces de la ecuación

sin x + sin2 x = 0

esto es
(sin x) (1 + sin x) = 0

luego x = kπ con k ∈ Z o bien x = 2
+ 2lπ con l ∈ Z.

Teorema 1.5.2. Las soluciones de la ecuación (1.34) son soluciones de equilibrio o


funciones monótonas estrictas.

Si x (t) es una solución definida en su intervalo maximal I, mostraremos que la derivada


no se puede anular a menos que la solución sea de equilibrio. Supongamos que x0 (tc ) = 0
para algún tc ∈ I y definamos φ (t) = x (tc ) = α una función constante, como

0 = φ0 (t)

y
x0 (tc ) = f (x (tc ))

se sigue
0 = φ0 (t) = f (φ (t))

luego φ (t) es solución de equilibrio pero


dx
= f (x)
dt
x (tc ) = α

tiene 2 soluciones, por el teorema de existencia y unicidad la solución debe ser la misma,
es decir x (t) = α para todo t.

Esta demostración tiene otra consecuencia

57
Apuntes Mat023 (versión preliminar actualizada 23-05-2014)

Teorema 1.5.3. Las gráficas de dos soluciones distintas de (1.34) no se intersectan.

Teorema 1.5.4. Sean Ω = ]α, β[, x0 ∈ Ω, t0 ∈ R. Si x = x (t) es solución de (1.34) con


x (t0 ) = x0 y su intervalo maximal de definición es ]a, b[ entonces los lı́mites

lı́m x (t) = A y lı́m− x (t) = B


t→a+ t→b

existen o son ±∞, más aún, A debe tomar el valor α o β si a =


6 −∞, y B debe tomar uno
de los valores α o β si b 6= ∞.

Ejemplo 1.5.3. Consideremos la ecuación

dx
= x (1 − x)
dt

en este caso Ω = R ası́ α = −∞ y β = +∞. Para 0 < x0 < 1 sean x (t) definida en ]a, b[ la
solución que cumple x (t0 ) = x0 , note que x1 (t) = 0 y x2 (t) = 1 son soluciones de equilibrio
luego
0 < x (t) < 1

se sigue que 0 ≤ A, B ≤ 1 entonces a = −∞ y b = +∞. La solución estará definida en


todo R.

Teorema 1.5.5. Si c ∈ Ω y x (t) es una solución de (1.34) tal que lı́mt→+∞ x (t) = c o
lı́mt→−∞ x (t) = c entonces φ (t) = c, t ∈ R es una solución de equilibrio.

Demostración. Tenemos que probar que f (c) = 0. Supongamos que f (c) > 0 entonces
por la continuidad de f existirı́a un intervalo ]c − δ, c + δ[ tal que x ∈ ]c − δ, c + δ[ implica
f (c)
f (x) > 2
> 0, como
lı́m x (t) = c
t→+∞

58
Apuntes Mat023 (versión preliminar actualizada 23-05-2014)

se sigue que existe un t0 tal que t ≥ t0 implica |x (t) − c| < δ se sigue


Z t
x (t) = x (t0 ) + x0 (u) du
t0
Z t
= x (t0 ) + f (x (u)) du
t0
Z t
f (c)
> x (t0 ) + du
t0 2
f (c)
= x (t0 ) + (t − t0 )
2

para t ≥ t0 pero esto contradice lı́mt→+∞ x (t) = c (se puede llegar a una contradicción
similar si f (c) < 0).

Equilibrio y estabilidad

Notemos que en la ecuación autónoma

dx
= f (x)
dt

se nos indica la pendiente de la recta tangente


a la gráfica de la función solución x = x (t) es-
ta viene dada por f (x), esto es, dado un valor
de x las pendientes siempre son las mismas
(independiente de t), las curvas isoclinas co-
rresponde x = c, además en los intervalos en
los cuales f (x) > 0 la solución es estrictamen-
te creciente y en los intervalos en los cuales
f (x) < 0 la función solución es estrictamente
decreciente.
Si x (t) es una solución de
dx
= f (x)
dt
entonces la función ϕ (t) = x (t + c) también es solución, en efecto

ϕ0 (t) = x0 (t + c) = f (x (t + c)) = f (ϕ (t))

59
Apuntes Mat023 (versión preliminar actualizada 23-05-2014)

esto significa que las traslaciones de una solución de la ecuación autónoma también es
solución, note que si se conoce la solución x (t) de

dx
= f (x)
dt
x (t0 ) = x0

entonces la solución de

dx
= f (x)
dt
x (µ0 ) = x0

es ϕ (t) = x (t + (t0 − µ0 )), esto nos dice que en cada banda limitada por las las soluciones
de equilibrio las soluciones son traslaciones de una solución dada.
Ejemplo 1.5.4. Considere la ecuación

dx
=x
dt

en este caso la solución de equilibrio es


x (t) = 0. Por teorema las demás soluciones
son monótonas estrictas. Para x > 0 las so-
luciones son estrictamente crecientes y para
x < 0 estrictamente decrecientes. En este caso
es fácil resolver explı́citamente

x1 (t) = et

es una solución en la región x > 0 las demás


soluciones son xc (t) = et+k = ek et = Cet
donde C > 0 son traslaciones de la solución
anterior. Note también que x (t) = −et es
una solución en la región x < 0 y las demás
soluciones en esa región son xK (t) = −et+k =
−Ket
En el ejemplo anterior el comportamiento de las soluciones en el entorno de la solución

60
Apuntes Mat023 (versión preliminar actualizada 23-05-2014)

de equilibrio es como indica el diagrama

diremos en este caso que el punto de equilibrio es un repulsor, las soluciones se alejan de
esta solución de equilibrio.

Llamaremos diagrama de fases o lı́neas de fases al gráfico del comportamiento de las


soluciones en el plano xt.

Definición 1.5.4. Dependiendo del comportamiento local de las soluciones de la ecuación


alrededor de un punto de equilibrio aislado x0 en las lı́neas de fases, se distinguen los
siguientes tipos:

1. x0 es llamado Repulsor si existe δ > 0 tal que

x0 − δ < x < x0 x = x0 x0 < x < x 0 + δ


signo f (x) −−− 0 +++

2. x0 es llamado Atractor si existe δ > 0 tal que

x0 − δ < x < x0 x = x0 x0 < x < x 0 + δ


signo f (x) +++ 0 −−−

3. x0 es llamado Atractor-repulsor si existe δ > 0 tal que

x0 − δ < x < x0 x = x0 x0 < x < x 0 + δ


signo f (x) +++ 0 +++

61
Apuntes Mat023 (versión preliminar actualizada 23-05-2014)

4. x0 es llamado Repulsor-atractor si existe δ > 0 tal que

x0 − δ < x < x0 x = x0 x0 < x < x 0 + δ


signo f (x) −−− 0 −−−

Para analizar entonces el tipo de solución de equilibrio tenemos que analizar el signo
de la función en el entorno de la solución de equilibrio, adicionalmente se cuenta con el
siguiente teorema:

Teorema 1.5.6. Si x (t) = c es una solución de equilibrio de


dx
= f (x)
dt
entonces:

1. Si f 0 (c) < 0, c es un atractor

2. Si f 0 (c) > 0, c es un repulsor.

Ejemplo 1.5.5. Bosquejar el diagrama de


fases de la ecuación

dx
= x2 (2 − x) (x − 3)
dt

Desarrollo: La función x2 (2 − x) (x − 3) esta


bien definida y es de clase C ∞ (R) existe so-
lución única en cada punto (t0 , x0 ) del plano
xt. Las soluciones de equilibrio corresponden
a x1 (t) = 0, x2 (t) = 2 y x2 (t) = 3, en el
siguiente diagrama se analiza el signo de f

0 2 3
x2 +++ 0 +++ + +++ + +++
x−2 −−− − −−− 0 +++ + +++
x−3 −−− − −−− − −−− 0 +++
f (x) = −x2 (x − 2) (x − 3) −−− 0 −−− 0 +++ 0 −−−

62
Apuntes Mat023 (versión preliminar actualizada 23-05-2014)

se sigue que x1 (t) = 0 es un repulsor-atractor, x2 (t) = 2 un repulsor y x2 (t) = 3 un


atractor.

Ejercicios de la sección

1. En una red de computadores sea x (t) la proporción de máquinas infectadas con el


virus MTA320 en un instante t dado (note que 0 ≤ x (t) ≤ 1). x0 (t) mide la rapidez
de propagación sobre la red. Los estudios de expertos de internet determinan que el
virus satisface el problema con valores iniciales
  
0 1 2
x = αx −x − x (1 − x)
3 3
x (0) = x0

donde x0 es la proporción de computadores en que el virus se implanta inicialmente


y α > 0 es una constante.

a) Encuentre las soluciones de equilibrio, clasificarlas y dibujar el diagrama de fase


del sistema.

b) Demostrar que si 0 < x0 < 2/3 entonces el virus alcanzará finalmente a un


tercio del total.

63
Apuntes Mat023 (versión preliminar actualizada 23-05-2014)

c) ¿Cuál debe ser la cantidad de máquinas infectada inicialmente para que todas
se infecten?

2. Sea x : I → R la solución maximal del P.V.I.

dx 2
= x2 (1 − x) (2 − x) ex
dt
3
x (0) =
2

muestre que I = R y determine lı́mt→+∞ x (t), lı́mt→−∞ x (t).

3. Considere la ecuación diferencial

dy
= y 2/3
dt

a) Demuestre que y1 (t) = 0 para todo t ∈ R es solución.

b) Comprobar que y2 (t) = t3 /27 es también solución.

c) Notar que y1 (0) = y2 (0) pero ambas funciones no son iguales para toda t ¿Por
qué este ejemplo no contradice el teorema de unicidad?

4. Esboce las lı́neas de fase para la ecuación diferencial dada. Identifique los puntos de
equilibrio:
dy dy dy
(a) dt
= 3y (1 − y) (b) dt
= y 2 − 6y − 16 (c) dt
= cos y

dw dw dy 1
(d) dt
= w cos w (e) dt
= (w − 2) sin w (f) dt
= y−2

dw dy
(g) dt
= w2 + 2w + 10 (h) dt
= tan y

5. Considere la ecuación diferencial

dy
= y 2 − 4y + 2
dt

con las siguientes condiciones iniciales:

a) y (0) = 0

b) y (0) = 1

64
Apuntes Mat023 (versión preliminar actualizada 23-05-2014)

c) y (0) = −1

d ) y (0) = −10

e) y (0) = 10

f ) y (3) = 1

describir el comportamiento a largo plazo de la solución.

6. Describir el diagrama de fases de la ecuación


dx
= (x + λ) x2 + λ (cos x + 2)

dt
para los distintos valores del parámetro λ.

1
7. Determine los valores de α ∈ R para los cuales la solución de equilibrio x (t) = 3
de
 
dx 1
− x x2 − α

= (x − α)
dt 3
es:

a) Un atractor.

b) Un repulsor.

o justifique la no existencia de tales valores.

8. Sea x (t) la solución del P.V.I.


dx 2
= ex−x sin (x) arctan x
dt
x (3) = 4

a) Determine su intervalo maximal de definición.

b) ¿Es x (t) estrictamente creciente?

c) Si existen, calcular el valor de lı́mt→+∞ x (t) y lı́mt→−∞ x (t)

9. Sean α, β > 0. La ecuación


dP
= αP 2/3 − βP
dt
modela el peso de un pez en el tiempo t. Sin resolver la ecuación, determine el peso
máximo del pez si P (0) = p0 > 0 (justificar sus cálculos).

65
Apuntes Mat023 (versión preliminar actualizada 23-05-2014)

10. Muestre que la solución de equilibrio y (t) ≡ 0 de la ecuación

dy
= y cos y 5 + 2y − 27πy 4
 
dt

corresponde a un repulsor.

66
Apuntes Mat023 (versión preliminar actualizada 23-05-2014)

Ejercicios del capı́tulo

1. Resuelva las siguientes ecuaciones diferenciales:


dy 1−x2 dy dy sec2 x
a) dx
= y2
b) dx
= y (2 + sin x) c) dx
= 1+x2

dy 1 dy dv 1−4v 2
d) dx
= xy 3
e) dx
= 3xy 2 f) x dx = 3v

dx dy dy2
g) dt
+ x2 = x h) dx
= 3x2 (1 + y 2 ) i) (x + xy 2 ) + ex y dx =0

2. Resuelva los siguientes problemas de valor inicial:


dy

a) y 0 = x3 (1 − y) , y (0) = 3 b) dx
= (1 + y 2 ) tan x, y (0) = 3

dy 3x2 +4x+2 dy √
c) dx
= 2y+1
, y (0) = −1 d) dx
= 2 y + 1 cos x, y (0) = 2

dy dy
e) dx
= y sin x, y (π) = −3 f) x2 + 2y dx = 0, y (1) = 2

3. Resuelva las siguientes ecuaciones lineales:


dy dy y dr
a) dx
− y = e3x b) dx
= x
+ 2x + 1 c) dθ
+ r tan θ = sec θ

dy dy dy
d) x dx + 3y + 2x2 = x3 + 4x e) (x2 + 1) dx + xy = x f) dx
= x2 e−4x − 4y

4. Considere el problema con valor inicial:

dy p
+ y 1 + sin2 x = x, y (0) = 2
dx

Utilice la integral indefinida para mostrar que el factor integrante para la ecuación
diferencial se puede escribir como:
Z x p 
2
µ (x) = exp 1 + sin t dt
0

5. Resuelva las siguientes ecuaciones diferenciales:

67
Apuntes Mat023 (versión preliminar actualizada 23-05-2014)

a) dy
(x − y) + x dx =0 b) dy
dx
= (3x + 2y − 1)2 + 2 c) dy
dx
= sin (x − y)

√ √
x2 +t t2 +x2 dy y(ln y−ln x+1)
d) y 0 + 1 = x+y e) dx
dt
= tx
f) dx
= x

6. Hallar la solución general de la ecuación de Bernoulli:

2y 0 + y tan x = (−2x sec x) y 3

7. Resuelva la ecuación:
3y 0 = (1 − 2t) y 4 − y

sabiendo que y (0) = 1.

8. Resolver:
dy 1
=y−x−1+
dx x−y−2
9. Considere la ecuación diferencial:

y − xy 0 = a 1 + x2 y 0 ,

a>1

a) Hallar la solución general de la ecuación.

b) Encuentre una solución particular de la ecuación, tal que y (0) = 1.

c) Hallar el intervalo más grande donde la solución particular anterior esté definida.

10. Demuestre que la ecuación diferencial:

2x4 y y 0 + y 4 = 4x6

se reduce a una ecuación homogénea mediante el cambio de variables:

y = zn

para cierto n ∈ Z. Determine el valor de n y resuelva la ecuación.

11. Muestre que la ecuación:

2 x3 y − y 3 dy = 3 x5 + x4 y 4 dx
 

68
Apuntes Mat023 (versión preliminar actualizada 23-05-2014)

se reduce a una ecuación homogénea, realizando el cambio de coordenadas:

x = up ∧ y = vq

para ciertas constantes adecuadas p, q ∈ R. Hallar tales constantes y resolver la


ecuación diferencial.

12. Hallar la solución general de la ecuación:


dy
(x − 2y − 4) + (2x − y + 2) =0
dx

13. Considere la ecuación de Ricatti:

y 0 + 2 (1 − x) y − y 2 = x (x − 2)

a) Hallar constantes A, B ∈ R de modo que:

y = Ax + B

sea una solución particular de la ecuación.

b) Determine la solución general de la ecuación diferencial.

14. Resuelva la ecuación:


dy y
= x3 (y − x)2 +
dx x
15. Sea x > 0. Considere la ecuación:
1 e2x
y 0 + e−2x y 2 − 1 + 4x + 2x2 y = − 1 + x + 2x2 + x3
 
x x

a) Hallar una solución particular de la forma:

y = e2x (Ax + B)

b) Resuelva la ecuación diferencial.

16. Un esquiador acuático ubicado en el punto (a, 0) es tirado por un bote que se
encuentra en el origen O y viaja hacia el norte en dirección OY . Hallar la trayectoria
que sigue el bote si este se dirige en todo momento hacia el bote.

69
Apuntes Mat023 (versión preliminar actualizada 23-05-2014)

17. Considere un tanque que contiene inicialmente 1000 litros de agua pura, dentro del
cual empieza a fluir una solución salina a razón de 6 litros por minuto. La solución
dentro del estanque se mantiene bien agitada y fluye hacia el exterior del tanque
a una velocidad de 5 litros por minuto. Si la concentración de sal en la salmuera
que entra en el estanque es de 1 kilogramo por litro, determine el instante en que la
63
concentración de sal dentro del tanque sea de 64
kilogramo por litro.

18. El modelo de Malthus para el crecimiento de poblaciones está dado por la ecuación:

dN
= rN, N (0) = N0
dt

donde r es una constante positiva intrı́nseca a la población y N0 es la población


inicial.

a) Resuelva la ecuación diferencial.

b) En 1790, la población de EE.UU. era de 3, 93 millones de habitantes, y en 1890


era de 62, 98 millones de habitantes. Estime la población para EE.UU para el
año 2000.

c) El censo realizado en el año 2000 en EE.UU. estimó en 281, 42 millones de


habitantes

d ) El modelo de Malthus sólo considera muertes por causas naturales, sin embargo,
un análisis más detallado indica que hay muertes debida a enfermedades, a
desnutrición, a crı́menes. En resumen, por la competencia entre los miembros
de la población. El modelo logı́stico implementa dichas consideraciones:

dN
= rN (N − K) , N (0) = N0
dt

para constantes positivas adecuadas r y K. Resuelva la ecuación diferencial.

e) Para la población de EE.UU. se sabe, además, que ésta alcanzó los 17, 07
millones de habitantes en 1840. Calcule, usando el modelo logı́stico, la población
de EE.UU. en el año 2000. Compare respecto al censo del año 2000.

f ) Considerando el modelo logı́stico, calcule la población lı́mite de EE.UU.

70
Apuntes Mat023 (versión preliminar actualizada 23-05-2014)

h i
cm3
19. La sangre conduce un medicamento a un órgano a razón de 3 seg
y sale de él a la
misma razón. El órgano tiene un volumen lı́quido de 125 [cm3 ]. Si la concentración
 gr 
del medicamento en la sangre que entra al órgano es de 0, 2 cm 3 :

a) ¿Cuál es la concentración del medicamento en el órgano en el instante t, si


inicialmente no habı́a rastros de dicho medicamento?

b) ¿En qué momento llegará la concentración del medicamento en el órgano a


 gr 
0, 1 cm 3

20. Una taza de café caliente, inicialmente a 95◦ C, se enfrı́a hasta 80◦ C en 5 minutos,
al estar en un cuarto con temperatura de 21◦ C. Use sólo la ley de enfriamiento de
Newton (ver MAT022) y determine el momento en que la temperatura del café estará a
unos agradables 50◦ C.

21. Era el mediodı́a en un frı́o dı́a de julio en Viña del Mar: 16◦ C. Un inspector de la
Policı́a de Investigaciones (PDI) llega a la escena de un crimen, encontrando un
detective sobre un cadáver. El detective dijo que habı́a varios sospechosos detenidos.
Dadas las coartadas se podrı́a hallar al culpable sabiendo el momento exacto de
la muerte. El inspector sacó un termómetro y midió la temperatura del cadáver:
34◦ C. Luego de realizar peritos en la escena que duraron exactamente una hora,
midió nuevamente la temperatura del cuerpo: 33, 7◦ C. Bajo que la hipótesis de que
la temperatura normal de un cuerpo humano es de 37◦ C. Determine el momento en
que ocurrió el crimen.

22. El economista inglés Thomas Malthus fue unos de los primeros en intentar modelar el
crecimiento poblacional humano por medio de matemáticas en 1978. Básicamente, el
fundamento del modelo maltusiano es la suposición de que la rapidez a la que crece
la población de un pais en cierto tiempo es proporcional a la población del paı́s en
ese momento.

a) Proponer una ecuación para este modelo.

b) Determine la ecuación diferencial si se permite inmigración a tasa constante


r > 0.

71
Apuntes Mat023 (versión preliminar actualizada 23-05-2014)

23. La ley de enfriamiento (o calentamiento) de Newton establece que la rapidez a la


que cambia la temperatura de un cuerpo es proporcional a la diferencia entre la
temperatura del cuerpo y la temperatura del medio circundante. Obtener una E.D.O.
para esta ley. Si la temperatura del medio cambia en el tiempo, escribir la ecuación
e identificarla.

24. Una enfermedad contagiosa se disemina en una comunidad por medio de la gente que
entra en contacto con otras personas. Suponer que la rapidez con que se disemina la
enfermedad es proporcional al número de interacciones entre las personas contagiadas
y no contagiadas (puede ser considerado como el producto de los contagiados y no
contagiados). Suponga que en una población pequeña de n personas se introduce un
enfermo obtener una EDO cuya solución permita obtener el número de contagiados
en un tiempo t.

25. Resolver las siguientes ecuaciones diferenciales ordinarias:

72
Apuntes Mat023 (versión preliminar actualizada 23-05-2014)

dy 1+y dy
1) = 2) − 2xy = x
dx 1+x dx

dy dy yx
3) − y tan x = cos x 4) = 2
dx dx x − y2

dy 2y + x − 2 dy 1 cos x
5) = 6) + y= 2
dx y−x+1 dx x y

dy 1 dy
7) + x2 y = 2 y 4 8) cos y + 2x sin y = −2x
dx x dx

e2x
 
dy dy 1
9) = y 2 + 2x − x4 10) + −1 y =
dx dx x x
 
dy dy y+1 y+1
11) − 4y = 2ex y 1/2 12) = + exp
dx dx x+2 x+2

2
dy dy e−y
13) = 3 |y|2/3 14) =
dx dx y (2x + x2 )

dy dy p
15) = (x − y + 3)2 16) x = y + x2 + y 2
dx dx

dy dy y
17) + y sin x = sin3 x 18) + = (1 + x) y 4
dx dx 1 + x
 2
dy dy
19) y = x +
dx dx

26. Suponga que un gran tanque de mezclado tiene inicialmente 600 galones de salmuera.
Otra solución de salmuera se bombea hacia en tanque a una rapidez de 4 galones
por minuto, la concentración de sal en el flujo de entrada es de 2 libras por galón.
Cuando la solución en el tanque esta bien agitada se bombea a 2 galones por minuto.
Determinar una EDO cuya solución corresponda a la cantidad de sal en el tanque en
el momento t.

73
Apuntes Mat023 (versión preliminar actualizada 23-05-2014)

27. Identificar el campo de direcciones con la ecuación correspondiente;

(A) y 0 = x + y

(B) y 0 = − xy

(C) y 0 = y + 1

(D) y 0 = y 2 − x2

Obs: el campo de direcciones nos indica la dirección de las rectas tangentes a las
soluciones en un punto (x, y) dado.

28. Considere la ecuación diferencial autónoma

dS
= S 3 − 2S 2 + S
dt

a) Hacer el diagrama de las lı́neas de fase..

b) Usando el dibujo anterior delinear las gráficas de las soluciones S (t) con las
condiciones iniciales siguientes: S (0) = 0; S (0) = 12 ; S (0) = 3
2
y S (0) = − 12 .

c) ¿A que tienden sus soluciones cuando t → +∞?

74
Apuntes Mat023 (versión preliminar actualizada 23-05-2014)

29. Considere la ecuación


dy 1
=
dt 1−y

a) Determine los valores de y ∈ R en los cuales es aplicable el teorema de existencia


y unicidad.

b) Obtener las soluciones explı́citamente y mostrar que llegan la y = 1 en un


tiempo finito, determine los intervalos maximales de solución

30. La ardilla negra es un pequeño mamı́fero nativo de las montañas rocallosas. Esas
ardillas son muy territoriales, por lo que si su población es grande, su razón de
crecimiento decrece y puede llegar a ser negativa. Por otra parte, si la población es
demasiado pequeña, los adultos fértiles corren el riesgo de no ser capaces de encontrar
parejas adecuadas y su tasa de crecimiento nuevamente es negativa. Muestre que el
modelo   
dP P P
= kP 1− −1
dt N M
donde k, M, N > 0 y M < N puede ser utilizado para describir la población de
Ardillas ¿Qué interpretación tienen N y M ?

75
Capı́tulo 2 : Ecuaciones diferenciales lineales de orden superior

Elementos de transformaciones lineales

Definiciones

En lo que sigue K representa R o bien C.

Definición 2.1.1. Sean U y V espacios vectoriales sobre K. Una transformación lineal


es una función T : U → V tal que para todo u, v ∈ U y α ∈ K cumple:

1. T (u + v) = T (u) + T (v)

2. T (α u) = α T (u)

Observación 2.1.1. Usualmente, los puntos (1) y (2) de la definición anterior, pueden
escribirse como

1. ∀u, v ∈ U, ∀α ∈ K, T (α u + v) = α T (u) + T (v)

o bien

2. ∀u, v ∈ U, ∀α, β ∈ K, T (α u + βv) = α T (u) + βT (v)

las tres definiciones obtenidas son equivalentes.

Observación 2.1.2. Note que si U, V son K espacios vectoriales, T : U → V es una


transformación lineal, para i = 1, · · · , n, αi son escalares en K y ui ∈ U entonces
n
! n
X X
T αi ui = αi T (ui )
i=1 i=1

luego las transformaciones lineales envı́an combinaciones lineales del espacio de partida a
combinaciones lineales del espacio de llegada.

76
Apuntes Mat023 (versión preliminar actualizada 23-05-2014)

Ejemplo 2.1.1. La función nula 0 y la función identidad 1 de un espacio vectorial en


sı́ mismo son transformaciones lineales. En efecto, Si V es un K espacio vectorial y
0 : V → V , v → 0 (v) = θV (el neutro de la adición) entonces

0 (αu + v) = θV
= αθv + θv
= α0 (u) + 0 (v)

de manera similar, si 1 : V → V , v → 1 (v) = v entonces

1 (αu + v) = αu + v
= α1 (u) + 1 (v)

Ejemplo 2.1.2. T : R2 → R4 , (x, y) → T (x, y) = (x − y, 2x, x + y, 2y) es una transfor-


mación lineal. En efecto:

1. Para cada (a, b), (u, v) ∈ R2 se tiene

T ((a, b) + (u, v))


= T (a + u, b + v)
= (a + u − b − v, 2a + 2u, a + u + v + b, 2b + 2v)
= ((a − b) + (u − v) , 2a + 2u, (a + b) + (u + v) , 2b + 2v)
= (a − b, 2a, a + b, 2b) + (u − v, 2u, u + v, 2v)
= T (a, b) + T (u, v)

2. Para cada (u, v) ∈ R2 y α ∈ R se cumple

T (α (u, v)) = T (αu, αv)


= (αu − αv, 2αu, αu + αv, 2αv)
= α (u − v, 2u, u + v, 2v)
= αT (u, v)

de esta forma T verifica las condiciones de transformación lineal.

77
Apuntes Mat023 (versión preliminar actualizada 23-05-2014)

Ejemplo 2.1.3. T : R → R2 , x → T (x) = (x, 2x + 1) no es una transformación lineal. En


efecto,

T (x + y) = (x + y, 2x + 2y + 1)
= (x, 2x + 1) + (y, 2y)
6= T (x) + T (y)

basta usar un contraejemplo:

T (1 + 2) = T (3) = (3, 7)

pero
T (1) + T (2) = (1, 3) + (2, 5) = (3, 8)

de esta forma
T (1 + 2) 6= T (1) + T (2)

luego no es una transformación lineal.

Ejemplo 2.1.4. Si R3 [x] es el espacio vectorial de los polinomios de grado menor igual a
3 entonces
T : R3 [x] → M (R)
 2×2 
p (0) p0 (0)
p → T (p) =  
p00 (0) p000 (0)
Es una transformación lineal. En efecto:

1. Si p, q ∈ R3 [x] entonces
 
(p + q) (0) (p + q)0 (0)
T (p + q) =  
(p + q)00 (0) (p + q)000 (0)

pero recordar que (p + q)0 (x) = p0 (x) + q 0 (x), (p + q)00 (x) = p00 (x) + q 00 (x) y

78
Apuntes Mat023 (versión preliminar actualizada 23-05-2014)

(p + q)000 (x) = p000 (x) + q 000 (x) se sigue


 
0
(p + q) (0) (p + q) (0)
T (p + q) =  00 000

(p + q) (0) (p + q) (0)
 
p (0) + q (0) p0 (0) + q 0 (0)
=  
p00 (0) + q 00 (0) p000 (0) + q 000 (0)
   
0 0
p (0) p (0) q (0) q (0)
=  + 
p00 (0) p000 (0) q 00 (0) q 000 (0)
= T (p) + T (q)

2. Si α ∈ R y p ∈ R3 [x] entonces
   
(αp) (0) (αp)0 (0) αp (0) αp0 (0)
T (αp) =  = 
(αp)00 (0) (αp)000 (0) αp00 (0) αp000 (0)
 
0
p (0) p (0)
= α  = αT (p)
p00 (0) p000 (0)

Note que no es necesario considerar la expresión del polinomio p (x) = a3 x3 + a2 x2 +


a1 x + a0 en esta demostración de que es transformación lineal, lo que utilizamos son
las propiedades de la derivada y matrices.

Ejemplo 2.1.5. Sea T : R3 → R2 la función definida por:

T (x, y, z) = (x + y + z, x − y)

muestre que T es una transformación lineal.

Solución. Sean α ∈ R, (u, v, w) , (x, y, z) ∈ R3 entonces

T (α (u, v, w) + (x, y, z))


= T ((αu, αv, αw) + (x, y, z))
= T (αu + x, αv + y, αw + z)
= ((αu + x) + (αv + y) + (αw + z) , (αu + x) − (αv + y))
= (αu + αv + αw, αu − αv) + (x + y + z, x − y)
= α (u + v + w, u − v) + (x + y + z, x − y)
= αT (u, v, w) + T (x, y, z)

79
Apuntes Mat023 (versión preliminar actualizada 23-05-2014)

Ejemplo 2.1.6. Sean I ⊆ R un intervalo abierto y C 1 (I) el espacio vectorial real de las
funciones de clase C 1 sobre I. Defina la transformación D : C 1 (I) → C (I) mediante:

df (x)
D (f ) (x) = , ∀x ∈ I
dx

Entonces, D es una trasformación lineal.

Solución. Por propiedades de la derivada, si α ∈ R, f, g ∈ C 1 (I) entonces

d
D (αf + g) (x) = {(αf + g) (x)}
dx
df (x) dg (x)
= α +
dx dx
= αD (f ) (x) + D (g) (x)
= (αD (f ) + D (g)) (x)

ası́
D (αf + g) = αD (f ) + D (g)

Ejemplo 2.1.7. Sea C el espacio vectorial de todas las funciones continuas de R en R y


sea T : C → C, f → T (f ) donde T (f ) es la función definida por:
Z x
T (f ) (x) = f (t) dt
1

Entonces, T es una transformación lineal.

Solución. Sean α ∈ R, f, g ∈ C entonces


Z x
T (αf + g) (x) = (αf (t) + g (t)) dt
1
Z x Z x
= αf (t) dt + g (t) dt
1 1
Z x Z x
= α f (t) dt + g (t) dt
1 1
= αT (f ) (x) + T (g) (x)
= (αT (f ) + T (g)) (x)

ası́
T (αf + g) = αT (f ) + T (g)

80
Apuntes Mat023 (versión preliminar actualizada 23-05-2014)

Ejemplo 2.1.8. Sean x, y ∈ Rn . Considere la función P : Rn → R definida por:


n
X
x 7→ P (x) = xi y i
i=1

Entonces, P es una transformación lineal. Esto es directo por las propiedades del producto
punto en Rn .

Observación 2.1.3. El conjunto de todas las transformaciones lineales de U en V se


denotará por LK (U, V ), o más sencillamente por L (U, V ) si el cuerpo de escalares no
presenta confusión. Es decir:
n o
L (U, V ) = T : U → V T es transformación lineal

Teorema 2.1.1. Sea T : U → V una transformación lineal, entonces T (θU ) = θV .

Demostración. Note que

T (θU ) = T (θU + θU )
= T (θU ) + T (θU )

sumando el inverso aditivo −T (θU ) ∈ V se tiene

T (θU ) + −T (θU ) = (T (θU ) + T (θU )) + −T (θU )

luego

θV = T (θU ) + (T (θU ) + −T (θU ))


= T (θU ) + θV

y ası́
T (θU ) = θV

Ejemplo 2.1.9. Considere la función T : R2 → R3 definida por la ecuación:

T (x, y) = (x + y, 1 − y, x)

En vista del teorema anterior, la función T no puede ser transformación lineal, pues:

T (0, 0) = (0, 1, 0)

no es el vector nulo de R3 .

81
Apuntes Mat023 (versión preliminar actualizada 23-05-2014)

Teorema 2.1.2. Sean U, V, W espacios vectoriales sobre K:

1. L (U, V ) es un espacio vectorial sobre K.

2. Si T ∈ L (U, V ) y S ∈ L (V, W ) entonces S ◦ T ∈ L (U, W ).

3. Si T ∈ L (U, V ) es biyectiva entonces T −1 ∈ L (V, U )

Demostración. En efecto:

1. Mostraremos que es un subespacio del espacio de las funciones de U en V . L (U, V ) no


es vacı́o pues existe la transformación lineal θ : U → V definida por u → θ (u) = θV .
Sean α, β ∈ K y T, L ∈ L (U, V ) entonces αT + βL es una transformación lineal pues
si γ ∈ K, u1 , u2 ∈ U entonces

(αT + βL) (γu1 + u2 ) = αT (γu1 + u2 ) + βL (γu1 + u2 )


= α (γT (u1 ) + T (u2 )) + β (γL (u1 ) + L (u2 ))
= γ (αT (u1 ) + βL (u1 )) + (αT (u2 ) + βL (u2 ))
= γ (αT + βL) (u1 ) + (αT + βL) (u2 )

luego L (U, V ) es no vacı́o, cerrado para la suma y producto escalar.

2. Sean u, v ∈ U y α ∈ K, entonces:

(S ◦ T ) (α u + v) = S (T (α u + v))
= S (α T (u) + T (v))
= α S (T (u)) + S (T (v))
= α (S ◦ T ) (u) + (S ◦ T ) (v)

Luego, S ◦ T es una transformación lineal.

3. Si T −1 : V → U existe entonces es una transformación lineal, en efecto, sean v1 , v2 ∈ V


y α ∈ K, por la biyectividad existen únicos u1 , u2 ∈ U tales que

T (u1 ) = v1
T (u2 ) = v2

82
Apuntes Mat023 (versión preliminar actualizada 23-05-2014)

luego

T −1 (αv1 + v2 ) = T −1 (αT (u1 ) + T (u2 ))


= T −1 (T (αu1 + u2 ))
= αu1 + u2
= αT −1 (v1 ) + T −1 (v2 )

Observación 2.1.4. Con respecto al teorema anterior, denotaremos la composición de


transformaciones lineales S ◦ T mediante la notación producto ST .

Teorema 2.1.3. Sean U un espacio vectorial sobre K tal que u1 , u2 , . . . , un es una base
para U y v1 , v2 , . . . , vn un subconjunto cualquiera de un espacio vectorial V arbitrario (se
pueden repetir elementos), entonces existe una única transformación lineal T : U → V tal
que:
T (ui ) = vi , ∀i = 1, 2, . . . , n (2.1)

Observación 2.1.5. Note que si u1 , u2 , . . . , un es una base para un espacio vectorial U y


u ∈ U , entonces existen únicos escalares x1 , x2 , . . . , xn ∈ K tales que:
n
X
u= xi u i
i=1

Si, además, T está definida por las ecuaciones (2.1), entonces se tiene que:
n
!
X
Tu = T xi ui
i=1
n
X
= xi T (ui )
i=1
n
X
= xi vi
i=1

Ejemplo 2.1.10. Determinaremos la fórmula T (x, y, z) de la transformación lineal T :


R3 → R3 [x] tal que:

T (1, 0, 2) = x3 + 2x2 + 1
T (−1, 1, 1) = −2x2 − 2 (2.2)
T (1, −2, 1) = x3 − 1

83
Apuntes Mat023 (versión preliminar actualizada 23-05-2014)

Notamos primeramente que u1 = (1, 0, 2) , u2 = (−1, 1, 1) y u3 = (1, −2, 1) forman una


base para R3 , pues:  
1 0 2
 
det 
 −1 1 =5
1 
1 −2 1
Por otro lado, si:

(a, b, c) = x1 · (1, 0, 2) + x2 · (−1, 1, 1) + x3 · (1, −2, 1)

entonces x1 = 35 a + 25 b + 15 c, x2 = 25 c − 15 b − 45 a y x3 = 15 c − 35 b − 25 a, ası́:
     
3 2 1 2 1 4 1 3 2
(x, y, z) = a + b + c (1, 0, 2)+ c − b − a (−1, 1, 1)+ c − b − a (1, −2, 1)
5 5 5 5 5 5 5 5 5

por la linealidad de T y su definición en la base, tenemos que:

 
3 2 1
T (a, b, c) = a + b + c T (1, 0, 2) +
5 5 5
   
2 1 4 1 3 2
+ c − b − a T (−1, 1, 1) + c − b − a T (1, −2, 1)
5 5 5 5 5 5
   
3 2 1 3 2 2 1 4
c − b − a −2x2 − 2 +
 
= a + b + c x + 2x + 1 +
5 5 5 5 5 5
 
1 3 2
c − b − a x3 − 1

+
5 5 5
     
1 1 2 3 14 6 2 2 13 7 4
= a− b+ c x + a+ b− c x + a+ b− c
5 5 5 5 5 5 5 5 5

Es decir, T : R3 → R3 [x] está definida por:


     
1 1 2 3 14 6 2 2 13 7 4
T (a, b, c) = a− b+ c x + a+ b− c x + a+ b− c
5 5 5 5 5 5 5 5 5

Ejemplo 2.1.11. Hallar la fórmula de una transformación lineal T de R2 en:

U = (x, y, z) ∈ R3 : x + y + z = 0


tal que T (−1, 1) = (1, 2, −3) y T (2, −1) = (5, −3, −2).

84
Apuntes Mat023 (versión preliminar actualizada 23-05-2014)

Solución. Note que (1, 2, −3), (5, −3, −2) pertenecen a U , como (−1, 1) y (2, −1) son
linealmente independiente, por el teorema existe una única transformación lineal T : R2 → U
tal que

T (−1, 1) = (1, 2, −3)


T (2, −1) = (5, −3, −2)

para determinar una fórmula, notemos que

(a, b) = α (−1, 1) + β (2, −1)

implica    
−1 2 a 1 0 a + 2b
 ∼ 
1 −1 b 0 1 a+b
ası́
(a, b) = (a + 2b) (−1, 1) + (a + b) (2, −1)

ası́

T (a, b) = (a + 2b) T (−1, 1) + (a + b) T (2, −1)


= (a + 2b) (1, 2, −3) + (a + b) (5, −3, −2)
= (6a + 7b, b − a, −5a − 8b)

Núcleo e imagen

Definición 2.1.2. Sean U y V espacios vectoriales sobre K y T : U → V una transforma-


ción lineal. Llamaremos:

1. Núcleo de T o Kernel de T al subconjunto de U definido como:

ker T = {u ∈ U : T (u) = 0}

2. Imagen de T al subconjunto de V definido como:

ImT = {v ∈ V : ∃ u ∈ U, T (u) = v}

85
Apuntes Mat023 (versión preliminar actualizada 23-05-2014)

Observación 2.1.6. El siguiente teorema indica que el núcleo y la imagen de T poseen


estructura de espacio vectorial.

Teorema 2.1.4. Sea T ∈ L (U, V ), entonces:

1. ker T ≤ U .

2. ImT ≤ V

Ejemplo 2.1.12. Hallar el núcleo y la imagen de T : R3 → R3 definida por:

T (x, y, z) = (2x − y + z, x − y + z, x)

Solución. Primero determinaremos el núcleo

(x, y, z) ∈ R3 : T (x, y, z) = (0, 0, 0)



ker (T ) =
(x, y, z) ∈ R3 : (2x − y + z,

= x − y + z, x) = (0, 0, 0)
 


 2x − y + z = 0  

3
= (x, y, z) ∈ R : x − y + z = 0

 

x = 0
 

note que    
2 −1 1 0 1 0 0 0
   
 1 −1 1 0  ∼  0 1 −1 0 
   
1 0 0 0 0 0 0 0
luego
 
 x = 0 
ker (T ) = (x, y, z) ∈ R3 :
 y−z = 0 
= (0, y, y) ∈ R3 : y ∈ R


= h{(0, 1, 1)}i

86
Apuntes Mat023 (versión preliminar actualizada 23-05-2014)

un espacio de dimensión 1. Determinemos la imagen

(a, b, c) ∈ R3 : ∃ (x, y, z) ∈ R3 , T (x, y, z) = (a, b, c)



ImT =
(a, b, c) ∈ R3 : ∃ (x, y, z) ∈ R3 , (2x − y + z,

= x − y + z, x) = (a, b, c)
 


 2x − y + z = a  

3 3
= (a, b, c) ∈ R : ∃ (x, y, z) ∈ R , x − y + z = b

 

x = c
 

en otras palabras, para que valores de a, b, c ∈ R el sistema

2x − y + z = a
x−y+z = b
x = c

tiene solución, notemos que


   
2 −1 1 a 2 −1 1 a
   
 1 −1 1 b  ∼  0 −1 1 2b − a 
   
1 0 0 c 0 0 0 −c − b + a

luego el sistema tiene solución si y solo si

−c − b + a = 0

se sigue

(a, b, c) ∈ R3 : −c − b + a = 0

ImT =
= (b + c, b, c) ∈ R3 : b, c ∈ R


= h{(1, 1, 0) , (1, 0, 1)}i

es un espacio de dimensión 2.

Definición 2.1.3. Sea T ∈ L (U, V ) con U y V espacios de dimensión finita, entonces:

1. Llamaremos nulidad de T , denotado η (T ), al número definido como:

η (T ) = dim ker T

87
Apuntes Mat023 (versión preliminar actualizada 23-05-2014)

2. Llamaremos rango de T , denotado ρ (T ) al número definido como:

ρ (T ) = dim ImT

Teorema 2.1.5. Sean U, V un espacios vectoriales sobre K, u1 , u2 , . . . , un una base para


U y T ∈ L (U, V ), entonces:

ImT = h{T (u1 ) , T (u2 ) , . . . , T (un )}i

Demostración. Suponga que v ∈ ImT entonces, por definición, existe un u ∈ U tal que
T (u) = v, como {u1 , u2 , . . . , un } es una base de U existirán escalares α1 , α2 , . . . , αn tales
que
n
X
u= αi ui
i=1

se sigue que !
n
X n
X
v = T (u) = T αi ui = αi T (ui )
i=1 i=1

es decir, v es una combinación lineal de los elementos T (u1 ) , T (u2 ) , . . . , T (un ), luego
{T (u1 ) , T (u2 ) , . . . , T (un )} genera ImT .

Ejemplo 2.1.13. Si T : M2×2 (R) → M2×2 (R)


      
x y x y 1 1 x y
 →T =  
z w z w 1 1 z w

sabemos que        
1 0 0 1 0 0 0 0
 , ,  y  
0 0 0 0 1 0 0 1

88
Apuntes Mat023 (versión preliminar actualizada 23-05-2014)

forman una base de M2×2 (R) luego


      
1 0 1 1 1 0 1 0
T  =   = 
0 0 1 1 0 0 1 0
      
0 1 1 1 0 1 0 1
T  =   = 
0 0 1 1 0 0 0 1
      
0 0 1 1 0 0 1 0
T  =   = 
1 0 1 1 1 0 1 0
      
0 0 1 1 0 0 0 1
T  =   = 
0 1 1 1 0 1 0 1

generan la imagen, se sigue


*     +
 
 1 0 0 1 1 0 0 1 
ImT =  , , , 
 1 0 0 1 1 0 0 1 
*   +
 1 0 0 1 
=  , 
 1 0 0 1 

de donde obtenemos ρ (T ) = 2.

Ejemplo 2.1.14. Demuestre que existe una transformación lineal T : M2 (R) → R3 tal
que:
ImT = {(x, y, z) : x − 2y + z = 0}

y la nulidad de T sea 2.

Solución. Notemos que

ImT = {(x, y, z) : x − 2y + z = 0}
= {(x, y, z) : x = 2y − z}
= {(2y − z, y, z) : y, z ∈ R}
= h{(2, 1, 0) , (−1, 0, 1)}i

este espacio tiene dimensión 2. Si necesitamos que la nulidad sea 2 entonces el Núcleo debe

89
Apuntes Mat023 (versión preliminar actualizada 23-05-2014)

estar generado por dos elementos L. I. Definamos


 
1 0
T  = (2, 1, 0)
0 0
 
0 1
T  = (−1, 0, 1)
0 0
 
0 0
T  = (0, 0, 0)
1 0
 
0 0
T  = (0, 0, 0)
0 1

existe una única T : M2 (R) → R3 que cumple lo anterior, para esta transformación se
tiene:
*         +
 1 0 0 1 0 0 0 0 
ImT = T   ,T   ,T   ,T  
 0 0 0 0 1 0 0 1 
= h{(2, 1, 0) , (−1, 0, 1)}i

Note que  
a b
T  = (2a − b, a, b)
c d
demuestre que el Núcleo tiene dimensión 2..

Observación 2.1.7. El siguiente resultado relaciona la nulidad y el rango de una trans-


formación lineal:

Teorema 2.1.6. Sean U y V espacios vectoriales sobre K con dim U < ∞. Considere
T ∈ L (U, V ), entonces:
dim U = η (T ) + ρ (T )

Demostración. Supongamos que dim U = n. Sea u1 , u2 , . . . , ur (r ≤ n) una base del núcleo


de T . Considere ur+1 , ur+2 , . . . , un una completación de la base u1 , u2 , . . . , ur del núcleo de
T al espacio vectorial U . Luego, si v ∈ U , entonces:
n
X
v= αi ui
i=1

90
Apuntes Mat023 (versión preliminar actualizada 23-05-2014)

Ası́:
n
!
X
T (v) = T αi ui
i=1
n
X
= αi T (ui )
i=1
Xn
= αT (ui )
i=r+1

y por tanto, T (ur+1 ) , T (ur+2 ) , . . . , T (un ) generan la imagen de T . Basta verificar, ahora,
que son linealmente independientes. En efecto, supongamos que:
n−r
X
αi T (ur+i ) = 0
i=1

Luego: !
n−r
X
T αi ur+i =0
i=1
Pn−r
es decir, si w = i=1 αi ur+i , se tiene entonces que w ∈ ker T . Por tanto, existen escalares
β1 , β2 , . . . , βr ∈ K tales que:
r
X
w= βi ui
i=1

Ası́:
r
X n−r
X
βi ui = αi ur+i
i=1 i=1

O bien:
r
X n−r
X
βi ui − αi ur+i = 0
i=1 i=1

Ahora bien, como u1 , u2 , . . . , ur , ur+1 , . . . , un es una base de U se tiene que:

β1 = β2 = · · · = βr = α1 = α2 = · · · = αn−r = 0

Ası́, T (ur+1 ) , T (ur+2 ) , . . . , T (un ) es una base para la imagen de T . Por tanto, η (T ) = r
y ρ (T ) = n − r.

Definición 2.1.4. Sea T ∈ L (U, V ). Diremos que:

91
Apuntes Mat023 (versión preliminar actualizada 23-05-2014)

1. T es inyectiva si:
T (u) = T (v) =⇒ u = v

para todos u, v ∈ U .

2. T es epiyectiva si:
ImT = V

3. T es biyectiva si es inyectiva y epiyectiva, simultáneamente.

Ejemplo 2.1.15. No existe transformación lineal T : R2 → R3 que sea sobreyectiva. En


efecto, si T es sobreyectiva ImT = R3 luego ρ (T ) = 3 pero del teorema de las dimensiones

η (T ) + ρ (T ) = Dim R2 = 2


como ρ (T ) = 3 se obtiene
η (T ) = −1

esto es una contradicción pues η (T ) ≥ 0.

Teorema 2.1.7. Sea T ∈ L (U, V ). Entonces, T es inyectiva, si y solo si, ker T = {θU }.

Demostración. Supongamos que ker T = {θU }. Sean u, v ∈ U tales que T (u) = T (v) .
Como T es una transformación lineal, se tiene T (u − v) = θV , es decir, u − v ∈ ker T . Ası́,
u − v = θU , y por tanto, T es inyectiva.
Por otro lado, supongamos que T es inyectiva, como {θU } ⊆ ker T , bastará verificar
que ker T ⊆ {0}. En efecto, sea u ∈ ker T , esto es T (u) = θV y como T es transformación
lineal, se tiene que T (θU ) = θV . Es decir, T (u) = T (θU ), luego u = θU , pues T es inyectiva.
Ası́, ker T = {θU }.

Isomorfismo

Definición 2.1.5. Dos espacios vectoriales U y V sobre K se dicen isomorfos si existe una
transformación lineal T : U → V biyectiva. Tal transformación T biyectiva se llamará iso-
morfismo entre U y V . El concepto de isomorfismo entre dos espacios se representará por:
U 'V.

92
Apuntes Mat023 (versión preliminar actualizada 23-05-2014)

Ejemplo 2.1.16. Sea U un espacio vectorial sobre R tal que u1 , u2 , . . . , un es una base de
U , entonces L : Rn → U definida por:
n
X
(x1 , x2 , . . . , xn ) 7→ L (x1 , x2 , . . . , xn ) = xi ui
i=1

es un isomorfismo entre U y Rn , esto se debe a que


n
X
(x1 , x2 , . . . , xn ) ∈ ker T ⇔ xi ui = θU
i=1
⇔ ∀i, xi = 0

(pues u1 , u2 , . . . , un es una base de U ) ası́ (x1 , x2 , . . . , xn ) ∈ ker T ⇔ (x1 , x2 , . . . , xn ) =


(0, 0, . . . , 0) luego L es inyectiva y por el teorema de las dimensiones

n = dim Rn = 0 + ρ (L)

ası́
ρ (L) = n

esto implica ImT = U .

Teorema 2.1.8. Sean U y V espacios vectoriales de dimensión finita sobre un cuerpo K.


Entonces, U ' V si y solo si dim U = dim V .

Ejemplo 2.1.17. M2 (R) ' R3 [x] ' R4

Ejemplo 2.1.18. Kmn ' Mm×n (K)

Matriz asociada a una transformación lineal

Definición 2.1.6. Sean U un espacio vectorial sobre K y B = {u1 , u2 , . . . , un } una base


de U . Diremos que B es una base ordenada de U , si se considera B como la sucesión
finita de vectores en U :
B = (u1 , u2 , . . . , un )

93
Apuntes Mat023 (versión preliminar actualizada 23-05-2014)

Definición 2.1.7. Sea B = {u1 , u2 , . . . , un } una base ordenada de un espacio vectorial U


y sea u ∈ U . Los coeficientes αi en la combinación

u = α1 u1 + α2 u2 + · · · + αn un

son llamadas coordenadas de u con respecto a B y desde ahora usaremos la notación


 
α1
 
 α 
 2 
[u]B =  . 
 .. 
 
αn
Observación 2.1.8. Estos escalares existen y son únicos por la definición de base, el orden
de los elementos de la base es importante.

Ejemplo 2.1.19. Calcular las coordenadas del vector −1 + 2x + 3x2 en la base de R2 [x]
dada por B = {1 + x, x2 , 1} .

Solución. Tenemos que encontrar escalares α, β, γ tales que

−1 + 2x + 2x2 = α (1 + x) + βx2 + γ1

desarrollando
−1 + 2x + 3x2 = (α + γ) + αx + βx2

ası́

−1 = α + γ
2 = α
3 = β

se sigue γ = −3 entonces

−1 + 2x + 3x2 = 2 (1 + x) + 3 x2 − 3 (1)


de donde obtenemos  
2
 
−1 + 2x + 3x2 B = 
 
 3 

−3

94
Apuntes Mat023 (versión preliminar actualizada 23-05-2014)

Observación 2.1.9. Dado en vector de coordenadas [v]B y conocida la base B se puede


recuperar el vector v.

Ejemplo 2.1.20. Si el determinado vector v en M2×2 (R) tiene vector de coordenadas


 
1
 
 −1 
[v]B = 
 

 2 
 
−2
       
 1 1 1 1 1 1 1 0 
en la base B =  , , ,  entonces
 1 1 1 0 0 0 0 0 
       
1 1 1 1 1 1 1 0
v = 1  + (−1)   + 2  + (−2)  
1 1 1 0 0 0 0 0
 
0 2
=  
0 1

Observación 2.1.10. Sean U y V espacios vectoriales sobre K tales que B = {u1 , u2 , . . . , un }


y D = {v1 , v2 , . . . , vm } son bases ordenadas de U y V , respectivamente. Note que para
cada i = 1, 2, . . . , n se tiene que T (ui ) ∈ V , por tanto, existen escalares Aji ∈ K tales que:
m
X
T (ui ) = A1 i v1 + A2 i v2 + · · · + Am i vm = Aji vj , ∀i = 1, 2, . . . , n
j=1

Es decir, los escalares A1 i , A2 i , . . . , Am i son las coordenadas de T (ui ) en la base D. Ası́:


 
A1 i
 
 A 
 2i 
[T (ui )]D =  .  , ∀i = 1, 2, . . . , n
 .. 
 
Am i

Definición 2.1.8. Con respecto a las notaciones de la observación anterior, se define la


matriz de T respecto de las bases B y D como la matriz A tal que A = (Ai j ) ∈ Mm×n (R).
Esto es:

95
Apuntes Mat023 (versión preliminar actualizada 23-05-2014)

 
[T ]D
B = [T (u1 )D ] [T (u2 )D ] · · · [T (un )D ]
 
A11 A12 · · · A1n
 
 A
21 A 22 · · · A 2n

=  .
 
 .. .. .. .. 
 . . . 
Am1 Am2 · · · Amn

Ejemplo 2.1.21. Dada la transformación lineal T : R3 → R2 definida por:

T (x, y, z) = (3x − 2y + z, x − y)

Determine la matriz asociada a T respecto de las bases:

1. canónicas de R3 y R2 , respectivamente.

Solución.

T (1, 0, 0) = (3, 1) = 3 (1, 0) + 1 (0, 1)


T (0, 1, 0) = (−2, −1) = −2 (1, 0) − 1 (0, 1)
T (0, 0, 1) = (1, 0) = 1 (1, 0) + 0 (0, 1)

ası́  
C 3 −2 1
[T ]CR23 =  
R
1 −1 0

2. las bases B = {(1, 1, 0) ; (1, 0, 1) ; (1, 2, 3)} y D = {(2, 1) ; (−1, 1)}

Solución.

T (1, 1, 0) = (3 − 2, 1 − 1) = (1, 0) = α1 (2, 1) + β 1 (−1, 1)


T (1, 0, 1) = (3 + 1, 1) = (4, 1) = α2 (2, 1) + β 2 (−1, 1)
T (1, 2, 3) = (3 − 4 + 3, 1 − 2) = (2, −1) = α3 (2, 1) + β 3 (−1, 1)

determinemos los escalares, los determinaremos en un solo sistema


   
1 5 1
2 −1 1 4 2 1 0 3 3 3 
 ∼
1 1 0 1 −1 0 1 − 3 − 3 − 43
1 2

96
Apuntes Mat023 (versión preliminar actualizada 23-05-2014)

se sigue  
1 5 1
[T ]D
B =
 3 3 3 
− 31 − 23 − 43

Ejemplo 2.1.22. Sea D : R3 [x] → R3 [x] el operador lineal derivada, es decir, p → D (p) =
p0 y considere las bases de R3 [x] dadas por

2x3 , 3x − x2 , 1 − x, 1

B1 =
= 1, x, x2 , x3

B2

luego

D 2x3 = 0 · 1 + 0x + 6x2 + 0x3




D 3x − x2 = 3 (1) − 2 (x) + 0x2 + 0x3




D (1 − x) = −1 (1) + 0x + 0x2 + 0x3


D (1) = 0 (1) + 0x + 0x2 + 0x3

entonces  
0 3 −1 0
 
 0 −2 0 0 
[D]BB21 =
 

 6 0 0 0 
 
0 0 0 0

usted puede calcular [D]B1


B2 y verá que es una matriz diferente.

Ejemplo 2.1.23. Si B = {v1 , v2 , . . . , vn } es una base del espacio vectorial V y I : V → V


la transformación identidad entonces

[I]BB = In×n (la matriz identidad)

En efecto, Iv1 = v1 = 1v1 + 0v2 + 0v3 + · · · + 0vn se sigue


 
1
 
 0 
[Iv1 ]B =  . 
 
 .. 
 
0

97
Apuntes Mat023 (versión preliminar actualizada 23-05-2014)

similarmente Iv2 = v1 = 0v1 + 1v2 + 0v3 + · · · + 0vn se sigue


 
0
 
 1 
 
 
[Iv2 ]B =   0 

 . 
 .. 
 
0

ası́ [I]BB es
[I]BB = In×n

Definición 2.1.9. Sean U un espacio vectorial sobre K tales que B = {u1 , u2 , . . . , un } y


D = {v1 , v2 , . . . , vn } son bases ordenadas distintas de U . Sea 1 : U → U la transformación
lineal identidad. Se define la matriz cambio de base como la matriz asociada a 1 respecto
de las bases B y D.

Teorema 2.1.9. Sean U y V espacios vectoriales finito dimensionales sobre K con bases B
y D, respectivamente. Sean, además, S, T : U → V dos transformaciones lineales, entonces:

[αT + S]D D D
B = α [T ]B + [S]B

Teorema 2.1.10. Sean U, V y W espacios vectoriales de dimensión finita sobre K tales que
B, D y E son bases de U, V y W , respectivamente. Suponga que T : U → V y S : V → W
son transformaciones lineales, entonces:

[ST ]EB = [S]ED · [T ]D


B

Observación 2.1.11. Sean U y V espacios vectoriales finito dimensionales sobre K con


bases B y E para U , y D y F para V . Considere, además, T : U → V una transformación
lineal. Note que en vista del teorema anterior el cambio de representación matricial puede
expresarse mediante el siguiente diagrama conmutativo:

E[T ]F
UE −→ VF
[1]EB ↑ ↓ [1]D
F
B[T ]D
UB −→ VD

98
Apuntes Mat023 (versión preliminar actualizada 23-05-2014)

donde la notación del tipo UE , por ejemplo, representa el espacio U considerado con la
base E. En efecto, utilizando la convención usual para la composición de funciones, tenemos
que:
[T ]D D F E
B = [1]F · [T ]E · [1]B

Teorema 2.1.11. Sean U y V espacios vectoriales finito dimensionales sobre K con bases
B y D, respectivamente. Sea, además, T : U → V una transformación lineal entonces T es
invertible si y solo si [T ]D
B es invertible, además

 −1 B  D −1
T D = [T ]B

Teorema 2.1.12. Sean U y V espacios vectoriales sobre el mismo cuerpo K tales que
B = {u1 , u2 , . . . , un } es una base ordenada para U y D = {v1 , v2 , . . . , vm } es una base
ordenada para V , entonces:
[T u]D = [T ]D
B [u]B

para todo u ∈ U .

Ejemplo 2.1.24. Considere las bases de R3

B1 = {(1, 1, 1) , (1, 1, 0) , (1, 0, 0)}

y
B2 = {(0, 0, 1) , (1, 0, 0) , (0, 1, 0)}

1. Calcular [I]BB21 y [I]BB12 .

2. Calcular [v]Bi para i = 1, 2 donde v = (3, 1, 3)

1. Notemos que

I (1, 1, 1) = (1, 1, 1) = 1 (0, 0, 1) + 1 (1, 0, 0) + 1 (0, 1, 0)


I (1, 1, 0) = (1, 1, 0) = 0 (0, 0, 1) + 1 (1, 0, 0) + 1 (0, 1, 0)
I (1, 0, 0) = (1, 0, 0) = 0 (0, 0, 1) + 1 (1, 0, 0) + 0 (0, 1, 0)

99
Apuntes Mat023 (versión preliminar actualizada 23-05-2014)

entonces  
1 0 0
[I]BB21 = 
 
 1 1 1 

1 1 0
 −1  −1
como [I]BB12 = [I]BB21 se sigue que [I]BB12 = [I]BB21
   
1 0 0 1 0 0 1 0 0 1 0 0
   
 1 1 1 0 1 0  ∼  0 1 0 −1 0 1 
   
1 1 0 0 0 1 0 0 1 0 1 −1
ası́  
1 0 0
[I]BB12 = 
 
 −1 0 1 

0 1 −1

2. Notemos que (3, 1, 3) = 3 (0, 0, 1) + 3 (1, 0, 0) + 1 (0, 1, 0)


 
3
 
[(3, 1, 3)]B2 =  3 

1
luego

[(3, 1, 3)]B1 = [I]BB12 [(3, 1, 3)]B2


  
1 0 0 3
  
= 
 −1 0 1   3 
 

0 1 −1 1
 
3
 
= 
 −2 

2

Ejemplo 2.1.25. Sean B = {(1, 1, 1) ; (0, 0, 1) ; (1, 0, 1)} y D = {(1, 1) ; (0, 1)} bases de
R3 y R2 , respectivamente. Sea T una transformación lineal tal que:
 
4 2 −2
[T ]D
B =
 
−3 −1 1

Hallar explı́citamente T (x, y, z).

100
Apuntes Mat023 (versión preliminar actualizada 23-05-2014)

Solución. Primero buscamos [(x, y, z)]B esto es

(x, y, z) = α (1, 1, 1) + β (0, 0, 1) + γ (1, 0, 1)

luego    
1 0 1 x 1 0 0 y
   
 1 0 0 y ∼ 0 1 0 z−x 
   
1 1 1 z 0 0 1 x−y
ası́  
y
 
[(x, y, z)]B = 
 z − x 

x−y
se sigue que

[T (x, y, z)]D = [T ]D
B [(x, y, z)]B
 
  y
4 2 −2  
=   z − x 
−3 −1 1
 
x−y
 
6y − 4x + 2z
=  
2x − 4y − z

de donde obtenemos

T (x, y, z) = (6y − 4x + 2z) (1, 1) + (2x − 4y − z) (0, 1)


= (6y − 4x + 2z, 2y − 2x + z)

Cálculo con coordenadas

Observación 2.1.12. Como se puede observar de la sección anterior, las matrices asociadas
a las transformaciones lineales dependen de las coordenadas. Es decir, dependen de las
bases consideradas en los espacios vectoriales involucrados. Por consiguiente, el cálculo
de los subespacios notables (núcleo e imagen) asociados a una transformación lineal no
es directo por definición. Sin embargo, con los procedimientos algebraicos adecuados se
pueden obtener los espacios notables asociados a las transformaciones a través de cálculos
con coordenadas. Veamos algunos ejemplos:

101
Apuntes Mat023 (versión preliminar actualizada 23-05-2014)

Ejemplo 2.1.26. Sean:

B = {(1, 1, 1) ; (0, 1, 1) ; (1, 0, 1)}

y:
D = {(1, 1, 1) ; (1, 0, 1) ; (1, 2, 3)}

bases de R3 y T ∈ L (R3 , R3 ) tales que:


 
4 2 −2
[T ]D
 
B =  −3 −1 1 
 
1 2 −2

Calcule el núcleo y la imagen de T , sin explicitar T (x, y, z).

Solución. Sea u un vector cualquiera en el núcleo de T , es decir T (u) = 0. Por tanto,


[T · u]D = [0]D = 0. Sin embargo, sabemos que [T · u]D = [T ]D
B [u]B , luego:
    
4 2 −2 a 0
    
 b  =  0 
 −3 −1 1     

1 2 −2 c 0
 T
en donde [u]B = a b c . Escalonando la matriz de coeficientes:
   
4 2 −2 1 0 0
   
 −3 −1 1  →  0 1 −1 
   
1 2 −2 0 0 0

se obtiene que a = 0 y b = c. Por lo tanto:


 
0
 
 c ,
[u]B =   c∈R
c

Luego:
u = 0 · (1, 1, 1) + c · (0, 1, 1) + c · (1, 0, 1) = (c, c, 2c) , c∈R

Por lo tanto, obtenemos de lo anterior que:

ker T = h (1, 1, 2) i

102
Apuntes Mat023 (versión preliminar actualizada 23-05-2014)

Para calcular la imagen de T , debemos notar primeramente que de:


 
4 2 −2
[T ]D
 
B =  −3 −1 1 
 
1 2 −2

se obtiene que:
     
4 2 −2
     
[T (1, 1, 1)]D = 
 −3 ,
 [T (0, 1, 1)]D = 
 −1 ,
 [T (1, 0, 1)]D = 
 1 

1 2 −2

Por consiguiente:

T (1, 1, 1) = 4 (1, 1, 1) + (−3) (1, 0, 1) + 1 (1, 2, 3) = (2, 6, 4)


T (0, 1, 1) = 2 (1, 1, 1) + (−1) (1, 0, 1) + 2 (1, 2, 3) = (3, 6, 7)
T (1, 0, 1) = (−2) (1, 1, 1) + 1 (1, 0, 1) + (−2) (1, 2, 3) = (−3, −6, −7)

Ahora bien:

ImT = hT (1, 1, 1) , T (0, 1, 1) , T (1, 0, 1)i


= h{(2, 6, 4) , (3, 6, 7) , (−3, −6, −7)}i
= h{(2, 6, 4) , (3, 6, 7)} i

note que el rango de T es 2.

Ejemplo 2.1.27. Sean:

B = {(1, 1, −1) ; (0, 2, −1) ; (1, 0, 1)}

y:
D = 1 + 2x + x2 ; 1 + 3x + 2x2 ; 2 + x + 3x2


bases de R3 y R2 [x], respectivamente. Sea T ∈ L (R3 , R2 [x]) tales que:


 
5 2 −2
[T ]D
 
B =  −3 −1 1 


1 4 −3

Verifique que T es un isomorfismo entre R3 y R2 [x]. Calcule, además, T −1 .

103
Apuntes Mat023 (versión preliminar actualizada 23-05-2014)

Solución. Para verificar que T es un isomorfismo, basta notar que det [T ]D 6 0. En efecto:
B =

5
2 −2

D

det [T ]B = −3 −1 1 =1


1 4 −3

Por otro lado, para calcular T −1 , procedemos por coordenadas. En efecto:


 −1 B
T (a, b, c) B = T −1 D [(a, b, c)]D
 
 −1
= [T ]D
B [(a, b, c)]D

Luego:
 −1
5 2 −2
B
T −1
  
D
= 
 −3 −1 1 
1 4 −3
 
−1 −2 0
 
= 
 −8 −13 1 

−11 −18 1

Necesitamos, ahora, las coordenadas de a + bx + cx2 respecto de la base D. Esto es:

α 1 + 2x + x2 + β 1 + 3x + 2x2 + γ 2 + x + 3x2 = a + bx + cx2


  

Resolviendo el sistema anterior, obtenemos:


 
α
 
a + bx + cx2 D = 
 
 β 

γ
 
7 1 5
a + b − c
 4 4 4

 1
=  4 b − 4 a + 43 c 
5

1 1 1
4
c + 4a − 4b

104
Apuntes Mat023 (versión preliminar actualizada 23-05-2014)

Luego:
  
7 1
−1 −2 0 4
a+ 4
b − 54 c
T −1 a + bx + cx2
    
1 5
B
= 
 −8 −13 1 
 4
b− 4
a + 43 c 

1 1 1
−11 −18 1 4
c+ 4
a − 4b
 
3 3 1
a − b − c
 4 4 4

=  2 a − 2 b + 21 c 
 5 11

7 15 1
2
a − 2 b + 2c

Ası́:
   
−1 2
 3 3 1 5 11 1
T a + bx + cx = a − b − c (1, 1, −1) + a − b + c (0, 2, −1)
4 4 4 2 2 2
 
7 15 1
+ a − b + c (1, 0, 1)
2 2 2
 
17 33 1 23 47 3 1 5 1
= a − b + c, a − b + c, a − b + c
4 4 4 4 4 4 4 4 4

Ejercicios resueltos de Transformaciones lineales

1. Considere la transformación

T : M2×2 (R) → R4
   
a b a b
  → T  = (a + b, a + c − b, c, d)
c d c d

a) Determine una base de Ker(T ) e Im(T ).

105
Apuntes Mat023 (versión preliminar actualizada 23-05-2014)

Solución. Vamos a buscar el Núcleo de la transformación,


    
 a b a b 
ker (T ) =   ∈ M2×2 (R) : T   = (0, 0, 0, 0)
 c d c d 
  
 a b 
=   ∈ M2×2 (R) : (a + b, a + c − b, c, d) = (0, 0, 0, 0)
 c d 
 


   a + b = 0 


 
a−b+c = 0 

 a b 
=   ∈ M2×2 (R) :


 c d c = 0  


 

 d = 0 
  
 a b 
=   ∈ M2×2 (R) : a = b = c = d = 0
 c d 
 
 0 0 
=  
 0 0 

del teorema de las dimensiones se sigue que

Dim (ker (T )) + Dim (Im (T )) = Dim (M2×2 (R))


0 + Dim (Im (T )) = 4

ası́ Dim (Im (T )) = 4 y luego Im (T ) = R4 (T es biyectiva)

b) Si        
 1 −1 0 −1 0 1 1 1 
B1 =   ,   ,   ,  
 2 0 2 1 1 1 0 3 
y
B2 = {(1, 0, 2, 0) , (1, 0, 2, 1) , (0, 0, 1, 1) , (1, 1, 0, −3)}

son bases de M2×2 (R) y R4 respectivamente determinar [T ]BB21 .

106
Apuntes Mat023 (versión preliminar actualizada 23-05-2014)

Solución. Como
 
1 −1
T  = (0, 4, 2, 0)
2 0
 
0 −1
T  = (−1, 3, 2, 1)
2 1
 
0 1
T  = (1, 0, 1, 1)
1 1
 
1 1
T  = (2, 0, 0, 3)
0 3
se sigue que necesitamos resolver los sistemas

(0, 4, 2, 0) = α1 (1, 0, 2, 0) + β1 (1, 0, 2, 1) + γ1 (0, 0, 1, 1) + δ1 (1, 1, 0, −3)


(−1, 3, 2, 1) = α2 (1, 0, 2, 0) + β2 (1, 0, 2, 1) + γ2 (0, 0, 1, 1) + δ2 (1, 1, 0, −3)
(1, 0, 1, 1) = α3 (1, 0, 2, 0) + β3 (1, 0, 2, 1) + γ3 (0, 0, 1, 1) + δ3 (1, 1, 0, −3)
(2, 0, 0, 3) = α4 (1, 0, 2, 0) + β4 (1, 0, 2, 1) + γ4 (0, 0, 1, 1) + δ4 (1, 1, 0, −3)

y ası́ la matriz asociada será


 
α1 α2 α3 α4
 
 β β β β 
 1 2 3 4 
[T ]BB21 = 
 γ1 γ2 γ3 γ4 
 
δ1 δ2 δ3 δ4
vamos a resolver los 4 sistema de una sola vez
   
1 1 0 1 0 −1 1 2 1 0 0 0 −6 −4 −1 −5
   
 0 0 0 1 4 3 0 0    0 1 0 0 2 0 2 7 


 
   
 2
 2 1 0 2 2 1 0    0 0
 1 0 10 10 −1 −4 

0 1 1 −3 0 1 1 3 0 0 0 1 4 3 0 0
ası́  
−6 −4 −1 −5
 
 2 0 2 7 
[T ]BB21 =
 

 10
 10 −1 −4 

4 3 0 0

107
Apuntes Mat023 (versión preliminar actualizada 23-05-2014)

c) Si la transformación es un isomorfismo (biyectiva) determine


B1
T −1

B2

Solución. ya sabemos que la transformación es biyectiva, como


 −1 B1  B2 −1
T B2 = [T ]B1

se sigue
 
3 3 9 17
22
− 22 − 22 11
 
 −1  
 
−6 −4 −1 −5 
 −2 2 6


 2
  11 11 11
− 19
11


0 2 7   
= 
  
 
 10 10 −1 −4  
 23


   21 8 4 
4 3 0 0  11 11 11 11 
 
 
 
7 4 1 6
− 11 − 11 − 11 − 11
B
= T −1 B12


2. Construir una transformación lineal T : R3 → R3 que cumpla las siguientes condicio-


nes (simultáneamente)

a) Ker(T ) = {(x, y, z) ∈ R3 : (x, y, z) = t (1, 0, −1) para t ∈ R}

b) Im(T ) = {(x, y, z) ∈ R3 : x − 2y + z = 0}

Solución. La transformación tiene que cumplir las dos condiciones, notamos que

ker (T ) = h(1, 0, −1)i

(x, y, z) ∈ R3 : x − 2y + z = 0

Im (T ) =
= (x, y, z) ∈ R3 : x + z = 2y

  
x+z 3
= x, , z ∈ R : x, z ∈ R
2
   
1 1
= 1, , 0 , 0, , 1
2 2

108
Apuntes Mat023 (versión preliminar actualizada 23-05-2014)

desde el punto de vista del teorema de las dimensiones estas dos condiciones no son
incompatibles puesto que

Dim (ker (T )) + Dim (Im (T )) = Dim R3 = 3




para la posible transformación. Vamos a utilizar el teorema que nos permite construir
transformaciones si la conocemos en una base
 
1
T (1, 0, 0) = 1, , 0
2
 
1
T (0, 1, 0) = 0, , 1
2
T (1, 0, −1) = (0, 0, 0)

notamos que los vectores (1, 0, 0) , (0, 1, 0) y (1, 0, −1) son linealmente independientes
pues
1 0 1


0 1 0 = −1 6= 0


0 0 −1

y luego forman una base de R3 estas condiciones determinan por completo una
transformación lineal, note también que

Im (T ) = h{T (1, 0, 0) , T (0, 1, 0) , T (1, 0, −1)}i


    
1 1
= 1, , 0 , 0, , 1 , (0, 0, 0)
2 2
   
1 1
= 1, , 0 , 0, , 1
2 2
3

= (x, y, z) ∈ R : x − 2y + z = 0

y (1, 0, −1) ∈ ker (T ) que tiene dimensión 1 (por el teorema de las dimensiones) ası́

ker (T ) = h{(1, 0, −1)}i


= (x, y, z) ∈ R3 : (x, y, z) = t (1, 0, −1) para t ∈ R


podemos también determinar en forma explı́cita esta transformación

(x, y, z) = α (1, 0, 0) + β (0, 1, 0) + γ (1, 0, −1)

109
Apuntes Mat023 (versión preliminar actualizada 23-05-2014)

α+γ = x
β = y
−γ = z

de donde α = x + z, β = y, γ = −z ası́

(x, y, z) = (x + z) (1, 0, 0) + y (0, 1, 0) + (−z) (1, 0, −1)

luego

T (x, y, z) = T ((x + z) (1, 0, 0) + y (0, 1, 0) + (−z) (1, 0, −1))


= (x + z) T (1, 0, 0) + yT (0, 1, 0) + (−z) T (1, 0, −1)
   
1 1
= (x + z) 1, , 0 + y 0, , 1 + (−z) (0, 0, 0)
2 2
 
1 1 1
= x + z, x + y + z, y
2 2 2

3. Sea T ∈ L (R3 , R2 ) con B = {(0, 0, 1) ; (0, 2, 1) ; (3, 2, 1)} y C = {(1, −3) ; (2, −5)}
bases ordenadas de R3 y R2 , respectivamente. Supongamos que:
 
−1 3 2
[T ]CB =   ∈ M2×3 (R)
4 −12 −8

Con respecto a las hipótesis anteriores:

a) Hallar ker T sin calcular T (x, y, z).

Solución. v ∈ ker (T ) ⇔ [v]B es solución de [T ]CB x = 0 luego


   
−1 3 2 0 1 −3 −2 0
 ∼ 
4 −12 −8 0 0 0 0 0

luego  
α
 
[v]B = 
 β 

γ
es solución si y solo si
α − 3β − 2γ = 0

110
Apuntes Mat023 (versión preliminar actualizada 23-05-2014)

de esta forma
   
α 3β + 2γ
   
[v]B =   para β, γ ∈ R
 β = β
 

γ γ

luego

v ∈ ker (T ) ⇔ (x, y, z) = (3β + 2γ) (0, 0, 1) + β (0, 2, 1) + γ (3, 2, 1)

para β, γ ∈ R esto es

(x, y, z) = (3γ, 2β + 2γ, 4β + 3γ) para β, γ ∈ R

ası́
ker (T ) = h(3, 2, 3) , (0, 2, 4)i

b) Determinar T (x, y, z).

Solución. Notemos que

[T (x, y, z)]C = [T ]CB [(x, y, z)]B

luego
(x, y, z) = α (0, 0, 1) + β (0, 2, 1) + γ (3, 2, 1)

ası́

3γ = x
2β + 2γ = y
α+β+γ = z

luego α = z − 21 y, β = 21 y − 13 x, γ = 13 x de donde tenemos


 
z − 12 y
 
[(x, y, z)]B =  1 1
 2 y − 3
x 

1
3
x

111
Apuntes Mat023 (versión preliminar actualizada 23-05-2014)

ası́
 
  z − 12 y
−1 3  2 
[T (x, y, z)]C =    1y − 1x 
 2 3
4 −12 −8

1
3
x
 
2y − 13 x − z
=  
4
3
x − 8y + 4z

ası́
   
1 4
T (x, y, z) = 2y − x − z (1, −3) + x − 8y + 4z (2, −5)
3 3
 
7 17
= x − 14y + 7z, 34y − x − 17z
3 3
 
x
c) ¿Por qué es falso que T (x, y, z) = [T ]CB · 
 
 y ?

z
Solución. Esta igualdad no es posible ni por los ordenes de las matrices.

d ) ¿Cuándo es válida la igualdad anterior?

Solución. Al considerar las bases canónicas en los espacios de partida y llegada


se cumple una desigualdad del tipo
 
x
T (x, y, z)t = [T ]canónica
 
·
canónica  y 
 

(además la transformación este definida de Rn en Rm )

e) Hallar la nulidad de T y el rango de T sin hallar la Im(T )?

Solución. El rango de la transformación lineal es igual al rango de la matriz


asociada en este caso es 1.

f ) Hallar Im(T ) y una base para Im(T ).

112
Apuntes Mat023 (versión preliminar actualizada 23-05-2014)

Solución. Como ya tenemos la transformación podemos determinar la imagen


como

Im (T ) = hT (1, 0, 0) , T (0, 1, 0) , T (0, 0, 1)i


  
7 17
= ,− , (−14, 34) , (7, −17)
3 3
= h(7, −17)i

esto es una recta que pasa por el origen con dirección (7, −17).

4. Sean U = {(x, y, z) ∈ R3 : 2x − 3y + 5z = 0} un espacio vectorial y A una base


ordenada de U y C la base canónica de R2 . Suponga que T : U → R2 es una
transformación lineal definida por T (x, y, z) = (x − y, x − z) tal que:
 
1 2
[T ]CA =   ∈ M2×2 (R)
1 1

Determine la base A.

Solución. Sea A = {(α, β, γ) , (ϕ, κ, λ)} la base de U del ejercicio (el espacio tiene
dimensión 2) entonces
 
1
[T (α, β, γ)]C =  
1
 
2
[T (ϕ, κ, λ)]C =  
1

además

2α − 3β + 5γ = 0
2ϕ − 3κ + 5λ = 0

pero
 
α−β
[T (α, β, γ)]C =  
α−γ
 
ϕ−κ
[T (ϕ, κ, λ)]C =  
ϕ−λ

113
Apuntes Mat023 (versión preliminar actualizada 23-05-2014)

de donde tenemos el sistema de ecuaciones

α−β = 1
α−γ = 1
ϕ−κ = 2
ϕ−λ = 1
2α − 3β + 5γ = 0
2ϕ − 3κ + 5λ = 0

que tiene solución α = 12 , κ = − 49 , β = − 12 , λ = − 54 , γ = − 12 , ϕ = − 14 ası́


   
1 1 1 1 9 5
A= ,− ,− , − ,− ,−
2 2 2 4 4 4

5. Considere la transformación lineal

T : M (R) → R4
 2×2   
x y x y
  → T  = (x − y, x − z, x − w, x)
z w z w

y suponga que
       
 1 0 −2 0 0 0 0 −1 
B1 =   ,   ,   ,  
 0 1 0 0 1 0 0 0 

B2 = {(1, 1, 1, 1) , (1, 1, 0, 0) , (1, 0, 0, 0) , (1, 1, 1, 0)}

a) Calcular [T ]BB21 , muestre que T es invertible (isomorfismo) y calcule T −1 en forma


explı́cita.

Solución. Como
 
x y
T  = (x − y, x − z, x − w, x)
z w

114
Apuntes Mat023 (versión preliminar actualizada 23-05-2014)

se sigue
 
1 0
T  = (1, 1, 0, 1)
0 1
 
−2 0
T  = (−2, −2, −2, −2)
0 0
 
0 0
T  = (0, −1, 0, 0)
1 0
 
0 −1
T  = (1, 0, 0, 0)
0 0

luego tenemos que resolver los sistemas

(1, 1, 0, 1) = α1 (1, 1, 1, 1) + β1 (1, 1, 0, 0) + γ1 (1, 0, 0, 0) + δ1 (1, 1, 1, 0)


(−2, −2, −2, −2) = α2 (1, 1, 1, 1) + β2 (1, 1, 0, 0) + γ2 (1, 0, 0, 0) + δ2 (1, 1, 1, 0)
(0, −1, 0, 0) = α3 (1, 1, 1, 1) + β3 (1, 1, 0, 0) + γ3 (1, 0, 0, 0) + δ3 (1, 1, 1, 0)
(1, 0, 0, 0) = α4 (1, 1, 1, 1) + β4 (1, 1, 0, 0) + γ4 (1, 0, 0, 0) + δ4 (1, 1, 1, 0)

resolveremos los 4 sistemas


   
1 1 1 1 1 −2 0 1 1 0 0 0 1 −2 0 0
   
 1 1 0 1 1 −2 −1 0 
  0 1 0 0 1
 0 −1 0 
∼
 
 
 1 0
 0 1 0 −2 0 0   0 0 1 0 0
 0 1 1 
1 0 0 0 1 −2 0 0 0 0 0 1 −1 0 0 0

se sigue  
1 −2 0 0
 
 1 0 −1 0 
[T ]BB21 =
 

 0 0 1 1 
 
−1 0 0 0

 −1
b) Encontrar [T ]BB21

115
Apuntes Mat023 (versión preliminar actualizada 23-05-2014)

Solución.
 −1 B
[T ]BB21 T −1 B12

=
 −1
1 −2 0 0
 
 1 0 −1 0 
= 
 

 0 0 1 1 
 
−1 0 0 0
 
0 0 0 −1
 
 −1 0 0 − 12 
=  2
 

 0 −1 0 −1 
 
0 1 1 1

c) Considere las bases canónicas deM2×2 (R) y R4


       
 1 0 0 1 0 0 0 0 
C1 =   ,   ,   ,  
 0 0 0 0 1 0 0 1 

C2 = {((1, 0, 0, 0)) , (0, 1, 0, 0) , (0, 0, 1, 0) , (0, 0, 0, 1)}

calcular [T ]CC21 usando la matrices de cambio de base adecuadas.

Solución. Para calcular [T ]CC21 necesitamos hacer

[I]CB22 [T ]BB21 [I]BC11

ası́
         
1 0 −2 0 0 0 0 −1 α − 2β −δ
α  +β   +γ   +δ  = 
0 1 0 0 1 0 0 0 γ α

de donde tenemos que resolver 4 sistemas


   
1 −2 0 0 1 0 0 0 1 0 0 0 0 0 0 1
   
 0 0 0 −1 0 1 0 0   0 1 0 0 − 12 0 0  1
2
∼
   
 
 0 0 1 0 0 0 1 0   0 0 1 0 0 0 1 0 
   
1 0 0 0 0 0 0 1 0 0 0 1 0 −1 0 0

116
Apuntes Mat023 (versión preliminar actualizada 23-05-2014)

de aquı́  
0 0 0 1
 
 −1 0 0 1 
[I]BC11 =  2 2 


 0 0 1 0 
 
0 −1 0 0
 −1
(note que es mas fácil pensarlo como [I]CB11 = [I]BC11 si la canónica esta en la
llegada es escribir los vectores como columna) y
 
1 1 1 1
 
 1 1 0 1 
C2
[I]B2 = 
 

 1 0 0 1 
 
1 0 0 0

ası́

[T ]CC21 = [I]CB22 [T ]BB21 [I]BC11


   
1 1 1 1 1 −2 0 0 0 0 0 1
   
 1 1 0 1  1 0 −1 0   −1 0 0 1 
 2 2 
= 
 
  
 1 0 0 1  0 0 1 1   0 0 1 0 
   
1 0 0 0 −1 0 0 0 0 −1 0 0
 
1 −1 0 0
 
 1 0 −1 0 
= 
 

 1 0
 0 −1 

1 0 0 0

6. Considere la transformación lineal T : R2 [x] → R2 [x] dada por:

T ax2 + bx + c = (a + b) x2 + (a + b + c) x + c


a) Encuentre una base para ker (T )

b) Encuentre una base para Im (T )

c) Considere en el dominio la base B = {1 + x2 , 1 + x + x2 , −1} y en el codominio


la base D = {1 − x, 1 + x, x2 } encontrar la matriz asociada a T respecto a estas
bases.

117
Apuntes Mat023 (versión preliminar actualizada 23-05-2014)

a) Por definición

ax2 + bx + c ∈ R2 [x] : T ax2 + bx + c = 0


 
ker (T ) =
ax2 + bx + c ∈ R2 [x] : (a + b) x2 + (a + b + c) x + c = 0

=
ax2 + bx + c ∈ R2 [x] : (a + b = 0) ∧ (a + b + c = 0) ∧ (c = 0)

=
      


 1 1 0 a 0  
2
     
= ax + bx + c ∈ R2 [x] :  1 1 1   b  =  0 
   

 
0 0 1 c 0
 

luego    
1 1 0 1 1 0
   
 1 1 1 ∼ 0 0 1 
   
0 0 1 0 0 0
ası́

ax2 + bx + c ∈ R2 [x] : (a + b = 0) ∧ (c = 0)

ker (T ) =
= ax2 + bx + c ∈ R2 [x] : (b = −a) ∧ (c = 0)


= ax2 − ax ∈ R2 [x] : a ∈ R


 2
= x −x

luego el kernel tiene dimensión 1 (la nulidad es 1).

b) Notemos que T (ax2 + bx + c) = (a + b) x2 + (a + b + c) x + c y luego

(a + b) x2 + (a + b + c) x + c = a x2 + x + b x2 + x + c (x + 1)
 

ası́
T ax2 + bx + c ∈ x2 + x, x2 + x, x + 1



como hay un elemento repetido, el conjunto es L.D. ası́

x2 + x, x2 + x, x + 1 x2 + x, x + 1



=

se sigue
x2 + x, x + 1


Im (T ) ⊆

118
Apuntes Mat023 (versión preliminar actualizada 23-05-2014)

por el teorema de las dimensiones se sigue

Dim (ker (T )) + Dim (Im (T )) = Dim (R2 [x])

ası́, de la parte a) se tiene

1 + Dim (Im (T )) = 3

de donde se obtiene
Dim (Im (T )) = 2

y ası́
x2 + x, x + 1


Im (T ) =

de esta forma, una base para la imagen es {x2 + x, x + 1}

c) Como T (ax2 + bx + c) = (a + b) x2 + (a + b + c) x + c se sigue que

T 1 + x2 = x2 + 2x + 1 = α (1 − x) + β (1 + x) + γx2


= γx2 + (β − α) x + (a + β)

por determinar a, β, γ.

T 1 + x + x2 = 2x2 + 3x + 1 = α (1 − x) + β (1 + x) + γx2


= γx2 + (β − α) x + (a + β)

por determinar a, β, γ.

T (−1) = −x − 1 = α (1 − x) + β (1 + x) + γx2
= γx2 + (β − α) x + (a + β)

por determinar a, β, γ.
Vamos a resolver esos 3 problemas de una sola vez, porque hay que resolver los
sistemas

γ = 1
(β − α) = 2
(a + β) = 1

119
Apuntes Mat023 (versión preliminar actualizada 23-05-2014)

γ = 2
(β − α) = 3
(a + β) = 1

γ = 0
(β − α) = −1
(a + β) = −1

en todas la matriz de coeficientes es la misma, formamos una tripe matriz


ampliada y escalonamos
   
0 0 1 1 2 0 1 0 0 − 21 −1 0
  Operaciones Elementales  
 −1 ∼ 3
 1 0 2 3 −1 

 0 1 0
 2
2 −1 

1 1 0 1 1 −1 0 0 1 1 2 0
 
− 12 −1 0
[T ]D
 
3
B =  −1  2

2 
1 2 0
(notar que T (1 + x2 ) = x2 + 2x + 1 = − 12 (1 − x) + 3
 
2
(1 + x) + 1x2 ,
T (1 + x + x2 ) = 2x2 + 3x + 1 = (−1) (1 − x) + 2 (1 + x) + 2x2 y T (−1) =
−x − 1 = 0 (1 − x) + (−1) (1 + x) + 0x2 )

7. Suponga que S : R2 [x] → R2 [x] es una transformación lineal con


 
− 21 −1 0
[T ]D
 
=  3 2 −1 
B  2 
1 2 0

B = {1 − x, 1 + x, x2 } y D = {1 + x2 , 1 + x + x2 , −1} (No es el ejercicio anterior ver


las bases)

a) Encontrar ker (T )

120
Apuntes Mat023 (versión preliminar actualizada 23-05-2014)

b) Encontrar Im (T )

c) Calcule T (2 + x − x2 )

d ) Encontrar una expresión para T (ax2 + bx + c)

a) Note que p ∈ ker (T ) si y solo si T (p) = 0 si y solo si


 
0
 
[T p]D = 
 0


0
pero
[T p]D = [T ]D
B [p]B

luego si resolvemos
    
0 − 21 −1 0 α
 = [T ]D [p] = 
    
3
 0
  B B  2
2 −1 
 β 
 

0 1 2 0 γ

estaremos encontrando las coordenadas


 
α
 
 β  = [p]
  B

de los vectores de ker (T ). Pues bien


   
− 12 −1 0 1 0 −2
   
 3 2 −1 ∼ 0 1 1 
 2   
1 2 0 0 0 0

se sigue que p ∈ ker (T ) si y solo si


 
α
 
 β  = [p]
  B

cumple
α = 2γ y β = −γ

121
Apuntes Mat023 (versión preliminar actualizada 23-05-2014)

ası́ p ∈ ker (T ) si y solo si

p (x) = (2γ) (1 − x) + (−γ) (1 + x) + γx2

para algún γ ∈ R es decir

−3x + x2 + 1


ker (T ) =

b) De la matriz asociada tenemos


 
− 21
 
[T (1 − x)]D =  3  ası́
 2 
1
   
1 2 3
1 + x + x2 + (1) (−1)
 
T (1 − x) = − 1+x +
2 2
3
= x2 + x
2

también
 
−1
 
[T (1 + x)]D = 
 2  ası́

2
T (1 − x) = (−1) 1 + x2 + (2) 1 + x + x2 + (2) (−1)
 

= x2 + 2x − 1

y
 
0
 
T x2
 
D
=  −1  ası́
 
0
2
= (0) 1 + x2 + (−1) 1 + x + x2 + (0) (−1)
  
T x
= −x2 − x − 1

luego  
32 2 2
Im (T ) = x + x, x + 2x − 1, −x − x − 1
2

122
Apuntes Mat023 (versión preliminar actualizada 23-05-2014)

del teorema de las dimensiones x2 + 32 x, x2 + 2x − 1, −x2 − x − 1 deberı́a ser




un conjunto L.D. en efecto


   
1 1 −1 1 0 −2
   
 3 2 −1  ∼  0 1 1 
 2   
0 −1 −1 0 0 0
 
3
(−2) x + x + (1) x2 + 2x − 1 = −x2 − x − 1
2

2
se sigue  
3
2 2
Im (T ) = x + x, x + 2x − 1
2
c) Para calcular T (2 + x − x2 ) podemos hacer lo siguiente
 
α
 
2 + x − x2
 
B
=
 β 

donde
2 + x − x2 = α (1 − x) + β (1 + x) + γx2

γ = −1
β−α = 1
α+β = 2

α = 21 , β = 23 , γ = −1 luego
    
− 21 −1 0 1
2
− 74
    
T 2 + x − x2
 
= 3 =3 19
D  2
2 −1 


 2 4


7
1 2 0 −1 2

se obtiene
     
2
 7 2
 19 2
 7
T 2+x−x = − 1+x + 1+x+x + (−1)
4 4 2
19 1
= 3x2 + x −
4 2

123
Apuntes Mat023 (versión preliminar actualizada 23-05-2014)

d ) Encontremos una expresión para

T ax2 + bx + c


notemos que
ax2 + bx + c = α (1 − x) + β (1 + x) + γx2

si y solo si

γ = a
β−α = b
α+β = c

esto es    
1
0 0 1 a 1 0 0 2
c − 21 b
   
1
 −1 1 0 b  ∼  0 1 0
   2
b + 12 c 

1 1 0 c 0 0 1 a
luego
   
2 1 1 1 1
ax + bx + c = c − b (1 − x) + b + c (1 + x) + ax2
2 2 2 2
podemos encontrar T (ax2 + bx + c) en dos formas
   
2 1 1 1 1
b + c T (1 + x) + aT x2
 
T ax + bx + c = c − b T (1 − x) +
2 2 2 2
    
1 1 2 3 1 1
b + c x2 + 2x − 1

= c− b x + x +
2 2 2 2 2
2

+a −x − x − 1
   
2 1 7 1 1
= (c − a) x + −a + b + c x + − b − c − a
4 4 2 2
la otra forma es
  
− 12 −1 1
− 12 b

0 2
c
  
T ax2 + bx + c
 
3 1
+ 21 c

D
= 
 2
2 −1  
 2
b 

1 2 0 a
 
− 14 b − 34 c
 
=  1 7 
 4
b − a + c
4 
1 3
2
b + 2c

124
Apuntes Mat023 (versión preliminar actualizada 23-05-2014)

y ası́
   
2 1 3 2 1 7
b − a + c 1 + x + x2
  
T ax + bx + c = − b− c 1+x +
4 4 4 4
 
1 3
+ b + c (−1)
2 2
   
2 1 7 1 1
= (c − a) x + −a + b + c x + − b − c − a
4 4 2 2

8. Muestre que T : M2×2 (R) → R3 [x] dada por


 
a b
T   = (a + b) x3 + cx2 + (a + d) x + (c − d)
c d

es una transformación lineal además determine el ker (T ) e Im(T ).

Solución. En general, si V y W son espacios vectoriales, una función F : V → W


es llamada transformación lineal si cumple: i) ∀α ∈ K,∀v ∈ V , F (αv) = αF (v) y ii)
∀v1 , v2 ∈ V , F (v1 + v2 ) = F (v1 ) + F (v2 ) (esto quiere decir, que envı́a combinaciones
lineales en combinaciones lineales). En nuestro ejercicio los elementos del espacio
de partida son matrices de orden 2 × 2 luego para probar la primera propiedad
necesitamos considerar un escalar arbitrario y una matriz arbitraria y en la segunda
propiedad, dos matrices arbitrarias como sigue:
 
a b
i) Sea α ∈ R y   ∈ M2×2 (R) se tiene
c d
    
a b αa αb
T α   = T  
c d αc αd
 
A B
= T   donde A = αa, B = αb, C = αc, D = αd
C D
= (A + B) x3 + Cx2 + (A + D) x + (C − D)
= ((αa) + (αb)) x3 + (αc) x2 + (αa + αd) x + (αc − αd)
= α (a + b) x3 + cx2 + (a + d) x + (c − d)

 
a b
= αT  
c d

125
Apuntes Mat023 (versión preliminar actualizada 23-05-2014)

(Explicación: tenemos que mostrar que el escalar se puede “sacar” de la función

a b
y para esto utilizamos solamente la definición, en la linea T α   =
c d
 
αa αb
T   no sabemos como actúa la función T sobre el producto escalar
αc αd
 
a b
α  es por eso que aplicamos la definición de producto escalar de matrices y
c d
ası́    
a b αa αb
α = 
c d αc αd
 
αa αb
y ahora podemos aplicar la definición de T a la matriz   (en este caso si
αc αd
sabemos como actúa T ), para que se vea mas claro agregué la linea
   
αa αb A B
 =  donde A = αa, B = αb, C = αc, D = αd
αc αd C D

pero no es necesario, entonces por definición


 
A B
T  
C D
= (A + B) x3 + Cx2 + (A + D) x + (C − D)

y ahora reemplazamos para obtener la igualdad deseada)


   
a11 a12 b b
ii) Sean   ,  11 12  ∈ M2×2 (R) matrices arbitrarias, se tiene
a21 a22 b21 b22
     
a11 a12 b11 b12 a11 + b11 a12 + b12
T  +  = T  
a21 a22 b21 b22 a21 + b21 a22 + b22
 
A B
= T  
C D

donde A = a11 +b11 , B = a12 +b12 , C = a21 +b21 , D = a22 +b22 , aplicando la definición

126
Apuntes Mat023 (versión preliminar actualizada 23-05-2014)

de T se sigue
 
A B
T   = (A + B) x3 + Cx2 + (A + D) x + (C − D)
C D
= ((a11 + b11 ) + (a12 + b12 )) x3 + (a21 + b21 ) x2 +
((a11 + b11 ) + (a22 + b22 )) x + ((a21 + b21 ) − (a22 + b22 ))
= ((a11 + a12 ) + (b11 + b12 )) x3 + (a21 + b21 ) x2 +
((a11 + a22 ) + (b11 + b22 )) x + ((a21 − a22 ) + (b21 − b22 ))
= (a11 + a12 ) x3 + a21 x2 + (a11 + a22 ) x + (a21 − a22 ) +


(b11 + b12 ) x3 + b21 x2 + (b11 + b22 ) x + (b21 − b22 )



   
a11 a12 b11 b12
= T   + T   
a21 a22 b21 b22

de esto concluimos
   
a11 a12 b11 b12
T  + 
a21 a22 b21 b22
   
a11 a12 b11 b12
= T   + T  
a21 a22 b21 b22

de i) y ii) T es una transformación lineal.

Vamos a buscar el ker (T )

ker (T ) = {v ∈ V : T (v) = θW }
    
 a b a b 
=   ∈ M2×2 (R) : T   = 0 (el polinomio 0)
 c d c d 
  
 a b 
=   ∈ M2×2 (R) : (a + b) x3 + cx2 + (a + d) x + (c − d) = 0
 c d 
  
 a b 
=   ∈ M2×2 (R) : (a + b) = 0 ∧ c = 0 ∧ (a + d) = 0 ∧ (c − d) = 0
 c d 

127
Apuntes Mat023 (versión preliminar actualizada 23-05-2014)

 
a b
de esta forma buscamos todas la matrices   tales que
c d

a+b = 0
c = 0
a+d = 0
c−d = 0

o matricialmente     
1 1 0 0
a 0
    
 0 0 1 0  b   0 
  
  = 
 
 
 1 0 0 1  c   0 
  
 
0 0 1 −1 d 0
resolvemos el sistema escalonando
   
1 1 0 0 0 1 1 0 0 0
   
 0 0 1 0 0   0 −1 0 1 0 
∼
   
 
 1 0 0 1 0   0 0 1 0 0 
 
 
0 0 1 −1 0 0 0 0 −1 0

de esto se obtiene a = b = c = d = 0 es decir


 
 0 0 
ker (T ) =  
 0 0 

(se concluye que la transformación es inyectiva) Por el teorema de las dimensiones se


tiene lo siguiente

Dim (ker (T )) + Dim (Im (T )) = Dim (M2×2 (R)) = 4

pero
Dim (ker (T )) = 0

ası́
Dim (Im (T )) = 4

128
Apuntes Mat023 (versión preliminar actualizada 23-05-2014)

como Dim (T ) < R3 [x] y R3 [x] tiene dimensión 4, se sigue Im (T ) = R3 [x]. Otra
forma para determinar la imagen es utilizar el resultado que la imágenes de una base
generan la imagen de la transformación, luego
*        +
 1 0 0 1 0 0 0 0 
Im (T ) = T ,T  ,T  ,T  
 0 0 0 0 1 0 0 1 

pero notemos que


 
1 0
T  = (1 + 0) x3 + 0x2 + (1 + 0) x + (0 − 0)
0 0
= x3 + x

 
0 1
T  = (0 + 1) x3 + 0x2 + (0 + 0) x + (0 − 0)
0 0
= x3

 
0 0
T  = (0 + 0) x3 + 1x2 + (0 + 0) x + (1 − 0)
1 0
= x2 + 1

 
0 0
T  = (0 + 0) x3 + 0x2 + (0 + 1) x + (0 − 1)
0 1
= x−1

ası́
x3 + x, x3 , x2 + 1, x − 1


Im (T ) =

y mostrar que {x3 + x, x3 , x2 + 1, x − 1} es un conjunto L.I. luego Im (T ) tiene di-


mensión 4 y por tanto tiene que ser R3 [x].

9. Encontrar por lo menos una transformación lineal T : M2×2 (R) → R3 tal que
  
 a b 
ker (T ) =   ∈ M2×2 (R) : a + d + c = 0
 c d 

129
Apuntes Mat023 (versión preliminar actualizada 23-05-2014)

Solución. Notemos que


  
 a b 
ker (T ) =   ∈ M2×2 (R) : a + d + c = 0
 c d 
  
 a b 
=   ∈ M2×2 (R) : a = −c − d
 c d 
  
 −c − d b 
=   ∈ M2×2 (R) : b, c, d ∈ R
 c d 
       
 −1 0 0 1 −1 0 
= c  + b  + d  ∈ M2×2 (R) : b, c, d ∈ R
 1 0 0 0 0 1 
*     +
 −1 0 0 1 −1 0 
 , , 
 1 0 0 0 0 1 

luego ker (T ) tiene dimensión 3 (no puede ser base de M2×2 (R) que tiene dimensión
4), sabemos que para conocer completamente una T.L. necesitamos conocerla sobre
una base, buscamos algún elemento L.I. con las tres matrices que forman el kernel,
para ello,
         
−1 0 0 1 −1 0 a b 0 0
α +β +γ +δ = 
1 0 0 0 0 1 c d 0 0

debe tener por única solución α = β = γ = δ = 0. Esto se traduce a

−α − γ + aδ = 0
β + bδ = 0
α + δc = 0
γ + δd = 0

matricialmente la ampliada
   
−1 0 −1 a 0 −1 0 −1 a 0
   
 0 1 0 b 0 
  0 1 0 0  b


 
   
 1 0 0
 c 0 
  0
 0 −1 a+c 0 

0 0 1 d 0 0 0 0 a+c+d 0

130
Apuntes Mat023 (versión preliminar actualizada 23-05-2014)

6 0, de esta forma, podemos considerar la


la solución es única si y solo si a + c + d =
matriz  
1 1
 
1 1
pues 1 + 1 + 1 = 3 6= 0. Ahora para definir la transformación (por ejemplo)
 
−1 0
T  = (0, 0, 0)
1 0
 
0 1
T  = (0, 0, 0)
0 0
 
−1 0
T  = (0, 0, 0)
0 1
 
1 1
T  = (1, 1, 1)
1 1
     
−1 0 0 1 −1 0
esto determina completamente quien es T (notar que  , , 
1 0 0 0 0 1
 
1 1
son elementos del kernel pero   no lo es). La T en forma explı́cita se deter-
1 1
mina de la siguiente manera, Como
       
 −1 0 0 1 −1 0 1 1 
B=  , , , 
 1 0 0 0 0 1 1 1 
 
x y
es base, dado una matriz   arbitraria, deben existir escalares α, β, γ, δ tales
z w
que
         
x y −1 0 0 1 −1 0 1 1
  = α +β +γ +δ 
z w 1 0 0 0 0 1 1 1

131
Apuntes Mat023 (versión preliminar actualizada 23-05-2014)

para buscar tales escalares resolvemos el sistema


   
2
−1 0 −1 1 x 1 0 0 0 3
z − 13 x − 13 w
   
 0 1 0 1 y    0 1 0
 0 y − 31 x − 13 w − 13 z 

 
   2

 1
 0 0 1 z    0 0 1
 0 3
w − 13 x − 13 z 

1
0 0 1 1 w 0 0 0 1 3
w + 31 x + 31 z
de donde se tiene
2 1 1
α = z− x− w
3 3 3
1 1 1
β = y− x− w− z
3 3 3
2 1 1
γ = w− x− z
3 3 3
1 1 1
δ = w+ x+ z
3 3 3
ası́
   
x y −1 0
 
2 1 1
  = z− x− w  
z w 3 3 3 1 0
 
0 1
 
1 1 1
+ y− x− w− z  
3 3 3 0 0
 
−1 0
 
2 1 1
+ w− x− z  
3 3 3 0 1
 
1 1
 
1 1 1
+ w+ x+ z  
3 3 3 1 1
luego
 
x y
 
2 1 1
T  = z − x − w (0, 0, 0)
z w 3 3 3
 
1 1 1
+ y − x − w − z (0, 0, 0)
3 3 3
 
2 1 1
+ w − x − z (0, 0, 0)
3 3 3
 
1 1 1
+ w + x + z (1, 1, 1)
3 3 3
 
1 1 1 1 1 1 1 1 1
= w + x + z, w + x + z , w + x + z
3 3 3 3 3 3 3 3 3

132
Apuntes Mat023 (versión preliminar actualizada 23-05-2014)

ası́

TM2×2 (R) → R3
:
   
x y x y
 
 → T = 1 1 1 1 1 1 1 1 1
 w + x + z, w + x + z , w + x + z
z w z w 3 3 3 3 3 3 3 3 3

10. Sea B la base canónica de R2 y considere las transformaciones T : R2 → R2 y


S : R2 → R2 definidas por

T (a, b) = (2a + 3b, 4a − 7b)

y
S (a, b) = (3a − 2b, a − b)

calcular [T ]B , [S]B y [T ◦ S]B comprobar que

[T ]B · [S]B = [T ◦ S]B

Solución. Al ser la matriz con respecto a la base canónica se tiene


   
2 3 3 −2
[T ]B =   y [S] = 
B

4 −7 1 −1

ası́     
2 3 3 −2 9 −7
[T ]B · [S]B =   = 
4 −7 1 −1 5 −1
por otro lado

T ◦ S (a, b) = T (3a − 2b, a − b)


= (2 (3a − 2b) + 3 (a − b) , 4 (3a − 2b) − 7 (a − b))
 
= 9a − 7b 5a − b

ası́  
9 −7
[T ◦ S]B =  
5 −1

133
Apuntes Mat023 (versión preliminar actualizada 23-05-2014)

11. Sea T : R4 → R4 una transformación lineal tal que

T (1, 1, 0, 0) = (0, 1, 0, −1)


T (1, 0, 1, 0) = (1, 1, 1, 0)

si además se tiene que T ◦ T = IR4 determinar la matriz asociada a T respecto a la


base canónica C de R4 . ¿Es T un isomorfismo? Justifique.

Solución. Por la propiedad T ◦ T = IR4 se sigue

T (T (1, 1, 0, 0)) = T (0, 1, 0, −1) = (1, 1, 0, 0)


T (T (1, 0, 1, 0)) = T (1, 1, 1, 0) = (1, 0, 1, 0)

ası́

T (1, 1, 0, 0) = (0, 1, 0, −1)


T (1, 0, 1, 0) = (1, 1, 1, 0)
T (0, 1, 0, −1) = (1, 1, 0, 0)
T (1, 1, 1, 0) = (1, 0, 1, 0)

queremos determinar la matriz asociada a la base canónica

T (1, 1, 1, 0) − T (1, 1, 0, 0) = T (0, 0, 1, 0)


= (1, 0, 1, 0) − (0, 1, 0, −1)
= (1, −1, 1, 1)

T (0, 0, 1, 0) = (1, −1, 1, 1)


T (1, 0, 0, 0) = T (1, 0, 1, 0) − T (0, 0, 1, 0)
= (1, 1, 1, 0) − (1, −1, 1, 1)
= (0, 2, 0, −1)
T (0, 1, 0, 0) = T (1, 1, 0, 0) − T (1, 0, 0, 0)
= (0, 1, 0, −1) − (0, 2, 0, −1)
= (0, −1, 0, 0)
T (0, 0, 0, 1) = T (0, 1, 0, 0) − T (0, 1, 0, −1)
= (0, −1, 0, 0) − (1, 1, 0, 0)
= (−1, −2, 0, 0)

134
Apuntes Mat023 (versión preliminar actualizada 23-05-2014)

ası́  
0 0 1 −1
 
 2 −1 −1 −2 
[T ]CC = 
 

 0 0 1 0 
 
−1 0 1 0
es fácil ver que es isomorfismo, pues

T ◦T = I
⇒ [T ◦ T ]CC = I4
⇒ [T ]CC [T ]CC = I4

ası́ [T ]CC es invertible y luego T es invertible. Otra forma



0 0 1 −1


2 −1 −1 −2
=1


0 0 1 0


−1 0 1 0

y luego T es invertible.

Otra forma es utilizar la matriz de cambio de base


 
1 1 0 1
 
 1 0 1 1 
[I]CB = 
 

 0 1 0 1 
 
0 0 −1 0

ası́  −1  
1 1 0 1 1 0 −1 0
   
 1 0 1 1   1 −1 0 −1 
[I]BC = =
   
 
 0 1 0 1 
 
 0
 0 0 −1 

0 0 −1 0 −1 1 1 1

135
Apuntes Mat023 (versión preliminar actualizada 23-05-2014)

luego

[T ]CC = [T ]CB [I]BC


  
0 1 1 1 1 0 −1 0
  
 1 1 1 0  1 −1 0 −1 
= 
  
 
 0 1 0 1 
  0 0 0 −1 

−1 0 0 0 −1 1 1 1
 
0 0 1 −1
 
 2 −1 −1 −2 
= 
 

 0 0 1 0 
 
−1 0 1 0

Ejercicios de la sección

1. Determine si las siguientes funciones son transformaciones lineales:

a) T : R2 → R, T (x, y) = |x + y|

b) T : R3 → R2 , T (x, y, z) = (2x + y, z − y)

c) T : R → R3 , T (x) = (x, 2x, x2 )

d ) T : R3 → R3 , T (x, y, z) = (cos x, sin y, z)

2. ¿Existe un transformación lineal de R2 en R2 que transforma los vectores (1, 4) y


(−3, 2) en los vectores (2, 1) y (4, 2), respectivamente? En caso afirmativo determine
la imagen del vector (−2, a), con a ∈ R.

3. Sea U un espacio vectorial tal que dim U = n < ∞. Suponga que u1 , u2 , . . . , un es


una base ordenada de U . Suponga que T : U → U es tal que:

T (ui ) = ui , ∀i = 1, 2, . . . , n (2.3)

Entonces:

a) Demuestre que existen funciones T : U → U definidas por la condición (2.3) que


son transformaciones lineales.

136
Apuntes Mat023 (versión preliminar actualizada 23-05-2014)

b) Verifique que si T es una transformación lineal que satisface la condición (2.3),


entonces T = IU , donde IU es la función identidad de U en U .

4. Sea T : R3 → R2 la transformación lineal definida por:

T (x, y, z) = (x + y − 2z, x + y)

Calcule ker T e ImT .

5. Sea S ∈ L (R3 , M2 (R)) definida por:


 
x − 3y y − 2z
S (x, y, z) =  
x z

a) Calcule condiciones sobre a ∈ R de modo que:

3e1 + ae2 − e3 ∈ kerT

donde ei es el i–ésimo vector de la base canónica de R3 .

b) Calcule ImT

6. Sea T : R2 [x] → M2 (R) una función definida por:


 R1 
00
 p (0) 0 p (x) dx
T p (x) =  
p (−1) 0

a) Demuestre que T es una transformación lineal.

b) Hallar una base para el kernel de T .

c) Hallar ImT y su dimensión.

7. Sea T ∈ L (U, V ) tal que ker T = {0}. Suponga que B es un conjunto linealmente
independiente en U . Demuestre que:

T (B) = {T (u) : u ∈ B}

es linealmente independiente en V .

8. Sea T ∈ L (U, U ). Entonces, las siguientes proposiciones son equivalentes:

137
Apuntes Mat023 (versión preliminar actualizada 23-05-2014)

(i) ker T ∩ ImT = {0}.

(ii) Si T 2 (u) = 0, entonces T (u) = 0.

9. Sea T : R3 → R3 la transformación lineal definida por:

T (x, y, z) = (3x, x − y, 2x + y + z)

¿Es T invertible? En caso afirmativo, hallar una fórmula para T −1 .

10. Sean U y V espacios vectoriales sobre el mismo cuerpo K, tales que dim U = dim V =
n < ∞. Sea, además, T ∈ L (U, V ) tal que:

T u = 0 =⇒ u = 0

Entonces, existe una base u1 , u2 , . . . , un de U tal que T u1 , T u2 , . . . , T un es una base


de V .

11. Sea B = (1, 2, 1) ; (2, 1, 1) , (1, 1, 2) una base de R3 y T : R3 → R3 una transfor-
mación lineal tal que:  
−3 −3 −5
[T ]BB = 
 
 1 1 1 

2 2 4

a) Determinar una base del ker T .

b) Determinar una base del ker T 2 .

c) Determinar una base de ImT .

12. Sea T : R3 → R3 tal que:  


1 2 0
[T ]D
 
B =  1 1 1 


3 −1 1
donde B = {(1, 1, 2) ; (0, 1, 1) ; (−1, 0, 0)} y D = {(1, 1, 1) ; (0, 1, 1) ; (0, 0, 1)} son
bases de R3 . Entonces:

a) Encuentre T (1, 3, 1), sin calcular T (x, y, z).

b) Determine T −1 (x, y, z), si acaso existe.

138
Apuntes Mat023 (versión preliminar actualizada 23-05-2014)

c) Hallar [T ]F
E , sin calcular T (x, y, z), si E = {(0, 1, 2) ; (1, 1, 1) ; (0, 1, 0)} y F =

{(1, 1, 0) ; (1, −1, 1) ; (1, 0, 0)} son, también, bases de R3 .

13. Sean T : R3 [x] → R3 [x] una función definida por:

T p (x) = p0 (x) − x · p00 (x)




y B1 = {1 − x2 , 1 + x2 , 1 − x3 , 2 + x}, B2 = {1, x2 + x3 , x3 − x2 , 1 − 2x} dos bases


ordenadas de R3 [x]. Entonces:

a) Pruebe que existe T es una transformación lineal.

b) Hallar ker T y una base para ImT . ¿Es T invertible?

c) Calcule [T ]BB21 .

14. Diremos que un espacio vectorial V es suma directa de los subespacios W1 y W2 ,


lo cual anotaremos, V = W1 ⊕ W2 , si W1 ∩ W2 = {0}, y para cada v ∈ V , existen
w1 ∈ W1 y w2 ∈ W2 tales que v = w1 + w2 . Sean V un espacio vectorial de dimensión
finita y T : V → V una transformación lineal. Pruebe que:

V = ker T ⊕ ImT ⇐⇒ ker T 2 = ker T




15. Sean U y V espacios vectoriales sobre el mismo cuerpo K y sea S un isomorfismo de


U en V . Demuestre que:
T 7→ S T S −1

es un isomorfismo de L (U, U ) en L (V, V ).

16. Hallar explı́citamente, esto es, calculando T (ax3 + bx2 + cx + d), una transformación
lineal T : R3 [x] → R3 tal que:

ImT = (x, y, z) ∈ R3 : x + 2y − z = 0


y cuya nulidad sea 2.

17. Sean:
B = {(1, −2, 1) , (0, 1, −2) , (1, 1, 1)}

139
Apuntes Mat023 (versión preliminar actualizada 23-05-2014)

y:        
 1 2 0 0 1 −2 2 1 
D=  ,  ,  ,  
 −2 1 1 −1 3 −3 2 1 

bases ordenadas de R3 y M2 (R), respectivamente. Considere T : R3 → M2 (R)


definida por:  
x + 2y x − y + 3z
T (x, y, z) =  
0 x+y+z

a) Hallar [T ]D
B.

b) Determine condiciones sobre a ∈ R para que (x, y, z) ∈ ker T , sabiendo que:


 
−1
 
[(x, y, z)]B = 
 a 

2

c) Hallar ImT , usando [T ]D


B.

18. Sean T : M2 (R) → M2 (R) una transformación lineal y:


       
 1 0 1 1 1 0 1 1 
B=  , , , 
 0 1 0 1 1 1 2 2 

una base ordenada de M2 (R) tal que:


 
1 0 1 0
 
 1 2 3 0 
[T ]BB = 
 

 1 1 2 0 
 
1 1 2 1

Determinar una base D de M2 (R) de modo que [T ]D


D sea una matriz diagonal.

(Ayuda: El polinomio caracterı́stico de T es fT (λ) = λ (λ − 4) (λ − 1)2 )

19. Sean:
B = 1 + x − x2 , 2x − x2 , 1 + x2


y:
D = {(1, 2, 1) , (1, 3, 2) , (2, 1, 3)}

140
Apuntes Mat023 (versión preliminar actualizada 23-05-2014)

bases ordenadas de R2 [x] y R3 , respectivamente. Consideremos T : R2 [x] → R3 tal


que:  
5 2 −2
[T ]D
 
B =  −3 −1 1 


1 4 −3

a) Demuestre que T es un isomorfismo.

b) Calcule ker T −1 e ImT −1 .

c) Hallar explı́citamente T −1 .

20. Sean B = {1 + x, 1 − x2 , 1} y D = {1, 1 + x, 1 + x + x2 } bases ordenadas de R2 [x] .


Considere T : R2 [x] → R2 [x] una transformación lineal tal que:
 
1 0 1
[T ]D
 
B =  0 1 1 
 

1 1 2

a) Hallar condiciones sobre a ∈ R para que:

1 − (a + 1) x − 2x2 ∈ ker T

b) Hallar condiciones sobre b ∈ R para que:

1 − 2x + (a − 1) x2 ∈ ker T −1

21. Sea:
W = (x, y, z) ∈ R3 : 2x − 3y + 5z = 0


un subespacio de R3 tal que B es una base W . Considere D = {(1, 1) , (2, 1)} una
base de R2 . Sea T ∈ L (W, R2 ) definida por T (x, y, z) = (x + y − z, 3x − y + 2z) tal
que:  
1 2
[T ]D
B =
 
−3 −1
Determine la base B.

141
Apuntes Mat023 (versión preliminar actualizada 23-05-2014)

22. Considere el plano en R3 dado por:

W = (x, y, z) ∈ R3 : ax + by + cz = 0


con a, b, c ∈ R. Para cualquier u ∈ R3 , denotaremos por PW (u) al punto del plano W


que se encuentra a menor distancia de u. Tal función PW (u) se llama la proyección
ortogonal de u sobre W . Entonces:

a) Dado u ∈ R3 , hallar una expresión para PW (u).

b) Demuestre que la función PW (u) : R3 → R3 definida por:

u 7→ PW (w)

es una transformación lineal.

c) Calcule [PW ]CC , donde C es la base canónica de R3 , y verifique que:


 2  T
[PW ]CC = [PW ]CC ∧ [PW ]CC = [PW ]CC

23. Considere el subespacio vectorial W < R2 [x] definido por:

W = 1 − x2 , x + 2x2

Demuestre que existe una transformación lineal T : R2 [x] → R2 [x] diagonalizable tal
que −1 y 1 sean sus valores propios, y que el espacio propio asociado a −1 sea W .

Ecuaciones diferenciales lineales

Definiciones

Definición 2.2.1. Sean I ⊆ R un intervalo, A0 , A1 , . . . , An y R funciones continuas sobre


I tales que A (x) 6= 0, para cada x ∈ I. Una ecuación diferencial de la forma:

An (x) y (n) + An−1 (x) y (n−1) + An−2 (x) y (n−2) + · · · + A1 (x) y 0 + A0 (x) y = R (x) (2.4)

se denomina ecuación diferencial lineal de orden n. Las funciones A0 , A1 , . . . , An son


llamados coeficientes de la ecuación diferencial. Si R ≡ 0, la ecuación lineal se denomina
ecuación homogénea.

142
Apuntes Mat023 (versión preliminar actualizada 23-05-2014)

Observación 2.2.1. En general, los puntos en los cuales An (x) = 0 se conocen como
puntos singulares y su estudio no será considerado en este curso.

Observación 2.2.2. La ecuación (2.4) puede escribirse como:

y (n) + pn−1 (x) y (n−1) + pn−2 (x) y (n−2) + · · · + p1 (x) y 0 + p0 (x) y = Q (x) (2.5)

al dividir la ecuación (2.4) por An (x), de esta forma


Ai (x)
pi (x) = para i = 0, . . . , n − 1
An (x)
y
R (x)
Q (x) =
An (x)
esta forma de la ecuación es llamada forma normal.

Ejemplo 2.2.1. Son ecuaciones diferenciales lineales las siguientes ecuaciones:

1. y (4) + 3xy 00 − 2 (cos x) y 0 + y = e2x cos 3x

2. y 00 + y = 0

3. Las ecuaciones de Euler, definidas como:

(ax + b)n y (n) + A1 (ax + b)n−1 y (n−1) + · · · + An−1 (ax + b) y 0 + An y = Q (x)

con ax + b > 0.

4. las ecuaciones de Legendre, definidas como:

1 − x2 y 00 − 2xy 0 + α (α + 1) y = 0


con α ∈ R.

Ejemplo 2.2.2. Considere la ecuación diferencial:

y 00 − 2y 0 + 2y = 0

1. Verifique que las funciones u1 (x) = ex cos x y u2 (x) = ex sin x son soluciones de la
ecuación diferencial.

143
Apuntes Mat023 (versión preliminar actualizada 23-05-2014)

2. Sean C1 , C2 ∈ R. Verifique que y (x) = C1 u1 (x) + C2 u2 (x) es solución de la ecuación


diferencial.

3. Hallar una solución y que satisfaga las condiciones iniciales y (0) = 1 e y 0 (0) = 4.

Solución. Por partes:

1. Notemos que
d x
u01 (x) = (e cos x) = ex cos x − ex sin x
dx
d x
u001 (x) = (e cos x − ex sin x) = −2ex sin x
dx
se sigue

u001 − 2u01 + 2u1 = (−2ex sin x) − 2 (ex cos x − ex sin x) + 2 (ex cos x)
= 0

de manera similar
d x
u02 (x) = (e sin x) = ex (cos x + sin x)
dx
d x
u002 (x) = (e (cos x + sin x)) = 2 (cos x) ex
dx
luego

u002 − 2u02 + 2u2 = (2 (cos x) ex ) − 2 (ex (cos x + sin x)) + 2 (ex sin x)
= 0

2. Notar que

y 0 (x) = C1 u01 (x) + C2 u02 (x)


y 00 (x) = C1 u001 (x) + C2 u002 (x)

luego

y 00 − 2y 0 + 2y = (C1 u001 (x) + C2 u002 (x)) − 2 (C1 u01 (x) + C2 u02 (x)) + 2 (C1 u1 (x) + C2 u2 (x))
= (C1 u001 (x) − 2C1 u01 (x) + 2C1 u1 (x)) + (C2 u002 (x) − 2C2 u02 (x) + 2C2 u2 (x))
= C1 (u001 (x) − 2u01 (x) + 2u1 (x)) + C2 (u002 (x) − 2u02 (x) + 2u2 (x))
= C1 0 + C2 0
= 0

144
Apuntes Mat023 (versión preliminar actualizada 23-05-2014)

3. Sabemos que y (x) = C1 ex cos x + C2 ex sin x es solución de la ecuación, para cumplir


las condiciones y (0) = 1 e y 0 (0) = 4 las constantes las debemos escoger en forma
adecuada

1 = y (0) = C1
4 = y 0 (0) = C1 + C2

ası́

1 = C1
4 = C1 + C2

que tiene solución C1 = 1, C2 = 3. Ası́

y (x) = ex cos x + 3ex sin x

Observación 2.2.3. Con el objeto de simplificar el estudio de las ecuaciones diferenciales


lineales y el de aprovechar la terminologı́a de las transformaciones lineales es que consi-
deraremos la noción de operador diferencial asociado a una ecuación diferencial lineal de
orden n. Para ello recordemos que C n (I) denota el espacio vectorial de todas las funciones
de clase C n definidas sobre I, esto es el conjunto de todas las funciones f : I → R que son
n veces derivables con f (n) continua sobre I.

Definición 2.2.2. Sean p0 , p1 , . . . , pn−1 funciones continuas sobre un intervalo I ⊆ R.


Llamaremos operador diferencial asociado a la ecuación (2.5) a la función L : C n (I) → C (I)
definida por:
L (f ) = f (n) + pn−1 f (n−1) + · · · + p0 f (2.6)

Observación 2.2.4. El operador diferencial dado por la ecuación (2.6) puede escribirse
como:
L = Dn + pn−1 Dn−1 + · · · + p0 Id

en donde Dk representa la derivada de orden k y Id representa la transformación lineal


identidad.

Teorema 2.2.1. Sea I ⊆ R un intervalo cualquiera. El operador diferencial L : C n (I) →


C (I) es una transformación lineal.

145
Apuntes Mat023 (versión preliminar actualizada 23-05-2014)

Observación 2.2.5. Con la introducción del operador diferencial L toda ecuación lineal:

y (n) + pn−1 (x) y (n−1) + · · · + p0 (x) y = Q (x)

puede escribirse como:


L (y) = Q (2.7)

Ası́, la ecuación:
L (y) = 0

se llamará ecuación homogénea asociada a la ecuación (2.7).

Observación 2.2.6. La ecuación (2.7) tiene un análogo finito dimensional con sistemas
de ecuaciones lineales y consecuentemente con la respectiva ecuación matricial asociada.
En efecto, sea S un sistema de m ecuaciones lineales con n incógnitas. Luego, existe una
matriz A ∈ Mm×n (K) que contiene los coeficientes del sistema, una matriz X ∈ Mm×1 (K)
con las incógnitas y además, una matriz B ∈ Mm×1 (K) con las constantes. El producto
de matrices permite representar el sistema S como la ecuación matricial siguiente:

AX = B

Observe que las propiedades del álgebra de matrices nos permiten escribir X como:

X = X h + Xp

donde Xh representa la solución general del sistema homogéneo AX = 0 y Xp representa


una solución particular del sistema que se asume conocida. En efecto:

AX = A (Xh + Xp )
= AXh + AXp
= 0+B
= B

Ahora bien, mediante el isomorfismo LK (U, V ) ' Mm×n (K), debe existir una transforma-
ción lineal T : U → V tal que:
A = [T ]CB

146
Apuntes Mat023 (versión preliminar actualizada 23-05-2014)

para ciertas bases ordenadas B y D, de U y V , respectivamente. Por tanto, el espacio


solución de la ecuación matricial es, salvo cambio de coordenadas, el núcleo o espacio nulo
de la transformación lineal T . Por esta razón, nos interesará calcular el núcleo o kernel del
operador diferencial L.

Teorema 2.2.2. Suponga que yh representa la solución de la ecuación homogénea asociada


a L (y) = Q y que yp es una solución particular (esto es, L (yp ) = Q) de la misma ecuación.
Entonces, la solución general de:
L (y) = Q

es de la forma:
y = yh + yp

En efecto, siguiendo los razonamientos de la observación anterior, tenemos que:

L (y) = L (yh + yp )
= L (yh ) + L (yp )
= 0+Q
= Q

Observación 2.2.7. Consideremos la ecuación diferencial:

y (n) + pn−1 (x) y (n−1) + pn−2 (x) y (n−2) + · · · + p1 (x) y 0 + p0 (x) y = 0

Por lo expuesto anteriormente, el operador lineal asociado L : C n (I) → C (I) definido por:

L = Dn + pn−1 Dn−1 + · · · + p1 D + p0 Id

permite expresar la ecuación como L (y) = 0. Por tanto, una función y es solución de la
ecuación diferencial homogénea si y solo si y ∈ ker L, notamos además que:

ker L ≤ C n (I)

y el espacio vectorial C n (I) no es de dimensión finita (basta ver que para cada n ≥ 1, el
conjunto 1, x, x2 , . . . , xn es linealmente independiente). Una aplicación muy importante
del Teorema de Existencia y Unicidad para ecuaciones diferenciales será la de mostrar que el

147
Apuntes Mat023 (versión preliminar actualizada 23-05-2014)

núcleo de un operador diferencial no sólo es de dimensión finita, sino que, además, es de


dimensión n, en donde n es el orden de la ecuación diferencial lineal. Es decir, si:

L = Dn + pn−1 Dn−1 + · · · + p1 D + p0 Id

es un operador diferencial asociado a una ecuación diferencial lineal, entonces:

dim ker L = n

Teorema de Existencia y Unicidad

Observación 2.3.1. El siguiente teorema juega un rol crucial en establecer que la dimen-
sión del núcleo del operador diferencial L asociado a una ecuación diferencial lineal de
orden n del tipo:
y (n) + pn−1 (x) y (n−1) + · · · + p0 (x) y = Q (x)

es de dimensión finita. En particular, en esta sección se utilizará el Teorema de Existencia


y Unicidad para demostrar que no sólo es finita esa dimensión, sino que además es de
dimensión coincide con el orden de la ecuación lineal. Introduzcamos primeramente la
siguiente definición:

Definición 2.3.1. Diremos que la función ϕ : I ⊆ R → R es solución del problema de


valores iniciales de orden n:

y (n) = f x, y, y 0 , y 00 , . . . , y (n−1)


y (x0 ) = y0 , y 0 (x0 ) = y1 , . . . , y (n−1) (x0 ) = yn−1

si ϕ : I ⊆ R → R es solución de la ecuación en su intervalo de definición, x0 ∈ I y:

ϕ (x0 ) = y0 , ϕ0 (x0 ) = y1 , . . . , ϕ(n−1) (x0 ) = yn−1

Observación 2.3.2. Para mayor claridad de los conceptos los teoremas serán expuestos
para ecuaciones de orden 2 y notar que la extensión se hace por inducción sobre n. Ası́, los
resultados se indicarán para orden 2 u orden n según convenga.

Teorema 2.3.1. Sean I ⊆ R un intervalo abierto y p0 , p1 dos funciones continuas sobre I.


Considere el operador diferencial:

L (y) = y 00 + p1 y 0 + p0 y

148
Apuntes Mat023 (versión preliminar actualizada 23-05-2014)

Si x0 ∈ I e y0 , y1 son números reales cualesquiera, entonces existe una única función


f : I → R que es solución de la ecuación diferencial:

L (y) = 0

y que satisface las condiciones iniciales:

f (x0 ) = y0 ∧ f 0 (x0 ) = y1

Teorema 2.3.2. Consideremos el operador diferencial L (y) = y 00 + p1 y 0 + p0 y, con p0 y


p1 funciones continuas sobre un intervalo abierto I ⊆ R. Suponga que u1 y u2 son dos
funciones no nulas, linealmente independientes sobre I, y que son solución de la ecuación
diferencial L (y) = 0. Entonces, para todo par de constantes c1 y c2 , la combinación lineal:

c1 u1 + c2 u2

es solución de la ecuación diferencial L (y) = 0 en I. Recı́procamente, si y es una solución


de L (y) = 0 en I, entonces existen constantes c1 y c2 tales que:

y = c1 u 1 + c2 u 2

Demostración. En primer lugar, note que como L es una transformación lineal, se tiene
que:
C1 u1 + C2 u2 ∈ ker L

luego, C1 u1 + C2 u2 es solución de L (y) = 0.


Por otro lado, como u1 y u2 son linealmente independientes en I, bastará verificar que:

ker L = hu1 , u2 i

En efecto, sean f ∈ ker L y x0 ∈ I. Por el Teorema de Unicidad, es suficiente verificar que


existen constantes C1 y C2 tales que:

 C u (x ) + C u (x ) = f (x )
1 1 0 2 2 0 0
 C1 u0 (x0 ) + C2 u0 (x0 ) = f 0 (x0 )
1 2

es decir, que las funciones C1 u1 + C2 u2 y f coinciden en las condiciones iniciales. Entonces,


para que tales constantes existan, es suficiente a su vez que el determinante:


u1 (x0 ) u2 (x0 )
= u1 (x0 ) u02 (x0 ) − u2 (x0 ) u01 (x0 )
0 0
u1 (x0 ) u2 (x0 )

149
Apuntes Mat023 (versión preliminar actualizada 23-05-2014)

sea no nulo. Consideremos la función W : I ⊆ R → R definida por:




u1 (x) u2 (x)
W (x) =
= u1 (x) u02 (x) − u2 (x) u01 (x)
0 0
u1 (x) u2 (x)

Supongamos que W (x) = 0, para todo x ∈ I, entonces:

u1 (x) u02 (x) − u2 (x) u01 (x)


 
d u2
(x) =
dx u1 u21 (x)
W (x)
=
u21 (x)
= 0

luego, por el Teorema del Valor Medio el cociente u2 /u1 debe ser constante, pues I es un
intervalo. Esto contradice la hipótesis de que u1 y u2 son linealmente independientes sobre
I. Por tanto, debe existir al menos un x0 ∈ I tal que W (x0 ) 6= 0, y esto último implica
que existen constantes C1 y C2 tales que:

f (x) = C1 u1 (x) + C2 u2 (x)

en otras palabras, u1 y u2 forman una base para el núcleo del operador L, y ası́:

dim ker L = 2

Observación 2.3.3. Ası́, entonces, el operador L asociado a la ecuación diferencial lineal:

y (n) + pn−1 (x) y (n−1) + · · · + p0 (x) y = Q (x)

tiene núcleo ker L con dimensión finita dado por el orden de la ecuación diferencial. Es
decir:
dim ker L = n

vemos entonces que para determinar la solución general de una ecuación diferencial
lineal homogénea, necesitamos una familia de n funciones linealmente independientes si
la ecuación es de orden n. Antes de continuar, veamos algunos ejemplos de funciones
linealmente independientes:

150
Apuntes Mat023 (versión preliminar actualizada 23-05-2014)

Ejemplo 2.3.1. Sean r1 , r2 , . . . , rn ∈ R tales que ri =


6 rj , para todo i 6= j. Considere la
familia de n funciones definidas por:

ui (x) = eri x , ∀i = 1, 2, . . . , n

con x ∈ U ⊆ R. Note que si n = 2, entonces u1 y u2 son linealmente independientes, pues:

u1 (x)
= e(r1 −r2 )x
u2 (x)

no es constante. Supóngase que el resultado es cierto para k funciones exponenciales. Sea:


k+1
X
Ci eri x = 0, ∀x ∈ U (2.8)
i=1

Luego, multiplicando por e−rk+1 x y derivando la ecuación anterior respecto de x, obtenemos:


k
X
Ci (ri − rk+1 ) e(ri −rk+1 )x = 0
i=1

Los k números ri − rk+1 , con 1 ≤ i ≤ k son distintos. Utilizando la hipótesis de inducción,


las k exponenciales de la ecuación anterior son linealmente independientes en U . Por tanto,
se debe tener que:
Ci (ri − rk+1 ) = 0, ∀i = 1, 2, . . . , k

6 rk+1 , para cada i ≤ k, se tiene que Ci = 0 para i ≤ k. Utilizando esto último


pero ri =
junto con la ecuación (2.8) se concluye también que Ck+1 = 0.

Ejemplo 2.3.2. Sea r ∈ R. Las n funciones:

ui (x) = xi−1 erx , ∀i = 1, 2, . . . , n

son linealmente independientes en todo intervalo I ⊆ R.

Observación 2.3.4. Si el cociente entre dos funciones u y v es idénticamente una constante,


entonces u y v son linealmente dependientes. En efecto, suponga que:

u
≡ λ, λ constante
v

entonces u = λv. Es decir, u ∈ hvi, y por tanto, no puede ser linealmente independiente.

151
Apuntes Mat023 (versión preliminar actualizada 23-05-2014)

Ejemplo 2.3.3. Tres polinomios de primer grado cualesquiera son linealmente dependientes
en (−∞, +∞).

Ejemplo 2.3.4. Cuatro polinomios de segundo grado cualesquiera son linealmente depen-
dientes en (−∞, +∞).

Definición 2.3.2. Diremos que las funciones u1 , u2 , . . . , un forman un sistema funda-


mental de soluciones de la ecuación diferencial L (y) = 0, de orden n, si:

1. L (ui ) = 0, para todo i = 1, 2, . . . , n.

2. u1 , u2 , . . . , un son linealmente independientes (sobre I).

Observación 2.3.5. Note que un sistema fundamental de soluciones u1 , u2 , . . . , un es una


base para el núcleo de L. Es decir:

ker L = h{u1 , u2 , . . . , un }i ≤ C n (I)

Ejemplo 2.3.5. Considere la ecuación y 00 +y = 0. Verifique que las funciones u1 (x) = cos x
y u2 (x) = sin x, forman un sistema fundamental de soluciones de la ecuación diferencial.

Solución. Las funciones sin x y cos x son linealmente independientes y son soluciones de
la ecuación y 00 + y = 0, se sigue que ker (D2 + 1) = h{sin x, cos x}i.

El wronskiano

Observación 2.4.1. En el teorema anterior, se demostró que el operador diferencial L,


asociado a la ecuación diferencial lineal homogénea de orden 2 siguiente:

y 00 + p1 (x) y 0 + p0 (x) y = 0

tiene núcleo:
ker L = h{u1 , u2 }i

en el cual el determinante:

u1 (x0 ) u2 (x0 )

0
u1 (x0 ) u02 (x0 )


juega un papel importante en relación con la independencia lineal de las funciones, tenemos
la siguiente definición:

152
Apuntes Mat023 (versión preliminar actualizada 23-05-2014)

Definición 2.4.1. Llamaremos wronskiano de las funciones u1 (x) , u2 (x) , . . . , un (x) a:



u (x)
1 u2 (x) ··· un (x)

u0 (x) u02 (x) ··· u0n (x)
1
W (x) =

.. .. .. ..

. . . .

(n−1) (n−1) (n−1)
u1 (x) u2 (x) · · · un (x)
n×n

Se anota también W [u1 , u2 , . . . , un ] (x).

Ejemplo 2.4.1. Calcule el wronskiano de u1 (x) = cos x y u2 (x) = sin x.




cos x sin x
Solución. W (x) =
= cos2 x + sin2 x = 1
− sin x cos x

Teorema 2.4.1. Sean u1 (x) y u2 (x) dos soluciones de la ecuación diferencial:

y 00 + p1 (x) y 0 + p0 (x) y = 0 (2.9)

Entonces, el wronskiano:

u1 (x) u2 (x)
W (x) =
u01 (x) u02 (x)


satisface la ecuación diferencial:

W 0 + p1 (x) W = 0

Demostración. Note que:


0
W 0 = (u1 u02 − u01 u2 )
= u01 u02 + u1 u002 − u001 u2 − u01 u02
= u1 u002 − u001 u2

luego:

W 0 + p1 (x) W = u1 u002 − u001 u2 + p1 (x) {u1 u02 − u01 u2 }


= u1 {u002 + p1 (x) u02 } − u2 {u001 + p1 (x) u01 }
= u1 {u002 + p1 (x) u02 + p0 (x) u2 } − u2 {u001 + p1 (x) u01 + p0 (x) u1 }
= u1 · 0 − u2 · 0
= 0

Por tanto, W 0 + p1 (x) W = 0, como se querı́a demostrar.

153
Apuntes Mat023 (versión preliminar actualizada 23-05-2014)

Observación 2.4.2. En vista del teorema anterior, si u1 (x) y u2 (x) son dos soluciones
de la ecuación (2.9), la ecuación diferencial asociada al wronskiano:

W 0 + p1 (x) W = 0

es de variable separable, luego:


Z Z
dW
=− p1 (x) dx + C
W
Por tanto:
R
W (x) = W (x0 ) e− p1 (x)dx

para algún x0 ∈ I. Por tanto se tiene el siguiente teorema:

Teorema 2.4.2 (Fórmula de Abel para el wronskiano). Sea W = W (x) el wronskiano


asociado a dos soluciones linealmente independientes de la ecuación diferencial de segundo
orden:
y 00 + p1 (x) y 0 + p0 (x) y = 0

entonces:
R
W (x) = W (x0 ) e− p1 (x)dx

Teorema 2.4.3. Sean u1 , u2 , . . . , un soluciones de la ecuación diferencial:

y (n) + pn−1 (x) y (n−1) + · · · + p0 (x) y = 0

donde las funciones p1 , p2 , . . . , pn−1 son continuas en un intervalo I ⊆ R. Entonces,


6
u1 , u2 , . . . , un son linealmente independientes sobre I si y solamente si W [u1 , u2 , . . . , un ] (x) =
0 en I.
1
Ejemplo 2.4.2. Sean u1 (x) = x y u2 (x) = x
soluciones de la ecuación diferencial:
1 1
y 00 + y 0 − 2 y = 0
x x
definida para x > 0. Hallar la solución general.

Solución. Para hallar la solución general, es suficiente probar que u1 (x) y u2 (x) son
linealmente independientes. Por el teorema anterior:

  1
x x

1 = −2 6= 0
W x, =
x 1
1 − 2 x

154
Apuntes Mat023 (versión preliminar actualizada 23-05-2014)

Por tanto:
1
y (x) = C1 x + C2
x
Teorema 2.4.4 (Fórmula de Abel para una segunda solución). Sea u1 (x) una solución no
trivial de la ecuación diferencial:

y 00 + p1 (x) y 0 + p0 (x) y = 0

entonces, una segunda solución u2 (x), linealmente independiente con u1 (x), está dada por:
Z − R p1 (x)dx
e
u2 (x) = u1 (x) dx
u21 (x)
Demostración. Como:


u1 (x) u2 (x)
W (x) =
u01 (x) u02 (x)


= u1 u02 − u01 u2

y:
R
W (x) = C e− p1 (x)dx

obtenemos la ecuación diferencial lineal de primer orden en la variable u2 siguiente:


R
u1 u02 − u01 u2 = C e− p1 (x)dx

Dividiendo la ecuación anterior por u21 , se tiene que:


u1 u02 − u01 u2 C − R p1 (x)dx
= e
u21 u21
Por consiguiente:  
d u2 C R
= 2 e− p1 (x)dx
dx u1 u1
integrando y eligiendo valores adecuados para las constantes de integración, se obtiene
finalmente que: R
e− p1 (x)dx
Z
u2 (x) = u1 (x) dx
u21 (x)

Ejemplo 2.4.3. Hallar la solución general de la ecuación de Legendre, con α = 1:

1 − x2 y 00 − 2xy 0 + 2y = 0,

|x| < 1

155
Apuntes Mat023 (versión preliminar actualizada 23-05-2014)

Solución. Notamos que u1 (x) = x es una solución particular de la ecuación diferencial.


Luego, de:
2x 0 2
y 00 − y + y=0
1 − x2 1 − x2
y de la fórmula de Abel, se obtiene que:
2x
R
Z dx
e 1−x2
y2 (x) = x dx
x2
Z − ln(1−x2 )
e
= x dx
x2
Z
dx
= x
x (1 − x2 )
2
Z  
1 1 1
= x + + dx
x2 2 (1 − x) 2 (1 + x)

mediante un desarrollo en fracciones parciales del penúltimo integrando. Por tanto:



1 1 + x
−1
y2 (x) = x ln

2 1 − x

Ası́, la solución general de la ecuación diferencial es:


 
x 1 + x
y (x) = C1 x + C2 ln −1
2 1 − x

Ejemplo 2.4.4. Hallar la solución general de la ecuación diferencial:

xy 00 − 2 (x + 1) y 0 + (x + 2) y = 0

para x > 0, bajo el supuesto de que la ecuación homogénea tiene una solución de la forma
y = emx , con m ∈ R una constante adecuada.

Solución. Por hipótesis, la ecuación diferencial posee una solución de la forma y = emx .
Ası́, al reemplazar en:
xy 00 − 2 (x + 1) y 0 + (x + 2) y = 0

obtenemos la ecuación:

xm2 emx − 2m (x + 1) emx + (x + 2) emx = 0

156
Apuntes Mat023 (versión preliminar actualizada 23-05-2014)

para todo x > 0. Pero:

xm2 − 2m (x + 1) + (x + 2) = xm2 − 2m (x + 1) + (x + 1) + 1
= xm2 + (x + 1) (1 − 2m) + 1
= xm2 + (x + 1) (1 − 2m) + 1 − m2 + m2
= (x + 1) m2 + (x + 1) (1 − 2m) + 1 − m2
= (x + 1) m2 − 2m + 1 + 1 − m2
 

= (x + 1) (m − 1)2 + 1 − m2


entonces:
(x + 1) (m − 1)2 + 1 − m2 = 0


para todo x > 0. Por tanto, m = 1. Ası́, y1 (x) = ex es una solución de la ecuación. Para
hallar una segunda solución linealmente independiente con y1 (x) = ex , utilizaremos la
fórmula de Abel. Primeramente escribimos la ecuación en su forma normal:
   
00 1 0 2
y −2 1+ y + 1+ y=0
x x

entonces, si p1 (x) = −2 1 + x1 , tenemos que:




R
e− p1 (x)dx
Z
y2 (x) = y1 (x) dx
[y1 (x)]2
Z 2 R (1+ 1 )dx
e x
= ex dx
e2x
Z 2(x+ln x)
x e
= e dx
e2x
Z 2 2x
xe
= ex dx
e2x
Z
= ex x2 dx
1 3 x
= xe
3

por tanto, la solución general de la ecuación diferencial está dada por:

y (x) = C1 ex + C2 x3 ex

157
Apuntes Mat023 (versión preliminar actualizada 23-05-2014)

Ecuaciones diferenciales a coeficientes constantes

Definiciones

Definición 2.5.1. Una ecuación diferencial lineal a coeficientes constantes es una ecuación
de la forma:

y (n) + an−1 y (n−1) + an−2 y (n−2) + · · · + a1 y 0 + a0 y = Q (x)

en donde a0 , a1 , . . . , an−1 ∈ R y Q es una función continua en R. Las constantes ai ∈ R,


i = 0, 1, . . . , n − 1, se llaman coeficientes de la ecuación.

Observación 2.5.1. Comenzamos el estudio de este tipo de ecuaciones con el caso n = 2.

La ecuación de orden 2

Observación 2.5.2. Consideremos la ecuación diferencial de segundo orden siguiente:

y 00 + ay 0 + by = 0 (2.10)

Supongamos que la ecuación (2.10) posee una solución y = y (x) de la forma y (x) = eλx ,
con λ ∈ R. Note que:
y 0 (x) = λeλx ∧ y 00 (x) = λ2 eλx

Por tanto, reemplazando lo anterior en la ecuación (2.10) obtenemos:

λ2 eλx + aλeλx + beλx = 0

Ası́, en vista de la ecuación anterior, las condiciones sobre λ ∈ R de modo que y (x) = eλx
sea solución es que:
λ2 + aλ + b = 0 (2.11)

Esto es, que λ sea raı́z de la ecuación de segundo grado anterior. La ecuación (2.11) se
llama ecuación caracterı́stica asociada a la ecuación (2.10).
Por tanto, tenemos tres casos dados por ∆ = a2 − 4b, el discriminante de (2.11). En
efecto:

158
Apuntes Mat023 (versión preliminar actualizada 23-05-2014)

1. Si ∆ = a2 − 4b > 0, entonces la ecuación (2.11), tiene dos soluciones reales λ1 y λ2


distintas, dadas por:
√ √
−a + a2 − 4b −a − a2 − 4b
λ1 = ∧ λ2 =
2 2

las que a su vez definen las funciones:

u1 (x) = eλ1 x ∧ u2 (x) = eλ2 x

Luego, como u2 /u1 no es constante, la solución general de la ecuación diferencial


lineal homogénea de orden 2 es:

y (x) = C1 eλ1 x + C2 eλ2 x

Ejemplo 2.5.1. Resuelva la ecuación diferencial:

y 00 + y 0 − 2y = 0

Solución. En este caso la ecuación caracterı́stica es

λ2 + λ − 2 = 0

(λ + 2) (λ − 1) = 0

luego tenemos dos soluciones u1 (x) = e−2x y u1 (x) = ex de y 00 + y 0 − 2y = 0 las


cuales son linealmente independientes pues

−2x x
−2x x
e e
= 3e−x

W e ,e =
−2x x
−2e e

la solución general de la ecuación será entonces

y (x) = c1 ex + c2 e−2x

2. Si ∆ = a2 − 4b = 0, entonces la ecuación (2.11) tiene una raı́z real λ de multiplicidad


2, dada por:
a
λ=−
2

159
Apuntes Mat023 (versión preliminar actualizada 23-05-2014)

Luego, u1 (x) = eλx es una solución, pero dim ker L = 2 por tanto, debemos hallar
otra solución u2 (x) linealmente independiente con u1 (x), podemos usar la fórmula
de Abel para la segunda solución
R
e− adx
Z
− a2 x
u2 (x) = e dx
e−ax
a
= xe− 2 x
a
se sigue que la segunda solución es u2 (x) = xe− 2 x y ası́ la solución general es

y (x) = c1 eλx + c2 xeλx

donde λ es la raı́z repetida de la ecuación caracterı́stica.

Ejemplo 2.5.2. Resuelva la ecuación diferencial:

y 00 + 4y 0 + 4y = 0

Solución. La ecuación caracterı́stica es

λ2 + 4λ + 4 = 0

la cual tiene raı́z repetida λ = −2 se sigue que la solución general de la ecuación es

y (x) = c1 e−2x + c2 xe−2x

3. Si ∆ = a2 − 4b < 0, entonces la ecuación (2.11) no tiene raı́ces reales. Sin embargo,


sea:
λ = α + iβ

una raı́z compleja de (2.11). Recordemos el siguiente teorema:

Teorema 2.5.1 (Fórmula de Euler). Sea θ ∈ R, entonces:

eiθ = cos θ + i sin θ

Entonces:

eλx = e(α+iβ)x
= eαx eiβx
= eαx {cos (βx) + i sin (βx)}
= eαx cos (βx) + i {eαx sin (βx)}

Consideremos ahora:

160
Apuntes Mat023 (versión preliminar actualizada 23-05-2014)

Teorema 2.5.2. Sean L (y) = 0 una ecuación diferencial lineal de orden 2 e y (x) =
6 v. Entonces, u (x) y
u (x) + iv (x) una solución compleja de la ecuación tal que u =
v (x) son dos soluciones reales y linealmente independientes de L (y) = 0.

De:
eλx = eαx cos (βx) + i eαx sin (βx)

se obtiene que:

u1 (x) = eαx cos (βx) ∧ u2 (x) = eαx sin (βx)

son dos soluciones reales y linealmente independientes de la ecuación diferencial


lineal:
y 00 + ay 0 + by = 0

Observación 2.5.3. Debemos notar que eλx y eλ̄x son efectivamente funciones linealmente
independientes como C espacio vectorial.

Ejemplo 2.5.3. Resuelva la ecuación:

y 00 + 2y 0 + 5y = 0

Solución. La ecuación caracterı́stica es

λ2 + 2λ + 5 = 0

la cual tiene raı́ces


λ = −1 ± 2i

se sigue que la solución general es

y (x) = c1 e−x cos (2x) + c2 e−x sin (2x)

Ahora daremos una segunda mirada a la ecuación de segundo orden, en relación al


operador diferencial asociado. En general los operadores diferenciales no conmutan, esto es,
si L1 y L2 son dos operadores diferenciales en general no es cierto que

L1 L2 = L2 L1

161
Apuntes Mat023 (versión preliminar actualizada 23-05-2014)

sin embargo, si los operadores diferenciales son con coeficientes constantes, estos se com-
portan igual que polinomios. Por ejemplo, considere los operadores diferenciales D − 1 y
D + 2 entonces

(D − 1) (D + 2) y = (D − 1) (y 0 + 2y)
= y 00 + 2y 0 − y 0 − 2y
= y 00 + y 0 − 2y
D2 + D − 2 y

=

por otro lado

(D + 2) (D − 1) y = (D + 2) (y 0 − y)
= y 00 − y 0 + 2y 0 − 2y
= y 00 + y 0 − 2y
D2 + D − 2 y

=

se sigue
(D − 1) (D + 2) = (D + 2) (D − 1) = D2 + D − 2


note que si (D − 1) y = 0 entonces (D + 2) (D − 1) y = 0 y ası́ y ∈ ker (D2 + D − 2) en


otras palabras una solución de la ecuación

y0 − y = 0

también es solución de
y 00 + y 0 − 2y = 0

de manera similar, si (D + 2) y = 0 entonces (D − 1) (D + 2) y = 0 y ası́ y ∈ ker (D2 + D − 2)


en otras palabras una solución de la ecuación

y 0 + 2y = 0

también es solución de
y 00 + y 0 − 2y = 0

pero la soluciones y 0 − y = 0 son u1 (x) = cex y las soluciones de y 0 + 2y = 0 son


u2 (x) = ke−2x (son de primer orden de variables separadas) ası́ podemos obtener las

162
Apuntes Mat023 (versión preliminar actualizada 23-05-2014)

soluciones ex y e−2x de la ecuación y 00 + y 0 − 2y = 0 y como son linealmente independientes,


se sigue que la solución general de la ecuación es dada por

y (x) = c1 ex + c2 e−2x

son estas ideas las que utilizaremos para resolver la ecuación diferencial lineal de orden n.

La ecuación de orden superior

Observación 2.5.4. En general, respecto de la ecuación diferencial:

y (n) + an−1 y (n−1) + an−2 y (n−2) + · · · + a1 y 0 + a0 y = Q (x)

tenemos:

Definición 2.5.2. Sean I ⊆ R un intervalo abierto y L : C (n) (I) → C (I) un operador


diferencial de orden n a coeficientes constantes, es decir:

L = Dn + an−1 Dn−1 + · · · + a1 D + a0 1 (2.12)

El polinomio caracterı́stico de la ecuación diferencial L (y) = 0 es el polinomio en R [λ]


siguiente:
fL (λ) = λn + an−1 λn−1 + · · · a1 λ + a0

Diremos, además, que la ecuación caracterı́stica asociada a la ecuación diferencial


L (y) = 0 es la ecuación siguiente:
fL (λ) = 0

O bien:
λn + an−1 λn−1 + · · · a1 λ + a0 = 0

Ejemplo 2.5.4. La ecuación diferencial:

y (4) − 3y 00 + y 0 − 3y = 0

tiene polinomio caracterı́stico:

f (λ) = λ4 − 3λ2 + λ − 3

Note que, el operador diferencial asociado es:

L = D4 − 3D2 + D − 3

163
Apuntes Mat023 (versión preliminar actualizada 23-05-2014)

Ejemplo 2.5.5. La ecuación diferencial:

y (5) + 2y (4) − 3y 000 − y 00 − 2y 0 + 3y = 0

tiene polinomio caracterı́stico:

f (λ) = λ5 + 2λ4 − 3λ3 − λ2 − 2λ + 3

Note que:

D5 + 2D4 − 3D3 − D2 − 2D + 3 = D2 + D + 1 (D − 1)2 (D + 3)




Teorema 2.5.3. Sean L y S dos operadores a coeficientes constantes con polinomios


caracterı́sticos fL y fS , respectivamente, y sea α ∈ R, entonces:

1. L = S, si y solo si, fL = fS

2. fL+S = fL + fS

3. fLS = fL · fS

4. fαL = αfL

Observación 2.5.5. Del teorema anterior se deduce que toda relación algebraica consti-
tuı́da por sumas, productos y productos por escalares de polinomios caracterı́sticos son
también válidas para los operadores L y S. En particular, se tienen las leyes conmutativas:

L+S =S+L ∧ LS = SL ∧ α (βL) = β (αL)

Note, además, que si L y S son operadores tales que LS = SL, entonces:

ker L + ker S ⊆ ker (LS) (2.13)

Por otro lado, recordemos que en el conjunto R [x] los elementos irreducibles mónicos
son los polinomios de la forma x − c y x2 + αx + β, con α2 − 4β < 0. Luego, el Teorema
Fundamental del Álgebra implica el siguiente resultado:

Teorema 2.5.4. Sea p (x) ∈ R [x]. Entonces, p (x) se factoriza como


r s
Y ni
Y mj
p (x) = (x − λi ) · x2 + αj x + βj (2.14)
i=1 j=1

donde λi son las raı́ces reales de p (x) con multiplicidad ni , para cada i = 1, 2, . . . , r,
αj2 − 4βj < 0 y mj es la multiplicidad de la raı́z compleja asociada, para cada j = 1, 2, . . . , s.

164
Apuntes Mat023 (versión preliminar actualizada 23-05-2014)

Obtenemos lo siguiente para los operadores diferenciales dados por la factorización de


la ecuación caracterı́stica en (2.14):

1. ker (D − λ)m =


eλx , xeλx , x2 eλx , . . . , xm−1 eλx

2.
m
ker (D − α)2 + β 2 eαx cos (βx) , xeαx cos (βx) , . . . , xm−1 eαx cos (βx) ; eαx sin (βx) ,


=

, xeαx sin (βx) , . . . , xm−1 eαx sin (βx)


Por tanto, mediante el uso del wronskiano y la ecuación (2.14), la solución general de la
ecuación diferencial está dada por las combinaciones lineales (con las constantes indexadas
adecuadamente) de los sistemas fundamentales de soluciones de cada factor.

Ejemplo 2.5.6. Resuelva la ecuación diferencial:

y 000 − y 00 − 8y 0 + 12y = 0

Solución. Consideremos la ecuación caracterı́stica:

λ3 − λ2 − 8λ + 12 = 0

Como:
λ3 − λ2 − 8λ + 12 = (λ + 3) (λ − 2)2

se sigue que

ker (D − 2)2 ⊂ ker D3 − D2 − 8D + 12




ker (D + 3) ⊂ ker D3 − D2 − 8D + 12


pero ker (D − 2)2 = h{e2x , xe2x }i y ker (D + 3) = h{e−3x }i luego

e2x , xe2x , e−3x ⊂ ker D3 − D2 − 8D + 12


 

sabemos que el espacio ker (D3 − D2 − 8D + 12) tiene dimensión tres luego, la solución
general de la ecuación
y 000 − y 00 − 8y 0 + 12y = 0

es dada por
yG (x) = c1 e2x + c2 xe2x + c3 e−3x

165
Apuntes Mat023 (versión preliminar actualizada 23-05-2014)

Ejemplo 2.5.7. Suponga que L (y) = 0 es una ecuación diferencial con operador:

L = D5 + 2D4 − 3D3 − D2 − 2D + 3

Hallar la solución general.

Solución. Note que:

L = D5 + 2D4 − 3D3 − D2 − 2D + 3
= D2 + D + 1 (D − 1)2 (D + 3)


Ahora bien:
* √ ! √ !+
3x 3x
ker D2 + D + 1 = e−x/2 cos ; e−x/2 sin

2 2

ker (D − 1)2 = hex ; xex i

y además:
ker (D + 3) = e−3x

Por tanto, la solución general de la ecuación L (y) = 0 está dada por:


√ ! √ !
3x 3x
y (x) = C1 e−x/2 cos + C2 e−x/2 sin + C3 ex + C4 xex + C5 e−3x
2 2

Ejemplo 2.5.8. Hallar la solución general de una ecuación diferencial lineal a coeficientes
constantes cuya ecuación caracterı́stica es:

λ5 − 2λ4 + 6λ3 − 9λ2 + 8λ − 4 = 0


√ 
3
sabiendo que y = ex/2 cos 2
x es una solución de dicha ecuación.

Solución. Sea ε una ecuación diferencial lineal a coeficientes constantes homogénea de la


cual se sabe que:
λ5 − 2λ4 + 6λ3 − 9λ2 + 8λ − 4 = 0 (2.15)

es la ecuación caracterı́stica asociada, y que además:


√ !
3
y = ex/2 cos x
2

166
Apuntes Mat023 (versión preliminar actualizada 23-05-2014)

√ 
x/2 3
es solución de la ecuación. Ahora bien, como y = e cos 2
x es solución:

1 3
λ= + i
2 2
es una raı́z de la ecuación caracterı́stica (2.15), entonces:

1 3
λ= − i
2 2
también es raı́z, luego la expresión:
( √ !) ( √ !)
1 3 1 3
λ− + i λ− − i = λ2 − λ + 1
2 2 2 2

factoriza a la ecuación caracterı́stica. Ası́:

λ5 − 2λ4 + 6λ3 − 9λ2 + 8λ − 4 = λ3 − λ2 + 4λ − 4 λ2 − λ + 1


 

Notamos, además, que λ = 1 es raı́z de la ecuación λ3 − λ2 + 4λ − 4 = 0. Entonces, por el


teorema del factor, se tiene que:

λ3 − λ2 + 4λ − 4 = λ2 + 4 (λ − 1)


entonces
λ5 − 2λ4 + 6λ3 − 9λ2 + 8λ − 4 = λ2 − λ + 1 λ2 + 4 (λ − 1)
 

Por tanto, la ecuación (2.15) tiene las siguientes raı́ces:


( √ √ )
1 3 1 3
+ i, − i, 1, 2i, −2i
2 2 2 2

Finalmente, la solución general de la ecuación ε está dada por:


( √ ! √ !)
3 3
y (x) = C1 ex + ex/2 C2 cos x + C3 sin x + {C4 cos 2x + C5 sin 2x}
2 2

Método de variación de parámetros

Observación 2.6.1. En las secciones anteriores obtuvimos métodos para encontrar el


espacio solución de ecuaciones diferenciales, para los siguientes casos:

167
Apuntes Mat023 (versión preliminar actualizada 23-05-2014)

1. Es conocida una solución particular y1 (x) de la ecuación:

y 00 + p1 (x) y 0 + p0 (x) y = 0

En particular, para la ecuación anterior, obtuvimos que la solución general está dada
por:
y (x) = C1 y1 (x) + C2 y2 (x)

donde: R
e− p1 (x)dx
Z
y2 (x) = y1 (x) dx
y12 (x)
2. La ecuación diferencial es de la forma:

y (n) + an−1 y (n−1) + · · · + a1 y 0 + a0 y = 0

con ai ∈ R, para cada i = 0, 1, . . . , n − 1. En este caso, las soluciones dependen de


las raı́ces del polinomio caracterı́stico:

fL (λ) = λn + an−1 λn−1 + · · · + a1 λ + a0

Por otro lado, dada la estructura lineal de la ecuación L (y) = Q, donde L es el operador
diferencial asociado a la ecuación y Q es una función continua, entonces la solución general
de la ecuación lineal se descompone como:

y = yh + yp

donde yh es la solución general de la ecuación L (y) = 0. El objetivo de esta sección es


hallar una solución particular yp para la ecuación L (y) = Q.

Observación 2.6.2. En este curso, se ven dos de tales métodos: el método de variación de
parámetros y el método de los anuladores. Como se acostumbra, haremos los razonamientos
para n = 2, y luego se presentará el resultado para n ≥ 2. Veamos primeramente:

Teorema 2.6.1 (Variación de parámetros). Sean u1 y u2 dos soluciones linealmente


independientes de la ecuación:

L (y) = y 00 + p1 (x) y 0 + p0 (x) y = 0

168
Apuntes Mat023 (versión preliminar actualizada 23-05-2014)

en un intervalo I ⊆ R. Entonces, la ecuación no homogénea L (y) = Q tiene una solución


particular de la forma:
yp (x) = C1 (x) u1 (x) + C2 (x) u2 (x)

donde:
Z Z
u2 (x) Q (x) u1 (x) Q (x)
C1 (x) = − dx ∧ C2 (x) = dx (2.16)
W (x) W (x)

y W (x) es el wronskiano de u1 (x) y u2 (x).

Demostración. Supongamos que yp (x) = C1 (x) u1 (x) + C2 (x) u2 (x), luego:

yp0 = C1 u01 + C2 u02 + (C10 u1 + C20 u2 )


0
yp00 = C1 u001 + C2 u002 + (C10 u01 + C20 u02 ) + (C10 u1 + C20 u2 )

Ahora bien, al evaluar yp en L se obtiene que:

L (yp ) = yp00 + p1 (x) yp0 + p0 (x) yp


0
= (C10 u01 + C20 u02 ) + (C10 u1 + C20 u2 ) + p1 (x) (C10 u1 + C20 u2 )

pues todos los términos que contienen a C1 (x) y C2 (x) desaparecen dado que L (u1 ) =
L (u2 ) = 0. Como deseamos que L (yp ) = Q, entonces de la última ecuación imponemos
que:
C10 u1 + C20 u2 = 0 ∧ C10 u01 + C20 u02 = Q

Ası́, las funciones C1 (x) y C2 (x), deben satisfacer que:



 C 0 (x) u (x) + C 0 (x) u (x) = 0
1 1 2 2
 C 0 (x) u0 (x) + C 0 (x) u0 (x) = Q (x)
1 1 2 2

para cada x ∈ I. Sin embargo, note que el último sistema tiene solución si y solo si la matriz
de coeficientes de él tiene determinante no nulo. Es decir, si:


u1 (x) u2 (x)
6 0
=
0
u1 (x) u02 (x)

169
Apuntes Mat023 (versión preliminar actualizada 23-05-2014)

para cada x ∈ I. Pero el determinante anterior es el wronskiano W (x) de u1 (x) y u2 (x), y


6 0, para cada x ∈ I. Por lo tanto, el
como u1 , u2 son linealmente independientes W (x) =
sistema tiene única solución, y además, por la regla de Cramer, obtenemos finalmente:


0 u2 (x)

Q (x) u02 (x)

u (x) Q (x)
C10 (x) = =− 2
u1 (x) u2 (x)
W (x)

0
u1 (x) u02 (x)

y:

u1 (x) 0

0
u1 (x) Q (x)

u (x) Q (x)
0
C1 (x) = = 1
u1 (x) u2 (x)
W (x)

u01 (x) u02 (x)


Integrando, se obtiene el resultado deseado.

Observación 2.6.3. En base a los razonamientos en la demostración del teorema anterior,


tenemos:

Teorema 2.6.2. Sean u1 , u2 , . . . , un una familia de n soluciones linealmente independientes


de la ecuación lineal de orden n:

L (y) = y (n) + pn−1 (x) y (n−1) + · · · + p1 (x) y 0 + p0 (x) y = 0

definida en un intervalo abierto I ⊆ R. Entonces, la ecuación no homogénea L (y) = Q,


tiene una solución particular:
n
X
yp (x) = Ci (x) ui (x)
i=1

donde Ci (x), con i = 1, 2, . . . , n son funciones que se obtienen como soluciones del sistema
de ecuaciones:



 C10 u1 + C20 u2 + · · · + Cn0 un = 0

0 0 0 0 0 0
 C1 u1 + C2 u2 + · · · + Cn un



 = 0
C10 u001 + C20 u002 + · · · + Cn0 u00n = 0 (2.17)
.. ..


. .





 C 0 u(n−1) + C 0 u(n−1) + · · · + C 0 u(n−1)

= Q (x)
1 1 2 2 n n

170
Apuntes Mat023 (versión preliminar actualizada 23-05-2014)

Ejemplo 2.6.1. Resuelva la ecuación diferencial:

y 00 + y = tan x

Solución. Consideremos un intervalo en el que el segundo miembro sea continuo, sea éste
(−a, a), siendo 0 < a < π/2. La ecuación homogénea tiene solución general:

yh (x) = C1 cos x + C2 sin x

pues la ecuación caracterı́stica de dicha ecuación homogénea es:

λ2 + 1 = 0

En particular, notamos que:




cos x sin x
W (x) = = cos2 x + sin2 x = 1
− sin x cos x

Entonces, en vista de las ecuaciones en (2.16) las funciones C1 (x) y C2 (x) satisfacen:
Z
sin x tan x
C1 (x) = − dx
W (x)
Z
= − sin x tan xdx
= sin x − ln |sec x + tan x|

y además:
Z
cos x tan x
C2 (x) = dx
W (x)
Z
= cos x tan xdx
= − cos x

Por tanto, por el método de variación de parámetros, una solución particular yp viene dada
por:

yp = C1 (x) cos x + C2 (x) sin x


= sin x cos x − cos x ln |sec x + tan x| − sin x cos x
= − cos x ln |sec x + tan x|

171
Apuntes Mat023 (versión preliminar actualizada 23-05-2014)

y la solución general es:

y = C1 cos x + C2 sin x + yp
= C1 cos x + C2 sin x − cos x ln |sec x + tan x|

Solución. Hallar la solución general de la ecuación diferencial:

00 0 ex
y − 2y + y =
(1 − x)2
Solución. Considere la ecuación diferencial:
ex
y 00 − 2y 0 + y =
(1 − x)2
Notamos que la ecuación anterior es una ecuación diferencial lineal a coeficientes constantes
no homogénea. Ası́, si y = y (x) es la solución general, entonces:

y (x) = yh (x) + yp (x)

donde yh (x) es la solución de la ecuación homogénea asociada e yp (x) es una solución


particular. Consideremos, entonces, la ecuación homogénea asociada:

y 00 − 2y 0 + y = 0

Luego, la ecuación caracterı́stica:

λ2 − 2λ + 1 = 0

tiene raı́z λ = 1 con multiplicidad 2. Por tanto, la solución homogénea yh (x) está dada por:

yh (x) = C1 ex + C2 xex

Ahora bien, por el método de variación de parámetros, tenemos que la solución particular
yp (x) tiene la forma:
yp (x) = C1 (x) ex + C2 (x) xex

donde C1 (x) y C2 (x) son soluciones del sistema de ecuaciones:



 C10 (x) ex + C20 (x) xex = 0
 C10 (x) ex + C20 (x) (1 + x) ex = ex
2
(1−x)

172
Apuntes Mat023 (versión preliminar actualizada 23-05-2014)

Ahora bien, como el wronskiano:



x
xex

e
W (x) =
= e2x
ex (1 + x) ex


es no nulo, por la regla de Cramer, se tiene que:

0 xex



ex
(1−x)2 (1 + x) ex

0
x
C1 (x) = =−
W (x) (x − 1)2
y además:
x
e 0



x ex
e

(1−x)2
1
C20 (x) = =
W (x) (x − 1)2
Ası́: Z
x 1
C1 (x) = − 2 dx = {ln (x − 1) − x ln (x − 1) + 1}
(x − 1) x−1
y: Z
1 1
C2 (x) = 2 dx = −
(x − 1) x−1
Por tanto, la solución general y (x) de la ecuación diferencial está dada por:
ex xex
y (x) = C1 ex + C2 xex + {ln (x − 1) − x ln (x − 1) + 1} −
x−1 x−1
x x x
= C1 e + C2 xe − e {ln (x − 1) + 1}

Observación 2.6.4. Las integrales que aparecen en las fórmulas (2.16) o al despejar
C1 (x) y C2 (x) del sistema (2.17), son integrales indefinidas, luego las funciones quedan
determinadas salvo una constante de integración. Suponga entonces que tenemos:

vi (x) = Ci (x) + Ki

con Ki ∈ R, i = 1, 2, constantes de integración. Luego, si u1 y u2 son las soluciones


linealmente independientes de L (y) = 0, considere:

y1 (x) = A1 u1 (x) + A2 u2 (x) + v1 (x) u1 (x) + v2 (x) u2 (x)


= (A1 + K1 ) u1 (x) + (A2 + K2 ) u2 (x) + C1 (x) u1 (x) + C2 (x) u2 (x)

Entonces:
y1 (x) = α1 u1 (x) + α2 u2 (x) + C1 (x) u1 (x) + C2 (x) u2 (x)
que era la solución que ya tenı́amos.

173
Apuntes Mat023 (versión preliminar actualizada 23-05-2014)

Método del anulador

Definición 2.7.1. Diremos que el operador diferencial L es un anulador o aniquilador de


ϕ si se cumple L (ϕ) = 0.

Ejemplo 2.7.1. El operador diferencial L = (D − 1)2 es un anulador de ϕ (x) = 2ex +5xex


pues ker (D − 1)2 = h{ex , xex }i, es decir, toda combinación lineal de ex y xex está en el
núcleo de (D − 1)2 . Notar que (D − 1)2 no es un anulador de x2 pues

(D − 1)2 x2 = (D − 1) (D − 1) x2
 

= (D − 1) 2x − x2


= (2 − 2x) − 2x − x2


= x2 − 4x + 2
6≡ 0

Observación 2.7.1. Recordemos que ker L + ker L ⊆ ker (LS) para los operadores con
coeficientes constantes.
2 2
Ejemplo 2.7.2. (D2 + 1) es anulador de x sin x y D2 es anulador de x luego D2 (D2 + 1)
es un anulador de 5x sin x − 3x

Suponga que buscamos resolver la ecuación L (y) = Q donde L es un operador con


coeficientes constantes y LQ es un anulador de Q (con coeficientes constantes también)
entonces

L (y) = Q

LQ L (y) = LQ (Q) = 0

luego la solución de L (y) = Q se encuentra entre las soluciones de la ecuación homogénea


LQ L (y) = 0.

Ejemplo 2.7.3. Resolver la ecuación diferencial:

y 00 − 5y 0 + 6y = xex

174
Apuntes Mat023 (versión preliminar actualizada 23-05-2014)

Solución. Notamos que el operador asociado a la ecuación diferencial es:

L = D2 − 5D + 6
= (D − 2) (D − 3)

Ası́, la ecuación diferencial puede escribirse como L (y) = Q, con Q (x) = xex . Por otro lado,
la ecuación homogénea L (y) = 0 tiene soluciones linealmente independientes u1 (x) = e2x
y u2 (x) = e3x . Sabemos que la solución general de la ecuación es de la forma:

y (x) = C1 e2x + C2 e3x + yp (x)

donde yp es una solución de la ecuación no homogénea L (y) = xex . Notamos que:

xex ∈ ker (D − 1)2

por tanto, aplicamos el operador diferencial (D − 1)2 a la ecuación diferencial para obtener
:
(D − 1)2 (D − 2) (D − 3) (y) = (D − 1)2 (xex ) = 0

Ası́, hemos obtenido la ecuación diferencial homogénea:

S (y) = (D − 1)2 (D − 2) (D − 3) (y) = 0

por tanto, la solución general de S (y) = 0 es:

yS (x) = aex + bxex + ce2x + de3x

donde a, b, c y d son constantes reales. Ahora bien, se deben elegir los valores de a, b, c y d
de tal modo que:
L aex + bxex + ce2x + de3x = xex


sin embargo, ce2x + de3x ∈ ker L, luego bastará elegir a y b tales que:

L (aex + bxex ) = xex

ası́, la ecuación:

L (aex + bxex ) = (D − 2) (D − 3) (aex + bxex )


= (2a − 3b) ex + 2bxex
= xex

175
Apuntes Mat023 (versión preliminar actualizada 23-05-2014)

implica que a = 3/4 y b = 1/2. Finalmente:


3 1
yp (x) = ex + xex
4 2
y por tanto:
yp
y
z }|h { z3 }|1 {
y (x) = C1 e + C2 e + ex + xex
2x 3x
4 2
Observación 2.7.2. Recalcamos que el operador (D − 1)2 se eligió de tal modo que:

xex ∈ ker (D − 1)2

Luego, al aplicar dicho operador a la ecuación diferencial L (y) = xex , vemos que el
término de la derecha se anula bajo la aplicación de (D − 1)2 . Por este hecho, el método
anterior se llama el método del anulador. El método del anulador, funciona correctamente
para ecuaciones que son anulables, en el sentido de que son funciones que aparecen como
soluciones de ecuaciones diferenciales a coeficientes constantes. En particular, funciones
que aparecen como combinaciones lineales de funciones de la forma:

xm−1 eαx ∨ xm−1 eαx cos βx ∨ xm−1 eαx sin βx

donde m es un número natural y α y β son constantes reales. A continuación detallamos


algunas de las funciones anteriores con sus respectivos anuladores:
Función Anulador

y = xm−1 Dm
y = eαx D−α
y = xm−1 eαx (D − α)m
y = cos βx o bien y = sin βx D2 + β 2
m
y = xm−1 cos βx o bien y = xm−1 sin βx (D2 + β 2 )
y = eαx cos βx o bien y = eαx sin βx D2 − 2αD + (α2 + β 2 )
m
y = xm−1 eαx cos βx o bien y = xm−1 eαx sin βx {D2 − 2αD + (α2 + β 2 )}

Observación 2.7.3. Un resultado que utiliza las propiedades lineales del operador dife-
rencial asociado a una ecuación, es el principio de superposición, que facilita en algunos
casos el cálculo de la solución particular. Tenemos, entonces:

176
Apuntes Mat023 (versión preliminar actualizada 23-05-2014)

Teorema 2.7.1 (Principio de superposición). Sean L un operador diferencial no necesaria-


mente a coeficientes constantes y f1 , f2 funciones continuas. Suponga que yi es solución de
la ecuación diferencial L (y) = fi , con i = 1, 2. Entonces:

y = y1 + y2

es solución de la ecuación L (y) = f1 + f2 .

Este principio nos permite separar la busqueda de la solución particular.

Ejemplo 2.7.4. Resolver


(D − 1) (D − 2) y = ex + x

o en otras palabras
d2 y dy
2
− 3 + 2y = ex + x
dx dx

Solución. Sabemos que


(D − 1) (ex ) = 0 y D2 (x) = 0

se sigue que
(D − 1) D2 (ex + x) = 0

ası́

(D − 1) D2 (D − 1) (D − 2) y = 0
D2 (D − 1)2 (D − 2) = 0

luego
yp (x) = c1 + c2 x + c3 ex + c4 xex + c5 e2x

y como
(D − 1) (D − 2) yp (x) = ex + x

177
Apuntes Mat023 (versión preliminar actualizada 23-05-2014)

se sigue

(D − 1) (D − 2) c1 + c2 x + c3 ex + c4 xex + c5 e2x
 

= (D − 1) (D − 2) [c1 + c2 x] + (D − 1) (D − 2) [c4 xex ]


= D2 − 3D + 2 [c1 + c2 x] + (D − 1) (D − 1 − 1) [c4 xex ]


= 2c1 − 3c2 + 2c2 x − (D − 1) [c4 xex ]


= 2c1 − 3c2 + 2c2 x − (c4 ex + c4 xex − c4 xex )
= 2c1 − 3c2 + 2c2 x − c4 ex

de esto obtenemos que las constantes cumplen

2c1 − 3c2 = 0
2c2 = 1
−c4 = 1

se sigue
c4 = −1, c2 = 1/2, c1 = 3/4

ası́

yG (x) = (3/4) + (1/2) x − xex + c3 ex + c5 e2x


= [(3/4) + (1/2) x − xex ] + c3 ex + c5 e2x
 

= yp (x) + yh (x)

es la solución general.
Note que
d2
(D − 1) (D − 2) ((3/4) + (1/2) x) = ((3/4) + (1/2) x)
dx2
d
−3 ((3/4) + (1/2) x) + 2 ((3/4) + (1/2) x)
dx
= x
x d2 d
(D − 1) (D − 2) (−xe ) = 2
(−xex ) − 3 (−xex ) + 2 (−xex )
dx dx
= 3ex (x + 1) − ex (x + 2) − 2xex
= ex

luego vale el principio de superposición.

178
Apuntes Mat023 (versión preliminar actualizada 23-05-2014)

Veamos como podemos trabajar, en algunos casos, si no conocemos el anulador:

Ejemplo 2.7.5. Hallar la solución general de la ecuación diferencial:


2
y 00 − 3y 0 + 2y = xex+x

Solución. Notamos que:

L = D2 − 3D + 2
= (D − 1) (D − 2)

Buscamos, primeramente, una solución particular yp . Considere el cambio de variables:

u = (D − 2) y (2.18)

la ecuación queda:
2
(D − 1) u = xex+x

Mediante la fórmula de Leibnitz, obtenemos:


Z
−x 2 1 2
ue = xex dx + C = ex + C
2
Ahora bien, como buscamos yp , podemos elegir C = 0. Ası́:
1 2
u = ex+x
2
Sustituyendo en (2.18), se obtiene:
1 2
(D − 2) y = ex+x
2
Luego, nuevamente por la fórmula de Leibnitz:
Z
1 2x 2
yp = e ex −x dx
2
Finalmente, la solución general de la ecuación diferencial es:
Z
1 2x 2
x 2x
y = C1 e + C2 e + e ex −x dx
2
Finalmente, como una aplicación de los métodos de variación de parámetros y del
anulador resolveremos la ecuación de Euler.

179
Apuntes Mat023 (versión preliminar actualizada 23-05-2014)

Definición 2.7.2. Una ecuación de Euler es una ecuación de la forma:

(ax + b)n y (n) + An−1 (ax + b)n−1 y (n−1) + · · · + A1 (ax + b) y 0 + A0 y = Q (x) (2.19)

donde ax + b > 0 y A0 , A2 , . . . , An−1 son constantes cualesquiera.

Observación 2.7.4. El método de resolución de la ecuación de Euler consiste en considerar


el cambio de variables:
ax + b = et

Como se puede observar, el cambio de variable es un cambio en la variable independiente.


Ası́, por la regla de la cadena:
dy dy dt dy
= = ae−t
dx dt dx dt 
d2 y
    2 
d dy d −t dy dt 2 −2t d y dy
= = ae =a e −
dx2 dx dx dt dt dx dt2 dt
3
 2   2  3
d2 y
  
dy d dy d 2 −2t d y dy 3 −3t dy dy
= = ae − =a e −3 2 +2
dx3 dx dt dt dt2 dt dt3 dt dt
y ası́ sucesivamente. Reemplazando todas las derivadas de orden superior en la ecuación de
Euler en (2.19) se obtiene la siguiente ecuación a coeficientes constantes:

y (n) (t) + Bn−1 y (n−1) (t) + · · · + B1 y 0 (t) + B0 y (t) = Q̃ (t)

Ejemplo 2.7.6. Resuelva la ecuación diferencial:

x2 y 00 + xy 0 + y = 1

Solución. Utilizando el cambio de variables x = et , se tiene que:


d2 y
 2 
dy −t dy −2t d y dy
=e ∧ =e −
dx dt dx dt2 dt
al reemplazar en la ecuación original, obtenemos:
d2 y
+y =1
dt2
luego:
y = C1 cos t + C2 sin t + 1

donde yp = 1, se obtiene inmediatamente por medio del método del anulador. Por tanto:

y = C1 cos (ln x) + C2 sin (ln x) + 1

es la solución general de la ecuación diferencial.

180
Apuntes Mat023 (versión preliminar actualizada 23-05-2014)

Ejemplo 2.7.7. Resolver las ecuación de Euler

x2 y 00 + 2xy 0 − 2y = 0

Solución. Haciendo el cambio de variables x = et se sigue


 2 
2 00 d y dy
xy = −
dt2 dt
dy
xy 0 =
dt

entonces la ecuación se transforma en

D (D − 1) y + 2Dy − 2y = 0

donde y = y (et ) entonces

D2 + D − 2 y = 0 ⇔ (D + 2) (D − 1) y = 0


se sigue
y et = c1 e−2t + c2 et


se sigue
c1
y (x) = + c2 x
x2
En las ecuaciones diferenciales de orden superior (al igual que en las de primer orden)
podemos hacer los cambios de variables que consideremos convenientes, sin embargo,
cualquier cambio de variables no asegura el poder resolver la ecuación:

Ejemplo 2.7.8. Haciendo el cambio de variables x = t3 resolver la ecuación

d2 y √ 2/3 dy
9x4/3 + 2y = x2/3

2
+ 6 3
x − 9x
dx dx

Solución. Usando el cambio de variables

dy dy dt
=
dx dt dx
2
 2
dy d2 y dt dy d2 t
= +
dx2 dt2 dx dt dx2

181
Apuntes Mat023 (versión preliminar actualizada 23-05-2014)

pero
dt 1 −2/3
= x
dx 3
d2 t −2 −5/3
2
= x
dx 9
se sigue


dy dy 1 −2
= t
dx dt 3
2
d2 y d2 y 1 −2
  
dy −2 −5
= t + t
dx2 dt2 3 dt 9
reemplazando
2 !
d2 y
    
4 1 −2 dy −2 −5 2 dy 1 −2
+ 2y = t2

9t t + t + 6t − 9t t
dt2 3 dt 9 dt 3
esto es
d2 y dy −1 dy dy
+ 2t−1 − 3 + 2y = t2

2
+ −2t
dt dt dt dt
es decir,
d2 y dy
2
− 3 + 2y = t2
dt dt
esta ecuación es
(D − 2) (D − 1) y = t2

aplicamos anuladores
D3 (D − 2) (D − 1) y = 0

la solución es de la forma

y = αe2t + βet + C1 + C2 t + C3 t2

determinamos las constantes C1 , C2 y C3

D2 − 3D + 2 C1 + C2 t + C3 t2 = t2


2C3 − 3C2 − 6C3 t + 2C1 + 2C2 t + 2C3 t2 = t2

obtenemos el sistema

2C3 − 3C2 + 2C1 = 0


−6C3 + 2C2 = 0
2C3 = 1

182
Apuntes Mat023 (versión preliminar actualizada 23-05-2014)

que tiene solución C1 = 74 , C2 = 32 , C3 = 1


2
ası́

7 3 1
y = αe2t + βet + + t + t2
4 2 2

volvemos a la variable original y la solución es



3 √
3 7 3√ 1
y (x) = αe2 x
+ βe x
+ + 3 x + x2/3
4 2 2

con α, β ∈ R.

Ejemplo 2.7.9. Use z = sin (x) para resolver la ecuación

y 00 + tan (x) y 0 + cos2 (x) y = sin (x) cos2 (x)

Solución. Las derivadas son con respecto a x la idea es cambiar las derivadas ahora con
respecto a z, para ello usamos la regla de la cadena: Notemos que

dy dy dx
=
dz dx dz

o bien
dy dz dy
=
dz dx dx
reemplazando
dy

dy dy
= dx = sec x
dz (cos x) dx
luego
 
d dy d
= ((sec x) y 0 )
dz dz dz
    2 
dx dy d y dx
= sec x tan x + sec x
dz dx dx2 dz
1 d2 y
 
sin x 1 dy
= +
cos2 x cos x dx cos2 x dx2

se sigue
d2 y dy d2 y
 
2
cos x = tan x +
dz 2 dx dx2
reemplazando en la ecuación original
 d2 y
 
2 2

cos x + cos x y=0
dz 2

183
Apuntes Mat023 (versión preliminar actualizada 23-05-2014)

luego
d2 y
+y =0
dz 2
se sigue
y (z) = c1 sin z + c2 cos z

es decir la solución general de la es

y (x) = c1 sin (sin x) + c2 cos (sin x) para c1 , c2 ∈ R

Movimiento vibratorio

Observación 2.8.1. Consideremos un sistema mecánico compuesto por un resorte de


longitud inicial l0 con su extremo superior sujeto firmemente y una masa m atada en su
extremo inferior. Nos interesa hallar la posición x (t) de la masa en cualquier instante
t. Más precisamente, nos interesa hallar la ecuación que describa el movimiento de la
partı́cula a la cual se le ha dado un desplazamiento x0 y una velocidad inicial v0 . Para ello
consideramos los siguientes supuestos:

1. La masa se mueve a lo largo de la vertical que pasa por el centro de gravedad de la


masa y en el sistema de referencia x = 0 indica la posición de reposo de la masa en
el resorte.

2. En cualquier tiempo t la magnitud de la fuerza ejercida sobre la masa es proporcional


a la diferencia entre la longitud l del resorte y su posición de equilibrio l0 . La constante
positiva de proporcionalidad k se llama constante del resorte, y el principio anterior
se conoce como la ley de Hooke.

Recordemos que la segunda ley del movimiento de Newton establece que la sumatoria
de fuerzas que actúa sobre la partı́cula es igual a la variación instantánea del momemtum
mv respecto del tiempo, pero como la masa m es constante, se tiene:
X
Fi = m a
i

d2 x(t)
donde a (t) = dt2
.

184
Apuntes Mat023 (versión preliminar actualizada 23-05-2014)

Suponemos, por ahora, que la única fuerza que actúa sobre la partı́cula es la fuerza
restauradora inducida por el resorte, es decir, no se consideran las fuerzas de fricción, por
ejemplo. Por la ley de Hooke, tenemos que:
d2 x
m = −kx
dt2
y el signo negativo es debido a que la fuerza restauradora del resorte se opone al movimiento.
Asumiendo, que la posición inicial (es decir, en t = 0) de la masa es x0 y que su velocidad
inicial es v0 , obtenemos el siguiente problema de valor inicial:
k
x00 + x = 0, x (0) = x0 , x0 (0) = v0 (2.20)
m
La ecuación caracterı́stica de la ecuación diferencial anterior es:
k
λ2 + =0
m
Ası́:
λ = ± ω0
p
en donde ω0 = k/m, luego:

x (t) = C1 cos ω0 t + C2 sin ω0 t

Usando las condiciones iniciales, obtenemos que C1 = x0 y C2 = v0 /ω0 . Por tanto:

x (t) = x0 cos ω0 t + (v0 /ω0 ) sin ω0 t


= A sin (ω0 t + φ)
q
donde A = x20 + (v0 /ω0 )2 y φ es tal que tan φ = x0 ω0 /v0 .

Definición 2.8.1. El movimiento de la masa m bajo las condiciones anteriores, se denomina


movimiento armónico simple. La frecuencia natural f del movimiento es el número
de oscilaciones completas por unidad de tiempo:
ω0
f=

Ejemplo 2.8.1. Considere un resorte, sujeto en su extremo superior, que sostiene una
pesa de 10 libras en su extremo inferior. Suponga que la pesa estira el resorte 6 pulgadas.

185
Apuntes Mat023 (versión preliminar actualizada 23-05-2014)

Halle la ecuación del movimiento de la pesa si ésta se lleva a una posición de 4 pulgadas
por debajo de su punto de equilibrio y se suelta. Calcule la ecuación del movimiento su
amplitud y la frecuencia natural.
h i
pie
Ayuda: Considere que 1 [pie] = 12 [pulgadas] y que g = 32 seg2
.

Observación 2.8.2. En los supuestos que dan origen al movimiento armónico simple
no se consideran las fuerzas, que en una situación más realista, actúan sobre la masa.
Considerando por ejemplo resistencia del medio, ya sea por acción del aire o realización
de la oscilación en un medio viscoso, es que se tienen las vibraciones amortiguadas. El
supuesto es simple, la fuerza amortiguadora del movimiento se presupone proporcional a
la velocidad del movimiento, ası́ el problema de valor inicial de la ecuación (2.20) queda
como:
k c
x00 = − x − x0 (2.21)
m m
con las condiciones iniciales x (0) = x0 y x0 (0) = v0 . La ecuación caracterı́stica de la
ecuación (2.21) es:
c k
λ2 + λ+ =0
m m
cuya solución es: √
−c ± c2 − 4km
λ=
2m
Por tanto, el comportamiento del movimiento depende del valor que tome el discriminante:

∆ = c2 − 4km

Tenemos, por tanto, tres casos:

Caso 2.8.1 (c2 − 4km > 0). La solución de la ecuación (2.21) en este caso está dada por:
 √   √ 
−c + c2 − 4km −c − c2 − 4km
x (t) = C1 exp + C2 exp
2m 2m

Note que 4km < c2 , lo que implica que c2 − 4km < c, y entonces:
√ √
−c + c2 − 4km < 0 ∧ −c − c2 − 4km < 0

Ası́, en este caso tenemos que:


lı́m x (t) = 0
t→∞

186
Apuntes Mat023 (versión preliminar actualizada 23-05-2014)

Caso 2.8.2 (c2 − 4km = 0). La solución de la ecuación (2.21) es:

x (t) = (C1 + C2 t) e(−c/2m)t

y al igual que en el caso anterior:


lı́m x (t) = 0
t→∞

Observación 2.8.3. Entonces, se observa por la forma de las soluciones que si:

c2 − 4km ≥ 0

no se producen oscilaciones en el sistema mecánico masa-resorte considerando amortiguación.


Este tipo de movimiento tı́picamente ocurre en un medio viscoso como agua o aceite.

Caso 2.8.3 (c2 − 4km < 0). Como (−c/2m) 6= 0, obtenemos:


 c  √ √ 
4km − c2 4km − c2
x (t) = exp − C1 cos t + C2 sin t
2m 2m 2m

Observación 2.8.4. Finalmente, podemos suponer una fuerza g = g (t) aplicada sobre
extremo superior del resorte (que inicialmente se supuso sujeto firmemente, esto es con
g ≡ 0) en la dirección del movimiento de la masa m. El movimiento de la masa obtenido bajo
este supuesto se conoce como vibración forzada. Una vibración forzada también puede
contener fuerzas amortiguadoras. Este es el caso más general del movimiento vibratorio, el
problema de valor inicial queda en su forma más general como:

c 0 k g (t)
x00 + x + x= (2.22)
m m m

con x (0) = x0 e x0 (0) = v0 .

Ejemplo 2.8.2. Considere un resorte que en su extremo superior está sometido a una
fuerza externa:
10
f (t) = sin t
32
y sostiene, en su parte inferior una masa de 10 libras. Suponga que la masa estira el resorte
6 pulgadas. Hallar la ecuación del movimiento de la masa si ésta se lleva a una posición de
4 pulgadas por debajo del punto de equilibrio y se suelta.

187
Apuntes Mat023 (versión preliminar actualizada 23-05-2014)

Observación 2.8.5. Un caso muy importante es cuando g es una función periódica, es


decir, supongamos que:
g (t) = F0 sin ωt

Ası́, la ecuación (2.22) queda:


c 0 k F0
x00 + x + x= sin ωt (2.23)
m m m
Sabemos que el anulador de g (t) = F0 sin ωt es el operador:

D2 + ω 2

Aplicando este último operador al operador D2 + mc D + m


k
, obtenemos:
 
2 2 2 c k  F0
· x = D2 + ω 2 ·

D +ω D + D+ sin ωt = 0
m m m
Por tanto, la ecuación (2.23) tiene una solución particular de la forma:

xp = b1 cos ωt + b2 sin ωt (2.24)

donde b1 y b2 son soluciones del sistema:



 (ω 2 − ω 2 ) b + cω b = 0
0 1 m 2
 − cω b1 + (ω 2 − ω 2 ) b2 = F0
m 0 m
q
k
donde ω0 = m
. Ası́:

F0 cω
b1 = − 2
m2 (ω02 − ω 2 ) + (cω)2
F0 m (ω02 − ω 2 )
b2 = 2
m2 (ω02 − ω 2 ) + (cω)2
Ası́, a partir de lo anterior y mediante el procedimiento de transformación a sinusoide de
la ecuación (2.24) encontramos que:

xp = A sin (ωt + φ)

donde la amplitud A está dada por:


F0 /k
A = s
 2 2  2
1 − ωω0 + 2 cc0 ωω0

188
Apuntes Mat023 (versión preliminar actualizada 23-05-2014)

y el ángulo de fase φ satisface:


2 cc0
tan φ =  2
ω
ω0
−1
con c0 = 2mω0 . Notamos que de la ecuación que define la amplitud los coeficientes
importantes son:

1. c/c0 llamado el cociente de amortiguación, y

2. ω/ω0 llamado el cociente de frecuencia del movimiento.

Se observa que el comportamiento de la solución particular xp depende entonces del


cociente de amortiguación si éste es cero o despreciable, entonces tenemos el movimiento
armónico simple:
x00 + ω02 x = 0

y por tanto, hay dos casos para el cociente de frecuencia:

Caso 2.8.4 (ω 2 6= ω02 ). En este caso se tiene:

F0 /k
x (t) = C1 cos ω0 t + C2 sin ω0 t + sin ωt
1 − (ω/ω0 )2
donde:
v0 (F0 /k) (ω/ω0 )
C 1 = x0 ∧ C2 = −
ω0 1 − (ω/ω0 )2
Caso 2.8.5 (ω 2 = ω02 ). En este caso debemos buscar una solución particular de la forma:

xp = b1 t cos ωt + b2 t sin ωt

pues la ecuación (2.24) se anula con el operador D2 + ω 2 , luego el anulador a considerar es


2
(D2 + ω 2 ) . Ası́, luego de los cálculo habituales del método del anulador se obtiene:
F0
b1 = − ∧ b2 = 0
2mω
Por tanto, la solución general tiene la forma:
F0
x (t) = C1 cos ωt + C2 sin ωt − t cos ωt
2mω
Observación 2.8.6. Observe en el caso anterior que al aumenta t, las vibraciones causadas
por el último término de la ecuación anterior aumentarı́an sin cota. En este caso, se dice
que la fuerza externa está en resonancia con la masa en vibración.

189
Apuntes Mat023 (versión preliminar actualizada 23-05-2014)

Ejercicios del capı́tulo

1. En los problemas siguientes se muestra que el principio de superposición generalmente


no se cumple para ecuaciones no lineales

a) Muestre que y = 1/x es solución de la ecuación y 0 + y 2 = 0 pero si c 6= 0 y c 6= 1


entonces y = c (1/x) no es solución.

b) Muestre que φ1 (x) = 1 e φ2 (x) = x son soluciones de yy 00 + (y 0 )2 = 0 pero la
suma no es solución.

2. Busque una solución polinomial de la ecuación de Legendre

1 − x2 y 00 − 2xy 0 + 2y = 0


y usando la fórmula de Abel determine la solución general de esta ecuación.

3. Verificar que φ1 (x) = x−1/2 cos x es solución (para x > 0) de la ecuación de Bessel
1
de orden 2  
2 00 20 1
x y + xy + x − y=0
4
y obtener su solución general.

4. Muestre que y1 (x) = x + 1 es solución de la ecuación

1 − 2x − x2 y 00 + 2 (1 + x) y 0 − 2y = 0


y resolverla completamente (encontrar solución general).

5. Considere la ecuación diferencial:

y 00 + P1 (x) y 0 + P2 (x) y = Q (x) (2.25)

y sean u1 (x) , u2 (x) soluciones linealmente independientes de la ecuación homogénea


asociada. Pruebe que la función:

f (x) = C1 (x) u1 (x) + C2 (x) u2 (x)

es una solución particular de la ecuación (2.25), si se satisface el sistema de ecuaciones:



 C 0 (x) u (x) + C 0 (x) u (x) = 0
1 1 2 2
 C 0 (x) u0 (x) + C 0 (x) u0 (x) = Q (x)
1 1 2 2

190
Apuntes Mat023 (versión preliminar actualizada 23-05-2014)

6. Compruebe que: Z x
1
y= f (t) sin ω (x − t) dt
ω 0

es una solución particular de y 00 + ω 2 y = f (x).

7. Suponga que ex y e−x son soluciones de una ecuación diferencial lineal ordinaria
homogénea, muestre que las funciones sinh x y cosh x también son soluciones de esa
ecuación.

8. Muestre que la solución particular de una EDO lineal de segundo orden no homogénea

y 00 + a1 (x) y 0 + a0 (x) y = f (x)

se puede escribir en la forma


Z x
yp (x) = G (x, t) f (t) dt
x0

donde G es una función adecuada.

9. Hallar la solución general de


d4 y d2 y
1) y 000 + 3y 00 − y 0 − 3y = 0 2) y (4) + 16y = 0 3) dx4
+ dx2
+y =0
4) y 00 − 36y = 0 5) y 00 + 2y 0 − 3y = 0 00 0
6) y + 8y + 16y = 0
7) y 0000 + 4y 00 + 3y = 0 8) y 00 − 4y 0 + 5y = 0 9) y 000 − 5y 00 + 3y 0 + 9y = 0

10. Resuelva la ecuación diferencial sujeta a las condiciones indicadas.

0
a) y 00 + 16y = 0 ; y(0) = 2, y (0) = −2

b) y 00 + 6y 0 + 25y = 0 ; y(0) = 2, y 0 (0) = 6

c) y 00 − 4y = 0 ; y(0) = 4, y 0 (0) = 16

d ) y 000 + 2y 00 − 5y 0 − 6y = 0 ; y(1) = 0, y 0 (1) = 0, y 00 (1) = 1

11. Resuelva las siguientes ecuaciones por anuladores.

a) y 00 + y 0 = 2t + sin t

b) y 00 + 4y 0 + 2y = te−2t

191
Apuntes Mat023 (versión preliminar actualizada 23-05-2014)


c) y 00 − 4y 0 + 5y = 4 + sin( 7t)

d ) y 00 + 3y = x2 e3x

e) y 00 − 2y 0 + 2y = e2x (cos x − 3 sin x)

f ) y 000 − y 00 − 4y 0 + 4y = 5 − ex + e2x

g) y 000 − 2y 00 + 2y 0 − y = ex + cos x

12. Resuelva la ecuación diferencial sujeta a las condiciones indicadas.

a) 2y 00 + 3y 0 − 2y = 14x2 − 4x − 11 , y(0) = 0, y 0 (0) = 0

b) y 00 + 4y 0 + 4y = (3 + x)e−2x , y(0) = 2, y 0 (0) = 5

c) y 00 − 2y 0 + 2y = 2x − 2 , y(π) = 0, y 0 (π) = 0
d2 x
d) + ω 2 x = F0 cos γt , x(0) = 0, x0 (0) = 1
dt2
d2 x
e) + ω 2 x = F0 sin ωt , x(0) = 0, x0 (0) = 0
dt2

13. Hallar una EDO lineal con coeficientes constantes que tenga por solución general:
1) y (x) = c1 xex + c2 ex + x + 1
√  √  √  √ 
2) y (x) = c1 e2x cos 3x + c2 e2x sin 3x + c3 xe2x cos 3x + c4 xe2x sin 3x + ex

14. Encontrar un anulador de


π

1) f (x) = x cos 2x + 3
2
2) f (x) = (x2 + 1) ex + x sinh (x)

2
3) f (x) = (x2 − 1) (1 + sin x) e 3x

15. Encontrar la solución general de


1) x2 y 00 + xy 0 + 9y = 0
2) x2 y 00 + xy 0 − p2 y = 0 con p ∈ R

16. Encontrar la solución general de

192
Apuntes Mat023 (versión preliminar actualizada 23-05-2014)

00 0
1) x2 y 00 + xy 0 − 9y = x2 + 1 2) x2 y + xy − 9y = x3 + 1
00 0
3) x2 y 00 + 4xy 0 + 2y = 2 ln x 4) x2 y + xy + 9y = sin(ln x3 )
5) x3 y 000 + 4x2 y 00 + xy 0 − y = x 6) x2 y 00 − xy 0 + 2y = 16
7) y 00 + x1 y 0 − 1
x2
y = 1
x2 +x3
8) y 00 − x2 y 0 + 2
x2
y = ln x
x

9) x4 y 00 + x3 y 0 − 4x2 y = 1 10) x3 y 000 + 6x y + 4xy − 4y = x


2 00 0

17. Encontrar una solución general de cada una de las ecuaciones diferenciales, usando el
método de variación de parámetros.

a) y 000 − y 00 + y 0 − y = 4xex

b) y 000 − y 0 = sin x

c) y 000 − 2y 00 = 4(x + 1)

d ) y 000 − 3y 00 − y 0 + 3y = 1 + ex

e) y 000 − 7y 0 + 6y = 2 sin x

f ) 3y 00 − 6y 0 + 30y = ex tan 3x
ex
g) y 00 − 2y 0 + y = 4x2 − 3 +
x
000 0 −x
h) y − 2y − 4y = 2e sec x (puede dejar integrales finales sin calcular)

18. Una solución y = u(x) de la ecuación diferencial y 00 − 3y 0 − 4y = 0 intersecta a una


solución de la ecuación diferencial y 00 + 4y 0 − 5y = 0 en el origen. Determinar las
funciones u y v si las dos soluciones tienen la misma pendiente en el origen y si
(v(x))4 5
lı́m = .
x→∞ u(x) 6

19. Sean y1 = u(x) y y2 = xu(x) dos soluciones de la ecuación y 00 + 4xy 0 + p(x)y = 0. Si


u(0) = 1, determine u(x) y p(x) explı́citamente.

20. Se sabe que para x > 0 la ecuación homogénea asociada a


√ 
4xy 00 + 2 − 8 x y 0 − 5y = 1

tiene una solución



y1 (x) = e− x

determine la solución general.

193
Apuntes Mat023 (versión preliminar actualizada 23-05-2014)

21. Determine la solución general de la ecuación diferencial


d2 y dy √
(2x − 1)2 2
+ 4 (2x − 1) − 8y = 2x − 1
dx dx
Ind.: Considere el cambio de variables u = 2x − 1.

22. Dada la ecuación diferencial


d2 y dy
(x − a) 2
−x + a2 y = a (x − 1)2 ex
dx dx
determine el valor del parámetro de forma que ex sea una solución de la homogénea
asociada y determine la solución general de la ecuación no homogénea para tales
valores del parámetro.

23. Use z = sin (x) para resolver la ecuación

y 00 + tan (x) y 0 + cos2 (x) y = sin (x) cos2 (x)

24. Considere la ecuación diferencial

x2 y 00 − 2xy 0 + 2y = 6

con condiciones iniciales y (0) = 3, y 0 (0) = 1 tiene soluciones yc (x) = cx2 + x + 3


para todo c ∈ R, este problema no tiene solución única, ¿contradice esto el teorema
de existencia y unicidad.?

25. Resolver
(xD + 1) (D − x) (xD) y = x
d
donde D = dx
.

1
26. Un cuerpo con masa m = 2
kilogramo (kg) está unido en el extremo de un resorte
estirado 2 metros (m) debido a una fuerza de 100 newtons (N) y es puesto en
movimiento a partir de la posición inicial x0 = 1 (m) y velocidad inicial v0 = 5 (m/s).
(Nótese que estas condiciones iniciales indican que el cuerpo se desplaza a la derecha
y a la izquierda en el tiempo t = 0). Encuéntrese la función de la posición del cuerpo,
ası́ como su amplitud, frecuencia, periodo de oscilación y el tiempo de retardo de su
movimiento.

194
Apuntes Mat023 (versión preliminar actualizada 23-05-2014)

27. Determine el periodo y la frecuencia del movimiento armónico simple de una masa
de 4 kg unida al extremo de un resorte con constante de 16 N/m.

28. Establezca el periodo y la frecuencia del movimiento armónico simple de un cuerpo


con una masa de 0.75 kg unida al extremo de un resorte con constante de 48 N/m.

29. Un cuerpo con masa de 250 g está unido al extremo de un resorte estirado 25 cm por
una fuerza de 9 N. En el tiempo t0 el cuerpo es movido 1 m a la derecha, estirando el
resorte y aplicando un movimiento con una velocidad inicial de 5 m/s a la izquierda.
(a) Encuentre x(t). (b) Obtenga la amplitud y el periodo de movimiento del cuerpo.

30. Asuma que la ecuación diferencial de un péndulo simple de longitud L es Lθ00 +gθ = 0,
donde g = GM/R2 es la aceleración gravitacional en el lugar donde éste se encuentra
(a una distancia R del centro de la Tierra; M significa la masa de la Tierra). Dos
péndulos de longitudes L1 y L2 —ubicados a una distancia R1 y R2 respecto del centro
de la Tierra— tienen periodos p1 y p2 . Muestre que

p1 R1 L1
= √
p2 R2 L2

195
Capı́tulo 3 : Sistemas de ecuaciones diferenciales

Definiciones

Considere los dos tanques que se ilustran en la figura. Suponga que el tanque A contiene
50 galones de agua en los que hay disueltas 25 libras de sal. Suponga que el tanque B
contiene 50 galones de agua pura. A los tanques entra y sale lı́quido como se indica en
la figura; se supone que tanto la mezcla intercambiada entre los tanques como el lı́quido
bombeado hacia fuera del tanque B están bien mezclados. Se desea construir un modelo
matemático que describa la cantidad de libras x1 (t) y x2 (t) de sal en los tanques A y B
respectivamente en el tiempo t.

se tiene
            
dx1 gal lb gal x2 lb gal x1 lb
= 3 ·0 +1 · −4 ·
dt min gal min 50 gal min 50 gal
2 x2
= − x1 +
25 50

dx2 x1 x2 x2
= 4 −3 −1
dt 50 50 50
2 2
= x1 − x2
25 25

196
Apuntes Mat023 (versión preliminar actualizada 23-05-2014)

ası́ obtenemos el sistema

dx1 2 1
= − x1 + x2
dt 25 50
dx2 2 2
= x1 − x2
dt 25 25

y las condiciones x1 (0) = 25 y x2 (t) = 0


Un sistema de ecuaciones de la forma

dX
= A (t) X (t) + B (t)
dt

esto es
      
x01 (t) a11 (t) a12 (t) · · · a1n (t) x1 (t) b1 (t)
      
 x0 (t)   a (t) a (t) · · · a (t)   x (t)   b (t) 
 2   21 22 2n  2   2
= + .

 . . . . ..  .
 .. .. .. ..   ..   ..


 
  .   


x0n (t) an1 (t) an2 (t) · · · ann (t) xn (t) bn (t)

donde X (t) = (x1 (t) , x2 (t) , . . . , xn (t))T , A (t) = (aij (t)) es una matriz de orden n × n
y B (t) = (b1 (t) , b2 (t) , . . . , bn (t))T es llamado sistema de ecuaciones diferenciales
lineal de primer orden (se supone que las funciones aij (t) y bi (t) son continuas en un
intervalo abierto I). Si adicionalmente se debe cumplir X (t0 ) = X0 para t0 ∈ I, X0 ∈ Rn
entonces tenemos el P.V.I

dX
= A (t) X (t) + B (t)
dt
X (t0 ) = X0

Ejemplo 3.1.1. Son sistemas de ecuaciones lineales:

dx
dt
= x + 2y + et
1.
dy
dt
= −x + 2y + cos t

dx
dt
= et x + e−t y + t
2.
dy
dt
= −x + (sin t) y

197
Apuntes Mat023 (versión preliminar actualizada 23-05-2014)

dx
dt
= x+y+z

dy
3. dt
= −x + 2y + 3z

dz
dt
= 2x + 3y + z

mientras que el sistema

dx
= x2 + e t y 2
dt
dy p
= cos t + x2 + y 2
dt
no es lineal.

Teorema 3.1.1. Si las funciones aij (t) y bi (t) son continuas en el abierto I ⊆ R, t0 ∈ I,
X0 ∈ Rn entonces el problema de valores iniciales

dX
= A (t) X (t) + B (t)
dt
X (t0 ) = X0

tiene solución única.

Ejemplo 3.1.2. Resolver el sistema

dx1
= x2
dt
dx2
= −x1
dt
con x1 (0) = 1, x2 (0) = 0.

Solución. Note que derivando la primera ecuación respecto a t obtenemos

d 2 x1 dx2
2
= = −x1
dt dt
de donde x1 debe cumplir
d 2 x1
+ x1 = 0
dt2
198
Apuntes Mat023 (versión preliminar actualizada 23-05-2014)

ası́ x1 (t) = c1 cos t + c2 sin t para algunas constantes c1 y c2 ahora bien, x1 (0) = 1 = c1 se
sigue
x1 (t) = cos t + c2 sin t

además
dx1
x2 (t) = = − sin t + c2 cos t
dt
luego
0 = x2 (0) = c2

se sigue

x1 (t) = cos t
x2 (t) = − sin t

Observación 3.1.1. Toda ecuación diferencial ordinaria lineal de orden n puede ser
transformada en un sistema de ecuaciones lineales, en efecto si tenemos

y (n) + an−1 (t) y (n−1) + · · · + a1 (t) y 0 + a0 (t) y = h (t)

poniendo

x1 (t) = y (t)
x2 (t) = y 0 (t)
..
.
xn (t) = y (n−1) (t)

entonces
      
x1 0 1 0 0 ··· 0 x1 0
      
 x
 2
 
  0 0 1 0 ··· 0  x
 2
 
  0 

d  ...
 
= .. .. .. .. .. ..  .
  ..
 
+ .. 
. . . . . . . 
dt       
   ..    
 xn−1

 
  0 0 0 0 . 1   xn−1

 
  0 

xn −a0 −a1 · · · · · · · · · −an−1 xn h (t)

199
Apuntes Mat023 (versión preliminar actualizada 23-05-2014)

Ejemplo 3.1.3. Considere la ecuación


d2 x dx
2
+ (cos t) + 2x = et
dt dt
esta se transforma en el sistema
      
d  x1   0 1 x
 1  + 
0
= 
dt x2 −2 − cos t x2 et

donde x1 (t) = x (t) y x2 (t) = x0 (t)

La teorı́a de los sistemas lineales es similar a la que hemos visto hasta ahora en el caso
de una ecuación. La ecuación Homogénea asociada a
dX
= A (t) X (t) + B (t)
dt
es
dX
= A (t) X (t)
dt
su conjunto solución
 
1 n dX
S= X (t) ∈ C (I, R ) : = A (t) X (t)
dt
es un espacio vectorial de dimensión n (si el sistema es de orden n) y la solución de la no
homogénea será una solución particular más una combinación lineal arbitraria de elementos
de la base de S.

Ecuación con coeficientes constantes

Suponga que trabajamos con el sistema


dX
= AX
dt
donde A ∈ Mn×n (R) (coeficientes constantes).

Teorema 3.2.1. Si λ es un valor propio de A con vector propio v entonces

x (t) = veλt
dX
es solución de dt
= AX.

200
Apuntes Mat023 (versión preliminar actualizada 23-05-2014)

Matriz A diagonalizable

Si A es una matriz diagonalizable entonces

P −1 AP = D

donde D es una matriz diagonal, la ecuación


dX
= AX
dt
la podemos ver como
dX
= P DP −1 X
dt
multiplicando la ecuación por P −1 por la izquierda
d
P −1 X = DP −1 X

dt
cambiamos la variable
Y = P −1 X

nos queda
dY
= DY
dt
que es un sistema de la forma
dyi
= λi yi para i = 1, 2, . . . , n
dt
tiene soluciones
yi (t) = ci eλi t

luego la solución es
n
X
X (t) = P Y (t) = ci vi eλi t
i=1

donde vi son los vectores propios asociados a los valores propios correspondientes.

Observación 3.2.1. Si el valor propio λi es complejo vi eλi t y vi eλi t aparecerán como


 
soluciones, al buscar soluciones reales se tiene que Re vi eλi t y Im vi eλi t son soluciones
l.i. reales correspondientes a combinaciones lineales de las dos complejas luego generan lo
mismo.

201
Apuntes Mat023 (versión preliminar actualizada 23-05-2014)

Ejemplo 3.2.1. Resolver


    
x1 −1 1 1 x1
d   
 x2  =  1
 
−1 1   x2 
dt     
x3 1 1 1 x3
Solución. Primero buscamos el polinomio caracterı́stico de la matriz
 
−1 1 1
 
A=  1 −1 1 

1 1 1
esta dado por

−1 − λ 1 1


PA (λ) = |A − λI| = 1 −1 − λ 1 = (λ − 2) (λ + 2) (λ + 1)


1 1 1−λ

luego tiene 3 valores propios distintos y es una matriz de orden 3 luego es diagonalizable,
ahora buscamos los vectores propios:
     
* −1 + * −1 + * 21 +
     
 −1  ↔ −1,  1  ↔ −2,  1  ↔2
     2 
1 0 1
ası́ la solución del sistema es
       
−1 −1 1 1
c e2t
2 3
− c2 e−2t − c1 e−t
  −t   −2t  2  2t  
X (t) = c1   −1  e + c2  1  e + c3  2
    1 e = 
  c2 e−2t − c1 e−t + 21 c3 e2t 

−t 2t
1 0 1 c1 e + c3 e

Ejemplo 3.2.2. Resolver


    
x1 5 2 2 x1
d     
 x2  =  2 2 −4   x2 
dt     
x3 2 −4 2 x3
Si  
5 2 2
 
A=
 2 −4 
2 
2 −4 2

202
Apuntes Mat023 (versión preliminar actualizada 23-05-2014)

entonces es diagonalizable por ser una matriz simétrica



5−λ 2 2

2

|A − λI| = 2 2 − λ −4 = − (λ + 3) (λ − 6)

2 −4 2 − λ

luego hay 1 valor propio de multiplicidad 2 buscamos los vectores propios


     
* − 21 + * 2 2 +
     
 1  ↔ −3,  1  ,  0  ↔ 6
     
1 0 1

se sigue que la solución del sistema es


       
− 21 2 2 2c2 e6t − 21 c1 e−3t + 2c3 e6t
  −3t   6t   6t  
 1  e + c2  1  e + c3  0  e = 
X (t) = c1        c1 e−3t + c2 e6t 

1 0 1 c1 e−3t + c3 e6t

Ejemplo 3.2.3. Resolver


    
x1 0 2 0 x1
d     
 x2  =  −2 0 0   x2 
dt     
x3 0 0 1 x3

Solución. Si  
0 2 0
 
A=
 −2 0 0 

0 0 1
entonces la ecuación caracterı́stica es (λ − 1) (λ2 + 4) = 0 de donde tenemos que la matriz
es diagonalizable, 3 valores propios distintos aunque hay dos complejos, veamos los espacios
propios asociados
     
* 0 + * i + * −i +
     
 0  ↔ 1,  1  ↔ −2i,  1  ↔ 2i
     
1 0 0

203
Apuntes Mat023 (versión preliminar actualizada 23-05-2014)

el resultado nos dice que    


i −i
  −2it  
 1 e y  1  e2it
   
0 0
son soluciones pero buscamos soluciones reales y para ello
     
i i
  −2it    −2it 
Re   1  e  y Im 
    1 e 

0 0

generan lo mismo que las dos anteriores


   
i i
  −2it  
 1  (cos (2t) − i sen (2t))
 1 e =  
 
0 0
 
i cos (2t) + sen (2t)
 
=   cos (2t) − i sen (2t) 

0
   
sen (2t) cos (2t)
   
=   cos (2t)  + i  − sen (2t)
  

0 0

se sigue     
i sen (2t)
  −2it   
 1  e  =  cos (2t) 
Re      

0 0
y     
i cos (2t)
  −2it   
Im 
 1  e  =  − sen (2t) 
    

0 0
luego la solución es
       
0 sin (2t) cos (2t) c3 cos 2t + c2 sin 2t
  t      
X (t) = c1 
 0
 e + c2  cos (2t)  + c3  − sin (2t)  =  c2 cos 2t − c3 sin 2t 
      
t
1 0 0 c1 e

204
Apuntes Mat023 (versión preliminar actualizada 23-05-2014)

Si la matriz no es diagonalizable no hemos desarrollado una teorı́a para poder resolverla


(más adelante resolveremos estos sistemas utilizando la transformada de Laplace) sin
embargo podemos enfrentar directamente el sistema a través de eliminación, considere el
siguiente ejemplo:

Ejemplo 3.2.4. Resolver el sistema


    
d  x   −2 −1   x 
=
dt y 1 −4 y

Solución. En este caso el polinomio es



−2 − λ −1

= λ2 + 6λ + 9 = (λ + 3)2

|A − λI| =
1 −4 − λ

se sigue que hay un solo valor propio de multiplicidad algebraica 2, Si calculamos vemos
que la multiplicidad geométrica (dimensión del espacio propio asociado) es 1 luego la matriz
no es diagonalizable
* +
1
Wλ=−3 =  
1

Lo resolveremos directamente

x0 = −2x − y
y 0 = x − 4y

si derivamos la primera ecuación obtenemos

x00 = −2x0 − y 0

pero de la segunda ecuación sabemos

y 0 = x − 4y

reemplazando

x00 = −2x0 − (x − 4y)


= 4y − x − 2x0

205
Apuntes Mat023 (versión preliminar actualizada 23-05-2014)

pero de la primera ecuación


y = −x0 − 2x

ası́

x00 = 4 (−x0 − 2x) − x − 2x0


= −9x − 6x0

ası́
x00 + 6x0 + 9x = 0 ⇔ (D + 3)2 x = 0

luego
x (t) = c1 e−3t + c2 te−3t

además

y (t) = −x0 − 2x
d
c1 e−3t + c2 te−3t − 2 c1 e−3t + c2 te−3t
 
= −
dt
−3t
= e (c1 − c2 + tc2 )
= c1 e−3t + c2 (−1 + t) e−3t

luego
   
−3t −3t
x (t) c1 e + c2 te
X (t) =  = 
y (t) c1 e−3t + c2 (−1 + t) e−3t
   
1 t
= c1   e−3t + c2   e−3t
1 −1 + t

es la solución. Note que la estructura de la solución es similar a las ecuaciones con raı́z
repetida.

Variación de parámetros en sistemas

Suponga que queremos determinar la solución particular de la ecuación


dX
= AX + B (t)
dt
206
Apuntes Mat023 (versión preliminar actualizada 23-05-2014)

donde la solución de la homogénea es

Xh (t) = c1 X1 (t) + c2 X2 (t) + · · · + cn Xn (t)

entonces proponemos una solución particular de la forma


n
X
Xp (t) = ci (t) Xi (t)
i=1

se sigue que
n
X n
X
Xp0 (t) = c0i (t) Xi (t) + ci (t) Xi0 (t)
i=1 i=1
Xn Xn
= c0i (t) Xi (t) + ci (t) AXi (t)
i=1 i=1
n
X
= c0i (t) Xi (t) + AXp (t)
i=1

entonces n
X
c0i (t) Xi (t) = B (t)
i=1

se sigue

X1 (t) X2 (t) · · · Xi−1 (t) B (t) Xi+1 (t) · · · Xn (t)

c0i (t) =
X1 (t) X2 (t) · · · Xi−1 (t) Xi (t) Xi+1 (t) · · · Xn (t)

es un cociente de determinantes por la regla de Cramer.

Ejemplo 3.3.1. Determine la solución general del sistema


      
dx 2t
1 −3 x e
 dt  =    +  
dy 4t
dt
−3 1 y e

Solución. Primero resolvemos el sistema homogéneo, la matriz


 
1 −3
 
−3 1

207
Apuntes Mat023 (versión preliminar actualizada 23-05-2014)

tiene valores propios −2, 4 y


*  +
 1 
Wλ=−2 =  
 1 
* +
 −1 
Wλ=4 =  
 1 

se sigue que la solución general del sistema homogéneo es


   
1 −1
Xh (t) = α   e−2t + β   e4t para α, β ∈ R
1 1

para determinar la solkución particular usamos variación de parámetros, existe una solución
de la forma    
1 −1
Xp (t) = C1 (t)   e−2t + C2 (t)   e4t
1 1
donde

2t 4t
e −e


4t 4t
e e e6t + e8t
C10 (t) = =
e−2t −e4t
2e2t

−2t 4t
e e
1 4t 1 6t
= e + e
2 2

se sigue Z  
1 4t 1 6t 1 1
C1 (t) = e + e dt = e4t + e6t
2 2 8 12
por otro lado

−2t 2t
e e

−2t 4t
e e

e2t − 1
C20 (t) = =
e−2t −e4t

2e2t

−2t
e e4t


1 1 −2t
= − e
2 2

208
Apuntes Mat023 (versión preliminar actualizada 23-05-2014)

se sigue
Z  
1 1 −2t
C2 (t) = − e dt
2 2
1 1
= t + e−2t
2 4
ası́
   
1 −1
Xp (t) = C1 (t)   e−2t + C2 (t)   e4t
1 1
   
1 −1
   
1 4t 1 1 1
= e + e6t   e−2t + t + e−2t   e4t
8 12 1 2 4 1
   
1 2t 1 4t 1 4t 1 2t

e + e − te + e
=  8 12 + 2 4
 
1 2t 1 4t 1 4t 1 2t
8
e + 12 e 2
te + 4 e
 
1 2t 1 4t 1 4t 1 2t
e + e − te − e
=  8 12 2 4 
1 2t 1 4t 1 4t 1 2t
8
e + 12
e + 2
te + 4
e
 
1 2t 1 2t 1 4t 1 4t
e − 4 e + 12 e − 2 te
=  8 
1 2t 1 2t 1 4t 1 4t
8
e + 4
e + 12
e + 2
te

la solución general es
     
− 81 e2t + 12
1
− 12 t e4t

1 −1
XG = α   e−2t + β   e4t +  
3 2t 1
+ 2t e4t

1 1 8
e + 12
para α, β ∈ R

Ejemplo 3.3.2. Resolver el sistema


      
dx
dt
1 −4 0 x 1
 dy
     
= 4 1  y  +  0 
0 
    
 dt  
dz
dt
0 0 2 z t

Solución. Este sistema tiene la forma

dX
= AX + B (t)
dt
209
Apuntes Mat023 (versión preliminar actualizada 23-05-2014)

donde    
1 1 −4 0
   
B (t) =  0
 
 yA= 4
 1 0 

t 0 0 2
primero resolvemos el sistema homogéneo
dX
= AX
dt
es decir     
dx
dt
1 −4 0 x
 dy    
 dt  =  4 1 0
    y 
 
dz
dt
0 0 2 z
buscamos los valores y vectores propios asociados, en este caso son:
     

 
 0 



 −i 
  i 

 
 
 0  ↔ 2,  1  ↔ 1 − 4i,  1  ↔ 1 + 4i
        
     
1 0 0
     

se sigue que las solución general del sistema homogéneo es de la forma


       
0 −i −i
  2t   (1−4i)t    (1−4i)t 
 0  e + c2 Re  1  e
Xh (t) = c1      + c3 Im  1
 
e



1 0 0
donde
   
−i −i
 1  e(1−4i)t =  1  et e−4ti
   
   
0 0
 
−i
  t
=  1  e (cos 4t − i sin 4t)
 
0
 
−iet cos 4t − et sin 4t
 
=  t
 e (cos 4t − i sin 4t) 

0
   
− sin 4t − cos 4t
   
= et   t
 cos 4t  + ie  − sin 4t


0 0

210
Apuntes Mat023 (versión preliminar actualizada 23-05-2014)

entonces      
0 − sin 4t − cos 4t
  2t    
Xh (t) = c1  0  e + c2 et  cos 4t  + c3 et  − sin 4t 
     
1 0 0
ahora buscamos una solución particular de la ecuación no homogénea
dX
= AX + B (t)
dt
de la forma
Xp (t) = c1 (t) X1 (t) + c2 (t) X2 (t) + c3 (t) X3 (t)

(mediante variación de parámetros) donde


     
0 −et sin 4t −et cos 4t
     
X1 (t) =   t  y X3 (t) =  −et sin 4t 
 0  , X2 (t) =  e cos 4t

  
e2t 0 0
se sigue las funciones c1 , c2 y c3 deben cumplir
    
0 −et sin 4t −et cos 4t c01 (t) 1
    
 0 et cos 4t −et sin 4t   c0 (t)  =  0 
  2   
0
e2t 0 0 c3 (t) t
sistema que podemos resolver por ejemplo mediante determinantes:

1 −et sin 4t −et cos 4t


0 et cos 4t −et sin 4t


t 0 0 te2t
c01 (t) = = 4t = te−2t
0 −et sin 4t −et cos 4t e


0 et cos 4t −et sin 4t

2t
e 0 0
:
0 1 −et cos 4t


0 0 −et sin 4t

2t
e t 0 −e3t sin 4t
c02 (t) = = = −e−t sin 4t
0 −et sin 4t −et cos 4t e4t


0 et cos 4t −et sin 4t

2t
e 0 0

211
Apuntes Mat023 (versión preliminar actualizada 23-05-2014)

y
0 −et sin 4t 1


0 et cos 4t 0

2t
e 0 t −e3t (cos 4t)
c03 (t) = = 4t
= − (cos 4t) e−t
0 −et sin 4t −et cos 4t e


0 et cos 4t −et sin 4t

2t
e 0 0

integramos : Z
1
c1 (t) = te−2t dt = − e−2t (2t + 1)
4
Z
1
c2 (t) = −e−t sin 4tdt = e−t (4 cos 4t + sin 4t)
17
Z
1
c3 (t) = − (cos 4t) e−t )dt = e−t (cos 4t − 4 sin 4t)
17
entonces la solución particular es
   
  0  −et sin 4t
1 −2t   1 −t  
Xp (t) = − e (2t + 1)  0  +
  e (4 cos 4t + sin 4t)  et cos 4t 
4 17  
2t
e 0
 
−et cos 4t
1
+ e−t (cos 4t − 4 sin 4t) 
 
−et sin 4t 
17  
0
 
1
− 17
 
=  4 
 17 
1 1
−2t − 4

212
Apuntes Mat023 (versión preliminar actualizada 23-05-2014)

verifiquemos que es correcto


 
0
Xp0 (t) = 
 
 0 

− 21
    
1
1 −4 0 − 17 1
    
=  4 + 0 
 4 1 0  
 17   
0 0 2 − 12 t − 1
4
t
   
−1 1
   
= 
 0 + 0 
  
−t − 21 t

se sigue que la solución del sistema es

X (t) = Xh (t) + Xp (t)


       
1
0 − sin 4t − cos 4t − 17
  2t 
t
    
= c1   + c3 et  − sin 4t  +  4
 0  e + c2 e  cos 4t
 
    17 
1 0 0 − 21 t − 1
4

Análisis cualitativo de sistemas

Vamos a analizar el comportamiento de los sistemas de orden 2 de la forma


dx
= ax + by
dt
dy
= cx + dy
dt
donde a, b, c y d son constantes, basándonos en los valores propios de la matriz asociada al
sistema  
a b
A= 
c d
la cual supondremos diagonalizable y adicionalmente que det (A) = ad − bc 6= 0 de esta
forma la única solución constante del sistema corresponde al origen, esta solución es llamada
solución de equilibrio, las demás soluciones serán entonces curvas paramétricas (x (t) , y (t))
y según su comportamiento clasificaremos la solución de equilibrio.

213
Apuntes Mat023 (versión preliminar actualizada 23-05-2014)

Valores propios reales y distintos (no nulos)

1. Punto silla: En el caso en que los valores propios son reales y distintos con signos
contrarios, el origen se dice que es un punto de silla. Considere el sistema
    
x
d    −1 1 x
=  
dt y 3 1 y

En este caso la matriz de coeficientes es


 
−1 1
A= 
3 1

la cual tiene determinante distinto de cero. Buscamos sus valores propios y vectores
propios asociados, en este caso son:
    
 −1   1 
  ↔ −2,  3  ↔ 2
 1   1 

luego las soluciones del sistema son de la forma


   
1
−1
X (t) = c1  3  e2t + c2   e−2t
1 1

Note que si c1 = 1 y c2 = 0 obtenemos una solución recta


 
1
3  e2t
X1 (t) = 
1

en esta solución x (t) = 13 e2t y y (t) = e2t se sigue que los puntos de esta curva están
sobre la recta
y e2t
= 1 2t = 3
x 3
e
 cuando t → +∞ por el primer cuadrante. De
es decir y = 3x y se alejan delorigen
1
3  e2t es la curva paramétrica que esta sobre la
manera similar, −X1 (t) = − 
1
recta y = 3x y si t → +∞ la solución se aleja del origen por el tercer cuadrante.
Existe otra solución recta, al poner c1 = 0 y c2 = 1 se obtiene
 
−1
X2 (t) =   e−2t
1

214
Apuntes Mat023 (versión preliminar actualizada 23-05-2014)

es decir la curva paramétrica

x = −e−2t
y = e−2t

esta curva esta sobre la recta

y
= −1 es decir y = −x
x

note que cuando t → +∞ los puntos se acercan al origen sobre tal recta por el
segundo cuadrante. Note que −X2 (t) también es solución, en este caso

x = e−2t
y = −e−2t

esta curva también esta sobre la recta y = −x los puntos se acercan al origen pero
por el cuarto cuadrante. Las solución general es
   
1
−1
X (t) = c1  3  e2t + c2   e−2t
1 1
 
−1
note que si t → +∞ entonces c2   e−2t ≈ 0 y la solución se comporta como
1
 
1
3  e2t esto es, cuando el tiempo avanza la curva solución se acerca a la recta
c1 
1
y = 3x (el cuadrante depende de c1 ) y sianalizamos
 el comportamiento cuando
−1
t → −∞ las curvas soluciones tienden a c2   e−2t esto se interpreta como que
1
las soluciones parten cercana a esa recta y se aproximan a y = 3x. Recuerde que
por teorema de existencia y unicidad dos soluciones distintas no se pueden cortar.

215
Apuntes Mat023 (versión preliminar actualizada 23-05-2014)

Grafiquemos algunas soluciones y el campo de direcciones asociado a este sistema

2. Sumidero: Si los valores propios son reales y distintos, ambos negativos, el origen
es llamado sumidero. Considere el sistema
    
6 2
d  x   −5 5   x 
=
dt y 2
− 95 y
5

en este caso la matriz de coeficientes es


 
− 65 2
5
A= 
2
5
− 95

que tiene vectores y valores propios asociados


    
 2   −1 
  ↔ −1,  2  ↔ −2
 1   1 

luego las soluciones son de la forma


   
1
2 −
X (t) = c1   e−t + c2  2  e−2t
1 1

216
Apuntes Mat023 (versión preliminar actualizada 23-05-2014)

grafiquemos el campo de direcciones y diagrama de fase asociados. Note que en este


caso tenemos dos soluciones rectas y que las soluciones tienden al origen si t → +∞

3. Fuente: Si los valores propios son distintos pero ambos positivos entonces el origen
es llamado fuente (las soluciones se alejan del origen) Considere el sistema
    
x
d    0 6 x
=  
dt y −1 5 y

en este caso la matriz de coeficientes es


 
0 6
A= 
−1 5

que tiene vectores y valores propios


     
 3   2 
  ↔ 2,   ↔ 3
 1   1 

217
Apuntes Mat023 (versión preliminar actualizada 23-05-2014)

luego las soluciones son de la forma


   
3 2
X (t) = c1   e2t + c2   e3t
1 1

note que si t → ∞ entonces kX (t)k → ∞ se sigue que las soluciones se alejan del
origen y tenemos dos soluciones rectas.

Valores propios complejos

1. Sumidero espiral: En el caso en que los valores propios son complejos y la parte
real del valor propio es negativa, el origen es un sumidero espiral. Considere el sistema
    
d  x   1 2 x
=  
dt y −4 −3 y

en este caso la matriz asociada es


 
1 2
A= 
−4 −3

218
Apuntes Mat023 (versión preliminar actualizada 23-05-2014)

tiene por vectores y valores propios


   
 −1 + 1i   −1 − 1i 
 2 2  ↔ −1 − 2i,  2 2  ↔ −1 + 2i
 1   1 

se sigue que la solución del sistema esta generada por


     
1 1 1 1
− + i − + i
Re  2 2  e(−1−2i)t  e Im  2 2  e(−1−2i)t 
1 1

entonces
   
1 1 1 1
sin 2t − cos 2t sin 2t + cos 2t
X (t) = c1 e−t  2 2  + c2 e−t  2 2 
cos 2t − sin 2t

ahora el campo de vectores y algunas curvas (note que en este caso no hay soluciones
rectas) las soluciones se acercan al origen

2. Fuente espiral: En el caso en que los valores propios son complejos con parte real
positiva las soluciones se alejan del origen y este es llamado fuente espiral. Considere

219
Apuntes Mat023 (versión preliminar actualizada 23-05-2014)

el sistema     
d   x 4 3 x
=  
dt y −6 −2 y
en este caso la matriz asociada es
 
4 3
A= 
−6 −2

los valores y vectores propios son


   
 −1 + 1i   −1 − 1i 
 2 2  ↔ 1 − 3i,  2 2  ↔ 1 + 3i
 1   1 

luego la solución es generada por


     
1 1 1 1
− + i − + i
Re  2 2  e(1−3i)t  e Im  2 2  e(1−3i)t 
1 1

es decir las soluciones son de la forma


   
1 1 1
sin 3t − cos 3t sin 3t + 21 cos 3t
X (t) = c1 et  2 2  + c2 et  2 
cos 3t − sin 3t

grafiquemos el campo de direcciones y algunas soluciones el comportamiento es el

220
Apuntes Mat023 (versión preliminar actualizada 23-05-2014)

mismo pero las soluciones se alejan del origen. (no tenemos soluciones rectas)

3. Centro: Cuando los valores propios son complejos con parte real igual a cero el
origen es llamado centro, en este caso las soluciones no se acercan al origen sino que
se mantienen alrededor de él, el origen es estable en este caso pero no asintóticamente
estable. Considere el sistema
    
d  x   3 3 x
=  
dt y −6 −3 y

en este caso la matriz asociada es


 
3 3
A= 
−6 −3

que tiene por valores y vectores propios


   
 −1 + 1i   −1 − 1i 
 2 2  ↔ −3i,  2 2  ↔ 3i
 1   1 

221
Apuntes Mat023 (versión preliminar actualizada 23-05-2014)

luego la solución del sistema es generada por


     
1 1 1 1
− + i − + i
Re  2 2  e(−3i)t  e Im  2 2  e(−3i)t 
1 1

es decir las soluciones son de la forma


   
1 1 1
sin 3t − cos 3t sin 3t + 21 cos 3t
X (t) = c1  2 2  + c2  2 
cos 3t − sin 3t

grafiquemos el campo de vectores y algunas curvas solución.

Valores propios repetidos (no nulos)

1. Fuente: Cuando hay solo un valor propio y es positivo el origen es llamado fuente,
las soluciones se alejan del origen. Considere el sistema
    
d  x   2 0  x 
=
dt y 0 2 y

222
Apuntes Mat023 (versión preliminar actualizada 23-05-2014)

en este caso la matriz asociada es


 
2 0
A= 
0 2

y el valor propio es λ = 2 se ve que las soluciones son de la forma


   
1 0
X (t) = c1   e2t + c2   e2t
0 1

es decir  
2t
c1 e
X (t) =  
2t
c2 e
las soluciones son todas rectas que se alejan del origen.

2. Sumidero: Si hay solo un valor propio y es negativo las curvas se acercan al origen
y este es llamado sumidero. Considere el sistema
    
d  x   −2 0   x 
=
dt y 0 −2 y

en este caso la matriz asociada es


 
−2 0
A= 
0 −2

los valores propios son iguales a −3 pero la matriz no es diagonalizable, las soluciones
son de la forma  
c1 e−2t
X (t) =  
c2 e−2t
son rectas que se acercan al origen.

Teorema 3.4.1. Para un sistema lineal X0 = AX en el cual det (A) =


6 0, sea X = X (t)
la solución que satisface X (0) = X0 con X0 6= 0.

1. lı́mt→+∞ X (t) = 0 si y solo si los valores propios de A tienen partes reales negativas.

223
Apuntes Mat023 (versión preliminar actualizada 23-05-2014)

2. X (t) es periódica si y solo si las valores propios de A son imaginarios puros.

Ejemplo 3.4.1. Sea α ∈ R− {1, 2}. Considere el sistema de ecuaciones


    
d  x   α α−2 x
=  
dt y 1−α α y

1. Clasificar la solución de equilibrio (silla, atractor, repulsor, etc.) para los distintos
valores de α.

Primero notemos que



α−2

α
= 2α2 − 3α + 2 > 0
1−α α

para todo α ∈ R, luego la única solución de equilibrio es el origen. Para clasificar


calculamos los valores propios

λ2 − 2αλ + 2α2 − 3α + 2


luego
p
4α2 − 4 (2α2 − 3α + 2)
2α ±
λ =
p 2
2α ± 4α − 4 (2α2 − 3α + 2)
2
=
p 2
= α ± − (α − 1) (α − 2)

a) Si − (α − 1) (α − 2) > 0 es decir α ∈ ]1, 2[ tenemos soluciones reales y notemos


que el producto de las dos es 2α2 − 3α + 2 > 0 luego las dos son positivas o
negativas,
p
α− − (α − 1) (α − 2) > 0
⇔ (para α > 0)
α2 > − (α − 1) (α − 2)

α2 + (α − 1) (α − 2) > 0

224
Apuntes Mat023 (versión preliminar actualizada 23-05-2014)

lo que tiene solución todo R+ pero por la restricción se sigue α ∈ ]1, 2[ . Ası́,
para α ∈ ]1, 2[ tenemos dos valores propios reales y positivos, el punto es un
repulsor.

b) Si − (α − 1) (α − 2) < 0 esto es α ∈ ]−∞, 1[ ∪ ]2, +∞[ tenemos raı́ces complejas


conjugadas. Para α = 0 tenemos un centro, para α < 0 tenemos un atractor
espiral y para α ∈ ]0, 1[ ∪ ]2, +∞[ tenemos un repulsor espiral.

3
2. Para α = 2
determine la solución general del sistema y bosquejar el diagrama de
fases.
3
si α = 2
entonces
3 1
λ= ±
2 2
es decir, los valores propios son 2, 1 buscamos los espacios propios
   
3 3 3 1
−2 −2
 2 2 = 2 
1 − 32 3
2
− 1
2
3
2

se sigue
* +
 1 
Wλ=1 =  
 1 
y
*  +
 −1 
Wλ=2 =  
 1 

la solución general es
     
x (t) 1 −1
  = C1   et + C2   e2t
y (t) 1 1

225
Apuntes Mat023 (versión preliminar actualizada 23-05-2014)

el diagrama de fases es

Ejercicios del capı́tulo

1. Escribir los siguientes sistemas en forma matricial


dx dx
a) dt
= 3x − 2y b) dt
= x − 2ty + t2
dy dy
dt
= 4x + 8y dt
= 4t2 x + 8y + sin t

dx dx
c) dt
= 3x − y + 3z d) dt
= et x − cos ty + 3tz + cos t
dy dy
dt
= 4x + 3y + z dt
= x + et
dz dz
dt
= 2x + y − z dt
= 2x + y − z − 3t + 1

2. Escribir los siguientes sistemas sin usar matrices


      
d 
x −2 1 x sin t
a) dt  =    +  
y t 0 y et
     
−1 1 1 −1 −1 + t
      t
b) X0 =   0 1 2    t 
 X+  0  e +  1  e sin t

−1 −1 1 −1 −1

3. En los siguientes ejercicios verificar si el vector X dado es solución del sistema


  
1  dx = 3x − 4y
dt
a) X =   e−5t de dy
2 
dt
= 4x − 7y

226
Apuntes Mat023 (versión preliminar actualizada 23-05-2014)

  
dx
1 

 dt
= x + 2y + z
  dy
b) X = 
 6  de  dt =
 6x − y

 dz
13 dt
= −x − 2y − z
    
sin t 

 1 0 1
  0 
 
c) X =  1 1 de X =  1 1 0 X
 − 2 sin t − 2 cos t 

 

− sin t + cos t −2 0 −1

4. Determine si los vectores dados son linealmente independientes


     
1 2 8
a) X1 =   et y X2 =   et +   tet
−1 6 −8
     
1 1 2
     
b) X1 =  6 , X2 =  −2  e−4t y X3 =  3  e3t
     
−13 −1 −2
   
1 1
5. Muestre que la función Xp =   et +   tet es solución particular del
1 −1
sistema    
2 1 1
X0 =   X−   et
1 −1 7

6. Demostrar que la solución general del sistema


 
0 6 0
X0 = 
 
X
 1 0 1 
1 1 0

es de la forma
     
6 −3 2
 −t
 e + c2  1  e−2t + c2  1  e3t
    
X =c1 
 −1     
−5 1 1

7. Determinar la solución general de los sistemas:

227
Apuntes Mat023 (versión preliminar actualizada 23-05-2014)

dx dx
a) dt
= x + 2y b) dt
= 2x + 2y
dy dy
dt
= 4x + 3y dt
= x + 3y

dx dx
c) dt
= x+y−z d) dt
= 2x − 7y
dy dy
dt
= 2y dt
= 5x + 10y + 4z
dz dz
dt
= y−z dt
= 5y + 2z

dx dx
e) dt
= 6x − y f) dt
= 2x + y + 2z
dy dy
dt
= 5x + 2y dt
= 3x + 6z
dz
dt
= −4x − 3z

8. Resolver los siguientes sistemas usando variación de parámetros


dx dx
a) dt
= 3x − 3y + 4 b) dt
= 2x − y
dy dy
dt
= 2x − 2y − 1 dt
= 3x − 2y + 4t

dx dx
c) dt
= x+y−z+t d) dt
= −y + sec t
dy dy
dt
= 2y + t dt
= x
dz
dt
= y−z+t

dx dx
e) dt
= x + 2y + et csc t f) dt
= 3x − y − z
dy dy
dt
= − 21 x + y + et sec t dt
= x+y−z+t
dz
dt
= x − y + z + 2et

9. Sea X = X (t) la solución al sistema de ecuaciones

x0 = αx − βy
y 0 = βx + αy

que satisface la condición inicial X (0) = X0 . Determine condiciones sobre α, β que


aseguren  
0
lı́m X (t) =  
t→+∞
0

228
Apuntes Mat023 (versión preliminar actualizada 23-05-2014)

10. Determine condiciones sobre µ tal que (0, 0) sea un centro para el sistema lineal

x0 = −µx + y
y 0 = −x + µy

11. Sea A ∈ Mn×n (R) defina

t2 t3
eAt = I + At + A2 + A3 + · · ·
2! 3!

X tk
= Ak
k=0
k!
 
a c
a) Calcular eAt para A =  
0 b
b) Muestre que si A = PDP−1 con D =diag(λ1 , λ2 , . . . , λn ) diagonal entonces

etA = P diag eλ1 t , eλ2 t , . . . , eλn t P−1




c) Muestre que la solución general de


 
d 4 3
X = X
dt −2 −1

es  
c1
X =etA  
c2
 
4 3
donde A =  
−2 −1
d ) Conjeturar una fórmula para la solución general de

X0 = AX + B (t)

en términos de etA .

12. Clasificar la solución de equilibrio de los siguientes sistemas y bosquejar el diagrama


de fases:

229
Apuntes Mat023 (versión preliminar actualizada 23-05-2014)

dx dx
a) dt
= −x − 5y b) dt
= 4x + 3y
dy dy
dt
= x + 3y dt
= 2x − 3y

dx dx
c) dt
= −x + 2y d) dt
= x + 4y
dy dy
dt
= x − 4y dt
= −x + y

dx dx
e) dt
= −3x + y f) dt
= 2x + y
dy dy
dt
= −2x − y dt
= 3x + 6y

230
Capı́tulo 4 : Transformación integral de Laplace

Definiciones y teoremas fundamentales

Observación 4.1.1. Las ecuaciones diferenciales describen formas en las cuales ciertas
cantidades cambian respecto al tiempo, cantidades tales como la corriente en un circuito
eléctrico, las oscilaciones de una membrana, flujos de calor, etc. esas ecuaciones generalmente
están acompañadas de condiciones iniciales que describen el estado del sistema en el
tiempo t = 0. Una herramienta muy poderosa para resolver este tipo de problemas es la
transformada de Laplace la cual transforma la ecuación diferencial original en una ecuación
algebraica la cual puede ser transformada nuevamente en la solución del problema original.
Esta técnica es conocida como “el método de la transformada de Laplace”. La clave del
proceso anterior radica en la fórmula de integración por partes. Si f, g ∈ C 1 recordemos
que la fórmula de integración por partes corresponde a:
Z b b Z b
0
f (x) g (x) dx = f (x) g (x) − f 0 (x) g (x) dx

a a a

y se puede interpretar del modo siguiente: En un producto de funciones bajo el sı́mbolo de


la integral es posible intercambiar la derivada presente en una función. Sin embargo, tal
procedimiento implica al final el tener que evaluar el producto de tales funciones en los
extremos del intervalo y trabajar con una nueva derivada.
La idea, entonces, es trabajar con una función que tenga un buen comportamiento de la
derivada y la evaluación en los extremos del intervalo de integración, esto se puede lograr
mediante una función que no cambie esencialmente al derivarla i.e. una exponencial y que
se pueda anular en los extremos. Lo anterior motiva la siguiente definición:

Definición 4.1.1. Una función f : [0, +∞) → R se dice localmente integrable si para
RT
cada T > 0, la integral 0 f (t) dt existe. Llamaremos transformación integral de
Laplace o simplemente la transformada de Laplace a la función L [f (t)] : D ⊆ R → R
definida mediante la integral impropia:
Z ∞
s → L [f (t)] (s) = f (t) e−st dt
0

para todos los valores de s ∈ R para los cuales la integral sea convergente. El conjunto de

231
Apuntes Mat023 (versión preliminar actualizada 23-05-2014)

los valores de s ∈ R para los cuales la integral converge se llamará dominio de L [f (t)] y se
denotará por D (L [f ]).

Ejemplo 4.1.1. Sea f : R → R, t → f (t) = 1. Calcular su transformada de Laplace.

Solución. Note que:


Z ∞ Z T
−st
1e dt = e−st dt
lı́m
0 T →∞ 0
 
1 −st T
= lı́m − e
s

T →∞ 0
1
=
s
para cada s > 0. Por tanto:
1
L [1] (s) = , s>0
s
Ejemplo 4.1.2. Sea f (t) = eat , con a una constante real. Calcular su transformada de
Laplace

Solución. Por definición:


Z ∞ Z T
at at −st
e(a−s)t dt
 
L e (s) = e e
dt = lı́m
0 T →∞ 0
1 (a−s)t T
= lı́m e
T →∞ a − s

0
1
e(a−s)T − 1

= lı́m
T →∞ a − s

1 1

ası́, si a − s < 0, entonces lı́mT →∞ a−s e(a−s)T − 1 = s−a , luego:

1
L eat (s) =
 
Para s > a
s−a
Observación 4.1.2. Con respecto al ejemplo anterior, dos observaciones deben hacerse.
En primer lugar, respecto a lo notacional, en lo sucesivo escribiremos:
T ∞
lı́m f (t) = f (t)

T →∞ 0 0

para evitar anotar cada vez que se calcule una transformada de Laplace, el lı́mite relacionado
con la integral impropia. En segundo lugar, notamos que D (L [eat ]) = ]a, +∞[, luego la
transformada no necesariamente estará definida para todos los valores de s ∈ R.

232
Apuntes Mat023 (versión preliminar actualizada 23-05-2014)

Ejemplo 4.1.3. Calcule, si acaso existe, la transformada de Laplace de f (t) = 1t , con


t > 0.

Solución. Sea t > 0. Luego:


Z ∞ 1 ∞
e−st e−st e−st
Z Z
dt = dt + dt
0 t 0 t 1 t
pero si 0 < t ≤ 1 y s > 0, entonces e−s ≤ e−st y la integral:
Z 1 −s
e
dt
0 t
diverge. Por tanto, la transformada de Laplace de f (t) = 1/t no existe para ningún valor
de s ∈ R.

En vista del ejemplo anterior, trataremos de dar respuesta a la siguiente pregunta:


¿Para qué tipo de funciones podemos asegurar la existencia de la transformada de Laplace?

Definición 4.1.2. Sean f una función real. Anotaremos:

f a+ = lı́m+ f (t) f a− = lı́m− f (t)


 

t→a t→a

si acaso estos lı́mites existen. Diremos que una función f tiene una discontinuidad de
salto en a si:
f a+ − f a− < ∞
 

o bien si f (a+ ) = f (a− ), pero estos valores son distintos de f (a) (puede ser incluso que
f (a) no esté definida). Una función f : [0, +∞) → R se dice seccionalmente continua
si para cada T > 0 la función f |[0,T ] es continua y tiene a lo sumo un número finito de
discontinuidades de salto.

Ejemplo 4.1.4. La función 


 1/t t 6= 0
f (t) =
 0 t=0

no es seccionalmente continua en [0, +∞), lo mismo para



 cos (1/t) t > 0
f (t) =
 0 t≤0

mientras que g (t) = btc si es seccionalmente continua.

233
Apuntes Mat023 (versión preliminar actualizada 23-05-2014)

Definición 4.1.3. Diremos que una función f : [0, +∞) → R es de orden exponencial
β si existen constantes M > 0, β ∈ R y un valor t0 ≥ 0 tales que:

|f (t)| ≤ M eβt

para todo t ≥ t0 .

Ejemplo 4.1.5. Las siguientes funciones son seccionalmente continuas y de orden expo-
nencial:

1. f (t) = sinh t, pues:


et − e−t et
sinh t = <
2 2
2. f (t) = tn . En efecto, sabemos que et = ∞ tn tn
< et . Por
P
n=0 n! . Luego, para t > 0 , n!

tanto:
tn ≤ n! et

Teorema 4.1.1 (Existencia). Sea f : [0, +∞) → R una función seccionalmente continua y
de orden exponencial β entonces la transformada de Laplace L [f (t)] (s) existe para s > β.

Demostración. En primer lugar note que, como f : [0, +∞) → R es seccionalmente


continua, la función t 7→ f (t) e−st es localmente integrable en [0, +∞). Por otro lado, por
la desigualdad triangular tenemos que:
Z ∞ Z t0 Z ∞
−st −st
|f (t)| e−st dt

f (t) e dt ≤ f (t) e dt +

0 0 t0
R
t
Note que 0 0 f (t) e−st dt < ∞ y además, como f : [0, +∞) → R es de orden exponencial,

existen constantes M, β > 0 y un valor t0 > 0 tales que:

|f (t)| ≤ M eβt

para todo t ≥ t0 . Luego:


Z ∞ ∞ ∞
M e(β−s) ∞
Z Z
−st βt −st
|f (t)| e dt ≤ M e e dt = M e(β−s)t dt =
β − s t0

t0 t0 t0

en donde el último término converge si y solo si β − s < 0. Por tanto, como:


Z ∞ Z ∞
−st −st

f (t) e dt < f (t) e dt
0 0

se tiene que L {f (t)} (s) existe para cada s > β.

234
Apuntes Mat023 (versión preliminar actualizada 23-05-2014)

 
t2 t2
Observación 4.1.3. La función f (t) = 2te cos e es continua en [0, +∞[ pero no de
orden exponencial sin embargo su transformada de Laplace existe para s > 0, por otro
1
lado la función g (t) = √
t
no es seccionalmente continua en [0, +∞[ sin embargo posee
transformada de Laplace.

Teorema 4.1.2 (Linealidad de la transformada). Sean f, g : [0, +∞) → R dos funciones


localmente integrables y α ∈ R, entonces para cada s ∈ D (L [f ]) ∩ D (L [g]) se tiene que:

L [αf + g] (s) = αL [f ] (s) + L [g] (s)

Cálculo de transformadas

Observación 4.2.1. En esta sección haremos el cálculo de transformadas de Laplace


básicas, para ası́ obtener una tabla elemental de transformadas.

Ejemplo 4.2.1. Transformadas de f (t) = sin ωt y f (t) = cos ωt.

Solución. Integrando por partes obtenemos


α cos (βx) eαx β sin (βx) eαx
Z
eαx cos (βx) dx = + +C
α2 + β 2 α2 + β 2
β cos (βx) eαx α sin (βx) eαx
Z
eαx sin (βx) dx = − + +C
α2 + β 2 α2 + β 2
luego
Z ∞
L [sin ωt] (s) = sin (ωt) e−st dt
0
+∞
ω cos (ωt) e−st s sin (ωt) e−st
= − −
s2 + ω 2 s2 + ω 2 0
ω
= 2
s + ω2
si s > 0. De manera similar
Z ∞
L [cos ωt] (s) = cos (ωt) e−st dt
0
+∞
−s cos (ωt) e−st ω sin (ωt) e−st
= +
s2 + ω 2 s2 + ω 2 0
s
= 2
s + ω2
para s > 0.

235
Apuntes Mat023 (versión preliminar actualizada 23-05-2014)

Ejemplo 4.2.2. Usemos variable compleja para calcular las mismas transformadas: Por la
fórmula de Euler:
eiωt = cos ωt + i sin ωt

y la linealidad de la transformada, se obtiene que:

eiωt + e−iωt
cos ωt =
2

y:
eiωt − e−iωt
sin ωt =
2i
Entonces:

eiωt + e−iωt
 
L [cos ωt] (s) = L (s)
2
1
L eiωt (s) + L e−iωt (s)
    
=
2 
1 1 1
= +
2 s − iω s + iω
 
1 s + iω + s − iω
=
2 s2 − (iω)2
s
=
s + ω2
2

para s > 0. Análogamente, se obtiene que:

ω
L [sin ωt] (s) = , s>0
s2 + ω2

Ejemplo 4.2.3. Transformadas de f (t) = sinh ωt y f (t) = cosh ωt. Sabemos que:

eωt + e−ωt eωt − eωt


cosh ωt = ∧ sinh ωt =
2 2

Por la linealidad de la transformada de Laplace se obtiene que:

s
L [cosh ωt] (s) = , s > |ω|
s2 − ω2

y:
ω
L [sinh ωt] (s) = , s > |ω|
s2 − ω 2

236
Apuntes Mat023 (versión preliminar actualizada 23-05-2014)

Ejemplo 4.2.4. Transformada de f (t) = t. Notemos que:


Z ∞
1 ∞
Z
−st
0
te dt = − t e−st dt
0 s 0
1
 ∞ Z ∞ 
−st −st
= − te − e dt
s 0 0
 
1 1
= − 0−
s s
1
= 2
s

para todo s > 0. Por tanto:


1
L [t] (s) = , s>0
s2
usando integración por partes, es fácil mostrar que

n!
L [tn ] (s) = , s>0
sn+1

Observación 4.2.2. Resumimos las funciones con sus respectivas transformadas de Laplace
y sus dominios en la siguiente tabla:

f (t) L [f (t)] (s) Dominio

1
1 s>0
s
1
t s>0
s2
1
eαt s>α
s−α
s
cos ωt s>0
s + ω2
2
ω
sin ωt s>0
s2 + ω 2
s
cosh ωt s > |ω|
s − ω2
2
ω
sinh ωt s > |ω|
s − ω2
2

n!
tn n+1
s>0
s

237
Apuntes Mat023 (versión preliminar actualizada 23-05-2014)

Ejemplo 4.2.5. Calcule la transformada de Laplace de la función:

f (t) = 3 − 2t + 4 cos 2t

Solución. Por la linealidad de la transformada y la tabla anterior, se obtiene:

L [f (t)] (s) = L [3 − 2t + 4 cos 2t] (s)


= 3L [1] (s) − 2L [t] (s) + 4L [cos 2t] (s)
3 2 4s
= − 2+ 2
s s s +4

Ejemplo 4.2.6. si f (t) = a0 + a1 t + · · · + an tn entonces


n
X
ak L tk (s)
 
L [f (t)] (s) =
k=0
n
X k!
= ak
k=0
sk+1

Teorema 4.2.1. Si f es seccionalmente continua en [0, +∞) y de orden exponencial β


entonces
lı́m L [f (t)] (s) = 0
s→+∞

El resultado anterior se debe a la desigualdad


Z ∞
−st ≤ M


e f (t) dt s−β
0

tomando s → +∞ se obtiene el resultado.

s−1 es
Observación 4.2.3. Las funciones ,
s+1 s
no pueden ser transformadas de una función
seccionalmente continua de orden exponencial.

Primer Teorema de la Traslación

Observación 4.3.1. El siguiente resultado nos permite calcular la transformada de Laplace


de eαt f (t) cuando es conocida la transformada de Laplace de f (t), en este resultado se
pone de manifiesto la primera propiedad de la traslación de la transformada de Laplace.
En efecto, tenemos el siguiente teorema:

238
Apuntes Mat023 (versión preliminar actualizada 23-05-2014)

Teorema 4.3.1. Suponga que F (s) = L [f (t)] (s) existe para cada s > β. Si α ∈ R,
entonces:
L eαt f (t) (s) = F (s − α)
 

para todo s > α + β.

Ejemplo 4.3.1. Calcule L [e−3t cos 2t] (s)


s
Solución. Notamos que F (s) = L [cos 2t] (s) = s2 +4
. Luego:
s+3
L e−3t sin 2t (s) = F (s + 3) =
 
(s + 3)2 + 4
Observación 4.3.2. La primera propiedad de traslación de la transformada de Laplace,
deja nuestra tabla básica de transformadas como sigue:

f (t) L [f (t)] (s) Dominio

n!
eαt tn s>α
(s − α)n+1
s−α
eαt cos ωt s>α
(s − α)2 + ω 2
ω
eαt sin ωt s>α
(s − α)2 + ω 2
s−α
eαt cosh ωt s > α + |ω|
(s − α)2 − ω 2
ω
eαt sinh ωt s > α + |ω|
(s − α)2 − ω 2

Transformada de la derivada

Observación 4.4.1. El origen y el objetivo de la transformada de Laplace es la resolución


de ecuaciones diferenciales. Para lograr lo anterior, debemos calcular la transformada de
Laplace de la derivada de una función. Entonces, sea y = y (t) una función derivable, note
que:
Z ∞ ∞ Z ∞ 0
0 −st −st
y (t) e dt = y (t) e − y (t) e−st dt
0 0
Z ∞ 0
= y (0) + s y (t) e−st dt
0

239
Apuntes Mat023 (versión preliminar actualizada 23-05-2014)

bajo el supuesto que:


lı́m y (t) e−st = 0
t→∞

Ası́, si y = y (t) es una función derivable y de orden exponencial, se tiene que y 0 (t) existe
para cada t ≥ 0 y:
|y (t)| ≤ M eβt , t ≥ t0

Entonces:
−M e(β−s)t ≤ y (t) e−st ≤ M e(β−s)t

donde los extremos convergen a cero si β − s < 0, cuando t → ∞. Ası́, bajo las condiciones
anteriores se obtiene que:

L [y 0 (t)] (s) = sL [y (t)] (s) − y 0+




Por tanto, tenemos el siguiente teorema:

Teorema 4.4.1. Suponga que f es continua en ]0, ∞[ y de orden exponencial β, supongamos


además que f 0 es seccionalmente continua en [0, +∞) entonces:

L [f 0 (t)] (s) = sL [f (t)] (s) − f 0+




Corolario 4.4.1. Suponga que f (t) , f 0 (t) , · · · , f (n−1) (t) son continuas en ]0, ∞[ y de
orden exponencial, suponga también que f (n) (t) es seccionalmente continua en [0, +∞).
Entonces

L f (n) (t) (s) = sn L [f (t)] (s) − sn−1 f 0+ − sn−2 f 0 0+ − · · · − f (n−1) 0+


    

Ejemplo 4.4.1. Calcule la transformada de Laplace de f (t) = tn , para n ≥ 2.

Solución. Note que:


f 0 (t) = ntn−1

ası́, por la transformada de la derivada, se tiene que:

L ntn−1 (s) = L [f 0 (t)] (s) = sL [f (t)] (s) − f 0+ = sL [tn ] (s)


  

ası́:
n  n−1 
L [tn ] (s) = L t (s) , n≥2
s
240
Apuntes Mat023 (versión preliminar actualizada 23-05-2014)

entonces: 
1
n

s2
n=1
L [t ] (s) = s>0
n n−1

s
L [t ] (s) n > 1
se sigue:
n!
L [tn ] (s) = , s>0
sn+1
Ejemplo 4.4.2. Como aplicación de la transformada de la derivada, calcular la transfor-
mada de Laplace de sin2 (αt)

Solución. Derivando

f 0 (t) = 2α sin (αt) cos (αt)


= α sin (2αt)

luego
sL [f (t)] (s) − f (0) = L [α sin (2αt)] (s)

ası́
2α2
sL [f (t)] (s) =
s2 + 4α2
finalmente
2α2
L [f (t)] (s) =
s (s2 + 4α2 )
Ejemplo 4.4.3. Sea f : [0, +∞) → R una función continua. Se define:
Z t
F (t) = f (u) du
0

Calcule L [F (t)] (s).

Solución. Como f es continua, por el Teorema Fundamental del Cálculo, F es derivable


con:
F 0 (t) = f (t)

Entonces, por la transformada de la derivada tenemos que:

L (F 0 (t)) (s) = sL [F (t)] (s) − F 0+




Ası́: Z t 
1
L f (t) dt (s) = L [F (t)] (s) = L [f (t)] (s)
0 s
pues F (0) = 0.

241
Apuntes Mat023 (versión preliminar actualizada 23-05-2014)

Teorema 4.4.2. Sea n ∈ N, suponga que f es seccionalmente continua en [0, +∞) de


orden exponencial β y que F (s) = L [f (t)] (s) entonces:

dn
L [tn f (t)] (s) = (−1)n F (s) (4.1)
dsn

para s > β.

Observación 4.4.2. El siguiente teorema, habla sobre la inyectividad de la transformación


de Laplace, lo que nos permite hablar de inversa.

Teorema 4.4.3. Dos funciones con la misma transformada de Laplace no pueden diferir
en todo un intervalo de longitud positiva. Es decir, si L [f (t)] (s) = L [g (t)] (s) entonces
f (t) = g (t), para cada t tal que f y g simultáneamente sean continuas.

Observación 4.4.3. Funciones continuas diferentes tienen transformadas de Laplace


diferentes.

Definición 4.4.1. Sea F : [0, +∞) → R una función tal que lı́ms→∞ F (s) = 0. Diremos
que una función f : [0, +∞) → R, para la cual existe su transformada de Laplace, es una
transformada de Laplace inversa de F si satisface:

L [f (t)] (s) = F (s)

Por el Teorema de Lerch, tal transformada inversa está únicamente determinada sólo sobre
sus puntos de continuidad. Abusando del lenguaje, se escribe:

f (t) = L−1 [F (s)] (t)

Observación 4.4.4. El operador L−1 también es lineal. Es decir:

L−1 [αf + g] = αL−1 [f ] + L−1 [g]

Ejemplo 4.4.4. Calcule L−1 s+9


 
s2 +6s+13
(t).

242
Apuntes Mat023 (versión preliminar actualizada 23-05-2014)

Solución. Observe que:


s+9 s+9
=
s2 + 6s + 13 (s + 3)2 + 4
s+3 6
= +
(s + 3) + 4 (s + 3)2 + 4
2

Entonces:
     
−1 s+9 −1 s+3 −1 2
L (t) = L (t) + 3L (t)
s2 + 6s + 13 (s + 3)2 + 4 (s + 3)2 + 4
= e−3t cos 2t + 3e−3t sin 2t

Ejemplo 4.4.5. Calcular


 
−1 s
L (s)
(s + 1) (s + 2) (s + 3)
Solución. Usando la técnica de fracciones parciales
s 2 1 3
= − −
(s + 1) (s + 2) (s + 3) s + 2 2 (s + 1) 2 (s + 3)
luego
 
−1 s
L (s)
(s + 1) (s + 2) (s + 3)
 
−1 2 1 3
= L − − (s)
s + 2 2 (s + 1) 2 (s + 3)
1 3
= 2e−2t − e−t − e−3t
2 2
Ejemplo 4.4.6. Calcular   
−1 s+1
L ln (s)
s+2
Solución. Queremos determinar f (t) tal que
 
s+1
L [f (t)] (s) = ln
s+2
notamos que
L [f (t)] (s) = ln (s + 1) − ln (s + 2)

derivando
1 1
L [f (t)]0 (s) = −
s+1 s+2
243
Apuntes Mat023 (versión preliminar actualizada 23-05-2014)

pero
L [f (t)]0 (s) = L [−tf (t)] (s)

se sigue
L [−tf (t)] (s) = L e−t − e−2t (s)
 

ası́
−tf (t) = e−t − e−2t

de donde obtenemos
e−2t − e−t
f (t) =
t
Ahora veremos algunos ejemplos de como funciona el método de la transformada de
Laplace para resolver ecuaciones:

Ejemplo 4.4.7. Resuelva la ecuación diferencial:

ty 00 − ty 0 − y = 0, y (0) = 0, y 0 (0) = 3 (4.2)

Solución. Sea Y (s) = L [ y (t)] (s). Aplicando la transformada de Laplace a la ecuación


(4.2), obtenemos:
L [ty 00 ] (s) − L [ty 0 ] (s) − L [y] (s) = 0

pero:
d  2
L [ty 00 ] (s) = − s L [y] (s) − sy (0) − y 0 (0) = −s2 Y 0 − 2sY

ds
y
d
L [ty 0 ] (s) = − {sL [y] (s) − y (0)} = −sY 0 − Y
ds
reemplazando estas expresiones en la ecuación original obtenemos:

−s2 Y 0 − 2sY + sY 0 + Y − Y = 0

rs decir:
s (s − 1) Y 0 + 2sY = 0

entonces:
2
Y0+ Y =0
s−1

244
Apuntes Mat023 (versión preliminar actualizada 23-05-2014)

la cual es una ecuación de variable separable. Separando, entonces, las variables, obtenemos:

dY 2
=− ds
Y s−1

de donde se sigue que:


C
Y (s) =
(s − 1)2
se sigue que
C  t

L [y (t)] (s) = Y (s) = 2 = L Cte (s)
(s − 1)
usando el teorema de Lerch, se sigue

y (t) = Ctet

(note que se busca una función derivable, luego continua). Ahora bien, como y 0 (0) = 3, se
obtiene que C = 3. Por tanto, la solución de la ecuación diferencial es:

y (t) = 3tet

Ejemplo 4.4.8. Utilizando la transformada de Laplace, resuelva la siguiente ecuación


diferencial:
y 00 − 4y = 0, y (0) = 1, y 0 (0) = 2

Solución. Aplicando la transformada de Laplace

s2 L [y] (s) − sy (0) − y 0 (0) − 4L [y] (s) = 0

reemplazando los valores iniciales

s2 − 4 L [y] (s) = s + 2


ası́
s+2 1  2t 
L [y] (s) = = = L e (s)
s2 − 4 s−2
finalmente la solución de la ecuación es:

y (t) = e2t

245
Apuntes Mat023 (versión preliminar actualizada 23-05-2014)

Funciones escalonadas y Segundo Teorema de la Traslación

Observación 4.5.1. Cuando se estudio el movimiento vibratorio en el caso más general,


se obtuvo una ecuación diferencial de la forma:
c 0 k
x00 + x + x = F (t)
m m
para constantes fı́sicas c, k y m. La función forzadora F (t), que actuaba como fuerza
externa al sistema, se supuso continua. En las aplicaciones, no siempre se puede pedir tal
condición; muchas veces, tal función es una función escalonada o con discontinuidades de
salto.

Definición 4.5.1. La función de Heaviside con salto en 0 se define como la función:



 1 t≥0
H (t) =
 0 t<0

La función de Heaviside con salto en a > 0 está dada por Ha (t) = H (t − a), con
t ∈ R.

Observación 4.5.2. Para las funciones anteriores, se tiene:


Z ∞
1
L [H (t)] (s) = H (t) e−st dt = , s>0
0 s
y para todo a > 0:
Z ∞
L [Ha (t)] (s) = Ha (t) e−st dt
Z0 ∞
= e−st dt
a
e−as
=
s
para s > 0.

Observación 4.5.3. Note que si a < b la función:

f (t) = Ha (t) − Hb (t)

corresponde a la función 
 1 t ∈ [a, b[
f (t) =
 0 t ∈ [a, b[c

246
Apuntes Mat023 (versión preliminar actualizada 23-05-2014)

Observación 4.5.4. Para introducir la segunda propiedad de traslación de la transformada


de Laplace, requerimos truncar una función. Más precisamente, consideremos f : R → R
una función y a > 0. Entonces, Ha fa : R → R es la función definida como:

 f (t − a) t ≥ a
(Ha fa ) (t) =
 0 t<a

Note que (Ha fa ) (t) = f (t − a) H (t − a).


Tenemos ası́ el siguiente teorema, que muestra que multiplicar por una función escalón
unitario tiene el efecto de multiplicar su transformada por una función exponencial:

Teorema 4.5.1 (Segundo Teorema de la Traslación). Sea a > 0 y f una función con
transformada de Laplace, entonces:

L [(Ha fa ) (t)] (s) = e−as L [f (t)] (s)

ae−bs
Ejemplo 4.5.1. L [sin a (t − b) H (t − b)] (s) = e−bs L [sin at] (s) = s2 +a2
.

Ejemplo 4.5.2. Calcule L [f (t)] (s), si:



 et 0 ≤ t < 2π
f (t) =
 et + cos t t > 2π

Solución. Note que

f (t) = et + H (t − 2π) (cos t)


= et + H (t − 2π) (cos (t − 2π + 2π))
= et + H (t − 2π) (cos (t − 2π))

ası́
1 se−2πs
L [f (t)] (s) = + 2
s−1 s +1
Ejemplo 4.5.3. Calcule:
1 − e−πs/2
 
−1
L (t)
1 + s2

247
Apuntes Mat023 (versión preliminar actualizada 23-05-2014)

Solución. Notamos que

1 − e−πs/2 1 1
2
= 2 − e−πs/2 2
1+s s +1 s +1
−πs/2
= L [sin t] (s) − e L [sin t] (s)
h  π  π i
= L [sin t] (s) − L H t − sin t − (s)
2 2
h  π   π i
= L sin t − H t − sin t − (s)
2 2

luego
1 − e−πs/2
 
−1
 π
L (t) = sin t + H t − cos t
1 + s2 2
Ejemplo 4.5.4. Resolver el P.V.I. dado por

y 00 − 2y = f (t) , y (0) = y 0 (0) = 0

donde 
 t 0≤t<1
f (t) =
 t2 t>1

Solución. Usando la función salto podemos escribir la ecuación como

y 00 − 2y = t + H (t − 1) t2 − t


para poder usar el segundo teorema de la traslación manipulamos la función

t2 − t = (t − 1 + 1)2 − (t − 1 + 1)
= (t − 1)2 + 2 (t − 1) + 1 − (t − 1) − 1
= (t − 1)2 + (t − 1)

ası́
y 00 − 2y = t + H (t − 1) (t − 1)2 + H (t − 1) (t − 1)

se sigue
1 2 1
s2 − 2 L [y] (s) = 2 + e−s 3 + e−s 2

s s s
luego
1 2 1
L [y] (s) = + e−s + e−s
s2 (s2 − 2) s3 (s2 − 2) s2 (s2 − 2)

248
Apuntes Mat023 (versión preliminar actualizada 23-05-2014)

usando fracciones parciales


 
1 1 1 1
= −
s2 (s2 − 2) 2 s2 − 2 s2
y
2 1 s 1 1
= − − 3
s3 (s2 − 2) 2
2 s − 2 2s s
luego
 
1 1 1 1
2 2
= −
s (s − 2) 2 s2 − 2 s2

1 √  t 
= L √ sinh 2t − (s)
2 2 2
y
2 1 s 1 1
=− − 3
s3 (s2 − 2) 2
2 s − 2 2s s

1 √  1 t2 
= L cosh 2t − − (s)
2 2 2
se sigue
1 2 1
L [y] (s) = + e−s + e−s
s2 (s2 − 2) s3 (s2 − 2) s2 (s2 − 2)

 
1   t
= L √ sinh 2t − (s)
2 2 2

1 √  1 t2 
−s
+e L cosh 2t − − (s)
2 2 2

1 √  t 
−s
+e L √ sinh 2t − (s)
2 2 2

1 √  t 
= L √ sinh 2t − (s)
2 2 2
" !#
1 √  1 (t − 1)2
+L H (t − 1) cosh 2 (t − 1) − − (s)
2 2 2
 
1 √  (t − 1) 
+L H (t − 1) √ sinh 2 (t − 1) − (s)
2 2 2
la solución es entonces
1 √  t
y (t) = √ sinh 2t −
2 2 2
!
1  √  1 (t − 1)2
1  √  (t − 1)
+H (t − 1) cosh 2 (t − 1) − − + √ sinh 2 (t − 1) −
2 2 2 2 2 2

249
Apuntes Mat023 (versión preliminar actualizada 23-05-2014)

La Transformada de integrales de convolución

Definición 4.6.1. Sean f y g funciones seccionalmente continuas en [0, +∞). Se define la


convolución de f y g como la función:
Z t
(f ∗ g) (t) = f (t − u) g (u) du
0

Ejemplo 4.6.1. La convolución entre f (t) = t y g (t) = sin t, está dada por:
Z t
t ∗ sin t = (t − u) sin u du
0
Z t Z t
= t sin u du − u sin u du
0 0
= t − t cos t − sin t + t cos t
= t − sin t

Ejemplo 4.6.2. Calcular t ∗ t2

Solución.
Z t
2
t∗t = (t − u) u2 du
0
1 4
= t
12
Observación 4.6.1. Dentro de las propiedades algebraicas del producto de convolución,
destacaremos la propiedad de conmutatividad. En efecto, observe que:
Z t
(f ∗ g) (t) = f (t − u) g (u) du
0
Z 0
= − f (z) g (t − z) dz ←− z = t − u
t
Z t
= g (t − z) f (z) dz
0
= (g ∗ f ) (t)

otras de sus propiedades básicas son:

1. α (f ∗ g) = (αf ) ∗ g = f ∗ (αg) para α constante.

2. f ∗ (g ∗ h) = (f ∗ g) ∗ h

250
Apuntes Mat023 (versión preliminar actualizada 23-05-2014)

3. f ∗ (g + h) = (f ∗ g) + (f ∗ h)

Teorema 4.6.1. Sean f y g funciones seccionalmente continuas en [0, +∞) y de orden


exponencial β entonces:

L [(f ∗ g) (t)] (s) = L [f (t)] (s) · L [g (t)] (s)

o bien
L−1 [L [f ] (s) L [g] (s)] (t) = (f ∗ g) (t)

Observación 4.6.2. El resultado anterior, desde una perspectiva práctica se utiliza


diciendo que la transformada de Laplace inversa de un producto de transformadas es la
convolución.

Ejemplo 4.6.3. Calcule:


( )
s+1
L−1
(s − 2) (s + 1)2 + 32
2 

Solución. Note que:


s+1 1 s+1
2 2 = ·
(s − 2) (s + 1) + 32 (s − 2) (s + 1)2 + 32
2

Luego:
" #    
s+1 1 s+1
L−1  (t) = L −1
(t) ∗ L −1
(s)
(s − 2)2 (s + 1)2 + 32 (s − 2)2 (s + 1)2 + 32
= te2t ∗ e−t cos (3t)
Z t
= (t − u) e2(t−u) e−u cos (3u) du
0
1 2t 1
= te − e−t sin 3t
6 18
Ejemplo 4.6.4. Calcular
 
−1 1
L (t)
(s − 2) (s + 2) (s − 1)
Solución. Esto lo podemos calcular con fracciones parciales pero
1 1 1 1
=
(s − 2) (s + 2) (s − 1) s−2s+2s−1
= L e2t ∗ e−2t ∗ et (s)
 

251
Apuntes Mat023 (versión preliminar actualizada 23-05-2014)

donde
t
e−2t
Z
−2t 1
e ∗e2t
= e2(t−u) e−2u du = e2t −
0 4 4
y
t
1 2u e−2u
Z  
2t −2t t t−u
e ∗e ∗e = e e − du
0 4 4
−2t
e 1 1
= − et + e2t
12 3 4

se sigue
e−2t 1 t 1 2t
 
−1 1
L (t) = − e + e
(s − 2) (s + 2) (s − 1) 12 3 4
Definición 4.6.2. Sean f (t) , g (t) funciones continuas. Llamaremos ecuación de Volte-
rra a una ecuación de la forma:
Z t
x (t) = f (t) + g (t − u) x (u) du
0

Observación 4.6.3. Note que, mediante el producto de convolución, podemos escribir la


ecuación anterior como:
x (t) = f (t) + (g ∗ x) (t)

Ahora bien, si ponemos X (s) = L [x (t)] (s) , F (s) = L [f (t)] (s) y G (s) = L [g (t)] (s) y
aplicamos luego la transformada de Laplace a la ecuación anterior, obtenemos:

X (s) = F (s) + G (s) X (s)

de donde se obtiene finalmente:

F (s)
X (s) =
1 − G (s)

de donde podemos obtener


 
−1 F (s)
x (t) = L (t)
1 − G (s)

Ejemplo 4.6.5. Resuelva la ecuación integral:


Z t
2
x (t) = t + sin (t − u) x (u) du
0

252
Apuntes Mat023 (versión preliminar actualizada 23-05-2014)

Solución. Supongamos que X (s) = L [x (t)] (s). Entonces, aplicando la transformada de


Laplace a la ecuación anterior, tenemos:

2 1
X (s) = + X (s)
s3 s2 + 1

Es decir:

2 2
X (s) = +
s3 s5
2 2 4!
= 3+
s 4! s5

Por lo tanto:
   
2
−1 2 −1 4!
x (s) = L (t) + L (t)
s3 4! s5
t4
= t2 +
12

Ejemplo 4.6.6. Resolver el sistema de ecuaciones

x0 = x − y + t
y 0 = x + y + et

con las condiciones iniciales x (0) = y (0) = 0.

Solución. Pongamos X (s) = L [x (t)] (s) y Y (s) = L [y (t)] (s) entonces

1
sX = X − Y +
s2
1
sY = X +Y +
s−1

luego

2s − 1
X =
2s2
− 4s3 + 3s4 − s5
s − 12 1 1
= 2 − − 2
s − 2s + 2 s − 1 2s
s2 + 1
Y =
2s2 − 2s3 + s4
1
1 s − 32 1
= − 22 + 2
2s s − 2s + 2 2s

253
Apuntes Mat023 (versión preliminar actualizada 23-05-2014)

de donde obtenemos
1 1
x (t) = et cos t + et sin t − et − t
2 2
1 1 1 t
y (t) = t + − e (cos t − 2 sin t)
2 2 2
Observación 4.6.4. Como apéndice calcularemos la transformada de la “función” delta
de Dirac. Muy informalmente, la función delta de Dirac se describe como una función que
es cero en todas partes excepto en t = 0 y cumple además con:
Z ∞
δ (t) dt = 1
−∞
Claramente, ninguna función real puede satisfacer tales condiciones. Sin embargo, considere
la familia de funciones: 
1


, −ε < t < ε
fε (t) =
 0 , |t| > ε
Note que, para cada ε > 0 las funciones fε (t) son seccionalmente continuas, y además:
Z ∞ Z ε
fε (t) dt = fε (t) dt = 1
−∞ −ε
De este modo, podemos pensar a δ como:

δ (t) = lı́m+ fε (t)


ε→0

Definición 4.6.3. Sea a > 0. La función delta de Dirac con salto en t = a se define
como:
δa (t) = δ (t − a)

Observación 4.6.5. Notemos que


1 1
fε (t − a) = H (t − a + ε) − H (t − a − ε)
2ε 2ε
ası́
e(−a+ε)s − e−(ε+a)s
L [fε (t − a)] (s) =
2εs
se sigue

L [δ (t − a)] (s) = lı́m L [fε (t − a)] (s)


ε→0
eεs − e−εs
= e−as lı́m
ε→0 2εs
−as se + se−εs
εs
= e lı́m
ε→0 2s
−as
= e

254
Apuntes Mat023 (versión preliminar actualizada 23-05-2014)

Teorema 4.6.2. Sea a > 0, entonces:

L [δa (t)] (s) = e−as s>0

y
L [δ (t)] (s) = 1

Ejemplo 4.6.7. Resolver el P.V.I.

y 00 − y = δ2 (t)
y (0) = 0
y 0 (0) = 0

Solución. Aplicando la transformada de Laplace

s2 − 1 L [y] (s) = e−2s




ası́

e−2s
L [y] (s) = = e−2s L [sinh (t)] (s)
s2 − 1
= L [H (t − 2) sinh (t − 2)] (s)

ası́
y (t) = H (t − 2) sinh (t − 2)

Observación 4.6.6. Una aplicación interesante es estudiar la ecuación diferencial:

dI Q
L + R I + = δa (t)
dt C

que es la ecuación diferencial asociada a un circuito RLC en serie. Es decir, un circuito


eléctrico compuesto por una resistencia R [Ω], un capacitor C [F], un inductor L [H] y
una fuente de voltaje modelada por la función delta de Dirac como función forzadora.
Naturalmente, las herramientas de las ecuaciones diferenciales a coeficientes constantes no
bastan para resolver este sistema.

255
Apuntes Mat023 (versión preliminar actualizada 23-05-2014)

Ejercicios del capı́tulo

1. Sean α, β > 0. Encontrar la transformada de Laplace de:


 π
1) sin (t + α) + cos2 (βt) 2) (αt + β) e5t + t cos 2t +
4

3) t2 cos t + 2e−3t sin (2t) 4) t3 e2t cos (6t) + 7e−4t sin t cos2 t

Z t   Z t
2t d 3u 2
5) te u e cos u du 6) t cos2 (u) du
0 du 3

 
 t si t < 2  cos t si 0 ≤ t < 2π
7) f (t) = 8) f (t) =
 0 si t > 2  sin t si t > 2π


 cos t si
 0 ≤ t ≤ 2π
t
 Z
9) f (t) = sin t si 2π < t ≤ 4π 10) sinh [2 (t − u)] cosh (2u) du

 0
et si 4π < t

Z t Z t
3
11) (t − u) sin u du 12) eu−t cos (3u) du
0 0

2. Si ω > 0, calcular las transformadas de:


Z t
sin ωt sin (ωu) cos ωt − 1
1) 2) du 3)
t 0 u t

t t
1 − cosh ωt 1 − cos (ωu) 1 − cosh (ωu)
Z Z
4) 5) du 6) du
t 0 u 0 u

Z t ∞
2 2
X
7) erf (t) = √ e−u du 8) H (t − nω)
π 0 n=0

256
Apuntes Mat023 (versión preliminar actualizada 23-05-2014)

3. Encontrar la transformada inversa de las siguientes funciones:


2 3 s+1
1) 2) 3 3)
s (s + 1) (s + 1) s (s + 4)2

2s s 2s
4) 5) 6)
s (s − 7) s2 + 6s + 25 (s2 + 4)2

e−s e−3s 1
7) 8) 9)
(s2 + 1)2 (s − 1) (s + 2) s3 + 23

n
s2 e−ks s2 + 1
X  
10) 11) 12) ln
s3 + 2 3 k=1
s s (s − 3)

e−πs s
   
−2s 1 1 1
13) e + 14) arctan 15)
s2 s4 s s2 + 3

e−3s
 
s 1 1
16) 17) 18) arctan
s2 − 9 (s + 3) (s2 + 1)
2 s s

52
   
1 s
19) 20) ln 21) ln 1 + 2
(s2 + 1)3 s+1 s

4. Suponga que α, β ∈ R, n, m ∈ N. Calcular explı́citamente las siguientes convoluciones:

a) exp (αt) ∗ exp (βt)

b) tn ∗ exp (αt)

c) tn ∗ tm

d ) exp (αt) ∗ sin (βt)

e) exp (αt) ∗ cos (βt)

f ) sin (αt) ∗ sin (βt)

g) sin (αt) ∗ cos (βt)

h) cos (αt) ∗ cos (βt)

i ) tn ∗ cos (αt)

257
Apuntes Mat023 (versión preliminar actualizada 23-05-2014)

j ) tn ∗ sin (αt)

5. Escriba las transformadas inversas de las siguientes funciones en términos de convo-


luciones:

s3
a)
(s2 + 4) (s2 + 9) (s2 + 16)
s
b)
(s − 2) (s + 2) (s − 1)
     
s+1 1 1
c) ln ln 1 + 2 arctan
s+2 s s

6. Resuelva las siguientes ecuaciones integrales o integro-diferenciales.


Z t
−t
a) x (t) = e −2 cos (t − u) x (u) du
0
1 t
Z
b) x (t) = t + (t − u)3 x (u) du
6 0
Z t
c) x (t) = t + sin (t − u) x (u) du
0
Z t
d ) x (t) = sin t + cos (t − u) x (u) du
0
Z t
0
e) x + 2x + x (u) du = f (t) donde
0

 cos t si 0 ≤ t < 2π
f (t) =
 sin t si t > 2π

con la condición inicial x (0) = 1


Z t
f) sin (t − u) y (u) du = y (t) + et cos (3t)
0

7. Suponga que f, f 0 : [0, +∞[ → R Zsonlocalmente integrables y de orden exponen-


t Z u 
df
cial: Calcular la transformada de f (v) dv du y tet en términos de la
0 0 dt
transformada de f .

8. La función Gamma está definida por:


Z ∞
Γ (x) = e−u ux−1 du, x>0
0

258
Apuntes Mat023 (versión preliminar actualizada 23-05-2014)

R∞
a) Demuestre que Γ (x + 1) = 0
e−u ux dx.

b) Integrando por partes, muestre que:

Γ (x + 1) = xΓ (x)

c) Demuestre que Γ (1) = 1

d ) Usando los resultados de (b) y (c), pruebe que si n es un entero positivo,


entonces:
Γ (n + 1) = n!

e) Calcule, usando la sustitución u = st en la parte (a), demuestre que:

Γ (x + 1)
L [tx ] (s) = , s > 0, x > −1
sx+1
1
 √
f ) Se sabe que Γ 2
= π. Calcule:
n o
1) L √1t
√
2) L t
 5/2
3) L t

9. Resuelva las siguientes ecuaciones diferenciales mediante Transformada de Laplace:

a) y 00 + y = 0, y (0) = 1, y 0 (0) = 3

b) y 00 − 4y = 0, y (0) = 2, y 0 (0) = −5

c) y 00 + 4y 0 + 4y = 0, y (0) = −1, y 0 (0) = 2

d ) y 00 + 2y 0 + 2y = H (t − 3) , y (0) = y 0 (0) = 0

e) y 000 + y = 0, y (0) = y 00 (0) = 1, y 0 (0) = −1

f ) y (4) − y = 0, y (0) = y 00 (0) = 0, y 0 (0) = y 000 (0) = 1

g) y (4) − y = sinh t, y (0) = y 00 (0) = 0, y 0 (0) = y 000 (0) = 1

10. Resolver los siguientes problemas de valores iniciales

259
Apuntes Mat023 (versión preliminar actualizada 23-05-2014)

a) y 00 − 4y = f (t) donde



 t2 si 0 ≤ t < 1
f (t) = 1 − t si 1 < t < 3


et 3<t

con las condiciones iniciales y (0) = 0, y 0 (0) = 1.

b) y 00 + 4y 0 + 13y = et cos t con valores iniciales y (0) = 1, y 0 (0) = 0.

c) y 00 + y = 1 con y (1) = y 0 (1) = 1.

d)

y 00 + y = 3 sin (4t) − H (t − 2π) sin (8t)


y (0) = 1
y 0 (0) = −1

e)

y 00 + 3y 0 + 2y = 3e4t H (t − 2)
y (0) = 2
y 0 (0) = 1

f)

ty 00 + 2 (t − 1) y 0 − 2y = 0
y (0) = 0
y 0 (0) = 0

g)

ty 00 + 2 (t − 1) y 0 + (t − 2) y = 0
y (0) = 1
y 0 (0) = −1

260
Apuntes Mat023 (versión preliminar actualizada 23-05-2014)

h)

ty 00 − ty 0 + y = 0
y (0) = 0
y 0 (0) = 1

11. Resolver el siguiente sistema utilizando transformada de Laplace:

2x01 + x1 + x002 = e6t


2x1 + x02 = 0

donde x1 (0) = 1, x2 (0) = 2 y x02 (0) = −2.

12. Resolver los siguientes sistemas utilizando transformada de Laplace:



 x0 − 4x − y + t = 0
1) con x (0) = 0, y (0) = 1
 y 0 + 2x − y + et = 0


 x0 + x + y 0 + cos t = 0
2) con x (0) = 0, y (0) = 0
 y 0 + x − y + et = 0


 x0 + x + y = cos t
3) con x (0) = 1, y (0) = 2
 y 0 + x − y = sin t


 x00 + y 00 + y 0 = t2
4) con x (0) = 1, x0 (0) = 0, y (0) = 0, y 0 (0) = 1
 x00 − y 00 = 2t

13. Calcular la transformada de Laplace de f (t) = abtc donde btc es la parte entera de t
y a ∈ R+ y con esa informacion resolver la ecuación

y (t + 2) − 3y (t + 1) + 2y (t) = 0

si se sabe que y (t) es constante e igual a 0 en el intervalo [0, 1[ y es constante e igual


a 1 en [1, 2[.

261
Apuntes Mat023 (versión preliminar actualizada 23-05-2014)

14. Resolver el problema de valores iniciales

y 00 + ty 0 − 2y = 4
y (0) = −1
y 0 (0) = 0

15. Resolver la ecuación diferencial


d2 y dy
+ 2t − 4y = 2 − 2t
dt2 dt
con y (0) = 0 y y 0 (0) = 1.

16. Muestre que si f : R → R es una función periodica de periodo T > 0 entonces


Z T
1
L [f (t)] (s) = e−st f (t) dt
1 − e−sT 0
RT R +∞
Ind.: L [f (t)] (s) = 0 f (t) e−st dt + T f (t) e−st dt y cambie variable.

17. Calcular la transformada de las funciones f (t) = |sin ωt| y f (t) = t − btc

18. Calcular las transformadas de las funciones de la figura

262
Capı́tulo 5 : Series de Fourier

Definiciones

El espacio de las funciones continuas en un intervalo [a, b] a valores reales representado


por C [a, b], es un espacio de dimensión infinita, pues para todo m ∈ N se cumple

1.x.x2 , . . . , xm ⊂ C [a, b]


y el conjunto {1.x.x2 , . . . , xm } es linealmente independiente. Sabemos que si un espacio V


tiene una base B = {v1 , v2 , . . . , vm } entonces todo elemento del espacio se puede escribir
como combinación lineal, esto es,
m
X
v= α j vj
j=1

al ser un espacio de dimensión infinita, debemos esperar que los elementos sean mas
complicados que un polinomio, por ejemplo sabemos que ex no es un polinomio. Es natural
preguntarse entonces, si todos los elementos se pueden escribir como una “combinación
lineal infinita” de los elementos de este conjunto, esto es, una serie de la forma
+∞
X
f (x) = ak xk = a0 + a1 x + a2 x2 + · · ·
k=0

lo que llamamos serie de Taylor. Sabemos de Mat022 que las funciones que se pueden
escribir de esta forma son funciones de clase C ∞ en su intervalo de convergencia, luego
no es de esperar que toda función continua se pueda escribir como una serie de Taylor
¿existirá otra forma de escribir estas funciones en forma de serie? La respuesta a esta
pregunta es afirmativa y viene dada por las series de Fourier, en algún sentido que vamos a
especificar en breve, toda función continua f ∈ C [a, b] se puede escribir en la forma
+∞   X +∞  
a0 X 2nπx 2nπx
+ an cos + bn sin
2 n=1
b−a n=1
b−a
para elecciones adecuadas de las constantes a0 , an y bn .

El espacio SC [a, b]

Definición 5.1.1. Una función a valores reales f es llamada función seccionalmente


continua en [a, b] si:

263
Apuntes Mat023 (versión preliminar actualizada 23-05-2014)

1. f esta definida y es continua en todos los puntos de [a, b] salvo quizás un número
finito de ellos.

2. Los lı́mites

f x+

0 = lı́m f (x)
x→x+
0

f x−

0 = lı́m f (x)
x→x−
0

existen en cada punto de [a, b] (note que en los extremos solo un lı́mite es relevante).

Z b
Observación 5.1.1. Si f ∈ SC [a, b] entonces f (x) dx existe y es independiente de
a
los valores (si los toma) de la función en los puntos de discontinuidad. En particular, si
f, g ∈ SC [a, b] son iguales salvo en los puntos de discontinuidad entonces
Z b Z b
f (x) dx = g (x) dx
a a

diremos entonces que dos funciones en SC [a, b] son iguales si f (x) = g (x) salvo quizás en
los puntos de discontinuidad.

Observación 5.1.2. Si α ∈ R, f, g ∈ SC [a, b] entonces αf , f + g y f g son funciones en


SC [a, b].

Observación 5.1.3. C [a, b] ⊆ SC [a, b].

Definición 5.1.2. Sea V un espacio vectorial real. Diremos que la función h·, ·i : V ×V −→
R es un producto interior si cumple:

1. ∀f ∈ V , hf, f i ≥ 0, Además hf, f i = 0 ⇔ f = 0

2. ∀f, g ∈ V , hf, gi = hg, f i

3. ∀α ∈ R, ∀f ∈ V , hαf, gi = α hf, gi

4. ∀f, g, h ∈ V , hf + g, hi = hf, hi + hg, hi

264
Apuntes Mat023 (versión preliminar actualizada 23-05-2014)

Ejemplo 5.1.1. En V = Rn , la función h·, ·i : V × V −→ R definida por


n
X
< x, y >= xi y i
i=1

es un producto interior.

Ejemplo 5.1.2. En SC[a, b] la función h·, ·i : SC[a, b] × SC[a, b] −→ R definida por


Z b
(f, g) → hf, gi = f (x)g(x) dx
a

es un producto interior. Llamaremos a este producto, el producto usual en SC[a, b].

Ejemplo 5.1.3. En C[a, b], si ρ ∈ C [a, b] es tal que ρ (x) > 0 para x ∈ [a, b] la función
h·, ·i : C[a, b] × C[a, b] −→ R definida por
Z b
(f, g) → hf, gi = ρ (x) f (x)g(x) dx
a

es un producto interior. Este producto es llamado producto con peso ρ.

En todo espacio vectorial con producto interior es posible definir una norma

k · k : V −→ R

como
p
kf k = hf, f i

y esta a su vez, nos permite definir una función distancia en el espacio V mediante la
fórmula

d(f, g) = kf − gk

Teorema 5.1.1. Sea (V, h·, ·i) un espacio vectorial real con producto interior, para toda
f, g ∈ V se tiene
p p
|hf, gi| ≤ hf, f i hg, gi

265
Apuntes Mat023 (versión preliminar actualizada 23-05-2014)

Ejemplo 5.1.4. Si f (x) = ex , g(x) = e−x son funciones en C [0, 1], calcular d (f, g), con
la distancia inducida por el producto usual del ejemplo.
Tenemos
p
d (f, g) = hf − g, f − gi
s
Z 1
= (f (x) − g (x))2 dx
0
s
Z 1
= (ex − e−x )2 dx
0

= sinh 2 − 2

Proposición 5.1.1. Propiedades básicas del producto interior h·, ·i en un espacio vectorial
real V :

1. ∀f, g ∈ V, ∀α ∈ R, hf, αgi = α hf, gi

2. ∀f, g, h ∈ V, hf, g + hi = hf, gi + hf, hi

3. ∀fk , gj ∈ V, con k = 1, 2, . . . , n y j = 1, 2, . . . , m y ∀αk , βj ∈ R, con k = 1, 2, . . . , n y


j = 1, 2, . . . , m * n +
X m
X n X
X m
αk fk , βj gj = αk βj hfk , gj i
k=1 j=1 k=1 j=1

Definición 5.1.3. Sea V un espacio con producto interior < ·, · >.

1. Diremos que f y g en V son ortogonales si < f, g >= 0

2. Diremos que {fk }k∈I es un conjunto ortogonal si < fk , fj >= 0, para k 6= j

3. Diremos que {fk }k∈I es un conjunto ortonormal si es ortogonal y la norma de cada


elemento es 1

266
Apuntes Mat023 (versión preliminar actualizada 23-05-2014)

nπx
 
Ejemplo 5.1.5. En SC[0, L], la familia sin L
, n ∈ N es ortogonal.
En efecto, si n 6= m entonces
Z  nπx   mπx 
sin sin dx
L L
 
L (m − n) πx (m + n) πx
= (m + n) sin + (n − m) sin
2π (m2 − n2 ) L L

(usar prostaféresis) luego


Z L  nπx   mπx 
sin sin dx
0 L L
  L
L (m − n) πx (m + n) πx
= (m + n) sin + (n − m) sin
2π (m2 − n2 ) L L
 0
L (m − n) πL (m + n) πL
= (m + n) sin + (n − m) sin
2π (m2 − n2 ) L L
L
= ((m + n) sin [(m − n) π] + (n − m) sin [(m + n) π])
2π (m2 − n2 )
= 0

Ejemplo 5.1.6. En SC[a, b] la familia


           
2πx 2πx 4πx 4πx 6πx 6πx
{1, cos , sin , cos , sin , cos , sin ,...}
b−a b−a b−a b−a b−a b−a
=
     
2nπx 2nπx
1, cos , sin con n ∈ N
b−a b−a
es una familia ortogonal.

Ejemplo 5.1.7. Si (V, h·, ·i) es un espacio con producto interior y {u1 , u2 , u3 } ⊆ V es un
conjunto ortonormal, determine la norma del vector

u1 − u2 + u3

Solución. Usando las propiedades del producto punto y la ortogonalidad

ku1 − u2 + u3 k2 = hu1 − u2 + u3 , u1 − u2 + u3 i
= ku1 k2 + ku2 k2 + ku3 k2
= 3

267
Apuntes Mat023 (versión preliminar actualizada 23-05-2014)

(los vectores tienen norma 1) entonces



ku1 − u2 + u3 k = 3
n o
Ejemplo 5.1.8. Muestre que el conjunto B = √12π , cos
√ x , sin
π
√ 2x
π
es ortonormal en C [−π, π]
y calcular
2 !1/2
Z π 
1 cos x sin 2x
√ − √ + √ dx
−π 2π π π

Solución.
  Z π
1 cos x cos x
√ , √ = √ dx
2π π −π 2π
Z π
1
= √ cos xdx
2π −π
π
1
= √ sin (x) = 0
2π −π

y   Z π
1 sin x sin 2x
√ , √ = √ dx = 0
2π π −π 2π
(función impar)   Z π
sin 2x cos x sin 2x cos x
√ , √ = dx = 0
π π −π π
función impar. Es fácil ver que

1 1
√ = √ k1k = √2π = 1
2π 2π 2π
Z π 1/2
sin 2x
√ = √1 2
sin 2xdx
π π −π
 Z π 1/2
1 1 − cos 4x
= √ 2 dx
π 0 2
1 √ 1/2
= √ π =1
π
Z π 1/2
cos x 1 2
√ = √ cos xdx
π π −π
 Z π 1/2
1 2
= √ 2 cos xdx
π 0
 Z π 1/2
1 1 + cos 2x
= √ 2 dx
π 0 2
= 1

268
Apuntes Mat023 (versión preliminar actualizada 23-05-2014)

usando el ejercicio anterior


2 !1/2
Z π
1 cos x sin 2x √
√ − √ + √ dx = 3
−π 2π π π

Teorema de la mejor aproximación

Ahora vamos a analizar cuando tenemos familia ortonormales como se logra la mejor
aproximación de un elemento en un espacio dado. La idea es obtener una aproximación
con distancia cero.

Teorema 5.1.2. Sea {fk }N


k=1 un conjunto ortogonal en V . Si λk ∈ R, ∀k = 1, . . . N

entonces 2
XN N
X
λ f = |λk |2 kfk k2

k k

k=1 k=1
Demostración.
N 2 * N N
+
X X X
λk f k = λk f k , λj f j



k=1 k=1 j=1
N X
X N
= λk λj hfk , fj i
k=1 j=1
N
X
= λ2k kfk k2
k=1

Note que si el conjunto {fk }N


k=1 es ortogonal (y funciones no nulas) en V entonces es

linealmente independiente
N
X
λk fk = θV
k=1
implica
* N +
X
0 = hθV , fi i = λk fk , fi
k=1
N
X
= λk hfk , fi i
k=1
= λi hfi , fi i

269
Apuntes Mat023 (versión preliminar actualizada 23-05-2014)

de donde obtenemos λi = 0.
Sea f ∈ V dado, ¿Cuál es la mejor aproximación que podemos hacer en V mediante un
elemento de de la forma
N
X
λk fk
k=1

esto es un elemento del subespacio generado por {f1 , f2 , ..., fN }

S = h{f1 , f2 , ..., fN }i

donde {fk } es un conjunto ortonormal.

N
!2
N
2
X X
d f, λk f k = f − λk f k


k=1 k=1
N
X N
X
= kf k2 − 2 λk hf, fk i + |λk |2 kfk k2
k=1 k=1
XN XN
= kf k2 − 2 λk hf, fk i + |λk |2
k=1 k=1
N
X N
X
= kf k2 − hf, fk i2 + (hf, fk i − λk )2
k=1 k=1

se sigue, independiente de los escalares λ1 , λ2 , . . . , λN

! v
N
X
u
u N
X
2
d f, λk f k ≥ kf k −
t hf, fk i2
k=1 k=1

luego el mı́nimo valor de esta distancia corresponde a


v
u
u XN
tkf k2 − hf, fk i2
k=1

que se alcanza en la combinación lineal de la forma

N
X
hf, fk i fk
k=1

se sigue que

270
Apuntes Mat023 (versión preliminar actualizada 23-05-2014)

N
X
hf, fk i fk
k=1

es la mejor aproximación de f ∈ V en el espacio.

S = h{f1 , f2 , ..., fN }i

! v
N
X
u
u N
X
mı́n d f, λk fk = tkf k2 − < f, fk >2
(λ1 ,...,λN )∈Rn
k=1 k=1

Teorema 5.1.3. Sea {fk }N


k=1 un conjunto ortonormal en un espacio vectorial V con

producto interior, f ∈ V y S = h{f1 , f2 , . . . , fN }i se tiene


N
!
X
d (f, S) = mı́n d f, λk f k
(λ1 ,...,λN )∈Rn
k=1
v
u
u N
X
2
= tkf k − hf, fk i2
k=1

y esta distancia mı́nima se logra para λi = hf, fi i con i = 1, 2, . . . , N . Es decir, el elemento


de S a menor distancia de f es
N
X
hf, fk i fk
k=1

de este resultado obtenemos:

Teorema 5.1.4 (Desigualdad de Bessel). Si {fk }N


k=1 es un conjunto ortonormal en V

entonces ∀f ∈ V
N
X
hf, fk i2 ≤ kf k2
k=1

Demostración. De los cálculos anteriores


N
!2 N N
X X X
0 ≤ d f, λk fk = kf k2 − hf, fk i2 + (hf, fk i − λk )2
k=1 k=1 k=1

271
Apuntes Mat023 (versión preliminar actualizada 23-05-2014)

si tomamos hf, fk i − λk = 0 entonces


N
X
2
0 ≤ kf k − hf, fk i2
k=1

que es la desigualdad deseada.

Teorema 5.1.5. Si {fk }∞


k=1 es un conjunto ortonormal en V entonces para cada f en V

se cumple
lı́m hf, fk i = 0
k→∞

Demostración. Por la desigualdad de Bessel se tiene ∀N ∈ N


N
X
hf, fk i2 ≤ kf k2
k=1

P+∞
luego la serie k=1 hf, fk i2 converge (sucesión de sumas parciales creciente y acotada
superiormente) luego
lı́m hf, fk i = 0
k→∞

n o
Ejemplo 5.1.9. Muestre que el conjunto √1 , sin
√x es ortonormal en C [−π, π] y deter-
2π π

minar el elemento de la forma


α sin x
√ +β √
2π π
(con α, β ∈ R) mas cercano a f (x) = x. ¿Cuál es la mı́nima distancia?.

Solución. Primero calculamos las normas


Z π 1/2
1 1
√ = dx
−π 2π

 Z π 1/2
1
= dx
2π −π
 Z π 1/2
1
= dx
2π −π
= (1)1/2
= 1

272
Apuntes Mat023 (versión preliminar actualizada 23-05-2014)

y
Z π 1/2
sin2 x

sin x
√ = dx
π
−π π
 Z π 1/2
1 1 − cos 2x
= dx
π −π 2
 1/2
1
= π
π
= 1

que son ortogonales es fácil


  Z π
1 sin x 1
√ , √ = √ sin xdx
2π π 2π −π
= 0

(por imparidad). El elemento mas cercano es


   
1 1 sin x sin x
g (x) = √ , x √ + √ , x √
2π 2π π π
donde
  Z π
1 x
√ ,x = √ dx = 0
2π 2π
Z−π
π √
 
sin x x sin x
√ ,x = √ dx = 2 π
π −π π
ası́
g (x) = 2 sin x

y la distancia mı́nima es
√ 2
q
d = kxk2 − 02 − 2 π

donde Z π
2 2
kxk = x2 dx = π 3
−π 3
entonces r
2 3
d= π − 4π
3
Observación 5.1.4. El conjunto
( )
2kπx 2kπx
 
1 cos b−a
sin b−a
B= ,  ,
2kπx
2kπx
 con k ∈ N
k1k cos b−a
sin
b−a

273
Apuntes Mat023 (versión preliminar actualizada 23-05-2014)

es un conjunto ortonormal en SC[a, b], notemos que


Z b 1/2
p
dx = (b − a)
a

!
b 1 + cos 4kπx
b
Z   Z
2kπx b−a
cos2 dx = dx
a b−a a 2
1 b
   
b−a
Z
4kπx
= + cos dx
2 2 a b−a
     b
b−a 1 4kπx b − a
= + sin
2 2 b−a 4kπ a
        
b−a 1 4kπb 4kπa b−a
= + sin − sin
2 2 b−a b−a 4kπ
 
b−a
=
2

donde hemos usado la fórmula de prostaféresis


   
x−y x+y
sin (x) − sin (y) = −2 sin cos
2 2

para ver    
4kπb 4kπa
sin − sin =0
b−a b−a
ahora bien
Z b   Z b  
2 2kπx 2kπx 2
sin dx = 1 − cos dx
a b−a a b−a
 
b−a
= (b − a) −
2
 
b−a
=
2

ası́

k1k = b−a
  r
cos 2kπx =
b−a
b−a 2
  r
sin 2kπx =
b−a
b−a 2

274
Apuntes Mat023 (versión preliminar actualizada 23-05-2014)

Definición 5.1.4. Sea f ∈ SC [a, b]. Llamaremos serie de Fourier de f a la serie de la


forma ∞   X ∞  
a0 X 2kπ 2kπ
+ ak cos x + bk sin x
2 k=1
b−a k=1
b − a
donde los coeficientes son dados por la mejor aproximación.

Calculemos los coeficientes en forma explı́cita:

hf (x) , 1i
* ∞   X ∞   +
a0 X 2kπ 2kπ
= + ak cos x + bk sin x ,1
2 k=1
b−a k=1
b−a
*∞   + *X ∞   +
Da E 2kπ 2kπ
0
X
= ,1 + ak cos x ,1 + bk sin x ,1
2 k=1
b−a k=1
b−a
∞     X ∞    
a0 X 2kπ 2kπ
= h1, 1i + ak cos x ,1 + bk sin x ,1
2 k=1
b − a k=1
b − a
a0
= (b − a)
2

esto es Z b
2
a0 = f (x) dx
b−a a
de manera similar
       
2nπ 2nπ 2nπ
f, cos x = an cos x , cos x
b−a b−a b−a

ası́
2nπ


f, cos b−a x
an =
2nπ
 2nπ

cos b−a x , cos b−a x
Z b  
2 2nπ
= f (x) cos x dx para n ∈ N
b−a a b−a

y
2nπ


f, sin b−a x
bn =
2nπ
 2nπ

sin b−a x , sin b−a x
Z b  
2 2nπ
= f (x) sin x dx para n ∈ N
b−a a b−a

275
Apuntes Mat023 (versión preliminar actualizada 23-05-2014)

Teorema 5.1.6. Si f ∈ SC [a, b] entonces


!
N   XN 
a0 X 2kπ 2kπ
lı́m f (x) − + ak cos x + bk sin x =0

N →+∞ 2 k=1
b−a b−a k=1

si a0 , ak y bk corresponden a los coeficientes de Fourier de f .

Podemos entonces escribir


∞   X ∞  
a0 X 2kπ 2kπ
f (x) = + ak cos x + bk sin x
2 k=1
b−a k=1
b − a

pero la convergencia de la serie no es necesariamente al evaluar en los puntos, sino en el


sentido de la distancia definida.

Ejemplo 5.1.10. Determinar la serie de Fourier de f (x) = x en C [−π, π]

Solución. En este intervalo la serie tiene la forma


∞ ∞
a0 X X
+ ak cos (kx) + bk sin (kx)
2 k=1 k=1

note que al ser una función impar a0 = ak = 0 y

1 π
Z
bk = x sin (kx) dx
π −π
2 π
Z
= x sin (kx) dx
π 0
2
= 2
((sin kx − kx cos kx)|π0 )
πk
2  k

= −k (−1)
πk 2
2 (−1)k+1
=
πk

ası́

!
X 2 (−1)k+1
x= sin (kx)
k=1
πk

276
Apuntes Mat023 (versión preliminar actualizada 23-05-2014)

N
!
X 2 (−1)k+1
en media. Veamos algunos gráficos de la aproximación sin (kx)
k=1
πk
   
P3 2(−1)k+1 P6 2(−1)k+1
k=1 πk
sin (kx) k=1 πk
sin (kx)

P15  2(−1)k+1  P60  2(−1)k+1 


k=1 πk
sin (kx) k=1 πk
sin (kx)

Ejemplo 5.1.11. Determinar la serie de Fourier de

f (x) = ex

para x ∈ [−1, 1] .

Solución. La serie de Fourier de f en este intervalo es de la forma


∞   X ∞  
a0 X 2nπx 2nπx
+ an cos + an sin
2 n=1
2 n=1
2
∞ ∞
a0 X X
= + an cos (nπx) + an sin (nπx)
2 n=1 n=1

277
Apuntes Mat023 (versión preliminar actualizada 23-05-2014)

donde
Z 1
2
a0 = ex dx = 2 sinh 1
2 −1
1
(2 sinh 1) (−1)n
Z
an = ex cos (nπx) dx = para n ∈ N
−1 π 2 n2 + 1
y
1
(2nπ sinh 1) (−1)n+1
Z
bn = ex sin (nπx) dx = para n ∈ N
−1 π 2 n2 + 1
se sigue
∞ ∞
x
X (2 sinh 1) (−1)n X (2nπ sinh 1) (−1)n+1
e = (sinh 1) + cos (nπx) + sin (nπx)
n=1
π 2 n2 + 1 n=1
π 2 n2 + 1
∞ ∞
!
X 2 (−1)n X 2nπ (−1)n+1
= (sinh 1) 1 + cos (nπx) + sin (nπx)
n=1
π 2 n2 + 1 n=1
π 2 n2 + 1

ası́ por ejemplo, la gráfica de la aproximación


10 10
!
X 2 (−1)n X 2nπ (−1)n+1
(sinh 1) 1 + 2 n2 + 1
cos (nπx) + 2 n2 + 1
sin (nπx)
n=1
π n=1
π
es:

en rojo esta la gráfica de la exponencial.

Teorema 5.1.7 (Identidad de Parseval). Si f ∈ SC [a, b] entonces


Z b ∞ ∞
2 2 a20 X 2 X 2
f (x) dx = + an + bn
b−a a 2 n=1 n=1

donde Z b  
2 2nπ
an = f (x) cos x dx para n ∈ N∪ {0}
b−a a b−a

278
Apuntes Mat023 (versión preliminar actualizada 23-05-2014)

y Z b  
2 2nπ
bn = f (x) sin x dx para n ∈ N
b−a a b−a

Ejemplo 5.1.1. Calcular la serie de Fourier de f (x) = |x| para x ∈ [−1, 1] y usando
Parseval calcular ∞
X 1
n=1
(2n + 1)4

Convergencia Puntual de series de Fourier

Si bien hemos enunciado que la serie de Fourier converge en norma la función dada, se
presenta la siguiente situación: Determinemos la serie de Fourier de

 1 si x ∈ [0, π]
f (x) =
 −1 si x ∈ [−π, 0[

se tiene:
Z π
1
a0 = √ f (x) dx = 0 (la función es impar)
2π −π
Z π
1
an = √ f (x) cos (nx) dx = 0 (la función f (x) cos (nx) es impar) para cada n ∈ N
π −π
Z π
1 1
bn = √ f (x) sin (nx) dx = − √ (2 (−1)n − 2) para n ∈ N
π −π πn

se sigue que
∞  
X 1 n sin (nx)
f (x) = − √ (2 (−1) − 2) √ (en media)
n=1
πn π
∞  
2X 1 n
= − ((−1) − 1) sin (nx) (en media)
π n=1 n

pero
f (0) = 1

pero si evaluamos
∞  
2X 1 n
− ((−1) − 1) sin (nx)
π n=1 n

279
Apuntes Mat023 (versión preliminar actualizada 23-05-2014)

en x = 0 nos queda
∞  
2X 1 n
− ((−1) − 1) sin (n0) = 0
π n=1 n
luego el valor al cual converge la serie no es el valor de la función en el punto. Como se ve
en las siguiente gráficas, los combinaciones trigonométricas
N  
2X 1 n
− ((−1) − 1) sin (nx)
π n=1 n

cuando N crece se va pareciendo más a la gráfica pero todos estas funciones pasan por el 0.
El siguiente teorema nos dice a que valor converge la serie de Fourier
∞   X ∞  
a0 X 2kπ 2kπ
+ ak cos x + bk sin x
2 k=1
b−a k=1
b−a

de f ∈ SC [a, b] cuando evaluamos en un x0 ∈ [a, b].

Teorema 5.2.1 (Convergencia puntual). Sea f, f 0 ∈ SC [a, b] entonces la serie de Fourier


converge en todos los puntos del intervalo [a, b], además si x0 ∈ [a, b] la serie de Fourier
converge a

f x+
 
0 + f x0
2
cuando x0 ∈ ]a, b[ y a
f (b− ) + f (a+ )
2
si x0 = a o x0 = b.

Por convergencia puntual se entiende


N     !
a0 X 2kπ 2kπ
lı́m + ak cos x0 + bk sin x0 = lı́m SN (x0 )
N →∞ 2 k=1
b−a b−a N →∞


f x+
 
0 + f x0
=
2

(no es la convergencia en norma, la cual esta garantizada sin mirar la derivada).

280
Apuntes Mat023 (versión preliminar actualizada 23-05-2014)

Ejemplo 5.2.1. Sabemos que


∞ ∞
!
x
X 2 (−1)n X 2nπ (−1)n+1
e = (sinh 1) 1 + cos (nπx) + sin (nπx) (en media)
n=1
π 2 n2 + 1 n=1
π 2 n2 + 1

Usando esa serie, determinar el valor de



X (−1)n
n=1
π 2 n2 + 1

Solución. Como
∞ ∞
!
n n+1
X 2 (−1) X 2nπ (−1)
ex = (sinh 1) 1 + 2 n2 + 1
cos (nπx) + 2 n2 + 1
sin (nπx)
n=1
π n=1
π

evaluando en x = 0 que es un punto de continuidad se obtiene


+∞
!
X 2 (−1)n
1 = (sinh 1) 1 +
n=1
π 2 n2 + 1

ası́
+∞
(−1)n
  X
1 1
−1 =
2 sinh 1 n=1
π 2 n2 + 1

Note que si f, f 0 ∈ SC [a, b] la serie de Fourier


∞   X ∞  
a0 X 2kπ 2kπ
+ ak cos x + bk sin x
2 k=1
b − a k=1
b − a

converge en cada punto del intervalo, si denotamos por


∞   X ∞  
a0 X 2kπ 2kπ
g (x) = + ak cos x + bk sin x
2 k=1
b−a k=1
b−a

del resultado anterior se tiene que g (x0 ) = f (x0 ) en los puntos en los cuales f es continua,
2kπ 2kπ
 
pero se obtiene una propiedad adicional, dada que las funciones 1, cos b−a x , sin b−a x
son periódicas de periodo (b − a) se sigue que
∞   X ∞  
a0 X 2kπ 2kπ
g (x) = + ak cos x + bk sin x
2 k=1
b − a k=1
b − a

es una función periódica de periodo (b − a) y ası́ g esta bien definida en todo R cumpliendo

g : R→R
x → g (x) = fe(x)

281
Apuntes Mat023 (versión preliminar actualizada 23-05-2014)

donde fe : R → R es una función periódica de periodo (b − a) tal que


+ −
 
f x 0 + f x 0
fe(x0 ) = para x0 ∈ ]a, b[
2
f (b− ) + f (a+ )
= para x0 = a, b
2

Ejemplo 5.2.2. La serie de Fourier de



 1 si x ∈ [0, π]
f (x) =
 −1 si x ∈ [−π, 0[

es ∞  
2X 1 n
− ((−1) − 1) sin (nx)
π n=1 n
entonces
∞  
2X 1 n
1 = − ((−1) − 1) sin (nx) para x ∈ ]0, π[
π n=1 n
∞  
2X 1 n
−1 = − ((−1) − 1) sin (nx) para x ∈ ]−π, 0[
π n=1 n
1 + (−1)
= 0 para x = 0, −π, π
2

y por periodicidad podemos decir por ejemplo que


∞  
2X 1 n
  π 
− ((−1) − 1) sin n 2π +
π n=1 n 4
∞  
2X 1  πn 
= − ((−1)n − 1) sin
π n=1 n 4
= 1

Ejemplo 5.2.3. Determinar la serie de Fourier de f ∈ C [−2, 2] definida por



 1 si 0 ≤ x ≤ 2
f (x) =
 −1 si −2 ≤ x < 0

Determine el valor al cual converge la serie en x = 6 y bosquejar un gráfico de la serie


poniendo especial cuidado en los puntos de discontinuidad.

282
Apuntes Mat023 (versión preliminar actualizada 23-05-2014)

Solución. La función es impar, esto permitirá simplifica los cálculos, la serie de Fourier
tiene la forma
∞ 
 X ∞  
a0 X 2nπx 2nπx
F (x) = + an cos + bn sin
2 n=1
4 n=1
4
∞ ∞
a0 X  nπx  X  nπx 
= + an cos + bn sin
2 n=1
2 n=1
2

donde

2 2
Z
a0 = f (x) dx = 0
4 −2
1 2
Z  nπx 
an = f (x) cos dx = 0
2 −2 2

1 2
Z  nπx 
bn = f (x) sin dx
2 −2 2
Z 2  nπx 
= sin dx
0 2
2  nπx  2
= − cos
nπ 2 0
2
= − (cos (nπ) − 1)

(−1)n − 1
= −2

n+1
(−1) +1
= 2

ası́

!
X (−1)n+1 + 1  nπx 
F (x) = 2 sin
n=1
nπ 2
por los teoremas vistos en clases, la serie es periódica de periodo 4 se sigue que

1 + (−1)
F (6) = F (2) = =0
2

la gráfica de la serie es:

283
Apuntes Mat023 (versión preliminar actualizada 23-05-2014)

1.

Series de Fourier de senos y cosenos

Definición 5.3.1. Si f : [0, L] → R es una función en SC [0, L], llamaremos:

1. Extensión par de f a la función



 f (x) si x ∈ [0, L]
fp (x) =
 f (−x) si x ∈ [−L, 0]

2. Extensión impar de f a la función



 f (x) si x ∈ [0, L]
fI (x) =
 −f (−x) si x ∈ [−L, 0[

Observación 5.3.1. fp ∈ SC [−L, L] es una función par tal que

fp (x) = f (x) para x ∈ [0, L]

y fI (x) ∈ SC [−L, L] es una función impar tal que

fI (x) = f (x) para x ∈ [0, L]

Al desarrollar en serie de Fourier la función fp en [−L, L] se tiene


∞   X ∞  
a0 X kπx kπx
fp (x) = + ak cos + bk sin
2 k=1
L k=1
L

284
Apuntes Mat023 (versión preliminar actualizada 23-05-2014)

donde

1 L
Z
a0 = fp (x) dx
L −L
1 L
Z  
kπx
ak = fp (x) cos dx para k ∈ N
L −L L
1 L
Z  
kπx
bk = fp (x) sin dx para k ∈ N
L −L L

notemos que al ser fp una función par fp (x) cos kπx kπx
 
L
es par y fp (x) sin L
es impar,
luego

1 L 2 L
Z Z
a0 = fp (x) dx = fp (x) dx
L −L L 0
2 L
Z
= f (x) dx
L 0

1 L
Z  
kπx
ak = fp (x) cos dx para k ∈ N
L −L L
2 L
Z  
kπx
= fp (x) cos dx para k ∈ N
L 0 L
2 L
Z  
kπx
= f (x) cos dx para k ∈ N
L 0 L

y
Z L  
1 kπx
bk = fp (x) sin dx para k ∈ N
L −L L
= 0

ası́ la serie es de la forma


∞  
a0 X kπx
fp (x) = + ak cos
2 k=1
L

con

2 L
Z
a0 = f (x) dx
L 0
2 L
Z  
kπx
ak = f (x) cos dx para k ∈ N
L 0 L

285
Apuntes Mat023 (versión preliminar actualizada 23-05-2014)

de manera similar, si al desarrollar en serie de Fourier la función fI en [−L, L] se tiene


∞   X ∞  
a0 X kπx kπx
fI (x) = + ak cos + bk sin
2 k=1
L k=1
L

donde
1 L
Z
a0 = fI (x) dx
L −L
1 L
Z  
kπx
ak = fI (x) cos dx para k ∈ N
L −L L
1 L
Z  
kπx
bk = fI (x) sin dx para k ∈ N
L −L L
notemos que al ser fI una función impar fI (x) cos kπx kπx
 
L
es impar y fI (x) sin L
es par,
luego Z L
1
a0 = fI (x) dx = 0
L −L
y
1 L
Z  
kπx
ak = fI (x) cos dx para k ∈ N
L −L L
= 0 para k ∈ N

y
1 L
Z  
kπx
bk = fI (x) sin dx para k ∈ N
L −L L
2 L
Z  
kπx
= fI (x) sin dx para k ∈ N
L 0 L
2 L
Z  
kπx
= f (x) sin dx para k ∈ N
L 0 L
ası́ la serie es de la forma ∞  
X kπx
fI (x) = bk sin
k=1
L
con Z L  
2 kπx
bk = f (x) sin dx para k ∈ N
L 0 L
como fp (x) = fI (x) = f (x) para x ∈ [0, L] se obtiene que es posible desarrollar f ∈
SC [0, L] en series de la forma
∞  
a0 X kπx
f (x) = + ak cos
2 k=1
L

286
Apuntes Mat023 (versión preliminar actualizada 23-05-2014)

donde
2 L
Z
a0 = f (x) dx
L 0
2 L
Z  
kπx
ak = f (x) cos dx para k ∈ N
L 0 L
llamada serie cosenoidal de f o serie de cosenos de f y
∞  
X kπx
f (x) = bk sin
k=1
L

donde
2 L
Z  
kπx
bk = f (x) sin dx para k ∈ N
L 0 L
llamada serie senoidal de f o serie de senos de f .

Ejemplo 5.3.1. Obtener la serie senoidal de

f (x) = cos x

en [0, π].

Solución. Hacemos uso de la extensión impar de f al intervalo [−π, π] entonces





 cos x si x ∈ [0, π]
fI (x) =


− cos (−x) si x ∈ [−π, 0[

entonces ∞ ∞
a0 X X
fI (x) = + an cos (nx) + bn sin (nx)
2 n=1 n=1

donde
1 π
Z
a0 = fI (x) = 0
π −π
Z π
1
an = fI (x) cos (nx) dx = 0
π −π
2 π
Z
bn = cos x sin (nx) dx
π 0
si n = 1 entonces Z π
2
b1 = cos x sin xdx = 0
π 0

287
Apuntes Mat023 (versión preliminar actualizada 23-05-2014)

pero

sin (x + nx) = sin x cos nx + sin nx cos x


sin (x − nx) = sin x cos nx − sin nx cos x

ası́
sin (x + nx) − sin (x − nx) = 2 cos x sin nx

luego
cos ((n + 1) x) cos ((1 − n) x)
Z
− + =2 cos x sin nxdx
n+1 1−n
luego
  π π
cos ((n + 1) x) cos ((1 − n) x)
Z
− + = 2 cos x sin nxdx
n+1 1−n
0
   0 π
cos ((n + 1) π) cos ((1 − n) π)
Z
1 1
− + − − + = 2 cos x sin nxdx
n+1 1−n n+1 1−n 0
(−1)n (−1)n π
Z
2n
+ + 2 = 2 cos x sin nxdx
n+1 n−1 n −1 0
n Z π
((−1) + 1) 2n
= 2 cos x sin nxdx
n2 − 1 0

se sigue
+∞ 
((−1)n + 1) n
 X 
2
fI (x) = sin (nx)
π n=2 n2 − 1
ası́ en [0, π] obtenemos la representación
+∞ 
((−1)n + 1) n
 X 
2
cos x = sin (nx)
π n=2 n2 − 1

la gráfica de la aproximación (en verde)


100 
((−1)n + 1) n
 X 
2
sin (nx)
π n=2 n2 − 1

288
Apuntes Mat023 (versión preliminar actualizada 23-05-2014)

es

en rojo esta la gráfica de coseno en el intervalo deseado.

Ejemplo 5.3.2. Obtener la serie cosenoidal de la función

f (x) = x

para x ∈ [0, 4] .

Solución. En este caso hacemos uso de la extensión par de f al intervalo [−4, 4] se tiene

 x

 si x ∈ [0, 4]
fp (x) =


(−x) si x ∈ [−4, 0[

entonces ∞   ∞  
a0 X 2nπx X 2nπx
fp (x) = + an cos + an sin
2 n=1
8 n=1
8
donde
2 4
Z
a0 = fp (x) dx
8 −4
1 4
Z
= xdx = 4
2 0

289
Apuntes Mat023 (versión preliminar actualizada 23-05-2014)

1 4
Z  nπx 
an = fp (x) cos dx
4 −4 4
1 4
Z  nπx 
= x cos dx
2 0 4
8 ((−1)n − 1)
=
π 2 n2
y Z 4
1  nπx 
bn = fp (x) sin dx = 0
4 −4 4
se sigue
∞ 
8 ((−1)n − 1)
X   nπx 
fp (x) = 2 + cos
n=1
π 2 n2 4
en particular en [0, 4] se tiene
∞ 
((−1)n − 1)
X   nπx 
x=2+8 cos
n=1
π 2 n2 4

Ejemplo 5.3.3. Sea f : [0, π] → R definida por f (x) = x (π − x)

1. Encontrar una función F periódica y par tal que F (x) = f (x) para x ∈ [0, π] y
desarrollar en serie de Fourier F .

2. Encontrar una función G periódica e impar tal que G (x) = f (x) para x ∈ [0, π] y
desarrollar en serie de Fourier G.

3. Use los puntos anteriores para probar


∞ ∞
X (−1)n+1 π 2 X (−1)n+1 π3
= y =
n=1
n2 12 n=1 (2n − 1)3 32

Solución. Por punto pero calculando las sumas pedidas.

1. F es la serie de cosenos

a0 X
F (x) = + an cos (nx)
2 n=1

donde
π
π2
Z
2
a0 = x (π − x) dx =
π 0 3

290
Apuntes Mat023 (versión preliminar actualizada 23-05-2014)

π
2 ((−1)n + 1)
Z
2
an = x (π − x) cos (nx) dx = −
π 0 n2
ası́ ∞
π2 X ((−1)n + 1)
F (x) = −2 2
cos (nx)
6 n=1
n
si consideramos la suma sobre los pares (en los impares los coeficientes son nulos)

π2 X 2
F (x) = −2 2 cos (2nx)
6 n=1
(2n)

π 2 X cos (2nx)
= −
6 n=1
n2
π
evaluando en 2
obtenemos
∞ π

π   π π 2 X cos 2n 2
π− = −
2 2 6 n=1
n2

π 2 X (−1)n
= −
6 n=1
n2

esto es

π2 π2 X (−1)n+1
− =
4 6 n=1
n2

π2 X (−1)n+1
=
12 n=1
n2

2. G es la serie senoidal ∞
X
G (x) = bn sin (nx)
n=1

donde
Z π
2
bn = x (π − x) sin (nx) dx
π 0
2
= − 3
(2 (−1)n − 2)
πn
−4 ((−1)n − 1)
=
πn3

ası́ ∞ 
−4 ((−1)n − 1)
X 
G (x) = sin (nx)
n=1
πn3

291
Apuntes Mat023 (versión preliminar actualizada 23-05-2014)

note que ahora en los pares los coeficientes son nulos, al considerar en los impares
tenemos ∞
8 X sin ((2n − 1) x)
G (x) =
π n=1 (2n − 1)3
π
evaluando en 2
tenemos

8 X sin (2n − 1) π2
π   
π
π− =
2 2 π n=1 (2n − 1)3

8 X (−1)n+1
=
π n=1 (2n − 1)3

pues
 π
sin (2n − 1) = (−1)n+1
2
ası́ ∞
π2 8 X (−1)n+1
=
4 π n=1 (2n − 1)3
de donde obtenemos ∞
π 3 X (−1)n+1
=
32 n=1 (2n − 1)3

Ejemplo 5.3.4. Sea f (x) = exp (− [x]) definida en [0, 2]. Obtenga su serie de Fourier de
cosenos y use la serie para calcular el valor de

X (−1)n−1
n=1
2n − 1

Obs.: [x] es la parte entera de x.

Solución. Tenemos que encontrar la serie de cosenos, usamos la extensión par


∞  
a0 X 2πnx
fp (x) = + an cos
2 n=1
4

292
Apuntes Mat023 (versión preliminar actualizada 23-05-2014)

donde
2 2
Z
a0 = fp (x) dx
4 −2
Z 2
= f (x) dx
0
Z 2
= exp (− [x]) dx
0
Z 1 Z 2
= 1dx + edx
0 1
−1
= e +1

y
2 2
Z  
2πnx
an = fp (x) cos dx
4 −2 4
Z 2  πnx 
= exp (− [x]) cos dx
0 2
Z 1  πnx  Z 2  πnx 
−1
= cos dx + e cos dx
0 2 1 2
 
1 −1 −1 1 2 1
= 2e sin πn − 2e sin πn + sin πn
πn 2 πn 2
2  πn  −1 
= − sin e −1
πn 2
luego

e−1 + 1 X 2  πn  −1
  πnx 
fp (x) = + sin 1−e cos
2 n=1
πn 2 2
evaluando en x = 0 se tiene

e−1 + 1 X 2  πn 
1 − e−1

1= + sin
2 n=1
πn 2

de donde obtenemos ∞
e−1 + 1 2 (1 − e−1 ) X sin πn
2
1− =
2 π n=1
n
luego

π 12 − 12 e−1 sin πn

X
2
=
2 (1 − e−1 ) n=1
n
luego

π X sin πn2
=
4 n=1
n

293
Apuntes Mat023 (versión preliminar actualizada 23-05-2014)

si n es par sin πn
2
es cero, se sigue
∞ π(2n−1)
π X sin 2
=
4 n=1
2n − 1
pero
π (2n − 1)
sin = (−1)n+1
2
es decir ∞
π X (−1)n+1
=
4 n=1
2n − 1

Derivación e integración de Series de Fourier

Si para x ∈ ]−π, π[

X 2 (−1)k+1
x= sin (kx)
k=1
k
es cierto que

X
1= 2 (−1)k+1 cos (kx)
k=1

para x ∈ ]−π, π[? En otras palabras ¿es posible derivar término a término una serie de
Fourier y asegurar que converge a la derivada de la función?, la respuesta es no, ya que
2 (−1)k+1 no corresponde al coeficiente de Fourier de la función, el cual tiene que cumplir

lı́m Ck = 0
k→∞

Para que sea válida la derivación término a término tenemos el siguiente resultado:

Teorema 5.4.1. Si f ∈ C [a, b] , f (a) = f (b) y f 0 ∈ SC [a, b] entonces la serie de Fourier


para f 0 puede ser obtenida derivando término a término la serie de f . La serie obtenida
converge puntualmente a f 0 (x) en los puntos en los cuales f 00 existe.

para la integración de las series tenemos el siguiente resultado:

Teorema 5.4.2. Si f ∈ SC [a, b] y


∞     
a0 X 2nπx 2nπx
f (x) = + ak cos + bk sin (media)
2 n=1
b − a b − a

294
Apuntes Mat023 (versión preliminar actualizada 23-05-2014)

entonces
x
(x − a)
Z
f (t) dt = a0
a 2

X Z x  
2nπt
+ ak cos dt
n=1 a b−a
∞ Z x  
X 2nπt
+ bk sin dt (media)
n=1 a b − a

(Esto es, es posible integrar la serie término a término)

Ejercicios del capı́tulo

1. Suponga que
∞  
X kπx
x (L − x) = ak sin
k=1
L
para x ∈ [0, L]. Haciendo uso de la propiedad de ortogonalidad de la familia
sin kπx
 
L k∈N
en C[0, L] encontrar los coeficientes de la serie.

2. Muestre que en C[a, b] = V se cumple

a) ∀f, g ∈ V
kf − gk2 + kf + gk2 = 2 kf k2 + 2 kgk2

b) Si hf, gi = hf, hi para todo f ∈ V entonces g = h.

3. Muestre que para todo f ∈ C [−π, π]


Z π
lı́m sin (nx) f (x) dx = 0
n→∞ −π

Ind.: Identidad de Parseval.

4. (Calculando Productos) Calcular para n, m ∈ N


Z π Z π
sin (mx) sin (nx) dx, sin (nx) dx
−π −π
Z π Z π
cos (nx) dx, cos (nx) cos (mx) dx
−π −π
Z π
cos (nx) sin (mx) dx
−π

295
Apuntes Mat023 (versión preliminar actualizada 23-05-2014)

¿Qué dicen estos cálculos del conjunto B = {1, cos (nx) , sin (nx) con n ∈ N} en
SC[−π, π]?

5. (De lo general a lo particular) En SC [a, b] hemos definido el producto interior


Z b
hf, gi = f (x) g (x) dx
a

y enunciado que el conjunto


     
2nπx 2nπx
C = 1, cos , sin con n ∈ N
b−a b−a

es una conjunto ortogonal. Escribir en los casos especiales SC[−π, π], SC[−L, L] y
SC[0, L]

6. (Calculando series de Fourier) Encontrar las series de Fourier para las funciones
siguientes en SC[−π, π]

a) x + sin x

b) ex

 1 si x ≤ 0
c)
 x2 si x > 0

 x si x ≤ 0
d)
 x3 si x > 0
e) sin3 x

7. (La identidad de Parseval) Utilizar la serie de Fourier de f (x) = x en [−π, π]


para calcular el valor de

X 1
k=1
k2

8. (Funciones pares e impares) Clasificar las siguientes funciones en par, impar o


ninguna de las anteriores

A) tan x
2
B) xex

296
Apuntes Mat023 (versión preliminar actualizada 23-05-2014)

x+1
C)
x−1
D) ln |x|

E) arcsin x

F) f (|x|) definida en [−1, 1] donde f : R → R es una función cualquiera.

G) x cos x − cos 2x

9. (Parte par e impar de una serie) En Mat021 se mostró que toda función f :
[−a, a] → R (donde a ∈ R+ ) se puede descomponer en su parte par mas su parte
impar, esto es
f (x) = fp (x) + fi (x)

donde
f (x) + f (−x)
fp (x) =
2
f (x) − f (−x)
fi (x) =
2
Suponiendo que no hay problemas de convergencia, encontrar las partes par e impar
de ∞
a0 X
+ (ak cos (kx) + bk sin (kx))
2 k=1

donde ak , bk son constantes.

10. (Convergencia puntual) Encontrar el desarrollo en serie de Fourier de:



 −1 −π < x < 0
f (x) =
 1 0<x<π

y mostrar que la serie evaluada en x = 0 converge a 0. Utilizando esta serie mostrar


que
π 1 1 1
= 1 − + − + ···
4 3 5 7
11. (Series de Fourier en intervalos arbitrarios) Encontrar el desarrollo en serie de
Fourier para las siguientes funciones:

A) f (x) = x para x ∈ ]−π, π[

297
Apuntes Mat023 (versión preliminar actualizada 23-05-2014)

B) f (x) = ex para x ∈ ]0, 2π[



 1 −π < x < 0
C) f (x) =
 1/2 0 < x < π

D) f (x) = |sin x| para x ∈ ]−2π, 2π[

E) f (x) = x (L − x) para x ∈ ]0, L[

12. (Series de Fourier y series numéricas) Encontrar el desarrollo en serie de Fourier


de la función
f (x) = cos (αx) para x ∈ ]−π, π[

para todos los valores de α ∈ R. Usar esta serie para demostrar que

!
1 1 X 2α
cot (απ) = −
π α k=1 k 2 − α2

siempre que α 6∈ Z.

13. (Combinaciones lineales de series) Si f, g ∈SC[a, b] son tales que


∞     
a0 X 2kπx 2kπx
f (x) = + ak cos + bk sin
2 k=1
b−a b−a

y
∞     
A0 X 2kπx 2kπx
g (x) = + Ak cos + Bk sin
2 k=1
b−a b−a

A) Determinar la serie de Fourier de αf + βg.

B) Es posible mostrar que las series



1 2 X sin (2k − 1) x
+
2 π k=1 2k − 1

y

2 X (−1)k+1
sin (kx)
π k=1 k
son las series de Fourier de las funciones

 0 −π < x < 0
f1 (x) =
 1 0<x<π

298
Apuntes Mat023 (versión preliminar actualizada 23-05-2014)

y
x
f2 (x) = para x ∈ ]−π, π[
π
respectivamente, usar estas series y el la parte anterior del ejercicio para obtener
las series de las siguientes funciones:

 1/2 −π < x < 0
1) f (x) =
 −1/2 0 < x < π

 x + π −π < x < 0
2) f (x) =
 x 0<x<π

 −x −π < x < 0
3) f (x) =
 −x + 2π 0 < x < π

14. (Ojo serie truncada) ¿Cuál es el desarrollo en serie de Fourier de f (x) = 2 +


5 cos (3x) − 4 sin (7x) en SC[−π, π]?.

15. (Para trigonométricas mejor identidades) Encontrar la serie de Fourier de


f (x) = cos4 x en SC[−π, π]

16. (Extensiones pares e impares) Encontrar las extensiones pares e impares de las
siguientes funciones y trazar su gráfica:

A) f (x) = 1 definida en ]0, 5[

B) f (x) = x2 definida en ]0, π[

C) f (x) = sin (x) definida en 0, π4


 

D) f (x) = ex definida en ]0, π[

17. (Series de senos) Encontrar la serie de Fourier de senos de las funciones

A) f (x) = cos x para x ∈ ]0, π[

B) f (x) = ex para x ∈ ]0, π[

C) f (x) = x2 para x ∈ ]0, π[

D) f (x) = x2 − x para x ∈ ]0, 1[

299
Apuntes Mat023 (versión preliminar actualizada 23-05-2014)

18. (Series de cosenos) Encontrar la serie de Fourier de cosenos de las funciones

A) f (x) = sin x para x ∈ ]0, π[

B) f (x) = ex para x ∈ ]0, π[

C) f (x) = x2 para x ∈ ]0, π[

D) f (x) = x − x3 para x ∈ [0, 4]

19. (Derivando Series) Si para x ∈ ]−π, π[



X 2 (−1)k+1
x= sin (kx)
k=1
k

¿Es cierto que



X
1= 2 (−1)k+1 cos (kx)
k=1

para x ∈ ]−π, π[? En otras palabras ¿es posible derivar término a término una serie de
Fourier y asegurar que converge a la derivada de la función?. (Obs: Para poder derivar
una serie de Fourier y garantizar convergencia basta que la función sea continua
f (a) = f (b) y f 0 , f 00 ∈ SC[a, b] derivar por ejemplo la serie de Fourier de |x| para
x ∈ ]−π, π[)

20. (¿Cuál serie usar?) Calcular el valor de


∞ ∞ ∞
X 1 X 1 X 1
2
, 4
,
k=1
k k=1
k k=1
k6

usando series de Fourier adecuadas.

300
Parte II

Cálculo diferencial en varias variables

301
Capı́tulo 6 : Elementos de topologı́a de Rn

El espacio euclidiano Rn

Definición 6.1.1. Se define el espacio n−dimensional sobre el conjunto de los números


reales por:
Rn = {x = (x1 , x2 , . . . , xn ) : xi ∈ R, i = 1, 2, . . . , n}

junto con la suma en Rn dada por:

x + y = (x1 , x2 , . . . , xn ) + (y1 , y2 , . . . , yn )
= (x1 + y1 , x2 + y2 , . . . , xn + yn )

y la multiplicación por escalar definida por:

αx = α (x1 , x2 , . . . , xn )
= (αx1 , αx2 , . . . , αxn )

para todos x, y ∈ Rn y α ∈ R.

Teorema 6.1.1. El espacio Rn es un espacio vectorial real de dimensión n.

Producto interno y norma

Definición 6.2.1. Sean x = (x1 , . . . , xn ) e y = (y1 , . . . , yn ) en Rn . Se define el producto


interno o producto escalar de x e y como el número real:
n
X
hx, yi = xi yi
i=1

Observación 6.2.1. El producto interno es el que confiere la noción de distancia y de


perpendicularidad al espacio Rn .

Proposición 6.2.1. Sean x, y, z ∈ Rn y α ∈ R, entonces:

1. Bilinealidad: hαx + y, z i = α hx, zi + hy, zi y hx, αy + zi = α hx, yi + hx, zi

2. Simetrı́a: hx, yi = hy, xi

302
Apuntes Mat023 (versión preliminar actualizada 23-05-2014)

3. Definido positivo: hx, xi ≥ 0 y hx, xi = 0 ⇐⇒ x = 0.

Definición 6.2.2. Sea x = (x1 , . . . , xn ) ∈ Rn . Se define la norma (euclidiana) de x


como el número real:
p
kxk = hx, xi
( n )1/2
X
= x2i
i=1

Proposición 6.2.2. Sean x ∈ Rn y α ∈ R, entonces:

1. kxk ≥ 0

2. kαxk = |α| kxk

Teorema 6.2.1 (Desigualdad de Cauchy-Schwarz). Sean x, y ∈ Rn , entonces:

|hx, yi| ≤ kxk kyk

Demostración. Consideremos ϕ : R → R definida por:

ϕ (λ) = hx + λy, x + λyi

Note que:
ϕ (λ) = kxk2 + 2λ hx, yi + λ2 kyk2

Ahora bien, ϕ (λ) ≥ 0 es equivalente a que el discriminante de la expresión cuadrática


anterior sea negativa o cero. Es decir:

4 hx, yi2 − 4 kxk2 kyk2 ≤ 0

Luego, extrayendo raı́z cuadrada se obtiene:

|hx, yi| ≤ kxk kyk

303
Apuntes Mat023 (versión preliminar actualizada 23-05-2014)

Observación 6.2.2. La noción de perpendicularidad caracterı́stica de los espacios eucli-


dianos se obtiene de la desigualdad de Cauchy-Schwarz. En efecto, sean x, y ∈ Rn r {0},
luego:
|hx, yi| hx, yi
|hx, yi| ≤ kxk kxk ⇐⇒ ≤ 1 ⇐⇒ −1 ≤ ≤1
kxk kyk kxk kyk
Por tanto, debe existir un ángulo θ ∈ [0, π) tal que:
hx, yi
cos θ =
kxk kyk
O bien, en su forma clásica:
hx, yi = kxk kyk cos θ

Definición 6.2.3. Sean x, y ∈ Rn . Se define el ángulo entre x e y como:


hx, yi
∠ (x, y) = arc cos
kxk kyk
Además, diremos que x e y son perpendiculares u ortogonales si ∠ (x, y) = 0. En tal
caso, se anota x ⊥ y.

Teorema 6.2.2 (Pitágoras). Sean x, y ∈ Rn tales que x ⊥ y, entonces:

kx + yk2 = kxk2 + kyk2

Teorema 6.2.3. Sean x1 , x2 , . . . , xm ∈ Rn r {0} con xi ⊥ xj , i 6= j y m ≤ n, entonces


x1 , x2 , . . . , xm son linealmente independientes.

Observación 6.2.3. Otra consecuencia de la desigualdad de Cauchy-Schwarz es la de-


sigualdad triangular. La desigualdad triangular es una propiedad muy importante de la
norma euclidiana y es la piedra angular de la noción de distancia sobre el espacio Rn .

Teorema 6.2.4 (Desigualdad triangular). Sean x, y ∈ Rn , entonces:

kx + yk ≤ kxk + kyk

Demostración. Por la desigualdad de Cauchy-Schwarz:

kx + yk2 = hx + y, x + yi
= kxk2 + 2 hx, yi + kyk2
≤ kxk2 + 2 kxk kyk + kyk2
(kxk + kyk)2

Lo cual implica el resultado.

304
Apuntes Mat023 (versión preliminar actualizada 23-05-2014)

Definición 6.2.4. Sean x, y ∈ Rn . Se define la distancia entre x e y como el número real


dado por:
d (x, y) = kx − yk

Teorema 6.2.5. Sean x, y, z ∈ Rn , entonces:

1. d (x, y) ≥ 0; d (x, y) = 0 ⇐⇒ x = y

2. d (x, y) = d (y, x)

3. d (x, z) ≤ d (x, y) + d (y, z) (desigualdad triangular)

Observación 6.2.4. Como se sabe, el espacio euclidiano n−dimensional tiene una propie-
dad sorprendente. A saber, generalizando la noción de norma euclidiana se puede considerar
la noción de norma como una función más general N : Rn → R que satisfaga:

1. N (x) ≥ 0

2. N (αx) = |α| N (x)

3. N (x + y) ≤ N (x) + N (y)

A modo de ejemplo, las normas más comunes en Rn son, a modo de ejemplo:

kxk∞ = máx {|x1 | , |x2 | , . . . , |xn |}


n
X
kxk1 = |xi |
i=1
( n )1/p
X
kxkp = |xi |p
i=1

Además, se tiene que:


lı́m kxkp = kxk∞
p→∞

Ası́:

Teorema 6.2.6. Sean N y N1 dos normas cualesquiera sobre Rn . Entonces, existen


constantes α, β > 0 tales que:

αN1 (x) ≤ N (x) ≤ βN1 (x)

para todo x ∈ Rn .

305
Apuntes Mat023 (versión preliminar actualizada 23-05-2014)

Elementos de topologı́a de Rn

Definición 6.3.1. Sean a ∈ Rn y ε > 0. Llamaremos bola abierta de radio ε con


centro en a al conjunto definido por:

B (a, ε) = {x ∈ Rn : kx − ak < ε}

Ejemplo 6.3.1. Si n = 1, entonces:

B (a,ε) = (a − ε, a + ε)

Si n = 2 y a = (x0 , y0 ), entonces:
 q 
2 2 2
B (a, ε) = (x, y) ∈ R : (x − x0 ) + (y − y0 ) < ε

Definición 6.3.2. Sea U ⊆ Rn . Diremos que a ∈ U es un punto interior de U si:

∃ε > 0, B (a, ε) ⊆ U

Llamaremos interior de U al conjunto de todos los puntos interiores de U . Anotaremos:


Ů . Es decir, se tiene que:

Ů = {a ∈ U : a es punto interior a U }

Definición 6.3.3. Sea U ⊆ Rn . Diremos que U es un conjunto abierto en Rn , o


simplemente abierto en Rn si U = Ů . Es decir, si todo punto de U es interior a U .

Observación 6.3.1. En particular, U es abierto si:

∀a ∈ U, ∃ε > 0, B (a, ε) ⊆ U

Ejemplo 6.3.2. Los conjuntos ∅ y Rn son abiertos.

Ejemplo 6.3.3. Un conjunto finito A ⊆ Rn no es abierto.

Teorema 6.3.1. Toda bola abierta es un conjunto abierto.

306
Apuntes Mat023 (versión preliminar actualizada 23-05-2014)

Demostración. Sean a ∈ Rn y ε > 0. Considere x ∈ B (a, ε) y:

δ = ε − kx − ak > 0

Sea, ahora, y ∈ B (x, δ). Luego, ky − xk < δ y además:

ky − ak = k(y − x) + (x − a)k
≤ ky − xk + kx − ak
< δ + kx − ak
= ε − kx − ak + kx − ak = ε

Ası́, ky − ak < ε, y por tanto, y ∈ B (a, ε). Esto implica que, B (x, δ) ⊆ B (a, ε). Es decir,
que todo punto x ∈ B (a, ε) es interior.

Ejemplo 6.3.4. Todo intervalo abierto en R es un conjunto abierto.

Ejemplo 6.3.5. Sea:


U = (x, y) ∈ R2 : x > 2


Entonces, U es abierto.

Proposición 6.3.1. Los conjuntos abiertos de Rn poseen las siguientes propiedades:

1. ∅ y Rn son abiertos.

2. La unión arbitraria de conjuntos abiertos en Rn es un conjunto abierto en Rn .

3. La intersección de un número finito de conjuntos abiertos en Rn es un conjunto


abierto en Rn .

Ejemplo 6.3.6. Sea Hn = a − n1 , b + 1



n
⊆ R, para cada n ∈ N. Notamos que cada Hn
es un conjunto abierto en R, pero:

\
H= Hn = [a, b]
n=1

el cual no es abierto.

307
Apuntes Mat023 (versión preliminar actualizada 23-05-2014)

Definición 6.3.4. Sean U ⊆ Rn y a ∈ Rn . Diremos que a es un punto de adherencia


a U si:
∀ε > 0, B (a, ε) ∩ U 6= ∅

El conjunto de todos los puntos de adherencia de U se denomina la clausura de U y se


anota U .

Observación 6.3.2. Para todo U ⊆ Rn , Ů ⊆ U ⊆ U .

Ejemplo 6.3.7. La bola cerrada con centro en a y radio ε está dada por:

B (a, ε) = {x ∈ Rn : kx − ak ≤ ε}

Ejemplo 6.3.8. N = Z = ∅ = ∅.

Ejemplo 6.3.9. Q = R.

Definición 6.3.5. Sea U ⊆ Rn . Diremos que U es un conjunto cerrado si U = U .

Ejemplo 6.3.10. Los conjuntos ∅ y Rn son cerrados.

Teorema 6.3.2. Sea U ⊆ Rn . Entonces, U es cerrado en Rn , si y solo si, U C es un conjunto


abierto.

Definición 6.3.6. Sean U ⊆ Rn y a ∈ Rn . Diremos que a es un punto de acumulación


de U si:
∀ε > 0, (B (a, ε) r {a}) ∩ U 6= ∅

El conjunto de todos los puntos de acumulación de U se denomina el conjunto derivado


de U y se anota como U 0 .

Ejemplo 6.3.11. Sea U = (0, 1] ∪ {2, 3}. Luego, U 0 = [0, 1].

: n ∈ N . Luego, X 0 = {0}. Note que lo anterior quiere


1
Ejemplo 6.3.12. Sea X = n

decir que hay infinitos puntos distintos de X suficientemente cercanos a 0.

m 1
: m, n ∈ N, n 6= 0 ⊆ R2 . Calcule A0 .
 
Ejemplo 6.3.13. Sea A = ,
n n

Observación 6.3.3. Si U ⊆ Rn tal que U 0 6= ∅, entonces U es infinito. Ası́, para todo


conjunto finito A ⊆ Rn , se tiene que A0 = ∅.

308
Apuntes Mat023 (versión preliminar actualizada 23-05-2014)

Teorema 6.3.3. Sea U ⊆ Rn . Entonces, U es cerrado en Rn , si y solo si, U 0 ⊆ U .

Definición 6.3.7. Sean U ⊆ Rn y a ∈ Rn . Diremos que a es un punto frontera de U si:

∀ε > 0, B (a, ε) ∩ U 6= ∅ ∧ B (a, ε) ∩ U C 6= ∅

El conjunto de todos los puntos frontera de U se llama la frontera de U y se anota como


∂U .

Observación 6.3.4. Note que, desde la definición, se obtiene que ∂U = U ∩ U C .

Ejemplo 6.3.14. La esfera con centro en a y radio ε > 0 se obtiene como:

S (a, ε) = ∂B (a, ε)
= {x ∈ Rn : kx − ak = ε}

Ası́, la esfera unitaria S n−1 en Rn se obtiene como:

S n−1 = ∂B (0, 1)
= {x ∈ Rn : kxk = 1}

Definición 6.3.8. Diremos que un conjunto X ⊆ Rn es acotado si existe un número real


M > 0 tal que:
kxk ≤ M

para todo x ∈ X.

Observación 6.3.5. Note que la cota anterior es uniforme, es decir: sup kxk : x ∈

X ≤ M.

Definición 6.3.9 (Heine-Borel). Un conjunto K ⊆ Rn se dice compacto si es cerrado y


acotado.

Ejemplo 6.3.15. La bola cerrada B (a, ε) es un conjunto compacto sobre en Rn . En efecto,


como B (a, ε) es cerrado, basta verificar que es acotado. Note que para todo x ∈ B (x, ε),
se tiene que:
kxk ≤ kx − ak + kak < ε + kak

309
Apuntes Mat023 (versión preliminar actualizada 23-05-2014)

Ejemplo 6.3.16. Sean L ⊆ Rn y K ⊆ Rm conjuntos compactos, respectivamente. Entonces,


L × K es compacto en Rn+m = Rn × Rm

Definición 6.3.10. Un conjunto A ⊆ Rn se llama disconexo si existen dos conjuntos


abiertos U y V de Rn tales que:

1. U ∩ A 6= ∅ ∧ V ∩ A 6= ∅.

2. (U ∩ A) ∩ (V ∩ A) = ∅.

3. (U ∩ A) ∪ (V ∩ A) = A

Por otro lado, un conjunto A ⊆ Rn se dice conexo si no es disconexo.

Ejemplo 6.3.17. N y Q son disconexos.

Ejemplo 6.3.18. R es conexo.

Ejemplo 6.3.19. Todo intervalo real es conexo.

Teorema 6.3.4. Sean A ⊆ Rn un conjunto conexo y C ⊆ Rn tales que A ⊆ C ⊆ A,


entonces C es conexo.

Teorema 6.3.5. Sea {Aλ }λ∈Λ una familia de conjuntos conexos en Rn con al menos un
punto en común, entonces:
[

λ∈Λ
es conexo.

Ejercicios del capı́tulo

1. Hallar un subespacio vectorial W de R3 tal que:



 x = 2t + 1


L: y = −2 + t


z = 3t − 1

se escriba como:
L = W + {u0 }

con u0 ∈ R3 adecuado.

310
Apuntes Mat023 (versión preliminar actualizada 23-05-2014)

2. En los siguientes conjuntos, determine cuales son: Abiertos, Cerrados. Determine en


cada caso el interior, la clausura y frontera del conjunto.
1 1
a) {(x, y) ∈ R2 / y = 2x} b) {(x, y) ∈ R2 / x = , y = , n, m ∈ N
n m
c) {(x, y, z) ∈ R3 / z < x2 + y 2 } 3
d) {(x, y, z) ∈ R / x = 0}
e) Una recta en R2 o R3 f) Un plano en R3
g) {(x, y, z) ∈ R3 / x = y = z = (−1)n , n ∈ N} h) {(x, y, z) ∈ R3 / x < y}
i) {x ∈ Rn : kxk ≤ 1} j) {x ∈ Rn : kxk = 1}

3. Demuestre rigurosamente que:

a) (0, 1) no es un punto interior de A = {(x, y) ∈ R2 / |x| + |y| < 1}.

b) (0, 0) es un punto interior de A.

c) (1, 0) es un punto de acumulación de A.


 
1 1
d) , es un punto de frontera de A.
2 2

4. Probar que B (a; r) = {x ∈ Rn : kx − ak < r} es abierto en Rn .

5. Si A es un conjunto cerrado que contiene cada número racional r ∈ [0, 1], probar que
[0, 1] ⊆ A.

6. Encuentre un conjunto en R2 que no sea Abierto ni Cerrado.

7. Sea R = [−1, 2] × [3, 4[

a) ¿Es Abierto?, ¿Es Cerrado?

b) Determine R.

8. Si K = [0, 1] × [0, 1] ∪ {(2, 2)}

a) ¿Es Abierto?

b) ¿Es cerrado?

9. Sea {Ai : i ∈ I} una colección de subconjuntos abiertos de Rn muestre que:

311
Apuntes Mat023 (versión preliminar actualizada 23-05-2014)

a) ∪i∈I Ai es un conjunto abierto (la unión arbitraria de abiertos es un abierto)

b) ∩i∈I 0 Ai es un conjunto abierto, donde I 0 ⊆ I es un conjunto finito (la intersección


finita de abiertos es un abierto)

c) Muestre que la intersección arbitraria de abiertos no es necesariamente un


abierto, para ello considere en Rn la colección de bolas abiertas B θ; n1 con


n ∈ N.

10. Encontrar A◦ , A, A0 y ∂A si

A = (x, y) ∈ R2 : x > 0, y > 0 ∪ {(−1, −1)}




11. Si T : Rn → Rm es una transformación lineal y P ∈ Rn , r > 0 ¿En que consiste el


conjunto T (B (P ; r))?¿Es abierto?

12. Muestre que p ∈ A si y solo si existe una sucesión de puntos en A, digamos {an }n∈N
tal que an → p (esto es
lı́m d (an , p) = 0
n→∞

en el sentido de los limites de sucesiones en R)

13. Considere el subconjunto de R


 
1 1
U = x∈R:x= + para algún n, m ∈ N
n m

encontrar U ◦ , U , U 0 y ∂U .

14. Muestre que si U y V son abiertos en R entonces U × V es abierto en R2 .

15. Sean U, V subconjuntos de Rn probar:

a) U ⊂ V ⇒ U ⊂ V

b) U ∪ V = U ∪ V

16. Si U ⊆ Rn muestre que:

a) U ◦ y ∂U son disjuntos y U = U ◦ ∪ ∂U

312
Apuntes Mat023 (versión preliminar actualizada 23-05-2014)

b) Muestre que la frontera es vacı́a si y solo si el conjunto es abierto y cerrado.

c) Muestre que U es abierto si y solo si ∂U = U − U


◦
17. Si U es abierto ¿Es verdad que U = U ?

18. Muestre que todo abierto de R2 es una unión numerable de cajas abiertas de la forma
]a, b[ × ]c, d[ y también es una unión numerable de bolas abiertas.

313
Capı́tulo 7 : Funciones de varias variables

Definiciones básicas

Definición 7.1.1. Consideremos una función f : U ⊆ Rn → Rm . Diremos que:

1. f es una función real de varias variables si n ≥ 2 y m = 1.

2. f es una función vectorial de una variable real si n = 1 y m ≥ 2.

3. f es una función vectorial de varias variables si n ≥ 2 y m ≥ 2.

En general, las funciones reales de varias variables se denotan con letras minúsculas,
como por ejemplo: f, g, h, etc. y las funciones vectoriales se anotan con letras mayúsculas,
tales como: F, G, H, etc.

Ejemplo 7.1.1. Son funciones reales de varias variables las siguientes funciones:

1. a) f : R2 → R, (x, y) 7→ f (x, y) = x2 + 2y.

b) g : R3 → R, (x, y, z) 7→ g (x, y, z) = sin (xyz).


1
(x, y, z) 7→ h (x, y, z) = e− 2 (x ).
2 +y 2 +z 2
c) h : R3 → R,

Ejemplo 7.1.2. Un ejemplo importante de función de varias variables corresponde a la


k−ésima proyección sobre R de un vector x = (x1 , x2 , . . . , xn ) ∈ Rn . Más precisamente, se
define la k−ésima proyección de x ∈ Rn como la función πk : Rn → R definida por:

πk (x1 , x2 , . . . , xn ) = πk

Esta función, en particular, nos permitirá representar funciones tales como:

2x2 + y 3
f (x, y, z) = 2
x + z2 + 1

como álgebra de proyecciones. Es decir, podemos escribir f (x, y, z) en la forma:

2 πx2 (x, y, z) + πy3 (x, y, z)


f (x, y, z) =
πx2 (x, y, z) + πz2 (x, y, z) + 1

Ejemplo 7.1.3. Son funciones vectoriales las siguientes funciones:

314
Apuntes Mat023 (versión preliminar actualizada 23-05-2014)

1. F : R → R2 , t 7→ F (t) = (C1 et , C2 e−t ), con C1 , C2 ∈ R.

2. G : R → R3 , t 7→ G (t) = (α cos t, α sin t, βt), con α, β > 0.

3. H : R2 → R3 , (x, y) 7→ H (x, y) = (ln (x2 + y 2 + 1) , cos (x + y) , x + 3y).

Ejemplo 7.1.4. Notamos que, si en el ejemplo (3) anterior definimos:

f1 (x, y) = ln x2 + y 2 + 1


f2 (x, y) = cos (x + y)
f3 (x, y) = x + 3y

entonces:
H (x, y) = (f1 (x, y) , f2 (x, y) , f3 (x, y))

Ejemplo 7.1.5. Sean fi : Ui ⊆ Rn → R funciones de varias variables con dominio Ui , con


i = 1, 2, . . . , m. Suponga que:
m
\
U= Ui
i=1
n m
Entonces, la transformación F : U ⊆ R → R definida por:

F (x) = (f1 (x) , f2 (x) , . . . , fm (x)) , x = (x1 , x2 , . . . , xn ) ∈ U

es una función vectorial de varias variables. Las funciones reales de varias variables fi se
llaman funciones componentes de F .

Definición 7.1.2. Sea f : D ⊆ Rn → R, (x1 , x2 , . . . , xn ) → f (x1 , x2 , . . . , xn ). Llamaremos


dominio máximo de f al conjunto:

dom ( f ) = {x ∈ Rn : f (x) ∈ R}

Ası́ mismo, el dominio máximo de F : D ⊆ Rn → R, x → F (x) = (f1 (x) , f2 (x) , . . . , fm (x))


es dado por:
m
\
dom (F) = dom ( fi )
i=1

Ejemplo 7.1.6. Hallar el dominio máximo de:


p
x2 + y 2 − 1
f (x, y) =
ln (|x| − y)

315
Apuntes Mat023 (versión preliminar actualizada 23-05-2014)

Solución. El dominio máximo en R2 para que esta expresión represente una función a
valores reales en tal conjunto es

(x, y) ∈ R2 : x2 + y 2 − 1 ≥ 0 ∧ |x| − y > 0 ∧ |x| − y 6= 1




Ejemplo 7.1.7. Si   
sin (xyz) 1
F (x, y, z) = , y cos
xyz x
Determine el dominio máximo de F .

Solución. El dominio máximo corresponde a

(x, y, z) ∈ R3 : xyz 6= 0


Definición 7.1.3. El recorrido de f : U ⊆ Rn → Rm se define como:

rec ( f ) = {y ∈ Rm : ∃x ∈ U, x = f (x)}

Observación 7.1.1. En general, para funciones de varias variables es difı́cil calcular el


recorrido. Sin embargo, puede ser útil tomar restricciones. Esto es imágenes directas de
conjuntos adecuados.

Ejemplo 7.1.8. El recorrido de f : R2 → R, (x, y) → f (x, y) = xy 2 + yx2 es todo R. En


efecto, si

(x, y) ∈ R2 : x = y

T =
= {(x, x) : x ∈ R}

Entonces, f T = x · x2 + x · x2 = 2x3 , se sigue que f (T ) = R.

Gráficos, conjuntos de nivel y trazas

Definición 7.2.1. Sea f : U ⊆ Rn → R una función. Llamaremos gráfico de f al conjunto


definido por:
Graf (f ) = (x, f (x)) ∈ Rn+1 : x ∈ U


Observación 7.2.1. Evidentemente, esta definición posee un sentido geométrico en el


caso de funciones reales de varias variables sólo cuando n = 1, 2.

316
Apuntes Mat023 (versión preliminar actualizada 23-05-2014)

Observación 7.2.2. Si f : U ⊆ R2 → R es una función real de dos variables reales,


diremos que:
z = f (x, y) , (x, y) ∈ U

es una superficie en R3 .

Observación 7.2.3. Respecto de lo anterior, hay algunas superficies clásicas que es


conveniente reconocer. Tales superficies son conocidas como superficies cuádricas.

Definición 7.2.2. Una superficie cuádrica es una ecuación del tipo:

P (x, y, z) = 0

donde P (x, y, z) es un polinomio de segundo grado en tres variables. En particular, se


consideran la superficies cuádricas en su forma normal, es decir, considerando las ecuaciones:

Ax2 + By 2 + Cz 2 + D = 0

o bien:
Ax2 + By 2 + Cz = 0

Observación 7.2.4. Las superficies cuádricas más importantes son las siguientes:

1. Esfera:

x2 + y 2 + z 2 = r 2

317
Apuntes Mat023 (versión preliminar actualizada 23-05-2014)

2. Elipsoide:

x2 y2 z2
a2
+ b2
+ c2
= 1, a, b, c 6= 0

3. Hiperboloide de una hoja:

x2 y2 z2
a2
+ b2
− c2
= 1, a, b, c 6= 0

318
Apuntes Mat023 (versión preliminar actualizada 23-05-2014)

4. Hiperboloide de dos hojas:


2 y2 z2
− xa2 + b2
− c2
= 1, a, b, c 6= 0

5. Paraboloide:
x2 y2
a2
+ b2
= cz, a, b 6= 0 ∧ c > 0

319
Apuntes Mat023 (versión preliminar actualizada 23-05-2014)

6. Paraboloide hiperbólico:

x2 y2
a2
− b2
= cz, a, b 6= 0 ∧ c > 0

Definición 7.2.3. Sea f : U ⊆ Rn → R y c ∈ R. Llamaremos conjunto de nivel c de f


al conjunto definido por:

Lc (f ) = {x ∈ U : f (x) = c} ⊆ Rn

En particular, si n = 2 diremos que Lc (f ) es la curva de nivel c de f . Si n = 3, diremos


que Lc (f ) es la superficie de nivel c de f.

Observación 7.2.5. Note que, simplemente:

Lc (f ) = f −1 ({c})

Observación 7.2.6. De los conjuntos anteriores, nos interesa el aspecto gráfico para n = 2
y n = 3. La colección de los gráficos superpuestos de un número adecuado de curvas de
nivel o superficies de nivel para una función permite esbozar el gráfico de la función dada.

Ejemplo 7.2.1. Las curvas de nivel c de la función f (x, y) = x2 + y 2 son circunferencias



de centro en (0, 0) y radio c para c > 0. Para c = 0 es solo un punto y para c < 0 es
conjunto vacio.

320
Apuntes Mat023 (versión preliminar actualizada 23-05-2014)

p
Ejemplo 7.2.2. Las curvas de nivel c de la función f (x, y) = x2 + y 2 son circunferencias
de centro en (0, 0) y radio c para c > 0. Para c = 0 es solo un punto y para c < 0 es
conjunto vacio.

Observación 7.2.7. Se puede observar que para los ejemplos anteriores las curvas de nivel
son, básicamente, las mismas, es decir, cı́rculos concéntricos desde el origen. En este caso,
las curvas de nivel no son suficiente para determinar el gráfico de las funciones anteriores.
Debemos considerar la noción de traza.

Definición 7.2.4. Llamaremos traza de una superficie S : z = f (x, y) a la intersección


de dicha superficie con alguno de los planos coordenados. Denotaremos las trazas de una
superficie S mediante los sı́mbolos Ty y Tx , si la intersección se efectúa con los planos xz e
yz, respectivamente.

Ejemplo 7.2.3. Por ejemplo, las trazas de las superficies S1 : z = x2 + y 2 y S2 : z =


p
x2 + y 2 están dadas por:

(x, y, z) ∈ Ty ⇐⇒ (x, y, z) ∈ S1 ∩ {(x, y, z) : y = 0} ⇐⇒ z = x2 , y=0

para S1 , y para S2 por:

(x, y, z) ∈ Ty ⇐⇒ (x, y, z) ∈ S1 ∩ {(x, y, z) : y = 0} ⇐⇒ z = |x| , y=0

En particular, son algunas trazas las que nos permiten distinguir los gráficos de las
superficies S1 y S2 .

Ejercicios del capı́tulo

1. Encontrar los dominios de las siguientes funciones:


p
2 y − x2
a) f : D ⊆ R → R (x, y) → 2
x − y2
p
y − x2 ln (1 − x2 + y 2 )
b) f : D ⊆ R2 → R (x, y) →
ln (x2 − y 2 )

√ !
x x x2
c) f : D ⊆ R2 → R3 (x, y) → , , p
x + y x + y 2 |x| − x2 + y 2

321
Apuntes Mat023 (versión preliminar actualizada 23-05-2014)

2. Esbozar curvas de nivel y gráficas de las siguientes funciones:


a) f : R2 → R (x, y) → x − y + 2
b) f : R2 → R (x, y) → x2 + 9y 2
c) f : R2 → R (x, y) → máx {|x| , |y|}

3. Describir el comportamiento de las curvas de nivel f (x, y) = c para c en R con:


a) f (x, y) = x2 + 4y 2 + 5 b) f (x, y) = 4 − x2 − y 2
f (x, y) = (x2 + y 2 ) e−(x +y )
2 2
c) f (x, y) = |8 − |x2 + y 2 − 4|| + 1 d)

4. Considere la función f (x, y) = x2 + y 2 y defina para θ ∈ 0, π2 el conjunto


 

Sθ = (x, y, z) ∈ R3 : y = (tan θ) x


ver la forma de la curva G (f ) ∩ Sθ donde G (f ) corresponde a la gráfica de la función


2 2
p
f . Hacer lo mismo con la función f (x, y) = x2 + y 2 + ex +y .

5. Describir los conjuntos de nivel de las funciones:


a) f : R3 → R (x, y, z) → −x2 − y 2 − z 2
b) f : R3 → R (x, y, z) → 4x2 − y 2 + 9z 2
c) f : R3 → R (x, y, z) → z − x2 − 4y 2

6. Representar en R3 los conjuntos:

y2
 
3 2
a) A = (x, y, z) ∈ R : z + =1
4
b) B = (x, y, z) ∈ R3 : x = y 2


c) B = (x, y, z) ∈ R3 : x2 + y 2 = 1 ∧ x + y + z = 0


7. Utilizando coordenadas polares describir las curvas de nivel de la función


2xy

 x2 + y 2 si (x, y) 6= (0, 0)



f (x, y) =



 0 si (x, y) = (0, 0)

322
Capı́tulo 8 : Lı́mites y continuidad

Definiciones

Definición 8.1.1. Sean U ⊆ Rn , a ∈ U 0 y f : U → R una función. Diremos que el número


real L es el lı́mite de f (x) cuando x tiende a a si:

∀ ε > 0, ∃ δ > 0, 0 < kx − akRn < δ =⇒ |f (x) − L| < ε (8.1)

Lo anterior se representa mediante el sı́mbolo:

lı́m f (x) = L
x→a

Observación 8.1.1. En particular, si f : U ⊆ R2 → R y (a, b) ∈ U 0 , la definición de lı́mite


en (8.1) queda como:
q
∀ ε > 0, ∃ δ > 0, 0 < (x − a)2 + (y − b)2 < δ =⇒ |f (x, y) − L| < ε

y en sı́mbolos:
lı́m f (x, y) = L
(x,y)→(a,b)

Usualmente, a los lı́mites del tipo anterior, se les conoce como lı́mites dobles.

Ejemplo 8.1.1. Demuestre que:

lı́m (3x + 2y) = 2


(x,y)→(2,−1)

Solución. Por demostrar que, dado cualquier ε > 0, existe δ > 0 tal que si:
q
0 < (x − 2)2 + (y + 1)2 < δ

implica que:
|3x + 2y − 4| < ε

En efecto, note que:

|3x + 2y − 4| = |3 (x − 2) + 2 (y + 1)|
≤ 3 |x − 2| + 2 |y + 1|
< 3δ + 2δ = 5δ
q
Por tanto, dado ε > 0, existe δ = ε/5 tal que si 0 < (x − 2)2 + (y + 1)2 < δ, entonces
|3x + 2y − 4| < ε. Por tanto, se concluye que lı́m(x,y)→(2,−1) (3x + 2y) = 2.

323
Apuntes Mat023 (versión preliminar actualizada 23-05-2014)

Ejemplo 8.1.2. Demuestre que:


x2 cos (x2 + y 2 )
lı́m p =0
(x,y)→(0,0) x2 + y 2
Solución. Note que si (x, y) 6= (0, 0)

x2 cos (x2 + y 2 ) |x|2
cos x2 + y 2

− 0 = p

p 2
x + y2 x2 + y 2
|x| |x|
≤ p
x2 + y 2
p p
x2 + y 2 x 2 + y 2
≤ p
x2 + y 2
p
= x2 + y 2 = δ
q
por lo tanto, dado ε > 0 existe δ = ε tal que si 0 < (x − 0)2 + (y − 0)2 < δ entonces

x2 cos (x2 + y 2 )
− 0 < ε.

p 2
x + y2
Observación 8.1.2. Es importante destacar que la noción de lı́mite es un concepto que
puede tratarse de manera más general que en el caso de funciones reales de varias variables.
En particular, el concepto de lı́mite se puede extender a funciones vectoriales considerando
la norma correspondiente al espacio de llegada. Más precisamente, tenemos:

Definición 8.1.2. Sean U ⊆ Rn , a ∈ U 0 y F : U → Rm una función vectorial. Diremos


que el vector L ∈ Rm es el lı́mite de F (x) cuando x tiende a a si:

∀ ε > 0, ∃ δ > 0, 0 < kx − akRn < δ =⇒ kF (x) − LkRm < ε

Sin embargo, para nuestro propósitos de cálculo, es útil el siguiente teorema:

Teorema 8.1.1. Sean U ⊆ Rn , x ∈ U 0 y F : U → Rm una función vectorial tal que:

F (x) = (f1 (x) , f2 (x) , . . . , fm (x))

Entonces, el vector L = (L1 , L2 , . . . , Lm ) ∈ Rm es el lı́mite de F (x) cuando x tiende a a,


si y solo si:
lı́m fi (x) = Li
x→a

para todo i = 1, 2, . . . , m. En otras palabras, F (x) tiende al vector L cuando x tiende a


a, si y solo si, la convergencia se da en cada coordenada de F (x) a la correspondiente
coordenada de L.

324
Apuntes Mat023 (versión preliminar actualizada 23-05-2014)

La demostración de este resultado se sigue de la desigualdad

|fi (x) − Li | ≤ k(f1 (x) , f2 (x) , . . . , fm (x)) − (L1 , L2 , . . . , Lm )k


m
√ X
≤ m |fi (x) − Li |
i=1

Observación 8.1.3. En vista del resultado anterior, centraremos nuestra atención en


funciones reales de varias variables.

Teorema 8.1.2. Sean F : U ⊆ Rn → Rm una función vectorial y a ∈ U 0 . Suponga que:

lı́m F (x) = L ∧ lı́m F (x) = M


x→a x→a

entonces, L = M.

Demostración. Sea ε > 0. Por hipótesis, existen δ1 , δ2 > 0 tales que:

ε
0 < kx − ak < δ1 =⇒ kf (x) − Lk <
2

y:
ε
0 < kx − ak < δ2 =⇒ kf (x) − M k <
2
Considerando, δ = mı́n {δ1 , δ2 }, se tiene que:

kL − M k = k(L − f (x)) + (f (x) − M )k


≤ kf (x) − Lk + kf (x) − M k
ε ε
< + =ε
2 2

Lo anterior implica que L = M .

Este resultado nos entregará una técnica para demostrar la no existencia de ciertos
lı́mites.

Ejemplo 8.1.3. Determine si acaso existe el siguiente lı́mite:

sin (xy) + z
lı́m
(x,y,z)→(0,0,0) x2 + y 2 + |z|

325
Apuntes Mat023 (versión preliminar actualizada 23-05-2014)

sin(xy)+z
Solución. Anotemos f (x, y, z) = x2 +y 2 +|z|
y consideremos el conjunto:

T = (x, y, z) ∈ R3 : x = y = 0

∧ z 6= 0

Ahora bien: 
z  1 , z>0
f T =
=
|z|  −1 , z<0
sin(xy)+z
Por unicidad del lı́mite, se concluye que el lı́mite lı́m(x,y,z)→(0,0,0) x2 +y 2 +|z|
no existe.

Ejemplo 8.1.4. Sea f : R2 r {(0, 0)} → R una función de dos variables definida por:

xy
f (x, y) =
x2 + y2

Considere:
Tm = (x, y) ∈ R2 : y = mx


con m 6= 0. Note que:


 
f Tm (x, y) = f (x, mx)
mx2
=
x2 + m 2 x2
m
=
1 + m2

Notamos que si (x, y) → (0, 0) a través de Tm , es decir, si consideramos el lı́mite


lı́m(x,y)→(0,0) f (x, y), se tiene que:
(x,y)∈Tm

lı́m f (x, y) = lı́m f (x, mx)


(x,y)→(0,0) x→0
(x,y)∈Tm
m
= lı́m
1 + m2
x→0
m
=
1 + m2

y por tanto, se puede concluir que el lı́mite de f (x, y) no existe, pues depende directamente
de la pendiente de la recta de aproximación al origen y = mx.

Lo anterior se puede generalizar introduciendo la idea de camino en Rn . Consideremos:

326
Apuntes Mat023 (versión preliminar actualizada 23-05-2014)

Definición 8.1.3. Un camino en Rn es una función ϕ : I → Rn , cuyo dominio es un


intervalo I ⊆ R. Es decir:

ϕ (t) = (ϕ1 (t) , ϕ2 (t) , . . . , ϕn (t)) , t∈I

y cada función componente ϕi : I ⊆ R → R es continua en I.

Definición 8.1.4. Sean U ⊆ Rn , a ∈ U 0 y un camino de la forma ϕ : [0, 1] → U . Diremos


que ϕ (t) converge propiamente a a, cuando t → 0+ si:

lı́m ϕ (t) = a
t→0+

p
y ϕ (t) 6= a, para todo t ∈ (0, 1]. Anotamos lo anterior mediante el sı́mbolo ϕ (t) → a.

Teorema 8.1.3. Sean L ∈ R, f : U ⊆ Rn → R y a ∈ U 0 tales que:

lı́m f (x) = L
x→a

entonces:
lı́m f (ϕ (t)) = L
t→0+
p
para todo camino ϕ : [0, 1] → U tal que ϕ (t) → a.

se sigue de este teorema, que si por dos caminos distintos el limite es diferente o bien,
por un camino no existe entonces el limite general no existe.

Ejemplo 8.1.5. Muestre que el lı́mite:


x3 + y 3
lı́m
(x,y)→(0,0) x − y

no existe

Solución. Note que:


x3 + y 3 x3 − y 3 + 2y 3
f (x, y) = =
x−y x−y
2y 3
= x2 + xy + y 2 +

x−y
Ahora bien, suponga que:
2y 3
=r
x−y

327
Apuntes Mat023 (versión preliminar actualizada 23-05-2014)

luego:
2
x = y + y3
r
Defina:  
2 3
ϕ (t) = t + t ,t
r
entonces
lı́m ϕ (t) = (0, 0)
t→0

y
   2 !
2 2
lı́m f (ϕ (t)) = lı́m t2 + t t + t3 + t + t3 + r
t→0 t→0 r r
= r

luego tomando r = 1 y r = 2 obtenemos lı́mites distintos. El lı́mite no existe. La técnica


empleada en este caso se conoce como la técnica de los conjuntos de nivel.

Observación 8.1.4. Otro procedimiento habitual para investigar acerca de la existencia


o inexistencia de un lı́mite es transformar mediante un cambio de variables adecuado el
espacio R2 completo. Es decir, considerando el cambio de coordenadas:

 x = r cos θ
 y = r sin θ
xy
la función f (x, y) = x2 +y 2
queda como:

f (x (r, θ) ; y (r, θ)) = f (r cos θ, r sin θ)


r2 cos θ sin θ
= 2
r cos2 θ + sin2 θ


= cos θ sin θ

Ahora bien, notando que si (x, y) → (0, 0) implica que r → 0, se observa que el lı́mite
anterior no existe, pues depende del ángulo θ de entrada al origen (0, 0).

Definición 8.1.5. Sean f : U ⊆ R2 → R una función de dos variables y (a, b) ∈ U 0 . Se


definen los lı́mites iterados de f como los lı́mites univariados:
 
lı́m lı́m f (x, y)
x→a y→b

y
 
lı́m lı́m f (x, y)
y→b x→a

328
Apuntes Mat023 (versión preliminar actualizada 23-05-2014)

Teorema 8.1.4. Sean f : U ⊆ R2 → R una función y (a, b) ∈ U 0 tales que:

lı́m f (x, y) = L
(x,y)→(a,b)

Entonces, lı́mx→a (lı́my→b f (x, y)) y lı́my→b (lı́mx→a f (x, y)) existen, y además:
   
lı́m lı́m f (x, y) = lı́m lı́m f (x, y) = L
x→a y→b y→b x→a

Ejemplo 8.1.6. Calcule los lı́mites iterados en (0, 0) de la función:

xy
f (x, y) = , (x, y) 6= (0, 0)
x2 + y2

Solución. Note que:


   
lı́m lı́m f (x, y) = lı́m 0 = 0 = lı́m lı́m f (x, y)
x→0 y→0 x→0 y→0 x→0

Por tanto, ambos lı́mites iterados existen y tienen valor cero, sin embargo, sabemos que
f (x, y) no posee lı́mite en (0, 0).

Ejemplo 8.1.7. Determine la existencia del lı́mite:

sin (x + y)
lı́m
(x,y)→(0,0) x + 3y

Solución. Usando lı́mites iterados


   
sin (x + y) sin x
lı́m lı́m = lı́m =1
x→0 y→0 x + 3y x→0 x
   
sin (x + y) sin y 1
lı́m lı́m = lı́m =
y→0 x→0 x + 3y y→0 3y 3

como los lı́mites son distintos el lı́mite no existe.

Cálculo de lı́mites

Observación 8.2.1. En la sección anterior, se establecieron procedimientos para indicar


que una función no posee lı́mite. Nuestro interés, ahora, se centra en aquellas funciones
que sı́ lo poseen. Por tanto, debemos entregar algunos elementos de cálculo:

329
Apuntes Mat023 (versión preliminar actualizada 23-05-2014)

Álgebra de lı́mites

Teorema 8.2.1. Sean f, g : U ⊆ Rn → R funciones tales que:

lı́m f (x) = L ∧ lı́m g (x) = M


x→a x→a

para x ∈ U 0 entonces:

1. ∀α ∈ R, (αf (x)) = α lı́m f (x) = αL,


x→a

2. lı́m (f (x) + g (x)) = lı́m f (x) + lı́m g (x) = L + M


x→a x→a x→a
  
3. lı́m (f (x) g (x)) = lı́m f (x) lı́m g (x) = LM
x→a x→a x→a

4. Si M 6= 0, lı́m (f (x) /g (x)) = lı́m f (x) / lı́m g (x) = L/M


x→a x→a x→a

Ejemplo 8.2.1. Calcule:


sin (3xy)
lı́m
(x,y)→(0,0) sin x sin y
sin u
Solución. Usaremos el lı́mite fundamental de una variable lı́mu→0 u
=1
sin(3xy)
sin (3xy) 3xy
lı́m = lı́m sin x sin y
(x,y)→(0,0) sin x sin y (x,y)→(0,0)
3xy
sin(3xy)
3xy
= lı́m   sin y 
(x,y)→(0,0) 1 sin x
3 x y
= 3

Ejemplo 8.2.2. Calcule:


1 − cos xy
lı́m
(x,y)→(0,0) x2 y sin y

Solución. Recordemos que


1 − cos u 1
lı́m 2
=
u→0 u 2
luego
1 − cos xy 1 − cos xy
lı́m = lı́m  
(x,y)→(0,0) x2 y sin y (x,y)→(0,0) 2 2 sin y
xy y
1−cos xy
(xy)2
= lı́m  
(x,y)→(0,0) sin y
y
1
=
2

330
Apuntes Mat023 (versión preliminar actualizada 23-05-2014)

Desigualdades y Teorema del Sandwich

Observación 8.2.2. Una herramienta muy importante para el cálculo de lı́mites el teo-
rema del sandwich o teorema del acotamiento. Este teorema requiere el uso adecuado de
desigualdades notables, las más usuales son:

1. Sea x = (x1 , x2 , . . . , xn ) ∈ Rn , entonces |xi | ≤ kxk, para todo i = 1, 2, . . . , n.

2. a) (a + b)2 ≤ 2 (a2 + b2 )

b) 2ab ≤ a2 + b2

3. |sin x| ≤ 1, para todo x ∈ R.

4. |sin x| ≤ |x|, para todo x ∈ R.

a a
5. a, b, c > 0 =⇒ b+c
≤ b

Teorema 8.2.2. Sean f, g, h : U ⊆ Rn → R y a ∈ U 0 tales que:

f (x) ≤ g (x) ≤ h (x) , ∀x ∈ U

Suponga que lı́mx→a f (x) = lı́mx→a h (x) = L, entonces:

lı́m g (x) = L
x→a

Ejemplo 8.2.3. Verificaremos que:


sin2 y
lı́m =0
(x,y)→(0,0) x2 + |y|

Solución. En efecto, basta notar que:


2
sin2 y |y|2

≤ |y|
x2 + |y| x2 + |y| ≤ |y| = |y|

se sigue que
sin2 y
− |y| ≤ 2 ≤ |y|
x + |y|
como lı́m(x,y)→(0,0) |y| = 0 se sigue que

sin2 y
lı́m =0
(x,y)→(0,0) x2 + |y|

331
Apuntes Mat023 (versión preliminar actualizada 23-05-2014)

Ejemplo 8.2.4. Calcular


x sin3 y
lı́m
(x,y)→(0,0) |x| + y 2

Solución. Note que

x sin3 y |x| |sin y|3 |x| |y|3



=
|x| + y 2 ≤ ≤ |x| |y|
|x| + y 2 |x| + y 2

ası́
x sin3 y
− |x| |y| ≤ ≤ |x| |y|
|x| + y 2
se sigue por el teorema de acotamiento que

x sin3 y
lı́m =0
(x,y)→(0,0) |x| + y 2

Continuidad

Definición 8.3.1. Sean U ⊆ Rn , a ∈ U 0 ∩ U y f : U → R una función real de varias


variables. Diremos que f es continua en a si:

∀ ε > 0, ∃ δ > 0, kx − akRn < δ =⇒ |f (x) − f (a)| < ε (8.2)

Además, diremos que f es continua en U si f es continua en cada punto de U .

Observación 8.3.1. Note que la definición dada anteriormente y por la proposición en



(8.2) , dice que f es continua en a si:

lı́m f (x) = f (a)


x→a

Definición 8.3.2. Diremos que f : U ⊆ Rn → R es discontinua en a ∈ U ∩ U 0 , si f no


es continua en a.

Ejemplo 8.3.1. Sea πk : Rn → R la k−ésima proyección de x ∈ Rn . Entonces, πk es


continua en Rn . En efecto, para cada a ∈ Rn , tenemos que:

|πk (x) − πk (a)| = |xk − ak | ≤ kx − ak

bastará tomar δ = ε.

332
Apuntes Mat023 (versión preliminar actualizada 23-05-2014)

Ejemplo 8.3.2. Demuestre que f : R2 → R definida por:



2
 xy

, (x, y) 6= (0, 0)
f (x, y) = x4 + y 2
0 , (x, y) = (0, 0)

es continua en (0, 0).

Solución. Basta notar que:


xy 2 |x| y 2

x4 + y 2 ≤ y 2 = |x|

Ası́, si (x, y) → (0, 0), entonces |x| → 0. Por el Teorema del Sandwich se concluye que:
xy 2
lı́m =0
(x,y)→(0,0) x4 + y 2

como f (0, 0) = 0, se tiene que f es continua en (0, 0).

Ejemplo 8.3.3. Sean U = {(x, y) : |x| < y 2 } ⊆ R2 y f : U → R definida por:


 xy
 , (x, y) ∈ U
f (x, y) = x + y2
2
 0 , (x, y) ∈/U

¿Es continua en (0, 0)?

Solución. Note que si (x, y) ∈ U

|y|3

xy |x| |y|
|f (x, y) − 0| = 2
= ≤ 2
x + y 2 x2 + y 2 x + y2
|y|3
≤ = |y|
|y|2
y si (x, y) 6∈ U entonces
|f (x, y) − 0| = 0 ≤ |y|

se sigue que, para todo (x, y) ∈ R2

|f (x, y) − 0| ≤ |y|

y por el teorema de acotamiento

lı́m f (x, y) = 0
(x,y)→(0,0)

La función es continua en (0, 0).

333
Apuntes Mat023 (versión preliminar actualizada 23-05-2014)

Definición 8.3.3. Se dice que una función f : U ⊆ Rn → Rm es Lipschitziana en U si:

∃K > 0, ∀x, y ∈ U, kf (x) − f (y)k ≤ K kx − yk

La constante K se llama constante de Lipschitz.

Toda función lipschitziana es continua en U . En efecto, supongamos que f : U ⊆ Rn →


Rm es una función lipschitziana en U y que ε > 0, entonces:

kf (x) − f (a)k ≤ K kx − ak < Kδ

ε
Ası́, tomando δ = K
, se verifica que f es continua en a. Como a ∈ U es cualquiera, se
obtiene, finalmente, la continuidad de f en U .

Ejemplo 8.3.4. Toda transformación lineal T : Rn → Rm es una función lipschitziana.

Álgebra de funciones continuas

El siguiente teorema facilita la identificación de funciones continuas:

Teorema 8.4.1 (Álgebra de funciones continuas). Sean f, g : U ⊆ Rn → R funciones


continuas en a ∈ U 0 ∩ U y α ∈ R, entonces αf, f + g, f − g y f g son continuas en a. Si
g (a) 6= 0, entonces f /g es continua en a.

Ejemplo 8.4.1. Son funciones continuas en R3 :

1. f (x, y, z) = 3xy 2 + z 2

sin x+sin y+sin z


2. g (x, y, z) = x2 +y 2 +z 2 +1

2
3. h (x, y, z) = ex sin (y) + z 2

Teorema 8.4.2. Sean f : U ⊆ Rn → Rm y g : V ⊆ Rm → Rp tales que f (U ) ⊆ V .


Suponga, además, que f es continua en a ∈ U 0 ∩ U y que g es continua en b = f (a) ∈ V 0 ,
entonces:
g ◦ f : U ⊆ Rn → Rp

es continua en a.

334
Apuntes Mat023 (versión preliminar actualizada 23-05-2014)

Demostración. Sea ε > 0. Como g es continua en b = f (a), existe η > 0 de modo que si
y ∈ V , entonces:
ky − bk < η =⇒ kg (y) − g (b)k < ε

Ahora bien, para y = f (x), existe δ > 0 tal que:

kx − ak < δ =⇒ kf (x) − f (a)k < η

Ası́, para x ∈ U tal que:

kx − ak < δ =⇒ f (x) ∈ V ∧ kf (x) − f (a)k < η =⇒ kg (f (x)) − g (f (a))k < ε

Ejemplo 8.4.2. Una aplicación inmediata del teorema anterior es el cambio de variables.
Ilustramos lo anterior con el siguiente ejemplo: sea f : R2 → R definida por:

 sin(x2 +y2 ) , si x2 + y 2 6= 0
x2 +y 2
f (x, y) =
 1 , si x2 + y 2 = 0

Estudie la continuidad de f en (0, 0).

Solución. Naturalmente, haciendo u = x2 + y 2 , tenemos que:

(x, y) → (0, 0) =⇒ u → 0

Ası́:
sin (x2 + y 2 ) sin u
lı́m 2 2
= lı́m =1
(x,y)→(0,0) x +y u→0 u
Como f (0, 0) = 1, obtenemos que f es continua en (0, 0).

Ejemplo 8.4.3. Usando compuesta de continuas podemos garantizar la continuidad de:


p
1. f (x, y) = x2 + sin2 y + 1

|x|+y 2 +z 2
2. g (x, y, z) = z2 +y2 +x|y|

335
Apuntes Mat023 (versión preliminar actualizada 23-05-2014)

Continuidad de funciones vectoriales

Observación 8.5.1. La continuidad para una función vectorial de varias variables F : U ⊆


Rn → Rm en a ∈ U ∩ U 0 se define por:

∀ε > 0, ∃δ > 0, kx − akRn < δ =⇒ kF (x) − F (a)kRm < ε

Observación 8.5.2. Al igual que en los lı́mites para funciones vectoriales, tenemos que la
continuidad de F : U ⊆ Rn → Rm definida por x 7→ F (x) = (f1 (x) , f2 (x) , . . . , fm (x)) en
a ∈ U 0 ∩ U se obtiene a través de la continuidad de las funciones componentes en x. Más
precisamente:

Teorema 8.5.1. Sean F : U ⊆ Rn → Rm una función vectorial definida por:

F (x) = (f1 (x) , f2 (x) , . . . , fm (x))

y a ∈ U ∩ U 0 . Entonces, F es continua en a ∈ U ∩ U 0 , si y solo si, fi es continua en a, para


cada i = 1, 2, . . . , m.

Ejemplo 8.5.1. La función F : R2 → R3 definida por:

F (x, y) = x2 y, sin x, 3x2 cos y




es continua en el plano.

Ejemplo 8.5.2. Sea f : R2 → R la función definida por:



 √2x−y2 +2 , si x2 + y 2 ≤ 1
1−x2 −y 2
f (x, y) =
 0 , si x2 + y 2 > 1

Analizar, completamente, la continuidad de f (x, y).

Solución. Consideremos los siguientes subconjuntos del plano:

(x, y) ∈ R2 : x2 + y 2 > 1

A =
B = (x, y) ∈ R2 : x2 + y 2 = 1


C = (x, y) ∈ R2 : x2 + y 2 < 1


Tenemos los siguientes casos:

336
Apuntes Mat023 (versión preliminar actualizada 23-05-2014)

1. Si (x, y) ∈ A, entonces f (x, y) = 0. Luego, f es continua en A por ser una función


constante en A.
2
2. Si (x, y) ∈ C, entonces f (x, y) = √2x−y 2+2 2 , con 1 − x2 − y 2 > 0. Luego, f es continua
1−x −y
en C por álgebra de funciones continuas y composición de funciones continuas.

3. Si (a, b) ∈ B, entonces, f (x, y) no puede ser continua en (a, b) ∈ B si:

2a − b2 + 2 6= 0

pues, si:

(x, y) → (a, b) =⇒ 1 − x2 − y 2 → 0 2x − y 2 + 2 → k, con k 6= 0




2
con lo cual la expresión √2x−y 2+2 2 se indefine. Por tanto, los puntos (a, b) ∈ B tales
1−x −y
que:
2a − b2 + 2 = 0

están dados por el sistema:



 2a − b2 + 2 = 0
 a2 + b2 = 1

Es decir, (a, b) = (−1, 0). Por tanto, investigamos el lı́mite:


2x − y 2 + 2
lı́m p
(x,y)→(−1,0) 1 − x2 − y 2
Note que:
2x − y 2 + 2 1 − x2 − y 2 + x2 + 2x + 1
p = p
1 − x2 − y 2 1 − x2 − y 2
p
2 2
(x + 1)2
= 1−x −y + p
1 − x2 − y 2
Por tanto, para que el lı́mite:
2x − y 2 + 2
lı́m p
(x,y)→(−1,0) 1 − x2 − y 2
exista es necesario y suficiente que el lı́mite:
(x + 1)2
lı́m p
(x,y)→(−1,0) 1 − x2 − y 2

337
Apuntes Mat023 (versión preliminar actualizada 23-05-2014)

exista. Sin embargo, note que:


( )
(x + 1)2 (x + 1)2
lı́m lı́m p = lı́m − √ =0
x→ −1− y→0− 1 − x2 − y 2 x→ −1 1 − x2
y que además, al considerar la trayectoria:

ϕ = (x, y) ∈ C : − x4 + 4x3 + 7x2 + 4x = y 2


 

vemos que:
(x + 1)2 (x + 1)2
lı́m p = lı́m √ =1
(x,y)→(−1,0) 1 − x2 − y 2 x→−1 1 − x2 + x4 + 4x3 + 7x2 + 4x
Por tanto:
(x + 1)2
lı́m p
(x,y)→(−1,0) 1 − x2 − y 2
no existe. En este caso, el lı́mite:
2x − y 2 + 2
lı́m p
(x,y)→(−1,0) 1 − x2 − y 2
no existe. Ası́, f no es continua en (−1, 0). Por lo tanto, f es continua en:

R2 − (x, y) ∈ R2 : x2 + y 2 = 1 .


Finalmente enunciamos un teorema que nos habla sobre el comportamiento de las


funciones continuas sobre los conjuntos compactos y conexos:

Teorema 8.5.2. Sea f : U ⊆ Rn → Rm una función continua en U , entonces:

1. Si A ⊆ U es conexo, entonces f (A) es conexo.

2. Si K ⊆ U es compacto, entonces f (K) es compacto.

Ejercicios del capı́tulo

1. Determine el valor de α ∈ R de modo que el lı́mite:



x − x cos xy (x2 − xy) 1 − α2
 
lı́m +
(x,y)→(0,0) y sin2 (3x) x2 − y 2
exista.

338
Apuntes Mat023 (versión preliminar actualizada 23-05-2014)

2. Sea f : R2 − {(0, 0)} → R la función definida por


 
2 1
x sin x+y
f (x, y) = 2
x + |x| + 2

calcular lı́m f (x, y)


(x,y)→(0,0)

3. Considere la función definida por


 x
 si x 6= −y 2
 x + y2


f (x, y) =


si x = −y 2

 0

analizar la continuidad de f en R2 .

4. Sean f y g funciones definidas en R2 − {(0, 0)} por


p 
sin x2 + y 2
f (x, y) = p
x2 + y 2 + x2 + y 2
 3
x − y3

g (x, y) = α
|x| + |y|
que valor debe tener α para que

lı́m {f (x, y) + g (x, y)}


(x,y)→(0,0)

exista.

5. Estudiar la continuidad de la función





 x3 + sin (y 2 ) y≤x
f (x, y) =


 2
x − xy + y 3 + cos (y 2 ) y>x

6. Estudiar la continuidad de la función


 2 2
x −y
 x2 +y2 −1

 x2 + y 2 6= 1
f (x, y) =


0 x2 + y 2 = 1

339
Apuntes Mat023 (versión preliminar actualizada 23-05-2014)

7. Determine si los siguientes lı́mites existen


|sin x sin y| x2 y 2
1) lı́m 2) lı́m 2
(x,y)→(0,0) |xy| (x,y)→(0,0) x2 y 2 + (y − x)

xy sin (y 3 ) sin (xy 4 )


3) lı́m 4) lı́m
(x,y)→(0,0) x4 + y 4 (x,y)→(0,0) x2 + y 8

p
y 3 |x| x3 + y 2
5) lı́m 6) lı́m
(x,y)→(0,0) |x| + y 4 (x,y)→(0,0) |x| + |y|

p 
sin x + sin y cos |xy| − 1
7) lı́m 8) lı́m
(x,y)→(0,0) x+y (x,y)→(1,0) y

(x − 1)4 y 2
p
x3 y x2 + y 2
9) lı́m 8 10) lı́m
(x,y)→(1,0) (x − 1) + y 4 (x,y)→(0,0) sin (xy)

x2 y 2x2 − y 2
11) lı́m 12) lı́m
(x,y)→(0,0) x2 + y (x,y)→(0,0) x2 + 2y 2

x4 + 2x2 y 2 + 2y 4 x6 + y 6
13) lı́m 14) lı́m
(x,y)→(0,0) (x2 + y 2 )2 (x,y)→(0,0) x4 + y 4

340
Capı́tulo 9 : Diferenciación en varias variables

En secciones anteriores hemos estudiado algunos métodos para dibujar la gráfica de


funciones de varias variables. Utilizando solo esos métodos es muy difı́cil obtener suficiente
información para captar las caracterı́sticas generales de una función con expresión algo
complicada. Sabemos del cálculo de una variable que la derivada nos puede ayudar mucho
en esta tarea; por ejemplo, nos permite cuantificar los cambios en la gráfica, localizar
máximos, mı́nimos, estudian concavidad, etc. La derivada también tiene otras muchas
aplicaciones como el estudiante habrá descubierto en cálculo de una variable. Una función
diferenciable de R2 en R deberı́a ser una función con gráfica suave, que no tenga dobleces
bruscos.
Para hacer precisas estas ideas necesitamos una definición de lo que entenderemos por
diferenciable en el caso de funciones de varias variables. Comenzamos con una noción de
derivada que descansa en nuestros conocimientos de una variable.

Derivadas parciales

Observación 9.1.1. En lo que sigue denotaremos por ei , con i = 1, 2, . . . , n, a los n


vectores de la base canónica de Rn , más precisamente, ei representa el vector de Rn que
tiene todas su componentes cero salvo la i-ésima componente la cual es igual a uno.

Definición 9.1.1. Sean f : U ⊆ Rn → R y a ∈ U . Se define la derivada parcial de f en a
respecto a la i-ésima variable como el lı́mite:

∂f f (a + tei ) − f (a)
(a) = lı́m
∂xi t→0 t
f (a1 , a2 , . . . , ai + t, . . . , an ) − f (a1 , a2 , . . . , an )
= lı́m
t→0 t

si este existe.

Observación 9.1.2. Para f : U ⊆ Rn → R, las derivadas parciales se denotan por:


∂f
∂xi
, fxi , Di f , etc.

Observación 9.1.3. Si n = 2, f : U ⊆ R2 → R, (x, y) → f (x, y) y a = (a, b) ∈ U ,

341
Apuntes Mat023 (versión preliminar actualizada 23-05-2014)

entonces:

∂f f (a + te1 ) − f (a)
(a, b) = lı́m
∂x t→0 t
f ((a, b) + t (1, 0)) − f (a, b)
= lı́m
t→0 t
f (a + t, b) − f (a, b)
= lı́m
t→0 t

∂f f (a + te2 ) − f (a)
(a, b) = lı́m
∂y t→0 t
f ((a, b) + t (0, 1)) − f (a, b)
= lı́m
t→0 t
f (a, b + t) − f (a, b)
= lı́m
t→0 t

note que corresponden a lı́mites de una variable, a saber, la variable t.

∂f ∂f
Observación 9.1.4. En particular, si n = 2, 3 y 4, anotamos: ∂x1
= , ∂f
∂x ∂x2
= ∂f
, ∂f
∂y ∂x3
= ∂f
∂z
∂f ∂f
y ∂x4
= ∂w
, según corresponda respecto al número de variables de f y el nombre y orden
que a estas asignemos.

∂f
Ejemplo 9.1.1. Sea f : R2 → R, (x, y) → f (x, y) = x3 y + 2x2 y 3 + 2x, calcular ∂x
(−1, 1)
∂f
y ∂y
(1, 2) si estas existen.

Solución. Por definición

∂f f (−1 + t, 1) − f (−1, 1)
(−1, 1) = lı́m
∂x t→0 t
(−1 + t) 1 + 2 (−1 + t)2 13 + 2 (−1 + t) − (−1)3 1 + 2 (−1)2 13 + 2 (−1)
3  
= lı́m
t→0 t
2
t (t − t + 1)
= lı́m
t→0 t
2

= lı́m t − t + 1
t→0

= 1

342
Apuntes Mat023 (versión preliminar actualizada 23-05-2014)

∂f f (1, 2 + t) − f (1, 2)
(1, 2) = lı́m
∂y t→0 t
13 (2 + t) + 2 (12 ) (2 + t)3 + 2 (1) − (13 2 + 2 (12 ) (23 ) + 2)

= lı́m
t→0 t
t (2t2 + 12t + 25)
= lı́m
t→0 t
2

= lı́m 2t + 12t + 25
t→0

= 25

note que en estos lı́mites, una de las variables se deja fija y se calcula la derivada usual de
una variable respecto a la otra, en otras palabras

∂f f (x + t, y) − f (x, y)
(x, y) = lı́m
∂x t→0 t
h (x + t) − h (x)
= lı́m
t→0 t
= h0 (x)

donde hemos pensado h (x) = f (x, y), al estar y fija esta es solo una función de la variable
x. En el ejemplo

∂f ∂
x3 y + 2x2 y 3 + 2x

(x, y) =
∂x ∂x
= 3x2 y + 4xy 3 + 2

evaluando en (−1, 1)
∂f
(−1, 1) = 3 (−1)2 + 4 (−1) + 2 = 1
∂x
y

∂f ∂
x3 y + 2x2 y 3 + 2x

(x, y) =
∂y ∂y
= x3 + 6x2 y 2

luego
∂f
(1, 2) = 13 + 6 22 = 25

∂y

343
Apuntes Mat023 (versión preliminar actualizada 23-05-2014)

Ejemplo 9.1.2. Si f (x, y, z) = 2xyz 2 + sin (xy 2 ) + 2x entonces

∂f ∂
2xyz 2 + sin xy 2 + 2x = 2yz 2 + y 2 cos xy 2 + 2
  
(x, y, z) =
∂x ∂x
∂f ∂
2xyz 2 + sin xy 2 + 2x = 2xz 2 + 2xy cos xy 2
  
(x, y, z) =
∂y ∂y
∂f ∂
2xyz 2 + sin xy 2 + 2x = 4xyz
 
(x, y, z) =
∂z ∂z

Ejemplo 9.1.3. Verifique que:

∂u ∂u ∂u
+ + =1
∂x ∂y ∂z

si:
x−y
u=x+
y−z
Solución. Derivando
 
∂ x−y y−z+1
x+ =
∂x y−z y−z
 
∂ x−y x−z
x+ = −
∂y y−z (y − z)2
 
∂ x−y x−y
x+ =
∂z y−z (y − z)2
se sigue      
y−z+1 x−z x−y
+ − + =1
y−z (y − z)2 (y − z)2
Ejemplo 9.1.4. Verifique que:

xzx + yzy = xy + z

si:
y
z = xy + xe x

Solución. Derivando

∂  y
 1  1y 1
y

xy + xe x = xe − ye + xy
x x
∂x x
∂  y
 1
xy + xe x = x + exy
∂y

344
Apuntes Mat023 (versión preliminar actualizada 23-05-2014)

luego

xzx + yzy
  
1 1
y 1
y
 1

= x xe x − ye x + xy + y x + exy
x
1 1 1
= xe x y − ye x y + xy + xy + ye x y
1
= xe x y + 2xy
y
= xy + xy + xe x
= xy + z

Ejemplo 9.1.5. Sea f : R2 → R la función definida por:



 xy , si (x, y) 6= (0, 0)
x2 +y 2
f (x, y) =
 0 , si (x, y) = (0, 0)

Calcule las derivadas parciales de f en (0, 0).

Solución. Es importante recordar que f no es continua en (0, 0). Sin embargo, la conti-
nuidad no afecta la existencia de las derivadas parciales. En efecto:
∂f f (t, 0) − f (0, 0)
(0, 0) = lı́m
∂x t→0 t
0−0
= lı́m =0
t→0 t
Análogamente, se tiene que
∂f f (0, t) − f (0, 0)
(0, 0) = lı́m
∂y t→0 t
0t
2 2 − 0
= lı́m 0 +t
t→0 t
= lı́m 0
t→0

= 0

Ejemplo 9.1.6. Sea f : R2 → R la función definida por:



 x2 , y ≤ x
f (x, y) =
 xy , y > x

∂f
Calcule ∂y
(1, 1).

345
Apuntes Mat023 (versión preliminar actualizada 23-05-2014)

∂f
Solución. Calculamos ∂y
(1, 1). Note que:

f (1, 1 + t) − f (1, 1) 1 · (1 + t) − 1
lı́m+ = lı́m+
t→0 t t→0 t
t
= lı́m+
t→0 t
= 1

y, análogamente, se obtiene que:


f (1, 1 + t) − f (1, 1)
→ 0, si t → 0−
t
∂f
Por tanto, ∂y
(1, 1), no existe.

en la figura se muestra el dominio y en dos colores distintos la parte y ≤ x e y > x, al


aproximarnos al punto (1, 1) por los puntos (1, 1 + t) se obtienen dos valores distintos de
lı́mite para t > 0 y t < 0.

Observación 9.1.5. La sola existencia de las derivadas parciales no es un buen concepto


de ser derivable en el caso de funciones de varias variables. Por ejemplo, la función

 1 si xy = 0


f (x, y) =


0 si xy 6= 0

no es continua en (0, 0) pero las derivadas parciales existen en ese punto, luego la existencia
de las derivadas parciales no implica continuidad, si la existencia de las derivadas parciales
fuera el concepto de “ser derivable” no podrı́amos recuperar el teorema de una variable,

346
Apuntes Mat023 (versión preliminar actualizada 23-05-2014)

derivable en un punto implica continua en el punto. Veremos que el concepto de ser


derivable estudia las variaciones de la función en una forma más global y no en una
dirección especı́fica como las derivadas parciales.

Ejercicios de la sección

∂f ∂f
1. Hallar ∂x
y ∂y
para las siguientes funciones

a) f (x, y) = xy 2
3 +y 2
b) f (x, y) = ex

c) f (x, y) = x ln (x2 + y 2 )

2. Si 
x2 y 3


 x2 +y 2
si (x, y) 6= (0, 0)
f (x, y) =


0 si (x, y) = (0, 0)

n o
determine el conjunto D = (x, y) ∈ R2 : ∂f∂x
(x,y)
existe y estudiar si la función

G D ⊆ R2 → R
:
∂f (x, y)
(x, y) → G (x, y) =
∂x
es continua en todo D.
 2 2
−y
3. Si u (x, y) = arctan x xy hallar ux , uy y verificar que

xux + yuy = 0

4. Si u (x, y, z) = x2 y + y 2 z + z 2 x verificar que


∂u ∂u ∂u
+ + = (x + y + z)2
∂x ∂y ∂z

5. Determine todas las funciones f : R3 → R, (x, y, z) → f (x, y, z) que cumplen


∂f
= ex+y + 3x2 y + 4yz + y 2 + 2
∂x
∂f
= ex+y + 2yz 3 + 2xy + 4xz + x3
∂y
∂f
= 3y 2 z 2 + 4xy
∂z

347
Apuntes Mat023 (versión preliminar actualizada 23-05-2014)

6. Si  p
 |x| + y 2 + |x|

 si x+y >0
x+y


f (x, y) = x3 si x + y ≤ 0 ∧ x2 + y 2 ≤ 4



y2 + x si x + y ≤ 0 ∧ x2 + y 2 > 4


∂f ∂f
determine si ∂x
(0, 0) y ∂y
(0, 0) existen.

Interpretación de la derivada parcial

En esta sección veremos una interpretación de las derivadas parciales y buscaremos la


ecuación del plano tangente a la gráfica de una función de dos variables.
Considere una función f : D ⊆ R2 → R, su gráfica esta contenida en R3 . Suponga que
estamos interesados en estudiar el comportamiento de la función en un punto (x0 , y0 ) de
su dominio, como no contamos con técnicas de varias variables, intentamos estudiar el
comportamiento descendiendo a una variable, para ello, vamos a intersectar la gráfica con
dos planos x = x0 e y = y0 .
La intersección de la gráfica con el plano x = x0 es una curva, la cual puede ser
interpretada como el gráfico de la función de una variable

z = f (x0 , y) = g (y)

en el plano x = x0 (note que si x = x0 y z = f (x, y) entonces z = f (x0 , y)). En


este plano podemos aplicar las técnicas de una variable para conocer el crecimiento y
decrecimiento, concavidad y otros, sin embargo, este análisis solo es un comportamiento de
la curva, no necesariamente un comportamiento global, por ejemplo un máximo sobre esta
curva puede no ser un máximo de la función en su dominio. La recta tangente a la gráfica
de esta función de una variable es

z − g (y0 ) = g 0 (y0 ) (y − y0 )

esto es
∂f
z − f (x0 , y0 ) = (x0 , y0 ) (y − y0 )
∂y

348
Apuntes Mat023 (versión preliminar actualizada 23-05-2014)

pues
g (y0 + h) − g (y0 )
g 0 (y0 ) = lı́m
h→0 h
f (x0 , y0 + h) − f (x0 , y0 )
= lı́m
h→0 h
∂f
= (x0 , y0 )
∂y
note que la recta está en el plano x = x0 , se sigue que los puntos de ella cumplen

x = x0
y = y
∂f
z = f (x0 , y0 ) + (x0 , y0 ) (y − y0 )
∂y
o en ecuaciones paramétricas
     
x x0 0
     
 y = 0 +y 1 
     
z f (x0 , y0 ) − y0 ∂f
∂y
(x0 , y0 ) ∂f
∂y
(x0 , y0 )

donde y ∈ R, el vector director es en este caso


 
0
 

 1 

∂f
∂y
(x0 , y0 )

De manera similar, la intersección de la gráfica con el plano y = y0 es una curva, la


cual puede ser interpretada como el gráfico de la función de una variable

z = f (x, y0 ) = p (x)

sobre el plano y = y0 (note que si y = y0 y z = f (x, y) entonces z = f (x, y0 )). La recta


tangente a la gráfica de esta función de una variable es

z − p (x0 ) = p0 (x0 ) (x − x0 )

esto es
∂f
z − f (x0 , y0 ) = (x0 , y0 ) (x − x0 )
∂x
349
Apuntes Mat023 (versión preliminar actualizada 23-05-2014)

pues
p (x0 + h) − p (x0 )
p0 (x0 ) = lı́m
h→0 h
f (x0 + h, y0 ) − f (x0 , y0 )
= lı́m
h→0 h
∂f
= (x0 , y0 )
∂x
se sigue que los puntos de esta recta cumplen

x = x
y = y0
∂f
z = f (x0 , y0 ) + (x0 , y0 ) (x − x0 )
∂x
o en ecuaciones paramétricas
     
x 0 1
     
 y = y0  + x 0 
     
z f (x0 , y0 ) − x0 ∂f
∂x
(x0 , y0 ) ∂f
∂x
(x0 , y0 )

el vector director corresponde a  


1
 

 0 

∂f
∂x
(x0 , y0 )
de estos cálculos se sigue que en el punto (x0 , y0 , f (x0 , y0 )) de la gráfica los vectores
   
1 0
   
 0  y  1 
   
∂f ∂f
∂x
(x0 , y0 ) ∂y
(x0 , y0 )

son tangentes, se sigue que el vector


     
1 0 − ∂f
∂x
(x0 , y0 )
     ∂f 
 0 × 1  =  − (x0 , y0 ) 
     ∂y 
∂f ∂f
∂x
(x0 , y0 ) ∂y
(x0 , y0 ) 1

es perpendicular a la superficie, ası́


 
∂f ∂f
− (x0 , y0 ) , − (x0 , y0 ) , 1 · (x − x0 , y − y0 , z − f (x0 , y0 )) = 0
∂x ∂y

350
Apuntes Mat023 (versión preliminar actualizada 23-05-2014)

deberı́a corresponder a un plano tangente a la superficie, el problema para definir esto


directamente como el plano tangente a la gráfica es el de formalizar en que sentido queremos
decir que la función y el plano se parecen cerca del punto, el cálculo realizado depende solo
de la existencia de las derivadas parciales pero por ejemplo en el caso de la función

 1 si xy = 0


f (x, y) =


0 si xy 6= 0

los cálculos nos llevarı́an a que el plano tangente es z = 1 sin embargo los valores que toma
la función en todo entorno abierto de (0, 0) no se parecen a uno (tan cerca como queramos
existen valores cero).
Veamos un ejemplo concreto, consideremos la función f : R2 → R, (x, y) → f (x, y) =
2 −y 4 +2y
(x2 − 2xy) e−2x y estudiemos la función en el punto (x0 , y0 ) = (1, 0). La gráfica de f
es

351
Apuntes Mat023 (versión preliminar actualizada 23-05-2014)

2
si cortamos con el plano y = 0 se obtiene la curva z = f (x, 0) = x2 e−2x

la recta tangente en x = 1 es

d  2 −2x2 
z − f (1, 0) = xe (x − 1)
dx
x=1

esto es

2 2
z − e−2 = 2xe−2x − 4x3 e−2x (x − 1)
x=1
z − e−2 = 2e−2 − 4e−2 (x − 1)


es decir

x = x
y = 0
z = e−2 − 2e−2 (x − 1)

352
Apuntes Mat023 (versión preliminar actualizada 23-05-2014)

de manera similar, si cortamos con el plano x = 1 se obtiene la curva z = f (1, y) =


4 +2y
(1 − 2y) e−2−y la recta tangente en y = 0 es

d  
−2−y 4 +2y
z − f (1, 0) = (1 − 2y) e (y − 0)
dy y=0

esto es

4 +2y−2 
z − e−2 = −4ye−y −2y 3 + y 2 + 1 y
y=0
z − e−2 = 0

es decir

x = 1
y = y
z = e−2
   
1 0
   
los directores de estas rectas son 
 0  y  1  se sigue que el normal corresponde
  
−2
−2e 0
a      
1 0 2e−2
     
 0 × 1 = 0 
     
−2
−2e 0 1

353
Apuntes Mat023 (versión preliminar actualizada 23-05-2014)

y ası́ el plano tangente corresponde a

2e−2 , 0, 1 · x − 1, y − 0, z − e−2 = 0
 

es decir
2e−2 (x − 1) + z − e−2 = 0

o equivalentemente
z = e−2 − 2e−2 (x − 1)

en la figura, en blanco están las curvas y las rectas tangentes, la intersección de estas
rectas y curvas corresponde al punto (1, 0, f (1, 0)) y el plano que contiene a estas rectas
tangentes es el plano tangente, como se puede apreciar la propiedad de tangencia es local,
no significa que toque a la gráfica en un único punto.

Ejercicios de la sección

1. Considere la función f : R2 → R, (x, y) → f (x, y) = ex sin y + e2y cos x. Determine


las ecuaciones paramétricas de las rectas tangentes a la gráfica de f en el punto
(0, 0, 1) que son paralelas a los planos x = 0 e y = 0.

354
Apuntes Mat023 (versión preliminar actualizada 23-05-2014)

2. Determine una recta normal a la superficie

y2
x2 + + z2 = 1
4
 
en el punto √1 , 1, 1 .
2 2

2 −y 4 +2y
3. Si f : R2 → R, (x, y) → f (x, y) = (x2 − 2xy) e−2x estudiar los extremos
(máximos y mı́nimos) locales y globales de la función g : R → R, t → g (t) = f (t, 0).

Diferenciabilidad

Definición 9.3.1. Sean U ⊆ Rn y f : U → R una función. Diremos que f es diferenciable



en a ∈ U si existe una transformación lineal T : Rn → R tal que:

|f (a + h) − f (a) − T (h)|
lı́m =0
h→0 khk

Observación 9.3.1. Si n = 2, por ejemplo, el lı́mite en la definición anterior toma la


forma:
|f (a + h, b + k) − f (a, b) − T (h, k)|
lı́m √ =0
(h,k)→(0,0) h2 + k 2
Recordemos, además, que si T : Rn → R es una transformación lineal, entonces
existen constantes A1 , A2 , . . . , An ∈ R (respecto, por ejemplo, de las bases canónicas,
respectivamente) tales que:
 
[T ] = A1 A2 · · · An ∈ M1×n (R)

El siguiente teorema indica la condición necesaria que deben cumplir las constantes
A1 , A2 , . . . , An en caso de que una función f sea diferenciable en a:

Teorema 9.3.1. Sean U ⊆ Rn y f : U → R una función diferenciable en a ∈ U , entonces:

∂f
Ai = (a)
∂xi

para cada i = 1, 2, . . . , n.

355
Apuntes Mat023 (versión preliminar actualizada 23-05-2014)

Demostración. Como f : U → R es diferenciable en a, existe una transformación lineal


T : Rn → R tal que:
|f (a + h) − f (a) − T · h|
lı́m =0
h→0 khk
Note que haciendo:
h = t · ei

el lı́mite anterior implica el siguiente lı́mite:


|f (a + t · ei ) − f (a) − T (t · ei )|
lı́m =0 (9.1)
t→0 kt · ei k
Por otro lado, suponga que la transformación lineal T : Rn → R tiene representación
matricial:
 
[T ]1C = A1 A2 · · · An

donde C es la base canónica de Rn y 1 representa la base canónica de R. Ası́:

[T · (t · ei )]1 = [T ]1C [t · ei ]C
= t · Ai

Ası́:
|f (a + t · ei ) − f (a) − T (t · ei )| |f (a + t · ei ) − f (a) − t · Ai |
lı́m = lı́m
t→0 kt · ei k t→0 |t|

f (a + t · ei ) − f (a)
= lı́m − Ai
t→0 t

∂f
= (a) − Ai
∂xi

y como el valor del lı́mite en (9.1) es 0 (por la diferenciabilidad de f en a) se tiene que:



∂f
∂xi (a) − Ai = 0

Esto es:
∂f
(a) = Ai
∂xi
para cada i = 1, 2, . . . , n.

Observación 9.3.2. Se sigue entonces que si f es diferenciable en x = a las derivadas


parciales deben existir en el punto.

356
Apuntes Mat023 (versión preliminar actualizada 23-05-2014)

Podemos entonces reformular la definición como:

Definición 9.3.2. Sean U ⊆ Rn y f : U → R una función. Diremos que f es diferenciable



en a ∈ U si:

∂f
1. Para cada i = 1, 2, . . . , n, existen las derivadas parciales ∂xi
(a).

2. Asumiendo, h = (h1 , h2 , . . . , hn ) ∈ Rn , se tiene que:


Pn ∂f
f (a + h) − f (a) − i=1 ∂x (a) h

i
i
lı́m =0
h→0 khk

Observación 9.3.3. Con el cambio de variables h = x − a el lı́mite


Pn ∂f
f (a + h) − f (a) − i=1 ∂x (a) h

i
i
lı́m =0
h→0 khk
se puede escribir en la forma
Pn ∂f
f (x) − f (a) − i=1 ∂x (a) (x − a )

i
i i
lı́m =0
x→a kx − ak
Observación 9.3.4. Se debe hacer notar que la suma:
n
X ∂f
(a) · hi
i=1
∂xi

se obtiene vı́a el isomorfismo canónico que existe entre M1×1 (R) y el espacio vectorial real
unidimensional. En efecto sabemos que:
 
[T ]1C = ∂x
∂f
(a) ∂f
∂x2
(a) · · · ∂f
∂xn
(a)
1

y como h = (h1 , h2 , . . . , hn ), tenemos:

[T · h]1 = [T ]1C [h]C


 
h1
 

∂f ∂f ∂f
 h2 
= (a) (a) · · · (a) 
 
∂x1 ∂x2 ∂xn .. 

 . 

hn
n
!
X ∂f
= (a) · hi
i=1
∂x i
1×1

357
Apuntes Mat023 (versión preliminar actualizada 23-05-2014)

Ejemplo 9.3.1. Considere la función f : R2 → R definida por:



2 4
 xy

si (x, y) 6= (0, 0)
x2 + y 2
0 si (x, y) = (0, 0)

estudiar la diferenciabilidad de f en (0, 0).

Solución. Notemos que


∂f f (h, 0) − f (0, 0)
(0, 0) = lı́m
∂x h→0 h
h2 04
2 2 − 0
= lı́m h +0
h→0 h
= lı́m 0
h→0
= 0

de manera similar
∂f f (0, h) − f (0, 0)
(0, 0) = lı́m
∂y h→0 h
02 h4
2 2 − 0
= lı́m 0 +h
h→0 h
= lı́m 0
h→0
= 0

luego las derivadas parciales existen, notemos que



f (x, y) − f (0, 0) − ∂f (0, 0) (x − 0) − ∂f
(0, 0) (y − 0)

∂x ∂y
lı́m p
(x,y)→(0,0) x2 + y 2
2 4
xy
x2 +y2
= lı́m p
(x,y)→(0,0) x2 + y 2
x2 y 4
= lı́m (por polares)
(x,y)→(0,0) (x2 + y 2 )3/2

= 0

se sigue que f es diferenciable en (0, 0).

Ejemplo 9.3.2. Considere la función g : R3 → R definida por:



 sin (xyz)

6 (0, 0, 0)
, (x, y, z) =
g (x, y, z) = x2 + y 2 + |z|
0 , (x, y, z) = (0, 0, 0)

358
Apuntes Mat023 (versión preliminar actualizada 23-05-2014)

Demuestre que g es diferenciable en el punto (0, 0, 0).

Solución. Calculamos, primeramente, las derivadas parciales de f en (0, 0, 0). Esto es:
∂f f (t, 0, 0) − f (0, 0, 0)
(0, 0, 0) = lı́m
∂x t→0 t
0
= lı́m
t→0 t

= 0
∂f ∂f
Análogamente, se tiene que (0, 0, 0) = (0, 0, 0) = 0. Luego:
∂y ∂z

∂f ∂f ∂f
f (h, k, l) − f (0, 0, 0) − (0, 0, 0) · h + (0, 0, 0) · k + (0, 0, 0) · l

∂x ∂y ∂z
lı́m √ =
(h,k,l)→(0,0,0) h2 + k 2 + l2
|f (h, k, l)|
= lı́m √
(h,k,l)→(0,0,0) h2 + k 2 + l2
|sin (hkl)|
= lı́m √
(h,k,l)→(0,0,0) (h2 + k2 + |l|) h2 + k 2 + l2
|hkl|
≤ lı́m √
(h,k,l)→(0,0,0) |l| h2 + k 2 + l2
|hkl|
≤ lı́m
(h,k,l)→(0,0,0) |l| |k|

= lı́m |h|
(h,k,l)→(0,0,0)

=0

Por tanto, f es diferenciable en (0, 0, 0).

Ejemplo 9.3.3. Consideremos f : R2 → R la función definida por:



 sin(xy2 ) , (x, y) =
6 (0, 0)
x2 +|y|
f (x, y) =
 0 , (x, y) = (0, 0)

Verifique que f no es diferenciable en (0, 0).

Solución. Notamos, primeramente que:


∂f f (t, 0) − f (0, 0)
(0, 0) = lı́m
∂x t→0
 t 
1 0
= lı́m · 2 =0
t→0 t t +0

359
Apuntes Mat023 (versión preliminar actualizada 23-05-2014)

Análogamente se obtiene que:


∂f
(0, 0) = 0
∂y
Ahora bien, como:

f (h, k) − f (0, 0) − ∂f (0, 0) · h − ∂f
(0, 0) · k

∂x ∂y |f (h, k)|
lı́m √ = lı́m √
(h,k)→(0,0) h2 + k 2 (h,k)→(0,0) h2 + k 2
1 |sin (hk)|
= lı́m √
(h,k)→(0,0) h2 + k 2 h2 + |k|
Ahora bien, note que:
 
1 |sin (hk)|
lı́m lı́m √ = lı́m 0 = 0
h→0 k→0 h2 + k 2 h2 + |k| h→0

y si consideramos la trayectoria ϕ : h = k, tenemos que:


1 |sin (h2 )| 1 |sin (h2 )|
lı́m √ = lı́m √
h→0 h2 + h2 h2 + |h| h→0 2 h3 + h2
1 |sin (h2 )| 1
= √ lı́m
2 h→0 h2 1 + |h|
1
= √
2
Por tanto, f no es diferenciable en (0, 0).

Ejemplo 9.3.4. Toda transformación lineal es diferenciable, la transformación lineal que


mas se parece es la misma función.

Observación 9.3.5. Supongamos que f : U ⊆ R2 → R es una función diferenciable en



(a, b) ∈ U . Esto quiere decir que:

f (a + h, b + k) − f (a, b) − ∂f (a, b) h − ∂f
(a, b) k

∂x ∂y
lı́m √ =0 (9.2)
(h,k)→(0,0) h2 + k 2
Ahora bien, haciendo el cambio de variables:

 x=a+h
 y =b+k

tenemos que:
(h, k) → (0, 0) ⇐⇒ (x, y) → (a, b)

360
Apuntes Mat023 (versión preliminar actualizada 23-05-2014)

Por tanto, el lı́mite en (9.2) queda:



f (x, y) − f (a, b) − ∂f (a, b) · (x − a) − ∂f
(a, b) · (y − b)

∂x ∂y
lı́m q =0
(x,y)→(a,b)
(x − a)2 + (y − b)2

Intuitivamente, para (x, y) suficientemente cercano a (a, b), tenemos que:



∂f ∂f
f (x, y) − f (a, b) − ∂x (a, b) · (x − a) − ∂y (a, b) · (y − b)

q '0
2 2
(x − a) + (y − b)

Ası́:
∂f ∂f
f (x, y) − f (a, b) − (a, b) · (x − a) − (a, b) · (y − b) ' 0
∂x ∂y
y por tanto:

∂f ∂f
f (x, y) ' (a, b) · (x − a) + (a, b) · (y − b) + f (a, b)
∂x ∂y

Es decir, para (x, y) suficientemente cercano a (a, b), f (x, y) puede ser aproximada por el
plano:
∂f ∂f
z = f (a, b) + (a, b) · (x − a) + (a, b) · (y − b)
∂x ∂y
el cual es tangente a la superficie:

S : z = f (x, y)

en una vecindad de (a, b). Ası́, tenemos la siguiente definición:

Definición 9.3.3. Sea f : U ⊆ R2 → R una función diferenciable en (a, b). Llamaremos


plano tangente a la superficie z = f (x, y) en (a, b) al plano de ecuación:

∂f ∂f
z − f (a, b) = (x − a) (a, b) + (y − b) (a, b)
∂x ∂y

Ejemplo 9.3.5. Encontrar la ecuación del plano tangente a la superficie S dada por la
gráfica de la función:
z = (x + y)2 − 2x

en el punto (1, 1, 0).

361
Apuntes Mat023 (versión preliminar actualizada 23-05-2014)

Solución. Calculamos las derivadas parciales


∂z ∂
(x + y)2 − 2x = 2x + 2y − 2

=
∂x ∂x

∂z
(1, 1) = 2
∂x
y
∂z ∂
(x + y)2 − 2x = 2x + 2y

=
∂y ∂y

∂z
(1, 1) = 4
∂y
el plano tangente es
z = 0 + 2 (x − 1) + 4 (y − 1)

es decir
z = 2x + 4y − 6

Ejemplo 9.3.6. Sean a, b, c números reales positivos. Verifique si el cono:


x2 y 2 z2
+ =
a2 b2 c2
y la esfera:
2
b2 + c 2 b2

2 2
= 2 b2 + c 2

x +y + z−
c c
son tangentes entre sı́ en los puntos (0, ±b, c).

Solución. Notemos que los puntos entregados pertenecen a la intersección


02 (±b)2 c2
+ =
a2 b2 c2
y
2
b2 + c 2 b2

2 2
= 2 b2 + c 2

0 + (±b) + c −
c c
Notemos que
x2 y 2 z2
+ 2 =
a2 b c2
⇒ r
x2 y 2
z = c + 2
a2 b
362
Apuntes Mat023 (versión preliminar actualizada 23-05-2014)

(el punto (0, ±b, c) tiene tercera coordenada positiva, por eso consideramos solo la raı́z
positiva). El vector normal al plano tangente es
r ! r ! !
∂ x2 y 2 ∂ x2 y 2
− c + 2 ,− c + 2 ,1
∂x a2 b ∂y a2 b
 

−cx −cy
=  q , q , 1
 
a2 y 2 +b2 x2 a 2 y 2 +b2 x2
a2 a2 b2
b2 a2 b2

(x,y)=(0,±b) (x,y)=(0,±b)
 c 
= 0, ∓ , 1
b
para la superficie
2
b2 + c 2 b2

2 2
= 2 b2 + c 2

x +y + z−
c c
se tiene r 2
2 2


z − b + c = b (b2 + c2 ) − (x2 + y 2 )
c c2
luego r
b2 + c 2 b2 2
z− =± (b + c2 ) − (x2 + y 2 )
c c2
evaluando en (0, ±b, c) se tiene
r
b2 + c 2 b2 2
c− = ± (b + c2 ) − b2
c c2

2 2
b +c b2
c− = ±
c c
se sigue que el signo correcto es −. Luego
r
b2 + c 2 b2 2
z= − (b + c2 ) − (x2 + y 2 )
c c2
ası́ el normal corresponde a
(−zx , −zy , 1)|(0,±b)

donde
r !
2 2
∂ b +c b2
zx = − (b2 + c2 ) − (x2 + y 2 )
∂x c c2
x
= q
b4 +b2 c2 −c2 x2 −c2 y 2
c2

363
Apuntes Mat023 (versión preliminar actualizada 23-05-2014)

r !
2 2
∂ b +c b2
zy = − (b2 + c2 ) − (x2 + y 2 )
∂y c c2
y
= q
b4 +b2 c2 −c2 x2 −c2 y 2
c2

evaluando

zx = 0
±b ±b c
zy = q = b2

b4 +b2 c2 −c2 (±b)2
c
b
c2

ası́

(−zx , −zy , 1)|(0,±b)


 c 
= 0, ∓ , 1
b
se sigue que tienen los mismos planos tangentes.

Teorema 9.3.2. Sean U ⊆ Rn y f : U → R una función diferenciable en a ∈ U , entonces
f es continua en a.

Demostración. Dado ε > 0, existe δ > 0 tal que:



n
X ∂f
f (a + h) − f (a) − (a) · hi ≤ ε khk

∂xi


i=1

para todo khk < δ. Entonces, por la desigualdad triangular, tenemos que:

n
X ∂f
|f (a + h) − f (a)| ≤ ε khk + (a) · hi


i=1
∂x i
n
X ∂f
≤ ε khk + khk
∂xi (a)

i=1

≤ (ε + n A) khk
n o
∂f  ε

donde A = máx ∂x (a) : i = 1, 2, . . . , n . Ahora bien, eligiendo δ̃ = mı́n δ, se

i ε+n A

obtiene que:
khk < δ̃ =⇒ |f (a + h) − f (a)| < ε

364
Apuntes Mat023 (versión preliminar actualizada 23-05-2014)

Observación 9.3.6. Como ya es sabido desde el cálculo diferencial en una variable, el


teorema anterior dice que la continuidad es una condición necesaria para la diferenciabilidad,
pero no es suficiente. En efecto, considere:

Ejemplo 9.3.7. Verifique que f : R2 → R definida por:


 3/2
 x |y|

, si (x, y) 6= (0, 0)
f (x, y) = x2 + y 2

 0 , si (x, y) = (0, 0)

es continua, pero no diferenciable en (0, 0).

Solución. La continuidad es inmediata. En efecto:


|x| |y|3/2 |xy| p
= |y|
x2 + y 2 x2 + y 2
p
1 x2 + y 2 p |y|
≤ |y| =
2 x2 + y 2 2
donde la última expresión converge a 0, cuando (x, y) → (0, 0). Ahora bien, por el Teorema
del Sandwich, se obtiene que:

x |y|3/2
lı́m = 0 = f (0, 0)
(x,y)→(0,0) x2 + y 2

Por tanto, f es continua en (0, 0). Por otro lado, es fácil ver que:
∂f ∂f
(0, 0) = (0, 0) = 0
∂x ∂y
Luego:

f (h, k) − f (0, 0) − ∂f (0, 0) h − ∂f
(0, 0) k

∂x ∂y |f (h, k)|
lı́m √ = lı́m √
(h,k)→(0,0) h2 + k 2 (h,k)→(0,0) h2 + k 2
|h | |k|3/2
= lı́m
(h,k)→(0,0) (h2 + k 2 )3/2
Al tomar la recta de aproximación h = k en el último lı́mite se observa que no existe. Por
tanto, la función f no es diferenciable en el (0, 0).

Teorema 9.3.3 (Álgebra de funciones diferenciables). Sean f, g : U ⊆ Rn → R funciones



diferenciables en a ∈ U y α ∈ R una constante, entonces αf, f + g, f − g, f g son
diferenciables en a. Si, además, g (a) 6= 0, entonces f /g también es diferenciable en a.

365
Apuntes Mat023 (versión preliminar actualizada 23-05-2014)


∂f
Teorema 9.3.4. Sean f : U ⊆ Rn → R y a ∈ U . Si las derivadas parciales ∂xi
existen en
una bola alrededor de a y son continuas en a entonces f es diferenciable en a.

Observación 9.3.7. La condición en el teorema anterior es sólo suficiente, mas no necesaria


como puede verse con el siguiente ejemplo:

Ejemplo 9.3.8. Verificar que la función f : R2 → R definida por:


 !

 (x2 + y 2 ) sin p 1
, si (x, y) 6= (0, 0)

f (x, y) = x2 + y 2

0 , si (x, y) = (0, 0)

es diferenciable en todo punto de R2 pero las funciones derivadas parciales no son continuas
en R2 .

Solución. Del álgebra de funciones diferenciables se obtiene que f es diferenciable en todo


punto distinto de (0, 0). Notemos que para (x, y) 6= (0, 0) se cumple
!
∂ 1
x2 + y 2 sin p

∂x x2 + y 2
! ! 
1 2 2
 1 −1 2 2 −3/2

= 2x sin p + x + y cos p x +y 2x
x2 + y 2 x2 + y 2 2
! !
1 x 1
= 2x sin p −p cos p
x2 + y 2 x2 + y 2 x2 + y 2
y por simetrı́a
!
∂ 1
x2 + y 2 sin

p
∂y x2 + y 2
! !
1 y 1
= 2y sin p −p cos p
x2 + y 2 x2 + y 2 x2 + y 2
en (0, 0) calculamos las derivadas parciales
∂f f (h, 0) − f (0, 0)
(0, 0) = lı́m
∂x h→0
h
1
h2 sin |h|
= lı́m
h→0 h
 
1
= lı́m h sin
h→0 |h|
= 0

366
Apuntes Mat023 (versión preliminar actualizada 23-05-2014)

∂f
y por simetrı́a ∂y
(0, 0) = 0. La función derivada parcial esta definida por
    

 2x sin √ 2 2 − √ 2 2 cos √ 2 2
1 x 1
si (x, y) 6= (0, 0)
x +y x +y x +y

∂f 
(x, y) =
∂x 


0 si (x, y) = (0, 0)

esta función no es continua en (0, 0), en efecto, considere los puntos


 
1
(xn , yn ) = ,0
2nπ + π4

entonces si n → +∞, (xn , yn ) → (0, 0) pero


∂f 2  π  π
(xn , yn ) = π sin 2nπ + − cos 2nπ +
∂x 2nπ + 4
4 4
2 1 1
= π √ −√
2nπ + 4 2 2
∂f
luego cuando n → +∞ se sigue (xn , yn ) → (0, 0) pero ∂x
(xn , yn ) → − √12 , el lı́mite no es
cero (usando otra sucesión se muestra que en realidad el lı́mite no existe por ejemplo con
 
1
xn , yen ) = 2nπ+ π , 0 da otro lı́mite). Se sigue que f no es de clase C 1 sin embargo
(f
3

 
2
(x + y 2 ) sin √ 1
|f (x, y)| x2 +y 2
lı́m p = lı́m p
(x,y)→(0,0) x2 + y 2 (x,y)→(0,0) x2 + y 2
!
p 1
= lı́m x2 + y 2 sin p

2 2

(x,y)→(0,0) x +y
= 0

aquı́ hemos usado el acotamiento


!
p
2 2
1
p
0 ≤ x + y sin ≤ x2 + y 2

p
2 2
x +y

luego f es diferenciable en (0, 0) y en todos los otros puntos por álgebra de funciones
diferenciables pero f no tiene derivadas parciales continuas en el origen..

Ejercicios de la sección

1. Hallar la ecuación del plano tangente a la superficie z = x2 + y 3 en el punto (3, 1, 10).

367
Apuntes Mat023 (versión preliminar actualizada 23-05-2014)

2. ¿Por qué deben llamarse tangentes en (0, 0) las gráficas de f (x, y) = x2 + y 2 y


g (x, y) = −x2 − y 2 + xy 3 ?

3. Sea f : R2 → R definida por



x2 y 4


 x2 +y 2
si (x, y) 6= (0, 0)
f (x, y) =


0 si (x, y) = (0, 0)

Probar que f es de clase C 1 (R2 ).

4. Sea p ≥ 1. Estudiar la diferenciabilidad en el origen de la función

k·kp : Rn → R
n
!1/p
X p
x→ kxkp = |xi |
k=1

5. ¿Para que valores de α la función



x3 +xy 2


 |x|α +|y|α
si (x, y) 6= (0, 0)
fα (x, y) =


0 si (x, y) = (0, 0)

es diferenciable en (0, 0)?

6. Considere la superficie definida por


   
3 x
S = (x, y, z) ∈ R : x sin −z =0
y
Si (a, b, c) ∈ S determine el plano tangente a la superficie en ese punto y mostrar que
pasa por el origen.

7. Utilizar la aproximación mediante el plano tangente para estimar el valor de:


10
a) (0,99e0,1 )
q
b) (3,99)2 + (4,01)2 + (2,01)2

8. Sean α > 1 y f : Rn → R, x →f (x) una función tal que, para todo x, y ∈ Rn

|f (x) − f (y)| ≤ kx − ykα

Muestre que f es diferenciable.

368
Apuntes Mat023 (versión preliminar actualizada 23-05-2014)

Derivadas de orden superior y funciones de clase C n

Observación 9.4.1. Sean U ⊆ Rn un conjunto abierto no vacı́o y f : U → R una función


∂f
tal que las derivadas parciales ∂xi
(x) existen para todo x ∈ U . Entonces, podemos definir
∂f ∂f
la función derivada parcial de f como la función ∂xi
: U ⊆ Rn → R dada por x 7→ ∂xi
(x).
Las derivadas parciales de orden superior son, entonces, derivadas parciales de la función
∂f
∂xi
: U ⊆ Rn → R. Más precisamente:

Definición 9.4.1. Sean U ⊆ Rn un conjunto abierto no vacı́o y f : U → Rm una función


∂f
tal que las derivadas parciales ∂xi
(x) existen para todo x ∈ U . Se define la derivada parcial
de segundo orden para f como:

∂ 2f
 
∂ ∂f
(x) = (x)
∂xj ∂xi ∂xj ∂xi

Más precisamente:

∂ 2f
 
1 ∂f ∂f
(x) = lı́m (x + tej ) − (x)
∂xj ∂xi t→0 t ∂xi ∂xi
∂2f
Además, si i 6= j la derivada ∂xj ∂xi
(x) recibe el nombre de derivada parcial mixta. Si i = j,
∂2f
anotamos ∂x2i
(x).
La derivada parcial de tercer orden para f se define como:

∂ 3f
 2 
∂ ∂ f
(x) = (x)
∂xk ∂xj ∂xi ∂xk ∂xj ∂xi
∂2f
considerando ∂xj ∂xi
: U ∗ ⊆ Rn → R, en donde U ∗ corresponde al conjunto de puntos de
U para los cuales la segunda derivada parcial está definida. En particular, si i = j = k,
anotamos entonces:
∂ 3f
(x)
∂x3i
Finalmente, las derivadas parciales de orden superior se definen como las derivadas
∂f
parciales sucesivas de ∂xi
: U ⊆ Rn → R.

Ejemplo 9.4.1. Hallar las derivadas parciales de segundo orden de la función:

x
f (x, y) = arctan
y

369
Apuntes Mat023 (versión preliminar actualizada 23-05-2014)

Solución.
 
∂f ∂ x y
= arctan = 2
∂x ∂x y x + y2
 
∂f ∂ x x
= arctan =− 2
∂y ∂y y x + y2

y las de segundo orden

∂ 2f
 
∂ y −2xy
= =
∂x2 2
∂x x + y 2
(x + y 2 )2
2

∂ 2f x2 − y 2
 
∂ y
= =
∂y∂x ∂y x2 + y 2 (x2 + y 2 )2
∂ 2f x2 − y 2
 
∂ x
= − 2 =
∂x∂y ∂x x + y2 (x2 + y 2 )2
∂ 2f
 
∂ x 2xy
= − 2 =
∂y 2 ∂y x +y 2
(x2 + y 2 )2
notar que, en este caso,
∂ 2f ∂ 2f
+ =0
∂x2 ∂y 2
y
∂ 2f ∂ 2f
=
∂y∂x ∂x∂y
∂ 3z
Ejemplo 9.4.2. Hallar , si:
∂x∂y 2
z = sin xy

Solución.

zy = (sin xy) = x cos xy
∂y
luego

zyy = (x cos xy) = −x2 sin xy
∂y
y finalmente

−x2 sin xy = −2x sin xy − x2 y cos xy

zyyx =
∂x
Definición 9.4.2. Sean U ⊆ Rn un conjunto abierto no vacı́o. Diremos que la función
f : U → R es de clase C n sobre U, o bien que f ∈ C n (U ) si todas las derivadas parciales
hasta el orden n−ésimo son funciones continuas en U .

370
Apuntes Mat023 (versión preliminar actualizada 23-05-2014)

Observación 9.4.2. A modo de resumen, tenemos que:

f ∈ C 1 (U ) =⇒ f es diferenciable en U =⇒ f es continua en U

Teorema 9.4.1 (Schwarz). Sean U ⊆ Rn un conjunto abierto no vacı́o y f ∈ C 2 (U ),


entonces, para todo x ∈ U y para todo i 6= j
∂ 2f ∂ 2f
(x) = (x)
∂xj ∂xi ∂xi ∂xj
Observación 9.4.3. Del teorema anterior y bajo la hipótesis de clase C n adecuada se
puede establecer igualdad de derivadas parciales mixtas de tercer orden y superior, note
por ejemplo que
∂ 3f ∂2
 
∂f
2
=
∂y∂x ∂y∂x ∂x
∂2
 
∂f
=
∂x∂y ∂x
 2 
∂ ∂ f
=
∂x ∂y∂x
 2 
∂ ∂ f
=
∂x ∂x∂y
∂ 3f
=
∂x2 ∂y
si f ∈ C 3 .

Observación 9.4.4. La continuidad de las derivadas parciales de segundo orden es funda-


mental en el teorema anterior. Esto se puede ilustrar considerando el siguiente ejemplo:

Ejemplo 9.4.3. Sea f : R2 → R la función definida por:


  
 xy x22 −y22 si (x, y) 6= (0, 0)
x +y
f (x, y) =
 0 si (x, y) = (0, 0)

Verifique que:
∂ 2f ∂ 2f
(0, 0) 6= (0, 0)
∂y∂x ∂x∂y
Solución. Note que
  2
x − y2

∂f ∂
(x, y) = xy
∂x ∂x x2 + y 2
y (x4 + 4x2 y 2 − y 4 )
=
(x2 + y 2 )2

371
Apuntes Mat023 (versión preliminar actualizada 23-05-2014)

para (x, y) 6= (0, 0) y


∂f f (h, 0) − f (0, 0)
(0, 0) = lı́m =0
∂x h→0 h
luego 
∂f  y(x4 +4x2 y2 −y4 ) si (x, y) 6= (0, 0)
(x, y) = (x2 +y 2 )2
∂x  0 si (x, y) = (0, 0)
se sigue que
∂ 2f
 
1 ∂f ∂f
(0, 0) = lı́m (0, h) − (0, 0)
∂y∂x h→0 h ∂x ∂x
1 h (0 − h4 )
 
= lı́m
h→0 h (02 + h2 )2
= −1

por otro lado


  2
x − y2

∂f ∂
(x, y) = xy
∂y ∂y x2 + y 2
−x (−x4 + 4x2 y 2 + y 4 )
=
(x2 + y 2 )2
y
∂f
(0, 0) = 0
∂y
se sigue
∂ 2f
 
1 ∂f ∂f
(0, 0) = lı́m (h, 0) − (0, 0)
∂x∂y h→0 h ∂y ∂y
1 −h (−h4 )
 
= lı́m
h→0 h (h2 + 02 )2
= 1
∂2f ∂2f
se sigue ∂y∂x
(0, 0) 6= ∂x∂y
(0, 0) en este caso la función no es de clase C 2 (R2 ).

Ejemplo 9.4.4. Si u = ln (x2 + y 2 ) verificar que


∂ 2u ∂ 2u
+ =0
∂x2 ∂y 2
Solución. Derivando
∂u ∂ 2x
ln x2 + y 2 = 2

=
∂x ∂x x + y2
∂u ∂ 2y
ln x2 + y 2 = 2

=
∂y ∂y x + y2

372
Apuntes Mat023 (versión preliminar actualizada 23-05-2014)

y de segundo orden
∂ 2u x2 − y 2
 
∂ 2x
= = −2
∂x2 ∂x x2 + y 2 (x2 + y 2 )2
∂ 2u x2 − y 2
 
∂ 2y
= = 2
∂y 2 ∂y x2 + y 2 (x2 + y 2 )2
ası́
∂ 2u ∂ 2u x2 − y 2 x2 − y 2
+ = −2 + 2 =0
∂x2 ∂y 2 (x2 + y 2 )2 (x2 + y 2 )2

Ejercicios de la sección
 
−3/2
p
1. Si u = arctan xy/ 1 + x2 + y 2 probar que uxy = (1 + x2 + y 2 ) y

15xy
uxxyy =
(1 + x2 + y 2 )7/2

2. Sean a, b, c constantes positivas. Si z = z (x, y) es tal que a2 x2 + b2 y 2 = c2 z 2 entonces


muestre que
zxx zyy = (zxy )2

3. Si u = ln (x2 + y 2 + z 2 ) probar que

xuyz = yuzx = zuxy

xy
4. Si f (x, y) = x+y
demuestre que

∂ 2f ∂ 2f 2
2∂ f
x2 + 2xy + y =0
∂x2 ∂x∂y ∂y 2

Gradiente y matriz jacobiana

Definición 9.5.1. Sean U ⊆ Rn y f : U → R una función tal que todas las derivadas

∂f
parciales ∂xi
(a), i = 1, 2, . . . , n de f existen en a ∈ U . Se define el gradiente de f en a
como el vector en Rn dado por:
 
∂f ∂f ∂f
∇f (a) = (a) , (a) , . . . , (a)
∂x1 ∂x2 ∂xn

373
Apuntes Mat023 (versión preliminar actualizada 23-05-2014)

Ejemplo 9.5.1. Sea f (x, y, z) = 3x2 + sin y 2 + 2xz 2 . Calcule ∇f (x, y, z).

Solución. En este caso


 
∂f (x, y, z) ∂f (x, y, z) ∂f (x, y, z)
∇f (x, y, z) = , ,
∂x ∂y ∂z

es decir
∇f (x, y, z) = 2z 2 + 6x, 2y cos y 2 , 4xz


Ejemplo 9.5.2. Considere f (x, y, z) = 3x2 + 2xy − 4z 3 . Sean S la superficie dada por:

S : f (x, y, z) = 1

y Π el plano tangente a S en (1, 1, 1). Si u, v ∈ Π linealmente independientes, entonces


pruebe que existe λ ∈ R tal que:

u × v = λ · ∇f (1, 1, 1)

Solución. Si u, v ∈ Π son linealmente independientes entonces u × v es normal al plano,


pero de las secciones anteriores sabemos que el normal a la gráfica de una función z = h (x, y)
tiene la forma k (−hx , −hy , 1) donde k es una constante, la superficie f (x, y, z) = 1 la
podemos mirar como la gráfica de
r
3 3 2 1 1
z= x + yx −
4 2 4

se sigue

(−zx , −zy , 1)
 q 
3 3 2 1 1
2√ 3
3 1
x + 2y 4
x + 2
yx − 4
2
= − 2 2 , −2x 2
, 1
3 (3x2 + 2yx − 1) 3 9x + 6yx − 3

evaluando en (1, 1)
 q 
3
3
+ 12 − 1
r 3 1
43 3 1 1 + 2 2 4 4
(−zx , −zy , 1) = − + − , −2 , 1
3 4 2 43+2−1 9+6−3
 
2 1
= − ,− ,1
3 6

374
Apuntes Mat023 (versión preliminar actualizada 23-05-2014)

note que
∇f (x, y, x) = 6x + 2y, 2x, −12z 2


luego

∇f (1, 1, 1) = (6 + 2, 2, −12)
= (8, 2, −12)

se sigue que existe λ ∈ R tal que


 
2 1
u × v =k − , − , 1 = λ∇f (1, 1, 1)
3 6

Observación 9.5.1. El isomorfismo canónico ϕ : Rn ' M1×n (R) vuelve a jugar un rol
importante en relacionar el vector gradiente ∇f (a) y la matriz de la diferencial Df (a).
Sabemos que la diferencial de una función f : Rn → R está dada por la transformación
lineal Df (a) : Rn → R tal que:

|f (a + h) − f (a) − Df (a) · h|
lı́m =0
h→0 khk

pero ϕ−1 (Df (a)) = ∇f (a), por lo tanto, podemos escribir:

|f (a + h) − f (a) − ∇f (a) · h|
lı́m =0
h→0 khk

Observación 9.5.2. Nuestro propósito, ahora, es tratar la diferenciabilidad para funciones


vectoriales de la forma f : U ⊆ Rn → Rm .. Teniendo en cuenta que la topologı́a del espacio
euclidiano Rm se obtiene considerando la norma k·kRm , tenemos que:

Definición 9.5.2. Sean U ⊆ Rn y f : U → Rm una función vectorial. Diremos que f es



diferenciable en a ∈ U si existe una transformación lineal T : Rn → Rm tal que:

kf (a + h) − f (a) − T (h)k
lı́m =0
h→0 khk

Definición 9.5.3. Sean U ⊆ Rn y f : U → Rm una función tal que todas las derivadas
∂fi
parciales existen en a. Llamaremos matriz jacobiana de f en a a la matriz de derivadas
∂xj

375
Apuntes Mat023 (versión preliminar actualizada 23-05-2014)

parciales:
 
∂fi
J f (a) = (a)
∂xj m×n

∂f1 ∂f1 ∂f1


 
(a) (a) · · · (a)

 ∂x1 ∂x2 ∂xn 

 ∂f2 ∂f2 ∂f2 
 (a) (a) · · · (a) 
 ∂x1 ∂x2 ∂xn 
 ∈ Mm×n (R)
= 
 .. .. .. .. 

 . . . . 

 
 ∂fm ∂fm ∂fm 
(a) (a) · · · (a)
∂x1 ∂x2 ∂xn

Ejemplo 9.5.3. Sea f : R2 → R2 definida por f (x, y) = (2x2 y + y, 2xy − y 2 ). Luego, si


anotamos u = 2x2 y + y y v = 2xy − y 2 , entonces la matriz jacobiana de f está dada por:
∂u ∂u
 
 ∂x ∂y 
Jf (x, y) =   ∂v

∂v 
∂x ∂y
 
2
4xy 2x
=  
2y 2x − 2y

Teorema 9.5.1. Sean f : U ⊆ Rn → Rm una función definida por f (x) = (f1 (x) , f2 (x) , . . . , fm (x))

y a ∈ U . f es diferenciable en a, si y solo si, fi : U → R es diferenciable en a, para cada
i = 1, 2, . . . , m. Además:
 
∇f1 (a)
  
 ∇f (a)  ∂fi

2
Df (a) =  = (a) = Jf (a)
 
.. ∂xj

 . 
 m×n

∇fm (a)
 
ϕ ∂fi ∂fi ∂fi
donde ∇fi (a) ∼ ∂x1
(a) ∂x2
(a) · · · ∂xn
(a) , para cada i = 1, 2, . . . , m, mediante el
n
isomorfismo canónico ϕ : R → M1×n (R).
√ 
Ejemplo 9.5.4. ¿Es diferenciable g (x, y, z) = x + sin x cos y 2 , x2 y + 1, z 2 + 1 en R3 ?

Solución. La respuesta es si, pues las funciones componentes son todas diferenciables en
R3 .

376
Apuntes Mat023 (versión preliminar actualizada 23-05-2014)

Observación 9.5.3. Por definición, tenemos que si f : U ⊆ Rn → Rm es diferenciable en


a, entonces existe una transformación lineal T : Rn → Rm tal que:
kf (a + h) − f (a) − T (h)k
lı́m =0
h→0 khk
Además, note que para cada i = 1, 2, . . . , m, se tiene que:

|fi (a + h) − fi (a) − Ti (h)| ≤ kf (a + h) − f (a) − T (h)k

para todo h. Ası́, si h = t · ej , tenemos que:



fi (a + t · ej ) − fi (a)
0 ≤ − Ti (ej )
t

fi (a + t · ej ) − fi (a) − Ti (tej )
=
t
kf (a + t · ej ) − f (a) − T (tej )k
≤ →0
ktej k
si acaso t → 0. Lo anterior implica que:
∂fi
[Ti ]j = (a)
∂xj
en donde [Ti ]j representa el j−ésimo coeficiente de la matriz [Ti ], respecto de las bases
canónicas. Todo lo anterior implica que en caso de que f sea diferenciable en a, la matriz
jacobiana Jf (a) es la matriz de la diferencial Df (a). Desde ahora en adelante, anotamos:

Df (a) = Jf (a)

La regla de la cadena

Teorema 9.6.1. Sean f : U ⊆ Rn → Rm , con U abierto y g : V ⊆ Rm → Rp , con


V abierto, tales que f (U ) ⊆ V . Suponga que f es diferenciable en a ∈ U y que g es
diferenciable en b = f (a) ∈ V , entonces:

g ◦ f : U ⊆ Rn → Rp

es diferenciable en a. Además:

D (g ◦ f ) (a) = Dg (f (a)) Df (a)


= Dg (b) Df (a)

377
Apuntes Mat023 (versión preliminar actualizada 23-05-2014)

Observación 9.6.1. Es importante destacar que Dg (b) ∈ Mp×m (R) y que Df (a) ∈
Mm×n (R).

Observación 9.6.2. Supongamos que z = g (f (x)) e y = f (x), con b = f (a), entonces


la regla de la cadena en términos de las matrices jacobianas queda en la forma:

D (g ◦ f ) (a)

= Dg (b) · Df (a)
 ∂z ∂z1 ∂z1 ∂y1 ∂y1 ∂y1
  
1
(b) (b) ··· (b) (a) (a) · · · (a)
 ∂y1 ∂y 2 ∂ym   ∂x1 ∂x2 ∂xn 
   
 ∂z2 ∂z2 ∂z2   ∂y2 ∂y2 ∂y2 
 ∂y1 (b) ∂y2 (b)
 ··· (b)   (a) (a) · · · (a) 
∂ym  
· ∂x1 ∂x2 ∂x2 
=  
 .. .. ... ..   .. .. .. .. 

 . . .  
  . . . . 

   
 ∂z ∂zp ∂zp   ∂ym ∂ym ∂ym 
p
(b) (b) ··· (b) (a) (a) · · · (a)
∂y1 ∂y2 ∂ym ∂x1 ∂x2 ∂xn
m
!
X ∂zi ∂yk
= (b) (a) ∈ Mp×n (R)
k=1
∂y k ∂x j
i×j

Por tanto, en notaciones clásicas, mirando las componentes del producto, la regla viene
dado por las fórmulas:
m
∂zi X ∂zi ∂yk
=
∂xj k=1
∂yk ∂xj
∂zi ∂y1 ∂zi ∂y2 ∂zi ∂ym
= + + ··· +
∂y1 ∂xj ∂y2 ∂xj ∂ym ∂xj

para i = 1, 2, . . . , p y j = 1, 2, . . . , n.

Ejemplo 9.6.1. Sean g : R3 → R y f : R3 → R3 tales que:



f (x, y, z) = u (x, y, z) ; v (x, y, z) ; w (x, y, z)

Definamos F : R3 → R mediante la ecuación:

F (x, y, z) = (g ◦ f ) (x, y, z)

378
Apuntes Mat023 (versión preliminar actualizada 23-05-2014)

Por la regla de la cadena:

DF (x, y, z) = Dg (u, v, w) · Df (x, y, z)


∂u ∂u ∂u
 

  ∂x ∂y ∂z
 
 
∂g ∂g ∂g  ∂v ∂v ∂v 
= · ∂x

∂u ∂v ∂w  ∂y ∂z 

 ∂w ∂w ∂w 
∂x ∂y ∂z
Ası́, por ejemplo:
∂F ∂g ∂u ∂g ∂v ∂g ∂w
= + +
∂x ∂u ∂x ∂v ∂x ∂w ∂x
Ejemplo 9.6.2. Sea g : R3 → R y consideremos el cambio de variables a coordenadas
esféricas: 
 x = r cos θ sin φ


ϕ (r, θ, φ) : y = r sin θ sin φ


z = r cos φ

∂z ∂z ∂z

Defina z (r, θ, φ) = g x (r, θ, φ) , y (r, θ, φ) , z (r, θ, φ) . Calcule ,
∂r ∂θ
y ∂φ
.

Solución.

∂z ∂g ∂x ∂g ∂y ∂g ∂z
= + +
∂r ∂x ∂r ∂y ∂r ∂z ∂r
∂g ∂g ∂g
= (cos θ sin φ) + (sin θ sin φ) + (cos φ)
∂x ∂y ∂z

∂z ∂g ∂x ∂g ∂y ∂g ∂z
= + +
∂θ ∂x ∂θ ∂y ∂θ ∂z ∂θ
∂g ∂g ∂g
= (−r sin θ sin φ) + (r cos θ sin φ) + (0)
∂x ∂y ∂z
∂g ∂g
= (−r sin θ sin φ) + (r cos θ sin φ)
∂x ∂y

∂z ∂g ∂x ∂g ∂y ∂g ∂z
= + +
∂φ ∂x ∂φ ∂y ∂φ ∂z ∂φ
∂g ∂g ∂g
= (r cos θ cos φ) + (r sin θ cos φ) + (−r sin φ)
∂x ∂y ∂z

379
Apuntes Mat023 (versión preliminar actualizada 23-05-2014)

Ejemplo 9.6.3. Considere:


w = x3 yez

Suponga que: 
2 2
 x=s −t


y = s2 + t2


z =s+t

Pruebe que:
∂w ∂w
(1, 0) + (1, 0) = 10e
∂s ∂t
Solución. Usando la regla de la cadena

∂w ∂w ∂x ∂w ∂y ∂w ∂y
= + +
∂s ∂x ∂s ∂y ∂s ∂y ∂s
∂w ∂w ∂x ∂w ∂y ∂w ∂y
= + +
∂t ∂x ∂t ∂y ∂t ∂y ∂t

recordar que las derivadas de w quedan evaluadas en x = s2 − t2 , y = s2 + t2 , z = s + t ası́

∂w ∂w 2  ∂x
(s, t) = s − t2 , s2 + t2 , s + t (s, t)
∂s ∂x ∂s
∂w 2  ∂y
+ s − t2 , s2 + t2 , s + t (s, t)
∂y ∂s
∂w 2  ∂y
+ s − t2 , s2 + t2 , s + t (s, t)
∂y ∂s

∂w ∂w 2  ∂x
(s, t) = s − t2 , s2 + t2 , s + t (s, t)
∂t ∂x ∂t
∂w 2  ∂y
+ s − t2 , s2 + t2 , s + t (s, t)
∂y ∂t
∂w 2  ∂y
+ s − t2 , s2 + t2 , s + t (s, t)
∂y ∂t

evaluando en (1, 0) se obtiene

∂w ∂w ∂x ∂w ∂y ∂w ∂z
(1, 0) = (1, 1, 1) (1, 0) + (1, 1, 1) (1, 0) + (1, 1, 1) (1, 0)
∂s ∂x ∂s ∂y ∂s ∂z ∂s
∂w ∂w ∂x ∂w ∂y ∂w ∂z
(1, 0) = (1, 1, 1) (1, 0) + (1, 1, 1) (1, 0) + (1, 1, 1) (1, 0)
∂t ∂x ∂t ∂y ∂t ∂z ∂t

380
Apuntes Mat023 (versión preliminar actualizada 23-05-2014)

note que
∂w
3x2 yez

(x, y, z) =
∂x
∂w
x3 e z

(x, y, z) =
∂y
∂w
x3 yez

(x, y, z) =
∂z
evaluando
∇w (1, 1, 1) = (3e, e, e)

por otro lado


∂x ∂x
(s, t) = 2s y (s, t) = −2t
∂s ∂t
∂y ∂y
(s, t) = 2s y (s, t) = 2t
∂s ∂t
∂z ∂z
(s, t) = 1 y (s, t) = 1
∂s ∂t
evaluando
∂x ∂x
(1, 0) = 2 y (1, 0) = 0
∂s ∂t
∂y ∂y
(1, 0) = 2 y (1, 0) = 0
∂s ∂t
∂z ∂z
(1, 0) = 1 y (1, 0) = 1
∂s ∂t
se sigue
∂w ∂w ∂x ∂w ∂y ∂w ∂z
(1, 0) = (1, 1, 1) (1, 0) + (1, 1, 1) (1, 0) + (1, 1, 1) (1, 0)
∂s ∂x ∂s ∂y ∂s ∂z ∂s
= (3e) (2) + (e) (2) + (e) (1)
= 9e

y
∂w ∂w ∂x ∂w ∂y ∂w ∂z
(1, 0) = (1, 1, 1) (1, 0) + (1, 1, 1) (1, 0) + (1, 1, 1) (1, 0)
∂t ∂x ∂t ∂y ∂t ∂z ∂t
= (3e) (0) + (e) (0) + (e) (1)
= e

se sigue
∂w ∂w
(1, 0) + (1, 0) = 10e
∂s ∂t

381
Apuntes Mat023 (versión preliminar actualizada 23-05-2014)

p
Ejemplo 9.6.4. Sea w = f (r) una función diferenciable y r = x2 + y 2 + z 2 . Demuestre
que:  2
dw
= k∇f k2
dr
Solución. La derivada de w respecto a r es
dw
= f 0 (r)
dr
luego  2
dw 2
= (f 0 (r))
dr
por otro lado, usando la regla de la cadena
x
wx = f 0 (r) rx = f 0 (r)
r
0 0 y
wy = f (r) ry = f (r)
r
0 0 z
wz = f (r) rz = f (r)
r
se sigue
 x 2  0 y 2  0 z 2
k∇f k2 = f 0 (r) + f (r) + f (r)
r r r
 0 2
f (r)
x2 + y 2 + z 2

=
r
 0 2
f (r)
r2

=
r
2
= (f 0 (r))

como se querı́a probar.

Ejemplo 9.6.5. Sean f : [0, +∞) → R una función diferenciable y u : R → R definida


por:
u = f k(x, y, z)k2


donde: 
 x = ρ cos ϕ cos ψ


y = ρ cos ϕ sin ψ


z = ρ sin ϕ

∂u
Demuestre que = 0.
∂ϕ

382
Apuntes Mat023 (versión preliminar actualizada 23-05-2014)

Solución. Usando la regla de la cadena, note que u (ρ, ϕ, ψ) = f (g (h (ρ, ϕ, ψ))) donde

p → f (p)
(x, y, z) → g (x, y, z) = x2 + y 2 + z 2
(ρ, ϕ, ψ) → h (ρ, ϕ, ψ) = (ρ cos ϕ cos ψ, ρ cos ϕ sin ψ, ρ sin ϕ)

se sigue

∂u ∂g ∂x ∂g ∂y ∂g ∂z
= f0 + f0 + f0
∂ϕ ∂x ∂ϕ ∂y ∂ϕ ∂z ∂ϕ
0 2

= f ρ (2ρ cos ϕ cos ψ) (−ρ sin ϕ cos ψ)
+f 0 ρ2 (2ρ cos ϕ sin ψ) (−ρ sin ϕ sin ψ)


+f 0 ρ2 (2ρ sin ϕ) (ρ cos ϕ)




= 2ρf 0 ρ2 − cos ϕ sin ϕ cos2 ψ − cos ϕ sin ϕ sin2 ψ + sin ϕ cos ϕ


 

= 2ρf 0 ρ2 (0)


= 0

otra forma de verlo es


u = f k(x, y, z)k2


con 
 x = ρ cos ϕ cos ψ


y = ρ cos ϕ sin ψ


z = ρ sin ϕ

queda
u = f ρ2


luego la derivada es 0.

Ejemplo 9.6.6. Cambiar las variables x, y, z en la ecuación

∂u ∂u ∂u
x +y +z = nu
∂x ∂y ∂z

por ξ, η, ϕ de la forma
x y
ξ= ,η= yϕ=z
z z

383
Apuntes Mat023 (versión preliminar actualizada 23-05-2014)

y probar que la ecuación se transforma en

∂v
ϕ = nv
∂ϕ

deducir de esto que u tiene la forma


x y 
u (x, y, z) = z n F ,
z z

para una función F arbitraria.

Solución. Definimos

v (ξ (x, y, z) , η (x, y, z) , ϕ (x, y, z)) = u (x, y, z)

con el cambio de variable propuesto

∂u ∂v ∂ξ ∂v ∂η ∂v ∂ϕ
= + +
∂x ∂ξ ∂x ∂η ∂x ∂ϕ ∂x
∂u ∂v ∂ξ ∂v ∂η ∂v ∂ϕ
= + +
∂y ∂ξ ∂y ∂η ∂y ∂ϕ ∂y
∂u ∂v ∂ξ ∂v ∂η ∂v ∂ϕ
= + +
∂z ∂ξ ∂z ∂η ∂z ∂ϕ ∂z

se sigue
 
∂u ∂v 1 ∂v ∂v
= + (0) + (0)
∂x ∂ξ z ∂η ∂ϕ
 
∂u ∂v ∂v 1 ∂v
= (0) + + (0)
∂y ∂ξ ∂η z ∂ϕ
∂u ∂v  x  ∂v  y  ∂v
= − 2 + − 2 + (1)
∂z ∂ξ z ∂η z ∂ϕ

se sigue que
∂u ∂u ∂u
x +y +z = nu
∂x ∂y ∂z
se transforma en

∂v  x  ∂v  y  ∂v  x  ∂v  y  ∂v
+ + − + − +z = nv
∂ξ z ∂η z ∂ξ z ∂η z ∂ϕ

es decir
∂v
ϕ = nv
∂ϕ

384
Apuntes Mat023 (versión preliminar actualizada 23-05-2014)

esta ecuación es de variables separadas, la resolvemos

ln |v| = n ln |ϕ| + C

luego
v = Kϕn

note que la constante es respecto a la variable ϕ, luego puede depender de ξ, η, ponemos

v = K (ξ, η) ϕn

luego

u (x, y, z) = v (ξ (x, y, z) , η (x, y, z) , ϕ (x, y, z))


= K (ξ (x, y, z) , η (x, y, z)) ϕ (x, y, z)n
x y 
= K , zn
z z
Observación 9.6.3. Muchas veces abusando de la notación, en lugar de poner

v (ξ (x, y, z) , η (x, y, z) , ϕ (x, y, z)) = u (x, y, z)

simplemente se trabaja con u (ξ, η, ϕ) y se escribe


∂u ∂u ∂ξ ∂u ∂η ∂u ∂ϕ
= + +
∂x ∂ξ ∂x ∂η ∂x ∂ϕ ∂x
Observación 9.6.4. Otra manera de enfrentar el problema anterior, cuando es posible
despejar en forma explı́cita es la siguiente, note que
x y
ξ= ,η= yϕ=z
z z
implica

x = ξϕ
y = ηϕ
z = ϕ

luego
P (ξ, η, ϕ) = u (x (ξ, η, ϕ) , y (ξ, η, ϕ) , z (ξ, η, ϕ))

385
Apuntes Mat023 (versión preliminar actualizada 23-05-2014)

luego

∂P ∂u ∂x ∂u ∂y ∂u ∂z
= + +
∂ϕ ∂x ∂ϕ ∂y ∂ϕ ∂z ∂ϕ
∂u ∂u ∂u
= (ξ) + (η) +
∂x ∂y ∂z

luego

∂P ∂u ∂u ∂u
ϕ = (ξϕ) + (ηϕ) + ϕ
∂ϕ ∂x ∂y ∂z
∂u ∂u ∂u
= x +y +z
∂x ∂y ∂z
= nu
= nP

de donde
P (ξ, η, ϕ) = ϕn K (ξ, η)

y ası́
u (x (ξ, η, ϕ) , y (ξ, η, ϕ) , z (ξ, η, ϕ)) = ϕn K (ξ, η)

esto es  
n x z
u (x, y, z) = z K ,
y y
Estudiaremos el comportamiento de la regla de la cadena en derivadas de orden superior

Ejemplo 9.6.7. Suponiendo z ∈ C 2 (R2 ), reescriba la ecuación diferencial:

∂ 2z ∂2 ∂ 2z
+2 −3 2 =0
∂x2 ∂x∂y ∂y

en nuevas variables u y v, definidas por: u = 3x − y y v = x + y.

Solución. Definamos
P (u (x, y) , v (x, y)) = z (x, y)

entonces
zx = Pu ux + Pv vx

esto es
zx = 3Pu + Pv

386
Apuntes Mat023 (versión preliminar actualizada 23-05-2014)

derivando nuevamente respecto a x

zxx = 3 (Pu )x + (Pv )x


= 3 ((Pu )u ux + (Pu )v vx ) + ((Pv )u ux + (Pv )v vx )
= 3 ((Pu )u 3 + (Pu )v ) + ((Pv )u 3 + (Pv )v )
= 9Puu + 6Puv + Pvv

zxy = (3Pu + Pv )y
= 3 (Pu )y + (Pv )y
= 3 (Puu uy + Puv vy ) + (Pvu uy + Pvv vy )
= 3 (Puu (−1) + Puv ) + (Pvu (−1) + Pvv )
= −3Puu + 2Puv + Pvv

con respecto a la variable y

zy = Pu uy + Pv vy
= −Pu + Pv

derivando nuevamente

zyy = − (Puu uy + Puv vy ) + (Pvu uy + Pvv vy )


= − (−Puu + Puv ) + (−Pvu + Pvv )
= Puu − 2Puv + Pvv

se sigue que
∂ 2z ∂2 ∂ 2z
+ 2 − 3 =0
∂x2 ∂x∂y ∂y 2
queda reescrita como

(9Puu + 6Puv + Pvv ) + 2 (−3Puu + 2Puv + Pvv ) − 3 (Puu − 2Puv + Pvv ) = 0

es decir
(9 − 6 − 3) Puu + (6 + 4 + 6) Puv + (1 + 2 − 3) Pvv = 0

387
Apuntes Mat023 (versión preliminar actualizada 23-05-2014)

ası́
Puv = 0

esta ecuación implica


P (u, v) = F (u) + G (v)

donde F , G son funciones de clase C 2 de una variable se sigue

z (x, y) = P (u (x, y) , v (x, y))


= F (u (x, y)) + G (v (x, y))
= F (3x − y) + G (x + y)
∂2z ∂2 ∂2z
como ejercicio, verificar que si z (x, y) = F (3x − y)+G (x + y) entonces ∂x2
+2 ∂x∂y −3 ∂y 2 =
0.

Ejemplo 9.6.8. Las ecuaciones u = f (x, y) , x = X (s, t) e y = Y (s, t) definen u como


una función de s y t: u = F (s, t). Si las funciones son de clase C 2 pruebe que:
2 2
∂ 2F ∂f ∂ 2 X ∂ 2 f ∂X ∂X ∂Y ∂ 2 f ∂f ∂ 2 Y ∂ 2 f ∂Y
 
= + 2 +2 + + 2
∂s2 ∂x ∂s2 ∂x ∂s ∂x ∂s ∂x∂y ∂y ∂s2 ∂y ∂s
Solución. Por la regla de la cadena

F s = f x xs + f y y s
Fss = (fx )s xs + fx xss + (fy )s ys + fy ys
= (fxx xs + fxy ys ) xs + fx xss
+ (fyx xs + fyy ys ) ys + fy ys
= fxx (xs )2 + 2fxy ys xs + fyy (ys )2 + fx xss + fy ys

se sigue
2 2
∂ 2F ∂ 2f ∂x ∂y ∂ 2 f ∂ 2f ∂f ∂ 2 x ∂f ∂ 2 y
 
∂x ∂y
= +2 + 2 + +
∂s2 ∂x2 ∂s ∂s ∂s ∂x∂y ∂y ∂s ∂x ∂s2 ∂y ∂s2

Gradiente y planos tangentes

Observación 9.7.1. Sea f : U ⊆ Rn → R una función diferenciable tal que ∇f (a) =


6 0.
Consideremos el conjunto de nivel:

S : Lc (f ) = {x ∈ U : f (x) = c}

388
Apuntes Mat023 (versión preliminar actualizada 23-05-2014)

Suponga que λ : t 7→ λ (t) una curva diferenciable de R → Rn tal que Im(λ) ⊆ S. Note
que, en vista de las hipótesis para la curva λ, tenemos que:

f (λ (t)) = c, ∀t

entonces, por la regla de la cadena, tenemos que:


n
0
X ∂f
∇f (λ (t)) · λ (t) = (λ (t)) λ0i (t) = 0
i=1
∂x i

en particular, si existe t0 ∈ R tal que λ (t0 ) = a, entonces, de la ecuación anterior concluı́mos


que:
h∇f (a) , λ0 (t0 )i = 0

como λ es cualquier curva diferenciable tal que se tiene que:

∇f (a) ⊥ S

puesto que λ0 es tangente a S. Se tiene entonces:

Teorema 9.7.1. Sean U un conjunto abierto no vacı́o, f : U ⊆ Rn → R una función


diferenciable y S : Lc (f ) el conjunto de nivel de c para f . Suponga que que f (a) = c (es
decir, a ∈ S) y que ∇f (a) 6= 0 entonces:

∇f (a) ⊥a S

donde ⊥a significa que la ortogonalidad de ∇f (a) al conjunto de nivel S : Lc (f ) se da en


el punto a.

Ejemplo 9.7.1. Hallar un vector normal unitario a la superficie definida por:

2xy 3 z + z ln x + y sin y = 0

en (1, 2π, 0).

Solución. El punto dado pertenece a la superficie 2xy 3 z + z ln x + y sin y = 0 la cual


corresponde al conjunto de nivel cero de la función f : D ⊆ R3 → R dada por

f (x, y, z) = 2xy 3 z + z ln x + y sin y

389
Apuntes Mat023 (versión preliminar actualizada 23-05-2014)

Sabemos que el gradiente es normal a la superficie,


 
1 3 2 3
∇f (x, y, z) = z + 2y z, sin y + y cos y + 6xy z, 2xy + ln x
x
evaluando

0, 2π, 2 (2π)3

∇f (1, 2π, 0) =
0, 2π, 16π 3

=

es un vector normal a la superficie en el punto dado.

Observación 9.7.2. Recordemos que si u0 ∈ Rn la ecuación del plano Π que pasa por u0
y que es normal n es dada por:
n · (x − u0 ) = 0

Por lo tanto, el plano tangente Π a una superficie S : f (x) = c en a, está dado por:

Π : ∇f (a) · (x − a) = 0

pues ∇f (a) ⊥a S y a ∈ S

Ejemplo 9.7.2. Encontrar el plano tangente a la superficie:

x2 + y 2 + z 2 = 1

en (1, 0, 0).

Solución. La superficie es el conjunto de nivel 1 de la función h (x, y, z) = x2 + y 2 + z 2 y


el punto (1, 0, 0) pertenece a la superficie, se sigue que el vector normal al plano tangente
es dado por
∇h (x, y, z) = (2x, 2y, 2z)

ası́ el plano tangente es

∇h (1, 0, 0) · (x − 1, y − 0, z − 0) = 0

es decir
(2, 0, 0) · (x − 1, y − 0, z − 0) = 0

luego
x=1

es el plano tangente.

390
Apuntes Mat023 (versión preliminar actualizada 23-05-2014)

Observación 9.7.3. En secciones anteriores encontramos la ecuación del plano tangente


a la gráfica de una función diferenciable z = f (x, y) en un punto (x0 , y0 , f (x0 , y0 )) el cual
tiene ecuación
∂f ∂f
z = f (x0 , y0 ) + (x0 , y0 ) (x − x0 ) + (x0 , y0 ) (y − y0 )
∂x ∂y
Note que la gráfica corresponde al conjunto

S = (x, y, z) ∈ R3 : z − f (x, y) = 0


el cual puede ser interpretado como el conjunto de nivel cero de F (x, y, z) = z − f (x, y),
luego en el punto
(x0 , y0 , f (x0 , y0 )) = (x0 , y0 , z0 )

el plano tangente a S tiene ecuación

∇F (x0 , y0 , z0 ) · (x − x0 , y − y0 , z − z0 ) = 0

pero
∂F ∂f
(x0 , y0 , z0 ) = − (x0 , y0 )
∂x ∂x
∂F ∂f
(x0 , y0 , z0 ) = − (x0 , y0 )
∂y ∂y
∂F
(x0 , y0 , z0 ) = 1
∂z
ası́ el plano es
 
∂f ∂f
− (x0 , y0 ) , − (x0 , y0 ) , 1 · (x − x0 , y − y0 , z − z0 ) = 0
∂x ∂y
es decir
∂f ∂f
(x0 , y0 ) (x − x0 ) +
z = f (x0 , y0 ) + (x0 , y0 ) (y − y0 )
∂x ∂y
La propiedad del gradiente entonces nos permite obtener el resultado obtenido por otros
métodos.

Derivada direccional y direcciones de crecimiento máximo


Definición 9.8.1. Sean a ∈ U , f : U ⊆ Rn → R una función cualquiera y u ∈ Rn un
vector unitario en Rn (es decir, un vector tal que kuk = 1). Se define la derivada direccional

391
Apuntes Mat023 (versión preliminar actualizada 23-05-2014)

de f en a en la dirección de u como el lı́mite:


∂f f (a + tu) − f (a)
Du (a) = (a) = lı́m
∂u t→0 t
si acaso existe.

Ejemplo 9.8.1. Calcular la derivada direccional de f : R2 → R, (x, y) → f (x, y) =


 
3 2 3 1 √1
x y + 2x y en el punto a = (1, 2) en la dirección u = 2 , 2

Solución. Por definición


∂f f (a + tu) − f (a)
(a) = lı́m
∂u t→0
 t  
f (1, 2) + t √12 , √12 − f (1, 2)
= lı́m
t→0
 t
f 1 + √t2 , 2 + √t2 − f (1, 2)
= lı́m
t→0 t
 3    2  3
1 + √t2 2 + √t2 + 2 1 + √t2 2+ √t
2
− (2 + 24 )
= lı́m
t→0 t
63 √
= 2 (Usando la regla de L’ Hôpital)
2
Observación 9.8.1. Note que los vectores canónicos ei , con i = 1, 2, . . . , n, de Rn son
vectores unitarios, luego las derivadas parciales son casos particulares de la derivada
direccional. Más precisamente, las derivadas parciales son derivadas direccionales en la
dirección de los vectores de la base canónica de Rn .
∂f
Observación 9.8.2. Se debe hacer notar, que ∂u
(a) mide la razón de cambio de f en la
dirección de la recta vectorial:

x = a + tu, t∈R

ası́ como las derivadas direccionales miden la razón de cambio de f es la dirección de los
ejes coordenados. La interpretación de estas derivadas es la misma.

Ejemplo 9.8.2. Sea f : R2 → R la función definida por:


|x| y


 p , si x3 − y 2 =
6 0
2
x +y 2
f (x, y) =
, si x3 − y 2 = 0

 0

392
Apuntes Mat023 (versión preliminar actualizada 23-05-2014)

Verifique que f posee derivada direccional en (0, 0) en cualquier dirección y que no es


diferenciable en dicho punto.

Solución. Sea u = (u, v) un vector unitario


∂f f (tu, tv) − f (0, 0)
(0, 0) = lı́m
∂u t→0 t
|tu| tv
= lı́m q
t→0
t (tu)2 + (tv)2
|t| |u| tv
= lı́m √
t→0 t |t| u2 + v 2

= lı́m |u| v
t→0

= |u| v

∂f ∂f
luego las derivadas direccionales existen en toda dirección. Note que ∂x
(0, 0) = ∂y
(0, 0) = 0
para ello tomar las direcciones (u, v) = (1, 0) y (0, 1) respectivamente.
Para la diferenciabilidad

f (x, y) − f (0, 0) − ∂f (0, 0) (x − 0) − ∂f
(0, 0) (y − 0)

∂x ∂y
lı́m
(x,y)→(0,0) k(x, y) − (0, 0)k
|xy|
p
x2 + y 2
= lı́m p
(x,y)→(0,0) x2 + y 2
|xy|
= lı́m
(x,y)→(0,0) x + y 2
2

note que este lı́mite no existe, usando las trayectorias y = x e y = 0 obtenemos lı́mites
distintos. La función no es diferenciable.

Observación 9.8.3. Del ejemplo anterior podemos afirmar que la existencia de todas las
derivadas direccionales en un punto no implica diferenciabilidad en el punto.

Teorema 9.8.1. Sea f : U ⊆ Rn → R una función diferenciable en a ∈ U . Si u =
(u1 , u2 , . . . , un ) es un vector unitario en Rn , entonces:
∂f
(a) = h∇f (a) , ui
∂u
n
X ∂f
= (a) ui
i=1
∂xi

393
Apuntes Mat023 (versión preliminar actualizada 23-05-2014)

Demostración. Definamos g (t) = f (a + tu), esta función nes diferenciable en 0 por se


compuesta de diferenciables, además por la regla de la cadena

g 0 (t) = ∇f (a + tu) · u

evaluando en t = 0 se obtiene
g 0 (0) = ∇f (a) · u

por otro lado

g (t) − g (0)
g 0 (0) = lı́m
t→0 t−0
f (a + tu) − f (a)
= lı́m
t→0 t
∂f
= (a)
∂u

ası́
∂f
(a) = ∇f (a) · u
∂u

y2
Ejemplo 9.8.3. Verifique que la derivada direccional de z = x
en cualquier punto de la
elipse:
2x2 + y 2 = c2

en la dirección de la normal a la curva es nula.

Solución. Note que la curva es el conjunto de nivel c2 de la función g (x, y) = 2x2 + y 2


luego en el punto (x0 , y0 ) de la elipse, la dirección normal es

∇g (x0 , y0 ) 1
n = =p (4x0 , 2y0 )
k∇g (x0 , y0 )k 16x20 + 4y02
1
= p 2 (2x0 , y0 )
4x0 + y02

394
Apuntes Mat023 (versión preliminar actualizada 23-05-2014)

luego la derivada direccional es


∂z
(x0 , y0 )
∂n
∇g (x0 , y0 )
= ∇z (x0 , y0 ) ·
k∇g (x , y0 )k
 2 0 
1 y0 2y0
= p 2 − , · (2x0 , y0 )
4x0 + y02 x20 x0
2y02 2y02
 
1
= p 2 − +
4x0 + y02 x0 x0
= 0

como se querı́a demostrar.

Observación 9.8.4. Suponiendo las condiciones del teorema anterior, utilizando las
propiedades euclidianas de Rn , obtenemos:
∂f
(a) = h∇f (a) , ui
∂u
= k∇f (a)k kuk cos θ
= k∇f (a)k cos θ

en donde θ = ∠ (∇f (a) ; u) ∈ [0, π]. Note que:


∂f
− k∇f (a)k ≤ (a) ≤ k∇f (a)k
∂u
Ası́, si en particular ∠ (∇f (a) ; u) = 0 entonces
∂f
(a) = k∇f (a)k
∂u
Es decir, la máxima razón de cambio de f se obtiene en la dirección del gradiente ∇f (a) y
de manera similar, si ∠ (∇f (a) ; u) = π entonces
∂f
(a) = − k∇f (a)k
∂u
máximo decrecimiento.

Teorema 9.8.2. Sean U ⊆ Rn abierto y f una función diferenciable en a ∈ U , entonces


∇f (a)
la dirección de máximo crecimiento de f es k∇f (a)k
. Además, obtenemos que:
 
∂f
k∇f (a)k = máx (a) : kuk = 1
∂u

395
Apuntes Mat023 (versión preliminar actualizada 23-05-2014)

De manera similar, la menor derivada direccional de f se obtiene en la dirección de


∇f (a)
− k∇f (a)k
y
 
∂f
− k∇f (a)k = mı́n (a) : kuk = 1
∂u

Ejercicios del capı́tulo

1. Calcule las derivadas parciales de las siguientes funciones:


 
a) f (x, y) = ln sin x+a

y
z
b) f (x, y, z) = xy
1
c) f (x, y) = y
arctan( x )
d ) f (x, y, z) = (x + z)x+y

e) f (x, y) = arcsin √ x
x2 +y 2

2. Suponga que g : R → R es una función continua. Calcule las derivadas parciales de f


si:
R x+y
a) f (x, y) = a
g (t) dt
Z sin(x sin(y+z 2 ))
b) f (x, y, z) = z g (t) dt
xy

3. Calcular las derivadas parciales de las siguientes funciones:

a) f (x, y) = 31 x3 − 3x2 y + 3xy 2 + y 3


p
b) f (x, y) = ln (x + 2 x2 + 3y 2 )
x2 − 2xy
c) f (x, y) =
x+y
d ) f (x, y) = x − x3 y + x2 y 2 − xy 3 + y 4
4

x+y
e) f (x, y) =
x−y
f ) f (x, y) = ln(x2 + y 2 )

g) f (x, y) = xy + y x

396
Apuntes Mat023 (versión preliminar actualizada 23-05-2014)

 
x
h) f (x, y) = x cos
y
 
2
i ) f (x, y) = arctan
x+y
j ) f (x, y, z) = xy+x − y z−y + z x+y

k ) f (x, y, z) = logx (x + y + z)

2x
4. Pruebe que si f (x, y) = entonces fx (3, 1) + fy (3, 1) = 1
xy
∂f ∂f
5. Pruebe que si f (x, y) = Ax4 + 28x2 y 2 + Cy 4 , entonces x +y = 4f (x, y)
∂x ∂y
∂ m+n f (x, y)
6. Sea f (x, y) = ex+y . Pruebe que: m n
= ex+y .
∂x ∂y
∂ 2z ∂ 2z
7. Sea z = f (x, y). Verifique que =
∂x∂y ∂y∂x

a) z = x2 − 4xy + 3y 2

b) z = ln(x + y)

c) z = (x3 + y 3 )10

d ) z = (x + y) sec(xy)

e) z = x2 cos(y −2 )
4
f ) z = arctan(xy)
π
y ∂ 2f ∂ 2f
8. Verifique que f (x, y) = arctan satisface la ecuación + =0
x ∂x2 ∂y 2
9. Hallar
∂ 2z
(x, y),
∂x∂y
p
si z = 2xy + y 2 , siendo 2xy + y 2 > 0.

−n2 kt ∂u ∂ 2u
10. Muestre que u(x, t) = e sin(nx) satisface la ecuación =k 2
∂t ∂x
11. Sea 
x2 y
si (x, y =6= (0, 0)


f (x) = x4 + y 2
0 si (x, y) = (0, 0)

397
Apuntes Mat023 (versión preliminar actualizada 23-05-2014)

Pruebe que existen todas las derivadas direccionales de f en (0,0) pero f no es


continua en el origen.
xy 3

 x2 + y 2 si (x, y) 6= (0, 0)



12. Sea f (x, y) =



0 si (x, y) = (0, 0)

a) ¿f es continua en (0,0)?
∂f ∂f
b) ¿Existen (0, 0), (0, 0)?
∂x ∂y
 4
x (y + 2)2


 x2 + y 2
 si (x, y) 6= (0, −2)
13. Sea f (x, y) =



0 si (x, y) = (0, −2)

a) Analice la continuidad de f en (0,-2).


∂f
b) Calcule, si existe, (0, −2)
∂x

14. Sea
 !
1
(x2 + (y − 1)2 )k sin si (x, y) 6= (0, 1)


 p
x2 + (y − 1)2


f (x, y) =




0 si (x, y) = (0, 1)

∂f ∂f
Determine todos los valores de k ∈ R de modo que (0, 1) y (0, 1) existan.
∂x ∂y
15. Sea y  
x

2 2

 x arctan − y arctan si xy 6= 0

 x y
f (x, y) =



0 si xy = 0

∂f ∂f
Calcule, si existen, (0, 1) y (1, 0)
∂x ∂y
16. Verifique que la función f : R → R2 definida por f (x) = (x2 , x3 ) es diferenciable en
R. Calcule Df (x).

398
Apuntes Mat023 (versión preliminar actualizada 23-05-2014)

17. Sea f : R3 → R2 la función definida por:

f (x, y, z) = xy 2 , yz 2


Demuestre, por definición, que f es diferenciable en R3 . Calcule Df (x, y, z)

18. Considere f : Rn → Rm . Suponga que existe M > 0 tal que:

kf (x) k < M kxk2

para todo x ∈ Rn . Verifique que f es derivable en 0 y que además Df (0) = 0.


p
19. Sea f : R2 → R la función f (x, y) = |xy|. ¿Es f diferenciable en (0, 0)?

20. Sea: 
x2 y 2
, si (x, y) 6= (0, 0)


f (x, y) = x2 + y 4
0 , si (x, y) = (0, 0)

∂f
a) Calcular ∂x
en cada punto donde exista.
∂f
b) Decidir ∂x
es continua en el punto (0, 0) y si acaso f es diferenciable en (0, 0).

21. Demuestre que las siguientes funciones son derivables y calcule su derivada:

a) f (x, y) = ey , sin xy, x2 + 2y 3

b) g (x, y, z, w) = xyzw2 , esin w , x2

22. Calcule ∇f (x, y), en términos de derivadas de g (x) y h (x) , para las funciones f (x, y)
siguientes:

a) f (x, y) = g (x)

b) f (x, y) = g (y)

c) f (x, y) = g (x + y)2 + g (x)h(y) − 3g (x2 − 3y 2 )

23. Verifique lo siguiente:

∂z ∂z
a) Si z = xy + xey/x , entonces x ∂x + y ∂y = xy + z.

399
Apuntes Mat023 (versión preliminar actualizada 23-05-2014)

∂u ∂u
b) Si u = (x − y) (y − z) (z − x), entonces ∂x
+ ∂y
= − ∂u
∂z
.

∂z x
a) Se supone que ∂y
= x2 +y 2
. Calcule z = z (x, y).
∂u x2 +y 2
b) Se supone que ∂x
= x
y que u (1, y) = sin y. Calcule u = u (x, y).

∂z
24. Sea z = logy x. Calcule ∂y .
 
x+y
25. Sea z = f x−y , con f una función clase C 1 (R). Verifique que:

∂z ∂z
+y =0 x
∂x ∂y
 
x+y
26. Sean f : R → R y u (x, y) = xy f xy . Hallar una función escalar g (x, y) tal que
se verifique la relación:
∂u ∂u
x2 − y2 = g (x, y) u (x, y)
∂x ∂y

27. Calcule:
d
(g ◦ f ) (t)

dt t=2
2 t−2 3
si f (t) = (t, t − 4, e ) y g : R → R es tal que:
∂g ∂g ∂g
(2, 0, 1) = 4 ∧ (2, 0, 1) = 2 ∧ (2, 0, 1) = 2
∂x ∂y ∂z

28. Calcule D h (x, y, z), si:

h (x, y, z) = f (u (x, y, z) , v (x, y) , w (y, z))


 
∂2u ∂ ∂u ∂2u
29. Se define ∂x∂y
= ∂x ∂y
. Considere z = u (x, y) eax+by tal que ∂x∂y
= 0. Hallar los
valores de las constantes a, b ∈ R de modo que:
∂ 2z ∂z ∂z
− − +z =0
∂x∂y ∂x ∂y

30. Sea f : Rp → R una función. Se dice que f es homogenea de grado m si, para cada
x ∈ Rp y cualquier t ∈ R se tiene que:

f (t x) = tm f (x)

Demuestre que si f es homogenea de grado m y diferenciable, entonces:

hDf (x) , xi = m f (x)

400
Apuntes Mat023 (versión preliminar actualizada 23-05-2014)

∂z
31. Calcule ∂x
, si:
z = f (u)

con u = xy + xy .

32. Hallar u0 (x), si:


u = f (x, y, z)

donde y = ϕ (x) y ψ (x, y).


 
33. Sea f (x, y, z) = x sin xy − z definida sobre:

A = (x, y, z) ∈ R3 : y 6= 0


Considere la superficie:

S = (x, y, z) ∈ R3 : f (x, y, z) = 0


Verifique que el plano tangente a S en cualquier punto de S pasa por el origen.

34. Encontrar la ecuación del plano tangente a la superficie S dada por la gráfica de la
función:

a) z = x2 + y 2 en el punto (0, 0, 0).

b) z = (x + y)2 − 2x en el punto (1, 1, 0).


p
c) f (x, y) = x2 + y 2 + (x2 + y 2 ) en (1, 0, 2).
p √ 
d ) f (x, y, z) = x2 + 2xy − y 2 + 1 en el punto 1, 1, −2, 3 .

35. Considere la superficie:


 π
S : z 2 = −3x2 y + cos 2xy +
4
√ 
a) ¿Se puede calcular el plano tangente a S en el punto 0, 0, 2 ? Explique.
 √ 
b) ¿Se puede calcular el plano tangente a S en el punto 1, − 4 , 23π ? Explique.
π

36. Sea f (x, y) = x2 − xy + 2y 2 . Hallar la derivada direccional de f en el punto (1, 2) y



en dirección que forma con el eje x positivo un ángulo de 60 .

401
Apuntes Mat023 (versión preliminar actualizada 23-05-2014)

37. Sea f (x, y) = x3 − 2x2 y + xy 2 + 1. Hallar la derivada direccional de f en el punto


(1, 2) en la dirección que va desde este punto al punto (4, 6).

38. Determinar los valores de las constantes a, b, c ∈ R de modo que la derivada direccional
de:
f (x, y, z) = a xy 2 + b yz + c z 2 x3

en el punto (1, 2, −1) tenga un valor máximo de 64 en una dirección paralela al eje


X.

39. Sea f : R2 → R la función definida por:



 x2 y , si x3 − y 2 6= 0
x3 −y 2
f (x, y) =
 0 , si x3 − y 2 = 0

Verifique que f tiene en (0, 0) derivada en cualquier dirección, pero que no es diferen-
ciable en dicho punto.

40. Calcule la derivada direccional de la función f en el punto P y en dirección del vector


v.

a) f (x, y) = 2x2 + 3xy + 4y 2 , P (2, 1), v = (1, 1).


 
x
b) f (x, y) = arctan , (−3, 3), v = (3, 4).
y
c) f (x, y, z) = x2 + 3xy + 4y 2 , (1, 1, 2), v = (2, 3, 0).
 z
x
d ) f (x, y, z) = , P (1, 1, 1), v = (2, 1, −1)
y

41. Calcule la derivada de f (x, y) = x4 + x3 y 2 + y en (1, 1) y en la dirección de la tangente


a la curva y = x4 .

42. Calcule la derivada direccional de f (x, y) = xy + x2 + x4 y, en (1, 1) y en la dirección


del vector que forma una ángulo de 60◦ con el eje x.

43. Encuentre la máxima derivada direccional de f en P .

a) f (x, y) = 2x2 + 3xy + 4y 2 , en P (1, 1).

b) f (x, y, z) = e−(x+y+z) , en P (5, 2, 3).

402
Apuntes Mat023 (versión preliminar actualizada 23-05-2014)

44. Dada la función f (x, y) se sabe que:

f 0 (x0 ; v) = 2 si x0 = (1, 2) y v = (2, 2)


f 0 (x0 ; v) = −2 si x0 = (1, 2) y v = (1, −1)

Calcule ∇f (1, 2) y f 0 (x0 ; v) cuando x0 = (1, 2) y v = (4, 6)

45. Suponga que la temperatura en el punto (x, y, z) está dada por T (x, y, z) = 3x2 +
2y 2 − 4z. Determine el valor de la razón de cambio de T en P (−1, −3, 2) en dirección
a Q(−4, 1, −2). ¿Cuál es la taza de máxima variación?.

46. Hallar los puntos (x, y) y las direcciones para las que la derivada direccional de
f (x, y) = 3x2 + y 2 tiene el valor máximo si (x, y) está en el cı́rculo x2 + y 2 = 1.

47. Calcule la derivada direccional de f (x, y, z) = x2 + y 2 − z 2 en (3, 4, 5) a lo largo de la


curva intersección de las superficies 2x2 + 2y 2 − z 2 = 25 y x2 + y 2 = z 2 .

48. Calcular la ecuación del plano tangente a la superficie dada por:

a) f (x, y) = 3x2 + 8xy en el punto (x0 , y0 ) = (1, 0).


p
b) f (x, y) = x2 + y 2 en el punto (x0 , y0 ) = (1, 2)
Z x2 +y2 √
c) f (x, y) = e− t dt en el punto (x0 , y0 ) = (1, 1)
0
 
1 π 1
d ) f (x, y, z) = sen(xyz) − en el punto 3, ,
2 6 3
49. Hallar una constante c tal que en todo punto de la intersección de las dos esferas:

(x − c)2 + y 2 + z 2 = 3 x2 + (y − 1)2 + z 2 = 1

los planos tangentes correspondientes sean perpendiculares el uno al otro.

50. Demuestre que la superficie x2 − 2yz + y 3 = 4 es ortogonal a cualquiera de las


superficies de la familia x2 + 1 = (2 − 4a)y 2 + az 2 en el punto de intersección
(1, −1, 2).

51. Determinar en que punto de la superficie z = 3xy − x3 − y 3 el plano tangente es


horizontal (paralelo al plano z = 0)

403
Apuntes Mat023 (versión preliminar actualizada 23-05-2014)

∂u ∂u ∂u
52. Pruebe que si u = x2 y + y 2 z + z 2 x, entonces + + = (x + y + z)2
∂x ∂y ∂z
53. Si w = x2 + y 2 + z 2 encuentre el diferencial total de w

xy ∂ 2f ∂ 2f 2
2∂ f
54. Sea f (x, y) = x+y
Demuestre que : x2 + 2xy + y =0
∂x2 ∂x∂y ∂y 2
55. Probar que el volumen formado por el plano tangente a la superficie xyz = m3 , en
cualquier punto, y los planos coordenados es constante.

56. Hallar
∂ 2z
(x, y),
∂x∂y
p
si z = 2xy + y 2 , siendo 2xy + y 2 > 0.

57. Calcular la ecuación del plano tangente a la superficie dada por

z = 3x2 + 8xy

en el punto (x, y) = (1, 0).

58. Calcular las derivadas de segundo orden:

a) f (x, y) = x3 + 3x2 y + 6xy 2 − y 3

b) 2x4 − 3x2 y 2 + y 4 = z

59. Sea f (x, y) = ϕ(x2 + y 2 ), considere ϕ diferenciable, pruebe que:

∂f ∂f
y −x =0
∂x ∂y

∂ 2u 2
2∂ u
60. Demuestre que la función u(x, t) satisface la ecuación 2 = a para:
∂t ∂x2

a) u(x, t) = ϕ(x − at) + ψ(x + at) donde ϕ y ψ son funciones de clase C 2 .

b) u(x, t) = sen(akt) sen(kx) con k ∈ Z.

404
Apuntes Mat023 (versión preliminar actualizada 23-05-2014)

61. Se dice que una función es armónica si verifica que


∂ 2f ∂ 2f
∇2 f = + =0
∂x2 ∂y 2

Sea f : R2 → R una función de clase C 2 armónica y sean

x = eu cos v y = eu sen v

Consideremos la función g : R2 → R definida como

g(u, v) = f (x(u, v) , y(u, v)).

a) Pruebe que "


 2  2 2  2 #
∂g ∂g ∂f ∂f
+ = e2u +
∂u ∂v ∂x ∂y

b) Pruebe que g es armónica. Es decir

∂ 2g ∂ 2g
∇2 g = + =0
∂u2 ∂v 2

62. Si x2 = y 2 + f (x2 + z 2 ), Calcular:


∂y ∂y ∂z ∂z
xy + yz − zx
∂x ∂x ∂x ∂x

∂f ∂f
63. Sea f (x, y) = x2 g(x2 y). Pruebe que x − 2y = 2f (x, y).
∂x ∂y
∂ 2g
64. Sea f (x, y) = g(x, y) eax+by con = 0. Determine los valores de las constantes a
∂x∂y
y b para que
∂ 2f ∂f ∂f
− − +f =0
∂x∂y ∂x ∂y
65. Sea f : R2 → R la función definida por:

 √|x|y , si x3 − y 2 6= 0
x2 +y 2
f (x, y) =
 0 , si x3 − y 2 = 0

Verifique que f es continua en (0, 0), que posee derivada direccional en (0, 0) en
cualquier dirección que no es diferenciable en dicho punto.

405
Apuntes Mat023 (versión preliminar actualizada 23-05-2014)

66. Sea f : R2 → R la función definida por:



 x , si y 6= 0
y
f (x, y) =
 0 , si y = 0

∂f ∂f
a) Calcule ∂x
(0, 0) y ∂y
(0, 0).
∂f
b) Sea u = (α, β) tal que αβ 6= 0. ¿Existe ∂u
(0, 0)?

67. Una mosca se encuentra volando, circularmente, en alguna cocina. La dueña de casa,
ante la presencia del insecto, rocı́a el aire con algún insecticida. La distribución del
veneno en el aire se puede modelar mediante la superficie:

z = 10 + 6 cos x cos y + 3 cos 2x + 4 cos 3y

Ante el peligro, la mosca establece (rápidamente) un sistema de referencia y determina


que se encuentra en el punto π3 , π3 , 6 . ¿En qué dirección deberı́a arrancar la mosca


si quiere sobrevivir?

a) Encontrar la ecuación del plano tangente a la superficie x = e2y−z en (1, 1, 2) y


un vector unitario normal a este plano.

b) Sea f (x, y, z) = z − ex sin y y sea a = ln 3, 3π



2
, −3 . Calcular el valor de c tal
que a ∈ S, donde:
(x, yz) ∈ S ⇐⇒ f (x, y, z) = c

Hallar, además, el plano tangente a esta superficie en a.

c) Sea f (x, y) = √ x2 2 . Hallar el plano tangente a la superficie f (x, y) = z en el


3
 x +y
punto 3, −4, 5 .

68. Una función u está definida por una ecuación de la forma:


 
x+y
u = xy f
xy
Demostrar que u satisface una ecuación en derivadas parciales de la forma:
∂u ∂u
x2 − y2 = u G (x, y)
∂x ∂y
Calcule, además, G (x, y).

406
Apuntes Mat023 (versión preliminar actualizada 23-05-2014)

1 √x
69. Sean k una constante positiva y g (x, t) = 2 kt
. Considere:
Z g(x,t)
2
f (x, t) = e−u du
0

Demuestre que:
∂ 2f ∂f
k 2
=
∂x ∂t
70. Las ecuaciones:

u = f (x, y) ∧ x = X (s, t) ∧ y = Y (s, t)

definen u como una función de s y t, digamos: u = F (s, t).

a) Emplee una forma adecuada de la regla de la cadena para expresar las derivadas
∂F ∂F ∂f ∂f ∂X ∂X ∂Y ∂Y
parciales ∂s
y ∂t
en función de , , , ,
∂x ∂y ∂s ∂t ∂s
y ∂t
.

b) Asumiendo que f ∈ C 2 , demuestre que:


2 2
∂ 2F ∂f ∂ 2 X ∂ 2 f ∂X ∂Y ∂ 2 f ∂f ∂ 2 Y ∂ 2f
 
∂X ∂Y
= + 2 +2 + +
∂s2 ∂x ∂s2 ∂x ∂s ∂s ∂s ∂x∂y ∂y ∂s2 ∂y 2 ∂s

71. Resuelva el ejercicio anterior para los casos:

a) X (s, t) = s + t e Y (s, t) = st.

b) X (s, t) = st e Y (s, t) = st .

c) X (s, t) = 21 (s − t) e Y (s, t) = 12 (s + t).

72. La sustitución x = es e y = et transforma f (x, y) en g (s, t) siendo g (s, t) = f (es , et ).


Se sabe, además, que f satisface la ecuación en derivadas parciales:

∂ 2f 2
2∂ f ∂f ∂f
x2 + y + x + y =0
∂x2 ∂y 2 ∂x ∂t

Demuestre que g satisface otra ecuación de la forma:

∂ 2g ∂ 2g ∂g ∂g
a + b + c + d =0
∂s2 ∂t2 ∂s ∂t

para ciertas constantes a, b, c, d ∈ R. Calcule los valores de dichas constantes.

407
Apuntes Mat023 (versión preliminar actualizada 23-05-2014)

73. Escriba la ecuación:


∂z ∂z
x +y = x2 y
∂x ∂y
en las variables u y v mediante el cambio de variables:

y
u=x ∧ v=
x

74. Represente la ecuación:


∂2 ∂ 2z ∂ 2z
+ 2 − 3 =0
∂x2 ∂x∂y ∂y 2
en las variables u y v definidas por el cambio de variables:

u = 3x − y ∧ v =x+y

75. Escriba la ecuación:


∂ 2z ∂ 2z ∂ 2z
y + (x + y) + x =0
∂x2 ∂x∂y ∂y 2
en términos de u y v las cuales vienen dadas por el cambio de variables:

u = x2 − y 2 ∧ v =y−x

408
Capı́tulo 10 : Máximos y mı́nimos

Extremos locales

Definición 10.1.1. Sean U ⊆ Rn un conjunto abierto y f : U ⊆ Rn → R una función.


Diremos que x0 ∈ U es un punto de:

1. Mı́nimo local, si existe un δ > 0 tal que ∀x ∈ B (x0 , δ) ∩ U, f (x0 ) ≤ f (x)

2. Máximo local, si existe un δ > 0 tal que ∀x ∈ B (x0 , δ) ∩ U, f (x0 ) ≥ f (x)

3. Extremo local si es máximo o mı́nimo local.

Cuando trabajamos con funciones diferenciables de una variable los extremos locales
ocurren en los puntos en los cuales f 0 (x) = 0, este resultado se extiende a funciones de
varias variables.

Suponga que x0 = (x01 , x02 , . . . , x0n ) es un punto de máximo local de f : U ⊆ Rn → R


entonces existe un δ > 0 tal que

∀x ∈ B (x0 , δ) ∩ U, f (x0 ) ≥ f (x)

409
Apuntes Mat023 (versión preliminar actualizada 23-05-2014)

es decir, ∀x ∈ U, kx − x0 k < δ ⇒ f (x0 ) ≥ f (x). Definamos la función

g : I⊆R→R
t → g (t) = f t, x02 , x03 , . . . , x0n


donde I es un intervalo abierto con x01 ∈ I entonces



t, x02 , x03 , . . . , x0n − x01 , x02 , . . . , x0n < δ ⇒ g x01 = f (x0 ) ≥ f t, x02 , x03 , . . . , x0n = g (t)
   

pero

t, x02 , x03 , . . . , x0n − x01 , x02 , . . . , x0n < δ ⇔ t − x01 < δ
 

luego |t − x01 | < δ ⇒ g (x01 ) ≥ g (t) de esta forma g tiene un máximo local en t = x01 se
sigue que como g es diferenciable
g 0 x01 = 0


pero

g (x01 + h) − g (x01 )
g 0 x01

= lı́m
h→0 h
f (x01 + h, x02 , x03 , . . . , x0n ) − f (x01 , x02 , x03 , . . . , x0n )
= lı́m
h→0 h
f (x0 + he1 ) − f (x0 ) ∂f (x0 )
= lı́m =
h→0 h ∂x1

se sigue
∂f (x0 )
=0
∂x1
el mismo procedimiento se puede realizar para todas las variables.

Teorema 10.1.1. Sea f : U ⊆ Rn → R una función diferenciable, donde U es un conjunto


abierto. Si x0 ∈ U es un extremo local de f entonces ∇f (x0 ) = 0.

Definición 10.1.2. Sea f : U ⊆ Rn → R una función diferenciable, donde U es un


conjunto abierto. Llamaremos puntos crı́ticos de f a todos aquellos puntos x0 ∈ U que
cumplen ∇f (x0 ) = 0. Si x0 es un punto crı́tico que no es un extremo local entonces x0 se
dice punto de silla.

410
Apuntes Mat023 (versión preliminar actualizada 23-05-2014)

Ejemplo 10.1.1. Considere la función f :


R2 → R, (x, y) → f (x, y) = x2 + y 2 , esta
función es diferenciable en todo R2 , sus puntos
crı́ticos son aquellos (x, y) tales que

∇f (x, y) = (0, 0)

pero
 
∂f ∂f
(x, y) , (x, y) = (2x, 2y) = (0, 0)
∂x ∂y

es decir, el único punto crı́tico es (0, 0). Por


la forma de la función sabemos que (0, 0) es
un punto de mı́nimo ya que

f (0, 0) = 0 ≤ x2 + y 2 = f (x, y)

más aún es un mı́nimo global (en él, la función


asume el menor valor que puede tomar en todo
su dominio)
Ejemplo 10.1.2. Considere la función f : R2 →
R, (x, y) → f (x, y) = −x2 − y 2 , esta función es
diferenciable en todo R2 , sus puntos crı́ticos son
aquellos (x, y) tales que

∇f (x, y) = (0, 0)

pero
 
∂f ∂f
(x, y) , (x, y) = (−2x, −2y) = (0, 0)
∂x ∂y
es decir, el único punto crı́tico es (0, 0). Por la forma de la función sabemos que (0, 0)
es un punto de máximo ya que

f (0, 0) = 0 ≥ −x2 − y 2 = f (x, y)

más aún es un máximo global (en él, la función asume el mayor valor que puede tomar en
todo su dominio).

411
Apuntes Mat023 (versión preliminar actualizada 23-05-2014)

Ejemplo 10.1.3. Considere la función f : R2 → R, (x, y) → f (x, y) = x2 − y 2 , esta


función es diferenciable en todo R2 , sus puntos crı́ticos son aquellos (x, y) tales que

∇f (x, y) = (0, 0)

pero  
∂f ∂f
(x, y) , (x, y) = (2x, −2y) = (0, 0)
∂x ∂y
es decir, el único punto crı́tico es (0, 0). Note que, si x, y 6= 0 entonces

−y 2 = f (0, y) < 0 = f (0, 0) < f (x, 0) = x2

de esta forma, arbitrariamente cerca de (0, 0) hay puntos en los cuales la función toma
valores mayores y menores. (0, 0) es un punto de silla.

Figura: La silla de montar

Claramente, no siempre es posible realizar este análisis para ver si los puntos son
máximos, mı́nimos o puntos de silla, necesitamos álgún criterio que nos permita distinguir
entre estos tipos de puntos.
En cálculo de una variable tenemos el siguiente criterio: Si x0 es un punto crı́tico de f ,
entonces f 00 (x0 ) > 0 implica que x0 es punto de mı́nimo local y si f 00 (x0 ) < 0 de máximo
local. Si f 00 (x0 ) = 0 no hay información. Este método requiere el signo de la segunda
derivada veamos como se puede generalizar a funciones de varias variables:

412
Apuntes Mat023 (versión preliminar actualizada 23-05-2014)

Proposición 10.1.1. Sea n ∈ N y f : [a, b] → R una función tal que f 0 , f 00 , . . . , f (n)


existen en [a, b] además f (n) es continua en [a, b] y diferenciable en ]a, b[ entonces existe
c ∈ ]a, b[ tal que
n
X f (k) (a) f (n+1) (c)
f (b) = (b − a)k + (b − a)n+1
k=0
k! (n + 1)!

Demostración. La demostración se basa en el teorema del valor medio, Para x ∈ [a, b]


defina n
X f (k) (a)
P (x) = (x − a)k
k=0
k!
y F : [a, b] → R
 
f (b) − P (b)
x → F (x) = f (x) − P (x) − (x − a)n+1
(b − a)n+1
la función esta bien definida además

F (a) = F (b) = 0

por el teorema de Rolle existe un c1 ∈ ]a, b[ tal que F 0 (c1 ) = 0 pero note que F 0 (a) = 0
entonces existe c2 ∈ ]a, c1 [ tal que
F 00 (c2 ) = 0

pero F 00 (a) = 0... este proceso continua hasta la existencia de un c = cn+1 ∈ ]a, cn [ tal que
F (n+1) (c) = 0 como P (n+1) (x) ≡ 0 se sigue
 
(n+1) f (b) − P (b)
f (c) = (n + 1)!
(b − a)n+1
es decir
f (n+1) (c)
(b − a)n+1 + P (b) = f (b)
(n + 1)!

Si f : [a, b] → R es una función tal que f 0 , f 00 existen en [a, b] además f 00 es continua en


[a, b] y diferenciable en ]a, b[ entonces existe c ∈ ]a, b[ tal que
2
X f (k) (a) f (3) (c)
f (b) = (b − a)k + (b − a)3
k=0
k! (n + 1)!

413
Apuntes Mat023 (versión preliminar actualizada 23-05-2014)

es decir
f 00 (a) f (3) (c)
f (b) = f (a) + f 0 (a) (b − a) + (b − a)2 + (b − a)3
2 3!
pongamos x0 y x0 + h en lugar de a, b entonces

0 f 00 (x0 ) 2 f (3) cxx00 +h 3
f (x0 + h) = f (x0 ) + f (x0 ) h + h + h
2 3!

si x0 es un punto crı́tico f 0 (x0 ) = 0 y ası́



f 00 (x0 ) 2 f (3) cxx00 +h 3
f (x0 + h) − f (x0 ) = h + h
2 3!
 !
2 f 00 (x0 ) f (3) cxx00 +h
= h + h
2 3!

Suponga que f 00 (x0 ) 6= 0, como



f (3) cxx00 +h
lı́m h=0
h→0 3!

se sigue que existe un δ > 0 tal que 0 < |h| < δ ⇒



f (3) cx0 +h  |f 00 (x )|
x0 0
<

3! 4

luego para 0 < |h| < δ ⇒



f 00 (x0 ) |f 00 (x0 )| f 00 (x0 ) f (3) cxx00 +h f 00 (x0 ) |f 00 (x0 )|
− < + h< +
2 4 2 3! 2 4
 
x +h
f 00 (x0 ) f (3) cx00
de donde podemos concluir que para 0 < |h| < δ la cantidad 2
+ 3!
h tiene el
mismo signo que f 00 (x0 ). Ası́, si f 00 (x0 ) > 0 entonces
 !
f 00 (x0 ) f (3) cxx00 +h
f (x0 + h) − f (x0 ) = h2 + h
2 3!
> h2

luego, para 0 < |h| < δ se cumple f (x0 + h) > f (x0 ) es decir x0 es un mı́nimo. El mismo
argumento entrega que x0 es un máximo local cuando f 00 (x0 ) < 0 (notar que el signo de h
no influye por estar al cuadrado).

414
Apuntes Mat023 (versión preliminar actualizada 23-05-2014)

Teorema 10.1.2 (Fórmula de Taylor de orden 2 en varias variables). Suponga que U ⊆ Rn


es un abierto y f : U → R tiene derivadas parciales continuas hasta el tercer orden entonces

1
f (x0 + h) = f (x0 ) + ∇f (x0 ) h + hT Hf (x0 ) h + R2 (h, x0 )
2

donde
R2 (h, x0 )
lı́m =0
khk→0 khk2
 
h1
 
 h 
 2 
h= . 
 .. 
 
hn
y

∂ 2f ∂ 2f ∂ 2f ∂ 2f
 
(x0 ) (x0 ) (x0 ) ··· (x0 ) 

 ∂x21 ∂x1 ∂x2 ∂x1 ∂x3 ∂x1 ∂xn 
 
 
 ∂ 2f ∂ 2f ∂ 2f 2
∂ f
 
···

(x ) (x0 ) (x0 ) (x0 ) 
 ∂x2 ∂x1 0

 ∂x22 ∂x2 ∂x3 ∂x2 ∂xn 

 
 
∂ 2f ∂ 2f ∂ 2f 2
∂ f
 
Hf (x0 ) =  (x ) (x0 ) (x0 ) ··· (x0 ) 

 ∂x3 ∂x1 0 ∂x23

∂x3 ∂x2 ∂x3 ∂xn 
 
 
 
 .. .. .. .. .. 

 . . . . . 

 
 
 
 ∂ 2f ∂ 2f ∂ 2f 2
∂ f 
(x0 ) (x0 ) (x0 ) ··· (x 0 )
∂xn ∂x1 ∂xn ∂x2 ∂xn ∂x3 ∂x2n

La demostración es el teorema de Taylor de una variable y la regla de la cadena, en


efecto, si ponemos
g (t) = f (x0 + ht)

entonces

g (1) = f (x0 + h)
g (0) = f (x0 )

415
Apuntes Mat023 (versión preliminar actualizada 23-05-2014)

y por la regla de la cadena g 0 (t) = Df (x0 + ht) D (x0 + ht) es decir

g 0 (t) = ∇f (x0 + ht) · h


n
X ∂f
= (x0 + ht) hi
i=1
∂xi

ası́ n
0
X ∂f
g (0) = (x0 ) hi = ∇f (x0 ) · h
i=1
∂x i

además
n
!
d X ∂f
g 00 (t) = (x0 + ht) hi
dt i=1
∂x i
n  
X d ∂f
= hi (x0 + ht)
i=1
dt ∂xi

usando la regla de la cadena


n  
00
X ∂f
g (t) = hi ∇ (x0 + ht) · h
i=1
∂xi
n n
!
X X ∂ 2f
= hi (x0 + ht) hj
i=1 j=1
∂xj ∂xi
n Xn
X ∂ 2f
= (x0 + ht) hj hi
i=1 j=1
∂xj ∂xi
n 2
X ∂ f
= (x0 + ht) hj hi
i,j=1
∂xj ∂xi

de manera similar n
X ∂ 2f
g 000 (t) = (x0 + ht) hj hi hk
i,j,k=1
∂x k ∂xj ∂xi

se sigue
g 00 (0) 2 g 000 (c) 3
g (1) = g (0) + g 0 (0) 1 + 1 + 1
2! 3!
reemplazando

f (x0 + h)
n n n
X ∂f 1 X ∂ 2f 1 X ∂ 2f
x0 + hc10 hj hi hk

= f (x0 ) + (x0 ) hi + (x0 ) hj hi +
i=1
∂xi 2! i,j=1 ∂xj ∂xi 3! i,j,k=1 ∂xk ∂xj ∂xi

416
Apuntes Mat023 (versión preliminar actualizada 23-05-2014)

note que
n
1 X ∂ 2f 1
(x0 ) hj hi = hT Hf (x0 ) h
2! i,j=1 ∂xj ∂xi 2
donde  
h1
 
 h2 
h=
 
.. 

 . 

hn
y
∂ 2f ∂ 2f ∂ 2f ∂ 2f
 
(x0 ) (x0 ) (x0 ) ··· (x0 ) 

 ∂x21 ∂x1 ∂x2 ∂x1 ∂x3 ∂x1 ∂xn 
 
 
 ∂ 2f ∂ 2f ∂ 2f 2
∂ f
 
···

(x ) (x0 ) (x0 ) (x0 ) 
 ∂x2 ∂x1 0

 ∂x22 ∂x2 ∂x3 ∂x2 ∂xn 

 
 
∂ 2f ∂ 2f ∂ 2f ∂ 2f
 
Hf (x0 ) =  (x ) (x0 ) (x0 ) ··· (x0 ) 

 ∂x3 ∂x1 0 ∂x23

∂x3 ∂x2 ∂x3 ∂xn 
 
 
 
 .. .. .. .. .. 

 . . . . . 

 
 
 
 ∂ 2f ∂ 2f ∂ 2f 2
∂ f 
(x0 ) (x0 ) (x0 ) ··· (x 0 )
∂xn ∂x1 ∂xn ∂x2 ∂xn ∂x3 ∂x2n

Observación 10.1.1. Bajo las hipótesis de continuidad del teorema se tiene


∂ 2f ∂ 2f
=
∂xi ∂xj ∂xj ∂xi
por lo tanto la matriz Hf (x0 ) llamada Matriz Hessiana es una matriz simétrica.

Ahora bien, si x0 es punto crı́tico entonces ∇f (x0 ) = 0 se sigue


n
X ∂f
(x0 ) hi = 0
i=1
∂xi
luego la expansión en Taylor quedará
1
f (x0 + h) = f (x0 ) + hT Hf (x0 ) h + R2 (h, x0 )
2
y podemos generalizar el criterio de la segunda derivada de funciones reales.

417
Apuntes Mat023 (versión preliminar actualizada 23-05-2014)

Definición 10.1.3. Sea A una matriz simétrica. Diremos que una forma cuadrática
CA : Rn → R, x → CA (x) = xT Ax es definida positiva si para todo x ∈ Rn se cumple
CA (x) = xT Ax ≥ 0 y CA (x) = 0 solo para x = 0. Diremos que es definida negativa si
para todo x ∈ Rn se cumple CA (x) = xT Ax ≤ 0 y CA (x) = 0 solo para x = 0.

Recordemos que toda matriz simétrica A es diagonalizable ortogonalmente, se sigue


que existe una matriz Q invertible QQT = I tal que

A = QT DQ

donde D = diag (λ1 , λ2 , . . . , λn ) es una matriz diagonal entonces

xT Ax = (Qx)T D (Qx)

si ponemos  
y1
 
 y2 
y = Qx = 
 
.. 

 . 

yn
entonces n
X
T
x Ax = λi yi2
i=1
T
la forma x Ax es definida positiva (negativa) si y solo si todos los valores propios de A
son positivos (negativos).

Proposición 10.1.2. Si la forma xT Ax es definida positiva entonces existe c > 0 tal que

xT Ax ≥ c kxk2

En efecto, basta considerar c como el mı́nimo de los valores propios de la matriz A.


entonces, siguiendo el cálculo anterior con

xT Ax = (Qx)T D (Qx)
 
y1
 
 y 
2
y = Qx =  . 
 
 .. 
 
yn

418
Apuntes Mat023 (versión preliminar actualizada 23-05-2014)

se tiene n n
X X
xT Ax = λi yi2 ≥ c yi2 = c kyk2
i=1 i=1
pero
kyk2 = kQxk2 = (Qx)T (Qx) = xT QT Qx = xT Ix = kxk2

dando el resultado deseado.

Teorema 10.1.3. Sean U ⊆ Rn abierto, f : U ⊆ Rn → R una función de clase C 3 (U ) y


x0 es un punto crı́tico de f . Si f (h) = 12 hT Hf (x0 ) h es definida positiva entonces x0 es
un mı́nimo relativo, si es definida negativa es un máximo relativo.

Demostración. Supongamos que es definida positiva, por la proposición anterior existe un


c > 0 tal que
1 T
h Hf (x0 ) h ≥ c khk2
2
y de la expansión de Taylor tenemos

hT Hf (x0 ) h
f (x0 + h) = f (x0 ) + ∇f (x0 ) h + + R2 (h, x0 )
2

donde
R2 (h, x0 )
lı́m =0
khk→0 khk2
como x0 es punto crı́tico ∇f (x0 ) · h = 0 y ası́

hT Hf (x0 ) h
f (x0 + h) = f (x0 ) + + R2 (h, x0 )
2
≥ f (x0 ) + c khk2 + R2 (h, x0 )

como
R2 (h, x0 )
lı́m =0
khk→0 khk2

existe un δ > 0 tal que 0 < khk < δ ⇒ R2khk
(h,x0 ) c
2 < 2 se sigue para 0 < khk < δ
 
R2 (h, x0 )
2
f (x0 + h) − f (x0 ) ≥ khk c +
khk2
 c  c khk2
≥ khk2 c − = >0
2 2

419
Apuntes Mat023 (versión preliminar actualizada 23-05-2014)

ası́ 0 < khk < δ implica


f (x0 + h) ≥ f (x0 )

luego x0 es un punto de mı́nimo local ( es lo mismo quer decir, si x ∈ B (x0 ; δ) entonces


f (x) ≥ f (x0 )). La demostración de Máximo local es similar.

Teorema 10.1.4. Sean U ⊆ Rn abierto, f : U ⊆ Rn → R una función de clase C 3 (U ) y


x0 es un punto crı́tico de f . Si Hf (x0 ) tiene todos sus valores propios no nulos y existen
dos con distinto signo entonces x0 es un punto silla.

Existen otros métodos para determinar si una matriz es definida positiva o negativa.
Sea  
a11 a12 . . . a1k . . . a1n
 

 a21 a22 . . . a2k . . . a2n 
 .. .. . . .. .. .. 
 . . . . . . 
A=
 


 ak1 ak2 . . . akk . . . akn 
 .. .. .. .. 

 . . ... . ... .  
an1 an2 . . . ank . . . ann
Denotaremos por Ak a la submatriz
 
a11 a12 . . . a1k
 
 a21 a22 . . . a2k 
Ak = 
 
.. .. . . .
. ..


 . . 

ak1 ak2 . . . akk
de esta forma

A1 = a11
 
a11 a12
A2 =  
a21 a22
 
a11 a12 a13
 
A3 =  a a a
 21 22 23 

a31 a32 a32


..
.
An = A

420
Apuntes Mat023 (versión preliminar actualizada 23-05-2014)

si los determinantes de todas estas submatrices son positivos la matriz generará una
forma cuadrática definida positiva y el punto crı́tico es un mı́nimo. Si los determinantes
se alternan en signo comenzando con el determinante de A1 negativo, A2 positivo, etc,
entonces la forma cuadrática será definida negativa y el punto crı́tico es un máximo local.
Si los determinantes son no nulos y no cumplen con los ordenes de signos anteriores, el
punto critico es punto silla.

Observación 10.1.2 (Importante). Los criterios anteriores no entregan información si


existen valores propios nulos o los subdeterminates son cero.

Ejemplo 10.1.4. Analizar los extremos locales de la función f : R2 → R, (x, y) →


f (x, y) = ln (x2 + y 2 + 1)

Solución. Los extremos locales están en los puntos crı́ticos. Los puntos crı́ticos de f son
aquellos puntos (x, y) tales que
∇f (x, y) = (0, 0)

pero
∂f 2x
(x, y) = 2
∂x x + y2 + 1
∂f 2y
(x, y) = 2
∂y x + y2 + 1
se sigue que el único punto crı́tico es (x, y) = (0, 0). Veamos si es un máximo, mı́nimo local
o un punto de silla. Calculemos la Hessiana, para ello necesitamos las derivadas de segundo
orden:
∂ 2f 2 (x2 + y 2 + 1) − 2x (2x) 2y 2 − 2x2 + 2
(x, y) = =
∂x2 (x2 + y 2 + 1)2 (x2 + y 2 + 1)2
∂ 2f −4xy
(x, y) =
∂x∂y (x2 + y 2 + 1)2
∂ 2f 2 (x2 + y 2 + 1) − 2y (2y) 2x2 − 2y 2 + 2
(x, y) = =
∂y 2 (x2 + y 2 + 1)2 (x2 + y 2 + 1)2
2y 2 − 2x2 + 2 −4xy
 
 (x2 + y 2 + 1)2 (x2 + y 2 + 1)2 
 
Hf (x, y) = 
 

2 2
 
 −4xy 2x − 2y + 2 
(x2 + y 2 + 1)2 (x2 + y 2 + 1)2

421
Apuntes Mat023 (versión preliminar actualizada 23-05-2014)

entonces  
2 0
Hf (0, 0) =  
0 2


2 0
aplicando el criterio de los subdeterminantes vemos que |A1 | = 2 > 0 y =4>0
0 2


luego el extremo local es un mı́nimo.

Ejemplo 10.1.5. Analizar los extremos de f (x, y) = x3 y + xy 5 + xy.

Solución. Buscamos los puntos crı́ticos

∇f (x, y) = (0, 0)

es decir

∂f
(x, y) = 3x2 y + y 5 + y = y 3x2 + y 4 + 1 = 0

∂x
∂f
(x, y) = x3 + 5xy 4 + x = x x3 + 5y 4 + 1 = 0

∂y

luego el único punto crı́tico es (x, y) = (0, 0). La Hessiana en este caso es
 
6xy 3x2 + 5y 4 + 1
 
Hf (x, y) = 



3x2 + 5y 4 + 1 20xy 3

luego  
0 1
 
Hf (0, 0) = 



1 0
entonces no funciona el método de los subdeterminantes pues el primero da cero. Note que
los valores propios de esta matriz son 1 y −1 por lo tanto es un punto de silla. Esto se
puede ver de la función misma pues

f (x, y) = xy x2 + y 4 + 1


cerca del (0, 0) hay puntos donde la función es negativa y positiva luego es un punto de
silla.

422
Apuntes Mat023 (versión preliminar actualizada 23-05-2014)

Ejemplo 10.1.6. Clasificar los puntos crı́ticos de f : R3 → R, (x, y, z) → f (x, y, z) =


x2 z + y 2 z + 23 z 3 − 4x − 4y − 10z

Solución. La función esta definida en un abierto y es de clase C ∞ , buscamos los puntos


crı́ticos
∇f (x, y, z) = 2xz − 4, 2yz − 4, x2 + y 2 + 2z 2 − 10 = (0, 0, 0)


esto es

2xz − 4 = 0
2yz − 4 = 0
x2 + y 2 + 2z 2 − 10 = 0

de las dos primeras ecuaciones se sigue que x, y, z 6= 0 y

2 2
x = ,y =
z z
reemplazado en la tercera
8
+ 2z 2 − 10 = 0
z2
ası́
z = ±1, ±2

tenemos 4 puntos crı́ticos

(2, 2, 1) , (−2, −2, −1) , (1, 1, 2) , (−2, −2, −1)

los clasificaremos usando el criterio de la hessiana y subdeterminantes


 
2z 0 2x
 
Hf (x, y, z) = 
 0 2z 2y 
2x 2y 4z

entonces

∆1 = 2z
∆2 = (2z)2
∆3 = −8z x2 + y 2 − 2z 2


423
Apuntes Mat023 (versión preliminar actualizada 23-05-2014)

y ası́
Punto/Determinante ∆1 ∆2 ∆3 Tipo de punto
P1 = (2, 2, 1) + + - Punto silla
P2 = (−2, −2, −1) - + + Punto silla
P3 = (1, 1, 2) + + + Punto de mı́nimo local
P4 = (−2, −2, −1) - + - Punto de máximo local
notemos que
2
f (0, 0, z) = z 3 − 10z
3
luego

lı́m f (0, 0, z) = +∞
z→+∞

lı́m f (0, 0, z) = −∞
z→−∞

luego la función no tiene máximo ni mı́nimo en todo R3 .

π π π

Ejemplo 10.1.7. Muestre que , ,
2 2 2
es un punto de máximo de la función f (x, y, z) =
sin x + sin y + sin z − sin (x + y + z)

π π π

Solución. Notemos que , ,
2 2 2
es un punto crı́tico pues

∇f (x, y, z)
= ∇ (sin x + sin y + sin z − sin (x + y + z))
= (cos x − cos (x + y + z) , cos y − cos (x + y + z) , cos z − cos (x + y + z))

ası́
π π π 
∇f , ,
2 2 2
 π π π  π π  π 
= cos − cos π + , cos − cos π + , cos − cos π +
2 2 2 2 2 2
= (0, 0, 0)

424
Apuntes Mat023 (versión preliminar actualizada 23-05-2014)

la Hessiana es

Hf (x, y, z)
 
sin (x + y + z) − sin x sin (x + y + z) sin (x + y + z)
 
 
 
 
= 
 sin (x + y + z) sin (x + y + z) − sin y sin (x + y + z) 

 
 
 
sin (x + y + z) sin (x + y + z) sin (x + y + z) − sin z
ası́
π π π 
Hf , ,
 2 2 2 
sin π + π2 − sin π π π
   
2
sin π + 2
sin π + 2
 
 
 
 
π π π π
   
= 
 sin π + 2
sin π + 2
− sin 2
sin π + 2


 
 
 
sin π + π2 π π π
   
sin π + 2
sin π + 2
− sin 2
 
−2 −1 −1
 
 
 
 
= 
 −1 −2 −1 

 
 
 
−1 −1 −2
los subdeterminantes son

∆1 = −2
∆2 = 3
∆3 = −4

luego el punto crı́tico es un punto de máximo local.

Máximos y mı́nimos en compactos y/o con restricciones

El siguiente teorema generaliza el teorema de una variable que afirma que toda función
continua f : [a, b] → R alcanza su máximo y mı́nimo absolutos, es decir, existen x0 , x1 ∈

425
Apuntes Mat023 (versión preliminar actualizada 23-05-2014)

[a, b] tales que f (x0 ) ≤ f (x) ≤ f (x1 ) para todo x ∈ [a, b].

Teorema 10.2.1 (De Weiertrass). Sean K ⊆ Rn un conjunto cerrado y acotado y f :


K → R una función continua entonces existen puntos x0 y x1 en K tales que

∀x ∈ K, f (x0 ) ≤ f (x) ≤ f (x1 )

esto es f alcanza su máximo y mı́nimos absolutos en K.

Para utilizar tal teorema es conveniente la siguiente proposición.

Proposición 10.2.1. La imagen inversa de un conjunto abierto por una función continua
es un conjunto abierto. La imagen inversa de un cerrado por una función continua es un
conjunto cerrado.

Por ejemplo, si consideramos la función continua f (x, y) = y − x2 entonces

f −1 (]0, +∞[) = f −1 ({u ∈ R : u > 0})


= (x, y) ∈ R2 : f (x, y) ∈ ]0, +∞[


= (x, y) ∈ R2 : y − x2 > 0


= (x, y) ∈ R2 : y > x2


es un conjunto abierto.
Si consideramos h (x, y) = x2 + y 2 entonces h es continua y

h−1 ([1, 2]) = (x, y) ∈ R2 : h (x, y) ∈ [1, 2]




= (x, y) ∈ R2 : 1 ≤ x2 + y 2 ≤ 2


es la imagen inversa de un cerrado por una función continua, entonces es un conjunto


cerrado.

Ejemplo 10.2.1. Hallar los valores extremos de f (x, y) = x2 + y 2 − x − y + 1 en el disco


D = {(x, y) ∈ R2 : x2 + y 2 ≤ 1}

426
Apuntes Mat023 (versión preliminar actualizada 23-05-2014)

Solución. La función f (x, y) = x2 + y 2 − x − y + 1 es continua. Note que el disco lo


podemos ver como
D = h−1 (]−∞, 1])

donde h (x, y) = x2 + y 2 entonces D es cerrado y esta contenido en la bola B ((0, 0) ; 2)


luego es acotado. Por el teorema de los extremos absolutos se tiene que f (x, y) alcanza su
máximo y mı́nimos absolutos en D, es decir, existen puntos (u0 , v0 ) y (u1 , v1 ) en D tales
que
∀ (x, y) ∈ D, f (u0 , v0 ) ≤ f (x, y) ≤ f (u1 , v1 )

Note que tales extremos pueden estar en la bola abierta B = {(x, y) ∈ R2 : x2 + y 2 < 1} o
en la circunferencia unitaria

S 1 = (x, y) ∈ R2 : x2 + y 2 = 1


Primero busquemos los puntos crı́ticos en B.


 
∂f ∂f
∇f (x, y) = (x, y) , (x, y) = (0, 0)
∂x ∂x
= (2x − 1, 2y − 1) = (0, 0)

1 1

el único punto crı́tico en B es (x, y) = ,
2 2
. El conjunto S 1 lo podemos parametrizar por
c : [0, 2π] → S 1 tal que t → c (t) = (cos t, sin t), de esta forma la función f en el conjunto
S 1 es
f (cos t, sin t) = 2 − cos t − sin t

para t ∈ [0, 2π] como es una función de una variable buscamos sus valores extremos con el
cálculo diferencial y el teorema de los extremos en R.

f 0 = 0 ⇔ sin t − cos t = 0

π π
tiene las soluciones t = 4
yt= 4
+ π en el intervalo ]0, 2π[ luego se tienen los puntos
crı́ticos
     
1 1 1 1 1 1
, , √ , √ , −√ , −√ y (1, 0) = (cos 0, sin 0) = (cos 2π, sin 2π)
2 2 2 2 2 2

427
Apuntes Mat023 (versión preliminar actualizada 23-05-2014)

evaluando en la función obtenemos


   2  2
1 1 1 1 1 1 1
f , = + − − +1=
2 2 2 2 2 2 2

 
1 1 1 1 1 1
f √ ,√ = + − √ − √ +1=2− 2
2 2 2 2 2 2

 
1 1 1 1 1 1
f −√ , −√ = + + √ + √ +1=2+ 2
2 2 2 2 2 2
f (1, 0) = 1 − 1 + 1 = 1
 
de donde − √12 , − √12 es el punto de máximo y 21 , 12 es el punto de mı́nimo en el disco.


2 2
Nota: La función es f (x, y) = x − 12 + y − 12 + 12 el valor mı́nimo se alcanza en
1 1
y el máximo en el disco será en el punto del disco que encuentre más lejos de 12 , 21
 
,
2 2  
este es − √12 , − √12 .

Extremos restringidos Multiplicadores de Lagrange

Es común en problemas querer maximizar o minimizar alguna función sujeta a res-


tricciones dadas. Por ejemplo; En economı́a suponga que se quiere vender dos tipos de
productos I y II. Sean x e y la cantidad de productos I y II respectivamente. Representemos
por f (x, y) la ganancia obtenida cuando se vende una cantidad x de I y y de II, nuestra
producción esta limitada o controlada por nuestro capital, de esta forma tenemos una
restricción del tipo g (x, y) = c. El problema podrı́a plantearse como

máx f (x, y)
(x,y)∈R2 ,g(x,y)=c

Supongamos que x0 es un punto extremo del problema

Optimizar f (x)
x∈Rn ,g(x)=c

donde f : U ⊆ Rn → R y optimizar es máximo o mı́nimo. Sea c : R → Rn donde

g (c (t)) = c

y c (0) = x0 entonces por la regla de la cadena

∇g (c (t)) · c0 (t) = 0

428
Apuntes Mat023 (versión preliminar actualizada 23-05-2014)

se sigue que
∇g (x0 ) · c0 (0) = 0

como x0 es extremo de la f se sigue que f (c (t)) tiene un máximo o mı́nimo en t = 0 luego

∇f (c (t)) · c0 (t) = D (f ◦ c) (t)


∇f (x0 ) · c0 (0) = 0

de esta forma ∇f (x0 ) y ∇g (x0 ) deben ser paralelos es decir ∃λ ∈ R tal que

∇f (x0 ) = λ∇g (x0 )

el número λ se llama multiplicador de Lagrange y la función de n + 1 variables

L (x, λ) = f (x) − λ (g (x) − c)

es llamado Lagrangiano del problema. Podemos entonces formular el siguiente teorema:

Teorema 10.3.1. Sean f, g : U ⊆ Rn → R funciones de clase C 1 . Sea x0 ∈ U un punto


tal que g (x0 ) = c y ∇g (x0 ) 6= 0. Si

S = {x ∈ U : g (x) = c}

y f |S tiene un extremo local en x0 entonces existe λ0 ∈ R tal que

∇f (x0 ) = λ0 ∇g (x0 )

Observación 10.3.1. Es lo mismo que buscar los puntos crı́ticos del Lagrangiano L (x, λ) =
f (x) − λ (g (x) − c)

Ejemplo 10.3.1. Sea S ⊆ R2 la recta que pasa por (−1, 0) inclinada en un ángulo de 45◦
y sea f : R2 → R, (x, y) → x2 + y 2 . Hallar los extremos de f |S .

Solución. La pendiente es tan (45) = 1 entonces

y−0=x−1

429
Apuntes Mat023 (versión preliminar actualizada 23-05-2014)

se sigue que la restricción es


g (x, y) = y − x + 1 = 0

buscamos los puntos crı́ticos del Lagrangiano

L (x, y, λ) = x2 + y 2 − λ (y − x + 1)

ası́
∂L
(x, y, λ) = 2x + λ = 0
∂x
∂L
(x, y, λ) = 2y − λ = 0
∂y
∂L
(x, y, λ) = y − x + 1 = 0
∂λ
Note que de la primera y segunda se obtiene
λ
λ = 2y = −2x ⇔ y = −x =
2
reemplazando en la tercera se obtiene
1 1
y=− ⇒x=
2 2
1
, − 12

geométricamente podemos deducir que este punto es un mı́nimo (x0 , y0 ) = 2
. Note
que el punto es un extremo de f |S pero no de f .

Ejemplo 10.3.2. Maximizar la función f (x, y, z) = x + z sujeta a la restricción x2 + y 2 +


z 2 = 1.

Solución. El conjunto S = {(x, y, z) ∈ R3 : x2 + y 2 + z 2 = 1} es cerrado y acotado y f es


una función continua en todo R2 de donde obtenemos que f |S debe tener un máximo y un
mı́nimo absolutos. Los buscamos con el método de Lagrange. Pongamos el Lagrangiano

L (x, y, z, λ) = x + z − λ x2 + y 2 + z 2 − 1


y debemos buscar sus puntos crı́ticos


∂L
(x, y, z, λ) = 1 − 2λx = 0
∂x
∂L
(x, y, z, λ) = −2λy = 0
∂y
∂L
(x, y, z, λ) = 1 − 2λz = 0
∂z
∂L
(x, y, z, λ) = x2 + y 2 + z 2 − 1 = 0
∂λ
430
Apuntes Mat023 (versión preliminar actualizada 23-05-2014)

note que λ = 0 o y = 0. λ = 0 no puede ser (sistema inconsistente) entonces y = 0 se sigue


de la primera y tercera ecuaciones
1
x =

1
z =

ası́ x = z y reemplazando en la última
1
2x2 = 1 ⇒ x = ± √
2
ası́ obtenemos dos puntos crı́ticos
   
1 1 1 1
√ , 0, √ y − √ , 0, − √
2 2 2 2
reemplazando se tiene que el primero es un máximo y el segundo un mı́nimo.

Ejemplo 10.3.3. Hallar el mayor volumen que pueda tener una caja rectangular con tapa
sujeta a la restricción de que el área de la superficie sea 10 m2 .

Solución. Sean x, y, z los lados de la caja entonces el volumen es V (x, y, z) = xyz, vemos
que esta función esta restringida a [0, 10] × [0, 10] × [0, 10] luego debe alcanzar el máximo
y mı́nimo absolutos. El área será

2 (xy + xz + zy) = 10

ası́ el Lagrangiano será

L (x, y, z, λ) = xyz − λ (xy + xz + zy − 5)

buscamos los puntos crı́ticos


∂L
(x, y, z, λ) = yz − λ (y + z) = 0
∂x
∂L
(x, y, z, λ) = xz − λ (x + z) = 0
∂y
∂L
(x, y, z, λ) = xy − λ (x + y) = 0
∂z
∂L
(x, y, z, λ) = xy + xz + zy − 5 = 0
∂λ
note que x 6= 0 pues si fuese cero zy = 5 pero λz = 0 se sigue λ = 0 y ası́ de la primera
yz = 0 contradicción, lo mismo para y, z.

431
Apuntes Mat023 (versión preliminar actualizada 23-05-2014)

como son cantidades positivas también las sumas serán distintas de cero
yz xz xy
= =
y+z x+z x+y
esto implica x = y = z sustituyendo en la última ecuación nos da

3x2 = 5

de donde obtenemos el punto crı́tico


r r r !
5 5 5
, ,
3 3 3
q 3
5
el volumen máximo es V = 3
.

Este resultado tiene una extensión a más restricciones:

Si tenemos las restricciones una restricción S dada por los puntos que satisfacen

g1 (x1 , x2 , . . . , xn ) = c1
g2 (x1 , x2 , . . . , xn ) = c2
..
.
gk (x1 , x2 , . . . , xn ) = ck

y f restringida a S tiene un extremo en x0 ∈ S entonces, si ∇g1 (x0 ) , ∇g2 (x0 ) , · · · , ∇gk (x0 )
son l.i. existirán λ1 , λ2 , . . . , λk ∈ R tales que

∇f (x0 ) = λ1 ∇g1 (x0 ) + λ2 ∇g2 (x0 ) + · · · + λk ∇gk (x0 )

Ejemplo 10.3.4. Determine los extremos absolutos de f (x, y, z) = x + 2y + z sujeto a


las restricciones x2 + y 2 = 1 y y + z = 1.

Solución. f (x, y, z) = x + 2y + z es una función continua. El conjunto definido por las


restricciones

(x, y, z) ∈ R3 : x2 + y 2 = 1 ∧ y + z = 1

S =
= (x, y, z) ∈ R3 : x2 + y 2 = 1 ∩ (x, y, z) ∈ R3 : y + z = 1
 

note que el primer conjunto restringe a x, y al conjunto [−1, 1] y eso restringe a z en el


segundo conjunto. Es un conjunto acotado e intersección de dos cerrados. Luego es cerrado
y acotado la función alcanza un máximo y mı́nimo absolutos en S.

432
Apuntes Mat023 (versión preliminar actualizada 23-05-2014)

El Lagrangiano será

L (x, y, z, λ, µ) = (x + 2y + z) − λ x2 + y 2 − 1 − µ (y + z − 1)


buscamos los puntos crı́ticos

∂L
= 1 − 2λx = 0
∂x
∂L
= 2 − 2λy − µ = 0
∂y
∂L
= 1−µ=0
∂z
∂L
= x2 + y 2 − 1 = 0
∂λ
∂L
= y+z−1=0
∂µ
resolvemos µ = 1 reemplazando en la segunda obtenemos

1 = 2λy

de donde λ ni y pueden ser cero. De la primera obtenemos

1
x= =y

de donde obtenemos en la cuarta

1
x = y = ±√
2
de donde podemos obtener z en la 5. Ası́
   
1 1 1 1 1 1
√ , √ ,1 − √ y −√ , −√ , 1 + √
2 2 2 2 2 2
evaluando en la función f (x, y, z) = x + 2y + z se tiene
 
1 1 1 1 2 1 2
f √ , √ ,1 − √ = √ + √ +1− √ = √ +1
2 2 2 2 2 2 2
 
1 1 1 1 2 1 2
f −√ , −√ , 1 + √ = −√ − √ + 1 + √ = 1 − √
2 2 2 2 2 2 2
el primer es máximo y el segundo un mı́nimo.

433
Apuntes Mat023 (versión preliminar actualizada 23-05-2014)

Criterio de la segunda derivada para extremos condicionados

En esta sección presentamos un teorema que nos permite clasificar el punto obtenido
con los multiplicadores de Lagrange:

Teorema 10.3.2 (Hessiana orlada). Sean f, g : U ⊆ Rn → R funciones de clase C 2 .


Sean v0 ∈ U, g (v0 ) = c, ∇g (v0 ) =
6 0 y existe un λ0 ∈ R tal que ∇f (v0 ) = λ0 g (v0 ). Si
L (λ, x) = f (x) − λ (g (x) − c) y

∂g ∂g ∂g
0
− ∂x 1
− ∂x2
··· − ∂x n


− ∂g ∂2L ∂2L 2
∂ L
∂x1 ∂x21 ∂x1 ∂x2
··· ∂x1 ∂xn

∂2L ∂2L ∂2L
∂g

H = HL (v0 ) = − ∂x ∂x1 ∂x2 ∂x22
··· ∂x2 ∂xn

. 2 .. .. .. ..
..

. . . .

− ∂g ∂2L ∂2L ∂2L

∂xn ∂x1 ∂xn ∂x2 ∂xn
··· ∂xn 2

entonces:

1. Si

∂g ∂g ∂g

∂g ∂g
0 − ∂x 1
− ∂x 2
− ∂x 3
− ∂x − ∂x
0
∂2L ∂2L ∂2L
∂g
1 2 −
∂2L ∂2L < 0, H4 = ∂x1 ∂x21 ∂x1 ∂x2 ∂x1 ∂x3
∂g
H3 = − ∂x ∂x21 ∂x1 ∂x2 < 0, . . .
− ∂g ∂2L ∂2L ∂2L
∂g1

− ∂2L ∂2L

∂x2 ∂x1 ∂x2 ∂x22 ∂x2 ∂x3
∂x2 ∂x1 ∂x2 ∂x22 ∂g
− ∂2L ∂2L ∂2L


∂x3 ∂x1 ∂x3 ∂x2 ∂x3 ∂x3 2

entonces v0 es mı́nimo local de f sujeto a g (x) = c.

2. Si los determinantes se alternan en la forma H3 > 0, H4 < 0, H5 > 0 etc, entonces


v0 es máximo local de f sujeto a g (x) = c

3. Si los determinantes anteriores son distintos de cero pero siguen los patrones anteriores
entonces el punto es de silla.

Ejemplo 10.3.5. Estudiar los extremos de la función f (x, y) = cos2 x + cos2 y sujeta a la
restricción x − y = π4 .

434
Apuntes Mat023 (versión preliminar actualizada 23-05-2014)

π

Solución. la función de Lagrange es F (λ, x, y) = cos2 x + cos2 y − λ x − y − 4
se sigue
∂F  π
= − x−y−
∂λ 4
∂F
= −2 cos x sin x − λ
∂x
∂F
= −2 cos y sin y + λ
∂y
luego

sin (2x) = −λ
sin (2y) = λ
π
x−y =
4
se sigue
  π 
sin 2 y + = − sin (2y)
4
cos 2y = − sin (2y)

ası́

cos 2y + sin 2y = 0

 π
sin 2y + = 0
4

k π
y = π − con k ∈ Z
2 8
se sigue  
k π k π
π+ , π− con k ∈ Z
2 8 2 8
son los puntos crı́ticos. Calculamos la Hessiana orlada
2
∂F ∂F ∂F
−1
∂λ2 ∂x∂λ ∂y∂λ 0 1


2F

∂F ∂ ∂F
∂x∂λ ∂x2 ∂x∂y = −1 −2 cos 2x 0


∂F ∂F ∂ 2F

∂y∂λ ∂x∂y ∂y 2
1 0 −2 cos 2y
pero

2 cos 2x = 2 (−1)k

2 cos 2y = 2 (−1)k

435
Apuntes Mat023 (versión preliminar actualizada 23-05-2014)

se sigue

∂2F ∂F ∂F

∂λ2 ∂x∂λ ∂y∂λ
0 −1 1



∂F ∂2F ∂F

= −1 √ k+1 = 2 (−1)k 2


∂x∂λ ∂x2 ∂x∂y
2 (−1) 0


∂2F
2 (−1)k+1
∂F ∂F

1 0
∂y∂λ ∂x∂y ∂y 2


∂2F ∂F ∂F
∂λ2 ∂x∂λ ∂y∂λ

se sigue que si k es impar ∂F ∂2F ∂F < 0 y el punto es mı́nimo y si k es par es
∂x∂λ ∂x2 ∂x∂y
∂2F
∂F ∂F


∂y∂λ ∂x∂y ∂y 2
un máximo. Note que

f (x, y) = cos2 x + cos2 y



     
k π k π 2 k π 2 k π
f π+ , π− = cos π+ + cos π−
2 8 2 8 2 8 2 8
π
1 + cos kπ − π4
 
1 + cos kπ + 4
= +
2 2
1  π  π 
= 1+ cos kπ + + cos kπ −
2 4 4
1 √ 
= 1+ 2 (−1)k
2√
2 (−1)k
= 1+
2

para k impar   √
k π k π 2
f π+ , π− =1−
2 8 2 8 2
para k par   √
k π k π 2
f π+ , π− =1+
2 8 2 8 2

Ejemplo 10.3.6. Hallar las dimensiones del mayor paralelepı́pedo rectangular de aristas
paralelas a los ejes coordenados que puede ser inscrito en el elipsoide de ecuación:
 x 2  y 2  z 2
+ + =1
3 4 5

436
Apuntes Mat023 (versión preliminar actualizada 23-05-2014)

Solución. El volumen del paralelepı́pedo es

V (x, y, z) = (2x) (2y) (2z) = 8xyz

pero (x, y, z) debe estar sobre el elipsoide


 x 2  y 2  z 2
+ + =1
3 4 5
y sus coordenadas son positivas o cero. Tenemos que resolver el problema

máx V (x, y, z)
r(x,y,z)=0
x,y,z≥0

x 2 y 2 z 2
  
donde V (x, y, z) = 8xyz y r (x, y, z) = 3
+ 4
+ 5
− 1. Este problema tiene
solución pues el elipsoide es un conjunto cerrado y acotado además la función es continua.
Determinaremos el valor máximo utilizando los multiplicadores de Lagrange

∇V (x, y, z) = λ∇r (x, y, z)


r (x, y, z) = 0

esto es
 
2x 2y 2z
(8yz, 8xz, 8xy) = λ , ,
32 42 52
 x 2  y 2  z 2
+ + = 1
3 4 5
437
Apuntes Mat023 (versión preliminar actualizada 23-05-2014)

el sistema a resolver es
2λx
8yz =
32
2λy
8xz =
42
2λz
8xy =
52
 x 2  y 2  z 2
+ + = 1
3 4 5
multiplicando las primeras tres ecuaciones por x, y, z respectivamente se obtiene
λx2
4xyz =
32
λy 2
4xyz =
42
λz 2
4xyz =
52
sumando las tres ecuaciones
λx2 λy 2 λz 2
12xyz = 2
+ 2 + 2
3 4 5
 x 2  y 2  z 2 
= λ + +
3 4 5
= λ

ası́
λ λx2 λy 2 λz 2
= 4xyz = 2 = 2 = 2
3 3 4 5
si λ 6= 0 entonces
32 3
x2 = ⇒x= √
3 3
42 4
y2 = ⇒y= √
3 3
52 5
z2 = ⇒z= √
3 3
si λ = 0 entonces xyz = 0 y

yz = 0
xz = 0
xy = 0
 x 2  y 2  z 2
+ + = 1
3 4 5
438
Apuntes Mat023 (versión preliminar actualizada 23-05-2014)

esto nos da los puntos (3, 0, 0) , (0, 0, 5) , (0, 4, 0). Estos 3 puntos son puntos de mı́nimo
V =0y
160 √
     
3 4 5 3 4 5
V √ ,√ ,√ =8 √ √ √ = 3
3 3 3 3 3 3 3
es el valor máximo.

Ejercicios del capı́tulo

1. Determine los valores de a, b ∈ R para los cuales la función f (x, y) = ax2 + by 2 tiene
un máximo, mı́nimo o punto silla en (0, 0).

2. Sea C ∈ R. Muestre que si f : U ⊆ Rn → R tiene un máximo (mı́nimo) local en


x = x0 entonces la función F : U → R definida por F (x) = f (x) + C también tiene
un máximo (mı́nimo) local en x = x0 .

3. Determinar los valores de a, b ∈ R para los cuales la función f : R2 → R definida por

f (x, y) = x3 + 3ax + y 3 + by + 1

no tiene extremos locales.

4. Para cada una de las siguientes funciones determinar los puntos crı́ticos (o puntos
estacionarios), clasificarlos en máximos, mı́nimos y puntos silla:
a) z = x2 + (y − 1)2 i) z = x2 − xy + y 2 − 2x + y
b) z = x2 − (y − 2)2 j) z = sin (x) sin (y) sin (x + y)
z = (5x + 7y − 25) e−(x +xy+y )
2 2
c) z = 1 − x2 − y 2 k)
z = (x2 + y 2 ) e−(x )
2 +y 2
d) z = (x − y − 1)2 l)
e) z = 2x2 − xy − 3y 2 − 3x + 7y m) z = xy 2 (x + 2y − 1)
f) z = x (x2 + y 2 − a2 ) n) z = x3 − x2 y − x2 + y 2
g) z = x3 + y 3 − 3 (x + y) + 1 o) z = (x + y) (x2 + y 2 − 6)
h) z = xy + (x − y)3 p) z = (x − y)3 + x4 + y 4

5. Sea f (x, y) = 3x4 − 4x2 y + y 2 . Demostrar que sobre toda recta de la forma y = mx la
función tiene un mı́nimo en (0, 0) pero no es mı́nimo en ningún entorno bidimensional
del origen. (Estudiar los puntos donde f (x, y) > 0 y f (x, y) < 0).

439
Apuntes Mat023 (versión preliminar actualizada 23-05-2014)

6. Considere la función z = f (x, y) que en los alrededores del punto (1, 1, 1) esta definida
implı́citamente por
z 3 + 3x2 y − y 3 z + y 2 − 3x − 1 = 0

obtener la expansión de Taylor para z en (1, 1).

7. Sea f : R4 − {(0, 0, 0, 0)} → R dada por


y z u 1
f (x, y, z, u) = x + + + +
x y z u
tiene un punto crı́tico en (1, 1, 1, 1) y clasificarlo.

8. Sea g : R → R una función diferenciable que no se anula. Analizar los extremos de


Z (x−1)2 Z (y−1)2
f (x, y) = g (t) dt + g (t) dt
0 0

si g (0) > 0.

9. Determine los puntos crı́ticos de la función

f (x, y) = 2x4 + y 4 − 4x2 − 2y 2

y clasificarlos (obs.: Son 9 puntos).

11.* Considere la función z = f (x, y) definida implı́citamente por la expresión

F (x, y, z) = x3 + y 3 + z 3 − 3x − 3y + z + 4 = 0

obtener sus puntos crı́ticos y clasificarlos.

10. Determine los valores de las constantes a, b ∈ R para los cuales la integral
Z 1
(a + bx − f (x))2 dx
0

sea mı́nimo (suponer f es continua en [0, 1]), estudiar los casos f (x) = x2 y f (x) =
x3 + x.

11. Sea f (x, y) = Ax2 + 2Bxy + Cy 2 + 2Dx + 2Ey + F donde A > 0 y B 2 < AC.

a) Demostrar que f tiene un mı́nimo.

440
Apuntes Mat023 (versión preliminar actualizada 23-05-2014)

b) Demostrar que en el punto de mı́nimo (x1 , y1 ) se cumple

f (x1 , y1 ) = Dx1 + Ey1 + F

c) Demostrar que
A B D

1
f (x1 , y1 ) = B C E
AC − B 2

D E F

12. Método de los mı́nimos cuadrados: Dados n números distintos x1 , x2 , . . . , xn y


otros n números (no necesariamente distintos) es en general imposible encontrar una
recta f (x) = ax + b que pase por todos los puntos (xi , yi ) esto es f (xi ) = yi ∀i, no
obstante se puede encontrar los valores de a y b que hacen que el error cuadrático
total n
X
E (a, b) = (f (xi ) − yi )2
i=1

sea mı́nimo. Encontrar tales valores!!!

13. Comprobar que f (x, y, z) = x4 + y 4 + z 4 − 4xyz tiene un punto crı́tico en (1, 1, 1)


y clasificarlo como máximo, mı́nimo o punto silla. Expandir esta función en Taylor
entorno al punto (1, 1, 1) para visualizar directamente la forma cuadrática asociada a
la Hessiana Hf (1, 1, 1).

14. Estudiar los extremos relativos de la función f (x, y, z) = xyz (1 − x − y − z)

15. Sea A ∈ Mn×n (R), B ∈ M1×n (R) y C ∈ R, se define F : Rn → R

F (X) = X T AX + BX + C

donde X ∈ Mn×1 (R) (el vector se mira como columna). Estudiar las condiciones
bajo las cuales esta función tiene máximos, mı́nimos o puntos silla.

16. Considere la función f : R3 → R dada por f (x, y, z) = sin x + sin y + sin z −


sin (x + y + z) determine la expansión de Taylor de f en π2 , π2 , π2 ´, muestre además


que el punto es un mı́nimo.

441
Apuntes Mat023 (versión preliminar actualizada 23-05-2014)

17. Considere la función z = f (x, y) que en los alrededores del punto (1, 1, 1) esta definida
implı́citamente por
z 3 + 3x2 y − y 3 z + y 2 − 3x − 1 = 0

obtener la expansión de Taylor para z en (1, 1).

18. La Hessiana de cierta función f : D ⊆ R3 → R es


 
x2 y xz
 
Hf (x, y, z) = 
 y y 2 yx 

2
xz yx z

¿En que subconjunto de R3 debe estar el punto critico (x, y, z) para que sea máximo,
mı́nimo o punto silla? ¿Cuando esta Hessiana no entrega información?.

19. Determine los extremos de la función f (x, y) = 2x − y con la restricción g (x, y) =


3x2 + 2y 2 − 33/2 = 0

20. Determine los extremos de la función f (x, y) = x2 + 8y si (x, y) se encuentra sobre


la elipse x2 + 4y 2 = 5.

21. Utilizar el método de los multiplicadores para determinar los semiejes de la elipse
5x2 − 6xy + 5y 2 − 32 = 0.

22. Usando multiplicadores de Lagrange, determinar la menor distancia entre las rectas

x+4 y−4 z+1


= =
2 −1 −2
x+5 y−5
= =z−5−5
4 −3

23. Demostrar que el paralelepı́pedo de mayor volumen que se puede inscribir en una
esfera es un cubo.

24. Hallar los puntos de la curva x2 + 4y 2 − 4 = 0 que se encuentren mas cercanos y mas
lejanos a los puntos de la curva

x2 + y 2 + 4x + 2y − 20 = 0

442
Apuntes Mat023 (versión preliminar actualizada 23-05-2014)

x3
25. Hallar los extremos absolutos de la función f (x, y) = 3
− 32 x2 + 2x + y 2 − 2y + 1 en
la región K = {(x, y) : x ≥ 0, y ≥ 0, x + y ≤ 1}.

26. Hallar los extremos absolutos de la función f (x, y, z) = (x − 1)2 + (y − 1)2 + (z − 1)2
en la región
K = (x, y, z) ∈ R3 : x2 + y 2 + z 2 ≤ 12


28.* Suponga que queremos determinar los extremos de la función f (x, y, z) sujeta a la
restricción g (x, y, z) = 0 y utilizando el teorema de Lagrange obtenemos el punto
crı́tico p0 = (x0 , y0 .z0 ), utilizando el teorema de la función implı́cita demostrar que
el problema se puede reducir a estudiar los extremos de una función de 2 variables
(con el mismo punto crı́tico), calcular la Hessiana de esta función de 2 variables y
expresarla en términos de f y g para obtener un criterio que permita determinar si
el punto crı́tico entregado por Lagrange es máximo, mı́nimo o silla (Esto es llamado
criterio de la hessiana reducida).

27. La fabrica de gelatinas BOB esta interesada en minimizar sus costos de producción.
Los ingredientes utilizados en la elaboración de su gelatina son tres: Grenetina, azúcar
y frutas. La fórmula de producción dice que el número de gelatinas que se puede
producir es
g (x, y, z) = xα y β z γ

donde x es la cantidad de grenetina, y es la cantidad de azúcar y z es la cantidad


de frutas (todas las cantidades en kilogramos) y las constantes α, β, γ son secretas.
Suponga que el precio por kilogramo de los ingredientes es: La grenetina $10,000,
azúcar $250 y fruta $500. Si se quiere producir 10,000 gelatinas, ¿Qué combinación
de ingredientes minimiza los costos c (x, y, z) = 10,000x + 250y + 500z? (la respuesta
puede quedar en términos de α, β, γ).

443
Capı́tulo 11 : Funciones implı́citas e inversas

El teorema de la función implı́cita

En los cursos anteriores, frecuentemente nos hemos encontrado con relaciones de la


forma
F (x, y) = 0

donde F : Ω ⊆ R2 → R es una función de clase C 1 (Ω), por ejemplo, una elipse

x2 y 2
+ 2 = 1 ⇔ F (x, y) = 0
a2 b

donde
x2 y 2
F (x, y) = + 2 −1
a2 b
o la ecuación cuadrática general

Ax2 + Bxy + Cy 2 + Dx + Ey + F = 0

Estas ecuaciones generalmente definen una curva en el plano que localmente (esto es, en
intervalos alrededor de un punto dado) puede ser descrita como la gráfica de una función
y = φ (x) o de una función x = ψ (y).

Por ejemplo considere la función F : R2 → R, (x, y) → F (x, y) = x2 + y 2 − 1, esta


función es clase C ∞ (R2 ) y
F (x, y) = 0 ⇔ x2 + y 2 = 1

Todos los puntos (x, y) con y > 0 que satisfacen

x2 + y 2 = 1

corresponden a puntos de la gráfica de la función



ψ (x) = 1 − x2 para x ∈ [−1, 1]

444
Apuntes Mat023 (versión preliminar actualizada 23-05-2014)


y= 1 − x2
1

0,5

−1 −0,5 0,5 1

−0,5

esto es, si (x, y) ∈ Graf (ψ), entonces F (x, y) = 0. En efecto

(x, y) ∈ Graf (ψ) ⇔ (x, y) = (x, ψ (x)) para algún x ∈ [−1, 1]


 √ 
⇔ (x, y) = x, 1 − x2 para algún x ∈ [−1, 1]

luego (x, y) ∈ Graf (ψ) implica


 √ 
F (x, y) = F x, 1 − x 2

√ 2
= x2 + 1 − x2 − 1
= 0

lo que demuestra que todos los puntos de la gráfica de ψ cumplen la relación F (x, y) = 0
(están sobre la circunferencia unitaria). De manera similar, los puntos (x, y) con y < 0 que
satisfacen F (x, y) = 0 corresponden a puntos de la gráfica de la función

y = ϕ (x) = − 1 − x2 para x ∈ [−1, 1]

sin embargo, cerca de los puntos (1, 0) y (−1, 0) no podemos interpretar, los puntos de
la curva como puntos del gráfico de una función de y = f (x), en este caso los puntos
corresponden a la gráfica de
p p
x = τ (y) = 1 − y 2 para y ∈ [−1, 1] ∧ x = η (y) = − 1 − y 2 para y ∈ [−1, 1]

respectivamente. En resumen, en cada punto (x0 , y0 ) ∈ R2 que cumple F (x0 , y0 ) = 0


existe una función ψ : ]x0 − ε, x0 + ε[ → R tal que ψ (x0 ) = y0 y que cumple para cada

445
Apuntes Mat023 (versión preliminar actualizada 23-05-2014)

x ∈ ]x0 − ε, x0 + ε[, F (x, ψ (x)) = 0 o bien existe una función ϕ : ]y0 − ε, y0 + ε[ → R tal
que ϕ (y0 ) = x0 y
F (ϕ (y) , y) = 0

para cada y ∈ ]y0 − ε, y0 + ε[.

y=ψ(x)
y0

x0 −ε x0 x0 +ε

Si tenemos la función G : R2 → R, (x, y) → G (x, y) = x7 + y 7 + xy − 1 y la ecuación

G (x, y) = 0
x7 + y 7 + xy = 1

el problema no es tan sencillo. ¿Cerca de que puntos (x, y) podemos encontrar una función
y = ψ (x) cuya gráfica coincida con la curva dada? es decir, los puntos del gráfico de
y = ψ (x) estén sobre la curva

x7 + ψ 7 (x) + xψ (x) = 1

446
Apuntes Mat023 (versión preliminar actualizada 23-05-2014)

supuesto que hemos resuelto el problema de la existencia de tal función, ¿Qué propiedades
de G hereda la función y = ψ (x)? ¿es diferenciable? si es ası́ ¿Cuanto vale su derivada?.
Supongamos que la función que encontramos y = ψ (x) es derivable, entonces tenemos

G (x, ψ (x)) = 0

por la regla de la cadena

∂G ∂ (x) ∂G
(x, ψ (x)) + (x, ψ (x)) ψ 0 (x) = 0
∂x ∂x ∂y

de esto se tiene
0 − ∂G (x, ψ (x))
ψ (x) = ∂G∂x
∂y
(x, ψ (x))
∂G
siempre que ∂y
(x, ψ (x)) 6= 0.

Definición 11.1.1. Sea F : U ⊆ Rn × Rm → Rm una función. Diremos que la función


ϕ : V ⊆ Rn → Rm , x → ϕ (x) esta definida implı́citamente por la ecuación

F (x, y) = 0

si ∀x ∈ V se cumple
F (x, ϕ (x)) = 0

Ejemplo 11.1.1. La función ϕ : R− → R definida por


sr
3 1 6 1 2x
ϕ (x) = x − 8x3 + x3 + rq
4 2 1
3
4
x6 − 8x3 + 12 x3

esta definida implı́citamente por la ecuación x3 + 6xy = y 3 . En la figura se muestra la curva


x3 + 6xy = y 3 y en verde la parte de ella descrita por ϕ. En este caso, puede resultar mas
simple trabajar directamente con la ecuación que define la función en lugar de trabajar
con la expresión explı́cita de la función.

447
Apuntes Mat023 (versión preliminar actualizada 23-05-2014)

El teorema de la función implı́cita nos entrega condiciones sobre la función que define
la ecuación, para garantizar todas estas propiedades

Teorema 11.1.1. Sean U abierto de Rn+1 y F : U → R es una función de clase C p (U ).


Denotemos por (x, z) los puntos de Rn+1 donde x ∈ Rn y z ∈ R. Supongamos que (x0 , z0 )
satisface
∂F
F (x0 , z0 ) = 0 y(x0 , z0 ) 6= 0
∂z
entonces existe una bola abierta U que contiene a x0 ∈ Rn , una vecindad V de z0 ∈ R
y una única función z = g (x) definida para x ∈ U y con recorrido en V que satisface
z0 = g (x0 ),
F (x, g (x)) = 0 para todo x ∈U

además z = g (x) es de clase C p (U) y


Dx F (x, g (x))
Dg (x) = − ∂F
∂z
(x, g (x))
donde Dx F es la derivada respeto a x es decir
 
∂F ∂F
Dx F = ,...,
∂x1 ∂xn

448
Apuntes Mat023 (versión preliminar actualizada 23-05-2014)

en términos de componentes
∂F
∂g − ∂x (x, g (x))
(x) = ∂F i
∂xi ∂z
(x, g (x))
para i = 1, 2, . . . , n.

Ejemplo 11.1.2. Sea G : R3 → R una función de clase C 1 (R3 ). Considere la superficie

G (x, y, z) = 0

suponga además que (x0 , y0 , z0 ) es un punto sobre la superficie, es decir

G (x0 , y0 , z0 ) = 0

si
∂G
(x0 , y0 , z0 ) 6= 0
∂z
por el teorema de la función implı́cita existe un conjunto abierto U que contiene (x0 , y0 ) y
una función z = z (x, y) definida en U tal que z0 = z (x0 , y0 ) además

G (x, y, z (x, y)) = 0 para (x, y) ∈ U

el plano tangente a la gráfica de z = z (x, y) en el punto (x0 , y0 , z0 ) es


∂z ∂z
z − z0 = (x0 , y0 ) (x − x0 ) + (x0 , y0 ) (y − y0 )
∂x ∂y
pero note que
∂z − ∂G (x0 .y0 , z0 )
(x0 , y0 ) = ∂x∂G(x0 ,y0 ,z0 )
∂x
∂z
y
∂z − ∂G
∂y
(x0 .y0 , z0 )
(x0 , y0 ) = ∂G(x0 ,y0 ,z0 )
∂y
∂z
reemplazando

− ∂G
∂x
(x0 .y0 , z0 ) − ∂G
∂y
(x0 .y0 , z0 )
z − z0 = ∂G(x0 ,y0 ,z0 )
(x − x0 ) + ∂G(x0 ,y0 ,z0 )
(y − y0 )
∂z ∂z

ası́
∂G ∂G ∂G (x0 , y0 , z0 )
(x0 .y0 , z0 ) (x − x0 ) + (x0 .y0 , z0 ) (y − y0 ) + (z − z0 ) = 0
∂x ∂y ∂z

449
Apuntes Mat023 (versión preliminar actualizada 23-05-2014)

que es
∇G (x0 , y0 , z0 ) ((x, y, z) − (x0 .y0 , z0 )) = 0

que es la ecuación del plano tangente a una superficie que obtuvimos por otros medios (el
gradiente es perpendicular a los conjuntos de nivel)

Ejemplo 11.1.3. Suponga que F : R3 → R es una función de clase C 1 (R3 ) que cumple
∂F
  ∂F  ∂F 
∂x ∂y ∂z
6= 0 muestre que las funciones z = z (x, y), x = x (y, z) y y = y (x, z)
definidas implı́citamente por
F (x, y, z) = 0

cumplen
∂z ∂x ∂y
= −1
∂x ∂y ∂z
∂F
Solución. Como ∂x
6= 0 se sigue que la ecuación define a x como función de y y z, es
decir, x = x (y, z) además
∂F
∂x ∂y
= − ∂F
∂y ∂x
 
de manera similar (usando ∂F ∂F ∂F
 
∂x ∂y ∂z
6= 0) se tiene y = y (x, z) y z = z (x, y) donde
∂F ∂F
∂z ∂x ∂y ∂z
= − ∂F y = − ∂F
∂x ∂z
∂z ∂y

entonces
! ∂F
! !
∂F ∂F
∂z ∂x ∂y ∂x ∂y ∂z
= − ∂F
− ∂F
− ∂F
∂x ∂y ∂z ∂z ∂x ∂y
= −1

Ejemplo 11.1.4. ¿Cerca de cuáles puntos es posible representar la superficie

x3 + 3y 2 + 8xz 2 − 3z 3 y = 1

como gráfica de una función diferenciable z = k (x, y)?

Solución. Son aquellos puntos para los cuales la derivada parcial respecto a la variable z
sea no nula, es decir

16xz − 9z 2 y 6= 0
z (16x − 9zy) 6= 0

450
Apuntes Mat023 (versión preliminar actualizada 23-05-2014)

Observación 11.1.1. También se puede intentar despejar las otras variables, formular el
teorema en tales casos.

Observación 11.1.2. De la ecuación

x3 + 3y 2 + 8xz 2 − 3z 3 y = 1

también se puede obtener una expresión para las derivadas parciales (derivando la ecuación
respecto a x)
∂z ∂z
3x2 + 8z 2 + 16xz − 9z 2 y =0
∂x ∂x
ası́
∂z −3x2 − 8z 2
=
∂x 16xz − 9z 2 y

Ejemplo 11.1.5. Sea f : R2 → R, (u, v) → f (u, v) una función de clase C 1 (R2 ). Supon-
∂f
gamos que ∂v
6= 0 en todo R2 . Muestre que la ecuación

f x2 − y 2 , y 2 − z 2 = 0


define implı́citamente una función z = z (x, y) para z 6= 0 y que la expresión


∂z ∂z
E (x, y, z) = yz + zx
∂x ∂y
no depende de f .

Solución. Definamos G (x, y, z) = f (x2 − y 2 , y 2 − z 2 ) entonces G es compuesta de funcio-


nes C 1 y por tanto es C 1 , además
∂G ∂f ∂u ∂f ∂v
= +
∂z ∂u ∂z ∂v ∂z
∂f
= (−2z) 6= 0
∂v
por el teorema de la función implı́cita existe una única función z = z (x, y) de clase C 1 tal
que

∂z − ∂G fu ux + fv vx
= ∂G∂x = −
∂x ∂z
fv (−2z)
  
fu 2x + fv 0 fu x
= − =
fv (−2z) fv z

451
Apuntes Mat023 (versión preliminar actualizada 23-05-2014)

similarmente
∂z − ∂G
∂y fu uy + fv vy
= ∂G = −
∂y ∂z
fv (−2z)
fu (−2y) + fv (2y) −fu y + fv y
= − =
fv (−2z) fv z
se sigue
     
∂z ∂z fu x −fu y + fv y
yz + zx = yz + zx
∂x ∂y fv z fv z
= xy

luego
∂z ∂z
yz + zx = xy
∂x ∂y
no depende de f .

Estudiaremos ahora el caso más general en el cual F : RN × Rm → Rm . Supongamos


que tenemos un sistema de ecuaciones lineales

2u + 3v + x − y = 0
u − v + 2x + 3y = 0

sabemos que este sistema tiene infinitas soluciones 4 variables y dos ecuaciones (es cuestión
de rangos), las infinitas soluciones son porque podemos dejar variables en función de otras,
notemos lo siguiente

2u + 3v = b1 = y − x
u − v = b2 = −2x − 3y

para resolver el sistema

2u + 3v = b1
u − v = b2

podemos utilizar Cramer y se obtiene



y−x

b1 3 3

b2 −1 −2x − 3y −1

7 8
u = = = − x − y
2 3

2 3

5 5

1 −1 1 −1

452
Apuntes Mat023 (versión preliminar actualizada 23-05-2014)

y
y−x

2 b1 2

1 b2 1 −2x − 3y

3 7
v = = = x+ y
2 3

2 3
5 5

1 −1 1 −1

luego las ecuaciones

2u + 3v + x − y = 0
u − v + 2x + 3y = 0

permiten despejar a u y v como funciones de x e y,


7 8
u (x, y) = − x − y
5 5
3 7
v (x, y) = x+ y
5 5
Ahora suponga que tenemos ecuaciones no lineales

xu + yvu2 = 2
xu3 + y 2 v 4 = 2

en este caso se nos hace mucho más difı́cil despejar u, v en función de x e y. Estas ecuaciones
las podemos reinterpretar como

F (x, y, u, v) = xu + yvu2 − 2, xu3 + y 2 v 4 − 2 = (F1 , F2 ) = (0, 0)




note que las ecuaciones se satisfacen en el punto (1, 1, 1, 1). Sabemos que una buena
aproximación de F (x, y, u, v) cerca de (1, 1, 1, 1) es dada por

F (1, 1, 1, 1) + DF (1, 1, 1, 1) (x − 1, y − 1, u − 1, v − 1)T

donde
 
∂F1 ∂F1 ∂F1 ∂F1
∂x ∂y ∂u ∂v
DF (x, y, u, v) =  
∂F2 ∂F2 ∂F2 ∂F2
∂x ∂y ∂u ∂v
 
2 2
u vu x + 2yvu yu
=  
u3 2yv 4 3xu2 4v 3 y 2

453
Apuntes Mat023 (versión preliminar actualizada 23-05-2014)

evaluando en el punto  
1 1 3 1
DF (1, 1, 1, 1) =  
1 2 3 4
luego
 
1 1 3 1
F (x, y, u, v) ≈   (x − 1, y − 1, u − 1, v − 1)T
1 2 3 4
 
3u + v + x + y − 6
≈  
3u + 4v + x + 2y − 10
luego la ecuación
F (x, y, u, v) = (0, 0)
debe ser similar a

3u + v + x + y = 6
3u + 4v + x + 2y = 10

en el cual podemos despejar u y v en función de x e y, para poder hacer esto necesitamos




3 1
= D(u,v) F (1, 1, 1, 1)
3 4



∂F1
∂u (1, 1, 1, 1) ∂F

∂v
1
(1, 1, 1, 1)
=
∂F2 ∂F2
∂u (1, 1, 1, 1) ∂v (1, 1, 1, 1)

6= 0

(permite usar Cramer para despejar las variables) Despejando



6−x−y 1


10 − x − 2y 4

14 2 1
u= = − y − x

3 1
9 9 3

3 4

y
3 6−x−y



3 10 − x − 2y

4 1
v= = − y

3 1
3 3

3 4

454
Apuntes Mat023 (versión preliminar actualizada 23-05-2014)

de donde deberı́a cumplirse


1 2
ux = − , uy = −
3 9
1
vx = 0, vy = −
3
(note que deberı́amos poner
14 2 1
u (x, y) ≈ − y− x
9 9 3
4 1
v (x, y) ≈ − y
3 3
(es una aproximación de primer orden de estas funciones) cerca del punto (1, 1, 1, 1)). Estas
ideas se resumen en el siguiente:

Teorema 11.1.2 (De la función implı́cita). Sean Ω abierto de RN , Λ abierto de Rm ,


F : Ω × Λ → Rm una función de clase C p (Ω × Λ). Pongamos (x, y) para los puntos en
RN × Rm . Suponga que (x0 , y0 ) es un punto tal que

F (x0 , y0 ) = 0

y que Dy F (x0 , y0 ) es invertible, entonces existe un abierto U ⊆ RN , una función y = g (x)


de U a Rm de clase C p (U ) tal que

F (x, g (x)) = 0 para todo x ∈ U

y0 = g (x0 ) además
Dg (x) = −Dy (x, y)−1 Dx (x, y)

Ejemplo 11.1.6. Mostrar que cerca del punto (x, y, u, v) = (1, 1, 1, 1) podemos resolver

xu + yvu2 = 2
xu3 + y 2 v 4 = 2

∂u

de manera única para u y v como funciones de x y y. Calcular ∂x
(1, 1)

Solución. Definamos

F : R2 × R2 → R2
xu + yvu2 − 2, xu3 + y 2 v 4 − 2

((x, y) , (u, v)) →

455
Apuntes Mat023 (versión preliminar actualizada 23-05-2014)

entonces
F ((1, 1) , (1, 1)) = (0, 0)

calculemos D(u,v) F (x, y, u, v)


 
∂ (xu+yvu2 −2) ∂ (xu+yvu2 −2)
∂u ∂v
D(u,v) F (x, y, u, v) =  ∂ (xu3 +y 2 v 4 −2) ∂ (xu3 +y 2 v 4 −2)

∂u ∂v
 
2
x + 2yuv yu
=  
3xu2 4y 2 v 3

en (1, 1, 1, 1) es  
3 1
D(u,v) F (1, 1, 1, 1) =  
3 4
el determinante es 12 − 3 = 9 6= 0 se sigue que es invertible y ası́ por el teorema de la
función implı́cita podemos despejar de manera única para u y v como funciones de x y y.
Además g (x, y) = (u (x, y) , v (x, y)) tiene por derivada
 −1
x + 2yuv yu2
Dg (x, y) = −   D(x,y) (x, y, u, v)
2 2 3
3xu 4y v

donde
 
∂ (xu+yvu2 −2) ∂ (xu+yvu2 −2)
∂x ∂y
D(x,y) (x, y, u, v) =  ∂ (xu3 +y 2 v 4 −2) ∂ (xu3 +y 2 v 4 −2)

∂x ∂y
 
2
u vu
=  
3 4
u 2yv

entonces  −1  
2 2
x + 2yuv yu u vu
Dg (x, y) = −    
3xu2 4y 2 v 3 u3 2yv 4
en el punto en cuestión
 −1    
3 1 1 1 − 31 − 29
Dg (1, 1) = −    = 
3 4 1 2 0 − 13

456
Apuntes Mat023 (versión preliminar actualizada 23-05-2014)

de donde obtenemos
   
∂u ∂u
∂x
(1, 1) ∂y
(1, 1) − 13 − 29
 = 
∂v ∂v
∂x
(1, 1) ∂y
(1, 1) 0 − 13

se sigue
∂u 1
(1, 1) = −
∂x 3
Ejemplo 11.1.7. Discutir sobre la resolubilidad del sistema

3x + 2y + z 2 + u + v 2 = 0
4x + 3y + z + u2 + v + w + 2 = 0
x + z + w + u2 + 2 = 0

para u, v, w en términos de x, y, z en el punto x = y = z = 0, u = v = 0 y w = −2. Si es


posible, obtener la ecuación del plano tangente a

w (x, y, z) = 0

en (0, 0, 0).

Solución. Definamos F : R6 → R3 dada por

F (x, y, z, u, v, w)
3x + 2y + z 2 + u + v 2 , 4x + 3y + z + u2 + v + w + 2, x + z + w + u2 + 2

=

el sistema pude verse como

F (x, y, z, u, v, w) = (0, 0, 0)

notamos que



1 2v 0
 
D(u,v,w) F (0, 0, 0) =  2u 1 1 
 

2u 0 1
(0,0,0)
 
1 0 0
 
= 
 0 1 1 

0 0 1

457
Apuntes Mat023 (versión preliminar actualizada 23-05-2014)

que tiene determinante 1, por el teorema de la función implı́cita se sigue que podemos
despejar (u, v, w) en términos de (x, y, z) además
   −1  
∂u ∂u ∂u 1 0 0 3 2 0
 ∂x ∂y ∂z 

   
 ∂v ∂v ∂v  = 
 0 1 1   4 3 1 
 ∂x ∂y ∂z    
∂w ∂w ∂w

∂x ∂y ∂z
0 0 1 1 0 1
(0,0,0)
 
3 2 0
 
= 
 3 3 0 

1 0 1
de esto obtenemos
∇W = (1, 0, 1)
y ası́ el plano es

(1, 0, 1) (x, y, z) = 0
x+z = 0

Ejercicios de la sección

1. Sea F : R2 → R, (u, v) → F (u, v) una función de clase C ∞ que satisface

F (0, 0) = 0
∂F ∂F
(0, 0) + (0,0) 6= 0
∂u ∂v
Muestre que la ecuación
F x3 − yz, y 3 − xz = 0


define una función z = z (x, y) de clase C ∞ en un entorno del punto (x, y) = (1, 1)
que satisface z (1, 1) = 1 y la ecuación
 ∂z  ∂z
xz + 3y 3 + yz + 3x3 = 9x2 y 2 − z 2
∂x ∂y
2. Probar que cerca del punto (x0 , y0 , z0 , u0 , v0 ) = 1, 1, 0, π2 , 0 se puede resolver el


sistema

x2 − y cos (uv) + z 2 = 0
x2 + y 2 − sin (uv) + 2z 2 − 2 = 0
xy − sin u cos v + z = 0

458
Apuntes Mat023 (versión preliminar actualizada 23-05-2014)

de manera única para x, y, z como funciones de u y v. Calcular ∂x π



∂v 2
,0
 
z z
3. Sea f : R → R definida por f (x, y, z) = g x + y , y + x donde g : R2 → R y
3

∂f
∂z
(x, y, z) 6= 0

a) Verificar que la ecuación f (x, y, z) = 0 define implı́citamente a z como una


función de clase C 1 de las variables x e y.

b) Comprobar que
∂z ∂z
x +y = z − xy
∂x ∂y

4. Mostrar que el sistema

x2 − y 2 + u2 + 2v 2 = 5
x2 + y 2 − u2 − v 2 = −4

define implı́citamente a u = u (x, y), v = v (x, y) con u (0, 1) = 2 y v (0, 1) = −1.


∂2v
Encontrar las diferenciales Du (0,1) y Dv (0, 1) además de la derivada ∂x2
(0, 1).

5. Suponga que F, G : R3 → R son dos funciones de clase C 1 (R3 ) y x0 = (x0 , y0 , z0 ) ∈ R3


es un punto en el cual F (x0 ) = 0, G (x0 ) = 0, esto es un punto de la intersección de
las superficies

S1 : F (x, y, z) = 0
S2 : G (x, y, z) = 0

Determine una condición que permita asegurar que es posible despejar x = x (y),
z = z (y) como funciones de clase C 1 en un entorno de y0 luego la intersección se
puede parametrizar como

x = x (y)
y = y
z = z (y)

para y en un entorno de y0 (el problema nos dice cuando podemos parametrizar con
un parámetro la intersección de dos superficies)

459
Apuntes Mat023 (versión preliminar actualizada 23-05-2014)

El teorema de la función inversa

Si f : R → R es una función de clase C 1 (R) y x0 es un punto al que f 0 (x0 ) 6= 0 entonces,


por la continuidad de la derivada, podemos garantizar la existencia de un intervalo abierto
I tal que f 0 (x) para todo x ∈ I tiene el mismo signo de f 0 (x0 ), entonces la función f
es estrictamente creciente o estrictamente decreciente en I (dependiendo si f 0 (x0 ) > 0 o
f 0 (x0 ) < 0).

f f0

f 0 (x0 )

x0

En la gráfica podemos observar en verde la gráfica de una función y en azul la de su


función derivada, en el punto x0 la derivada es no nula (estrictamente positiva), existe un
intervalo (marcado en rojo) en el cual la función derivada es positiva, note que en todo ese
intervalo la función f es estrictamente creciente.
Si restringimos nuestra función f al intervalo I entonces f es inyectiva (toda función
estrictamente creciente o estrictamente decreciente es inyectiva pues envı́a números distintos
del dominio en imágenes distintas), si además restringimos el recorrido a f (I) entonces es
sobreyectiva y podemos definir una inversa (no de la función completa, se trata de una
inversa local.

460
Apuntes Mat023 (versión preliminar actualizada 23-05-2014) f0

f −1

note que solo podemos garantizar la existencia, obtener una expresión para ella puede ser
muy complicado. ¿Como podemos trabajar entonces con tal función?, ¿Cuales propiedades
de f conserva su inversa?.
El teorema de la función inversa de una variable asegura que la inversa es una función
de la misma clase (en términos de derivadas) además

f ◦ f −1 (x) = x


entonces
d
f ◦ f −1 (x) = 1
 
dx
por la regla de la cadena
df −1  df −1
f (x) (x) = 1
dx dx
entonces
df −1 1
(x) = 0 −1
dx f (f (x))
si queremos calcular la derivada de segundo orden

d2 f −1 d 1
(x) =
dx2 dx f 0 (f −1 (x))
 
0 −1
−2 00 −1  1
= − f f (x) f f (x)
f 0 (f −1 (x))
−f 00 (f −1 (x))
=
(f 0 (f −1 (x)))3

461
Apuntes Mat023 (versión preliminar actualizada 23-05-2014)

Si T : Rn → Rn es una transformación lineal, sabemos que es invertible si y solo si su


matriz asociada (por ejemplo respecto a las bases canónicas) es invertible, sabemos que la
matriz asociada respecto a las bases canónicas corresponde a la matriz Jacobiana de la T .

JT (x0 ) = [T ]CC

luego, si det (JT (x0 )) 6= 0 la transformación lineal es invertible (derivada no nula implica
invertible), es razonable esperar que si una función se parece a una transformación lineal
localmente entonces podamos obtener información de ella a través de la transformación
lineal a la cual se parece, eso es justamente lo que afirma el teorema de la función inversa
de varias variables.

Observación 11.2.1. Supongamos que U, V son abiertos y F : U ⊆ Rn → V ⊆ Rm


(diferenciable) tiene inversa (diferenciable) entonces n = m. En efecto, si G : V → U es su
inversa entonces

F ◦G : V →V
x → (F ◦ G) (x) = x

G◦F : U →U
x → (G ◦ F ) (x) = x

luego

D (F ◦ G) = Im
D (G ◦ F ) = In

ası́

DF (G (x)) DG (x) = Im
DG (F (v)) DF (v) = In

si ponemos x =F (v) entonces G (x) = v entonces

DF (G (x)) DG (x) = Im
DG (x) DF (G (x)) = In

462
Apuntes Mat023 (versión preliminar actualizada 23-05-2014)

pongamos A = DF (G (x)) y B = DG (x) entonces

AB = In
BA = Im

definamos las transformaciones lineales

TA : Rn → Rm
x → TA (x) = Ax

TB : Rm → Rn
x → TB (x) = Bx

entonces TA ◦ TB es IRm y TB ◦ TA es IRn de la primera se obtiene que TB es inyectiva

TB (x) = TB (y) ⇒ TA (TB (x)) = TA (TB (y))


⇒ x=y

y de la segunda que TB es sobreyectiva (si y ∈ Rn entonces TA (y) = v ∈ Rm entonces

TB (v) = TB (TA (y)) = y

de donde Im(TB ) = Rn ) se sigue por el teorema de las dimensiones que

Dim (Rn ) = Dim (Rm )

ası́ n = m.

Teorema 11.2.1 (De la función inversa). Sean U un abierto de Rn , F : U ⊆ Rn → Rn


una función de clase C p (U ) y x0 ∈ U un punto en el cual det (DF (x0 )) 6= 0 entonces
existe un abierto U1 ⊆ U tal que x0 ∈ U1 y F : U1 → V = F (U1 ) es invertible, su inversa
F −1 : V → U1 es de clase C p (V ), además

DF −1 (F (x)) = (DF (x))−1 para x ∈ U1

en particular, si F (x0 ) = y0 entonces

DF −1 (y0 ) = (DF (x0 ))−1

463
Apuntes Mat023 (versión preliminar actualizada 23-05-2014)

Observación 11.2.2. Si x = (x1 , x2 , . . . , xn ) y F : D ⊆ Rn → Rn , x → F (x) =


(F1 (x) , F2 (x) , . . . , Fn (x)) entonces det (DF (x0 )) se representa por el sı́mbolo

∂ (F1 , F2 , . . . , Fn )
∂ (x1 , x2 , . . . , xn ) x=x0

y es llamado el Jacobiano (es el determinate de la matriz Jacobiana).

Ejemplo 11.2.1. Muestre que la función

F : R2 → R2
(x, y) → F (x, y) = (u (x, y) , v (x, y)) = (ex cos y, ex sin y)

no tiene una inversa global (definida en R2 ), sin embargo, cerca de todo punto es posible
definir una inversa local. Calcular
√ √ 
DF −1 2, 2
√ √ 
si F ln 2, π4 =

2, 2 y obtener una expresión para

DF −1 (u, v)

si (u, v) 6= 0.

Solución. Notemos
F (0, 2π) = F (0, 0) = (1, 0)

luego F no es inyectiva y por tanto no tiene inversa global.


Notemos que F ∈ C ∞ (R2 ) además
   
∂F1 ∂F1 ∂u ∂u
∂x ∂y ∂x ∂y
   
DF (x, y) = 

=
 


∂F2 ∂F2 ∂v ∂v
∂x ∂y ∂x ∂y
 
ex cos y −ex sin y
=  
ex sin y ex cos y
y  
x x
∂ (u, v) e cos y −e sin y
= det   = e2x 6= 0
∂ (x, y) x x
e sin y e cos y

464
Apuntes Mat023 (versión preliminar actualizada 23-05-2014)

ası́, por el teorema de la función inversa, F es invertible cerca de cada punto de R2 (inversa
local) además

DF −1 (F (x, y)) = (DF (x, y))−1


 −1
ex cos y −ex sin y
=  
x x
e sin y e cos y

de esto se obtiene
 π π −1
eln 2 cos −eln 2 sin
  π   4 4 
DF −1 F ln 2, = 
 
4


ln 2 π ln 2 π 
e sin e cos
4 4

1
√ 1
√ 
4
2 4
2
 
=  
√ √
 
1 1
−4 2 4
2

esto es  √ √ 
1 1
√ √   4
2 4
2
DF −1

2, 2 =  
√ √
 
− 41 2 1
4
2
si F (x, y) = (u, v) entonces F (u, v) = (x, y) donde x = x (u, v) e y = y (u, v) además

∂x
√ √  ∂x
√ √    1√ 1
√ 
∂u
2, 2 ∂v
2, 2 2 2
   4 4

 = 
√ √  √ √  √ √
   
∂y ∂y
∂u
2, 2 ∂v
2, 2 − 14 2 1
4
2

ası́ por ejemplo


∂x √ √  1 √
2, 2 = 2
∂u 4

465
Apuntes Mat023 (versión preliminar actualizada 23-05-2014)

en general

DF −1 (u, v) = DF −1 (F (x, y))


 −1
ex cos y −ex sin y
=  
ex sin y ex cos y
 −1
u −v
=  
v u
 u v 
 u2 + v 2 u2 + v 2 
= 
 

 v u 
− 2
u + v2 u2 + v 2
Observación 11.2.3. Note que el teorema de la función inversa afirma DF −1 (y0 ) =
(DF (x0 ))−1 luego
∂ (u, v) 1
= ∂(x,y)
∂ (x, y)
∂(u,v)

cuidando donde están evaluadas las funciones.

Ejemplo 11.2.2. Sea f : R2 → R2 la función definida por

f (u, v) = u2 + u2 v + 10v, u + v 3


1. Mostrar que tiene inversa cerca del punto (1, 1)

Solución. Notemos que la función es de clase C ∞ (R2 ) y


 
∂F1 ∂F1
∂u ∂v
Df (1, 1) =  
∂F2 ∂F2
∂u ∂v
 
∂ (u2 +u2 v+10v ) ∂ (u2 +u2 v+10v )
∂u ∂v
=  ∂ (u+v 3 ) ∂ (u+v 3 )

∂u ∂v
 
2
2u (v + 1) u + 10
=  
2
1 3v


(1,1)
 
4 11
=  
1 3

466
Apuntes Mat023 (versión preliminar actualizada 23-05-2014)

luego  
4 11
det   = 1 6= 0
1 3
y ası́, por el teorema de la función inversa, la función es localmente invertible, además

f (1, 1) = (12, 2)

2. Calcular la derivada de su inversa en el punto (12, 2).

Solución. Como f (1, 1) = (12, 2) se sigue


 −1
4 11
Df −1 (12, 2) = Df −1 (f (1, 1)) =  
1 3
 
3 −11
=  
−1 4

Ejemplo 11.2.3. Sea u : R2 → R, (x, y) → u (x, y) una función que satisface


∂u  ∂u
2xy − x2 − y 2 =0
∂x ∂y

1. Utilizar el teorema de la función inversa para determinar condiciones sobre la función


ζ de modo que

F : D ⊆ R2 → R2
F (x, y) = (µ (x, y) , ζ (x, y))
 
x
= , ζ (x, y)
x2 + y 2
sea invertible.

Solución. Por el teorema de la función inversa, si



∂µ ∂µ x2 −y 2 2xy
− (x2 +y2 )2 − (x2 +y2 )2

∂x ∂y
∂ζ ∂ζ =

∂ζ ∂ζ
∂x ∂y ∂x ∂y

x2 − y 2
   
∂ζ ∂ζ 2xy
= − +
∂y (x2 + y 2 )2 ∂x (x2 + y 2 )2
6= 0

467
Apuntes Mat023 (versión preliminar actualizada 23-05-2014)

2. Escribir la ecuación
∂u  ∂u
2xy − x2 − y 2 =0
∂x ∂y
en las nuevas variables (µ, ζ) mediante regla de la cadena, resolver esa nueva ecuación
y concluir que u debe tener la forma
 
x
u (x, y) = f
x + y2
2

donde f ∈ C 1 (R) es una función arbitraria.

Solución.

∂u ∂u ∂µ ∂u ∂ζ
= +
∂x ∂µ ∂x ∂ζ ∂x
∂u ∂u ∂µ ∂u ∂ζ
= +
∂y ∂µ ∂y ∂ζ ∂y
de donde

x2 − y 2
 
∂u ∂u ∂u ∂ζ
= − +
∂x ∂µ (x2 + y 2 )2 ∂ζ ∂x
 
∂u ∂u 2xy ∂u ∂ζ
= − +
∂y ∂µ (x2 + y 2 )2 ∂ζ ∂y

se sigue

∂u  ∂u
0 = 2xy − x2 − y 2
∂x ∂y
x − y2
2
   
∂u ∂u ∂ζ
= 2xy − +
∂µ (x2 + y 2 )2 ∂ζ ∂x
   
2 2
 ∂u 2xy ∂u ∂ζ
− x −y − +
∂µ (x2 + y 2 )2 ∂ζ ∂y
∂u ∂ζ  ∂u ∂ζ
= 2xy − x2 − y 2
∂ζ ∂x ∂ζ ∂y
 
∂u ∂ζ 2 2 ∂ζ

= 2xy − x −y
∂ζ ∂x ∂y
pero  
∂ζ 2 2 ∂ζ

2xy − x −y 6= 0
∂x ∂y
entonces
∂u
=0
∂ζ

468
Apuntes Mat023 (versión preliminar actualizada 23-05-2014)

se sigue
u (µ, ζ) = f (µ)

ası́  
x
u (x, y) = f
x + y2
2

donde f es una función arbitraria.

Ejercicios de la sección

1. Sea f : R → R una función de clase C 1 y sea

u = f (x)
v = −y + xf (x)

si f 0 (x0 ) 6= 0 probar que la función T (x, y) = (u, v) es invertible cerca de (x0 , y0 ) y


que la inversa tiene la forma

x = f −1 (u)
y = −v + uf −1 (u)

encontrar DT −1 .
 2

2. Considere la función F (x, y) = (x − y)2 , xy para y 6= 0.

a) Probar que F admite inversa local en una vecindad de (−1, 1)

b) Sea F −1 : V ⊆ R2 → R2 , (x, y) = (g (u, v) , h (u, v)) la inversa local de F ,


calcular la razón de cambio de h en (4, 1) en la dirección del vector (2, −1).

3. Sea F (x, y) = (f1 (x, y) , f2 (x, y)) = (x cos y, sin (x − y)). Mostrar que F tiene inversa
local en una vecindad del punto π2 , π2 y obtener la Jacobiana de la inversa en (0, 0).


4. Definimos x : R2 → R, (r, θ) → x (r, θ) = r cos θ y y : R2 → R, (r, θ) → y (r, θ) =


r sin θ.

469
Apuntes Mat023 (versión preliminar actualizada 23-05-2014)

a) Demostrar que
∂ (x, y)
= r0
∂ (r, θ) (r0 ,θ0 )

b) ¿cuándo se puede formar una función inversa suave de F : R2 → R2 , (r, θ) →


(x (r, θ) , y (r, θ))? Comprobarlo directamente y con el teorema de la función
inversa.

5. Definamos F : R3 → R3 , (ρ, φ, θ) → F (ρ, φ, θ) = (x (ρ, φ, θ) , y (ρ, φ, θ) , z (ρ, φ, θ))


donde

x (ρ, φ, θ) = ρ sin φ cos θ


y (ρ, φ, θ) = ρ sin φ sin θ
z (ρ, φ, θ) = ρ cos φ

a) Muestre que
∂ (x, y, z)
= ρ2 sin φ
∂ (ρ, φ, θ)
b) ¿Cuándo se puede despejar (ρ, φ, θ) en términos de (x, y, z)?

Ejercicios del capı́tulo

1. Si u1 , u2 son las raı́ces de la ecuación cuadrática en λ

x2 + y 2 z2
+ =1
λ−2 λ−1

y u3 es arctan (y/x) probar que

∂ (u1 , u2 , u3 ) 4z
=−
∂ (x, y, z) u1 − u2
√ √ 
2. La ecuación x3 + y 3 = 6xy define cerca del punto 2 3 2, 2 3 4 a y como función de x.

Muestre que x = 2 3 2 es un punto crı́tico de tal función y clasificarlo en máximo,
mı́nimo o punto silla.

470
Apuntes Mat023 (versión preliminar actualizada 23-05-2014)

3. La ecuación
y z 
, f =0
x x
define a z como función de x, y, z = g (x, y). Muestre que
∂g ∂g
x +y =g
∂x ∂y
4. Si ϕ, φ : D ⊆ R3 → R son funciones de clase C 1 tales que
∂ (ϕ, φ)
6= 0
∂ (y, z)
entonces las ecuaciones ϕ (x, y, z) = 0, φ (x, y, z) = 0 determinan y, z como funciones
de x y
∂(ϕ,φ) ∂(ϕ,φ)
dy ∂(z,x) dz ∂(x.y)
dx
= ∂(ϕ,φ) dx
= ∂(ϕ,φ)
∂(y,z) ∂(y,z)

5. Si z1 = er cos θ, z2 = er sin θ y x = r cos θ, y = r sin θ probar que


∂ (z1 , z2 ) e2r
=
∂ (x, y) r
6. Suponga que x, y, z, r, θ, φ están relacionados por

f (x, y, z) = 0

y
x = r sin θ cos φ y = r sin θ sin φ z = r cos θ
probar que
  
∂z ∂r ∂r
sin θ sin θ + r cos θ + sin φ
∂x ∂θ ∂φ
 
∂r
− sin θ cos φ cos θ − r sin θ = 0
∂θ

7. Sea a > 0. Si y 4 + 3a2 y 2 − 5a2 xy + 2a2 x2 = 0, probar que


dy dy 2
lı́m = 0 o lı́m =
x→0 dx x→0 dx 3
8. Si u, v son funciones de las variables x, y por medio de las ecuaciones f (x, y, u) = 0
y g (u, v, x) = 0 probar que
∂v fx gu − fu gx
=
∂x f u gv
∂v fy gu
=
∂y fu gv

471
Apuntes Mat023 (versión preliminar actualizada 23-05-2014)

∂f
9. La ecuación ∂z
(x, z) = u define a z como función de x y u. Si ponemos z = g (x, u)
y se define ϕ (x, u) = f (x, g (x, u)) probar que

∂ϕ ∂z
= u
∂u ∂u
∂ϕ ∂f ∂z
= +u
∂x ∂x ∂x

10. Suponiendo que u, v son funciones de x, y las cuales definen a u, v en términos de


x, y y satisfacen las ecuaciones

∂u ∂v ∂u
+ +u = 0
∂x ∂y ∂y
∂v ∂u ∂v
+ +u = 0
∂x ∂y ∂y

Probar que
∂ 2x ∂ 2x ∂x
2
− 2 =
∂u ∂v ∂v
11. Si u1 x1 = u2 x2 = u3 x3 = u4 x4 = (u1 u2 u3 u4 )1/2 entonces

∂ (u1 , u2 , u3 , u4 )
= −1
∂ (x1 , x2 , x3 , x4 )

12. Determinar el Jacobiano de la transformación

x = a (ρ cos θ sin φ)α


y = b (ρ sin θ sin φ)β
z = c (ρ cos φ)γ

donde a, b, c, α, β.γ son constantes positivas y determinar cuando es posible invertir


tal transformación. Ind.: Compuestas de funciones.

472
Parte III

Evaluaciones de años anteriores

473
Capı́tulo 12 : Controles

Control 1
q
x−y
1. Dada la función f : D ⊆ R2 → R definida por f (x, y) = x+y

a) Determine y grafique el dominio de f .

b) ¿El dominio de f es un conjunto abierto?

c) Determine la frontera del dominio de f y sus puntos de acumulación.


1
d ) Determine las curvas de nivel f (x, y) = c y grafique para c = 0, c = 1, c = 2
y
c = 2.

e) Analice que sucede cuando c crece indefinidamente.

2. Sea T : R3 [x] → R2 [x] definida por


Z 1
00
T [p (x)] = p (x) + p (x) dx
0

a) Pruebe que T es una transformación lineal.

b) Sean

1, x − 1, (x − 1)2 , (x − 1)3

B1 =
B2 = {1, x, x (x − 1)}

bases de R3 [x] y R2 [x] respectivamente. Determine [T ]BB21 y use esta matriz para
obtener el núcleo de T .

Control 2

1. Sea T : R4 → R3 una transformación lineal definida por

T (x, y, z, w) = (x − y + z + w, x + 2z − w, x + y + 3z − 3w)

Encuentre una base y la dimensión de Ker(T ) e Im(T ).

2. Describa explı́citamente una transformación lineal T : R3 → R2 tal que

Im (T ) = h{(1, 0, −1) , (1, 2, 2)}i

474
Apuntes Mat023 (versión preliminar actualizada 23-05-2014)

Control 3

1. Considere la función T : R2 [x] → R3 definida por T (p (x)) = (p (0) , p (1) , p (2)). Sean
B = {1, x, x2 } y C = {(1, 0, 0) , (0, 1, 0) , (0, 0, 1)} bases de R2 [x] y R3 respectivamente:

a) Pruebe que T es una transformación lineal.

b) Determine Ker(T ) y [T ]CB .

c) ¿T es un isomorfismo?.
p
2. Considere la función f : D ⊆ R2 → R, definida por f (x, y) = sin (x2 + y 2 ).

a) Encuentre el dominio de f y dibujarlo.

b) Determine si el dominio es abierto o cerrado.

Control 4

1. Sea 
 x + y si |x| + |y| ≥ 2


f (x, y) =


1 si |x| + |y| < 2

¿Existe lı́m f (x, y)? Justificar.


(x,y)→(1,1)

2. Demuestre o dé un contraejemplo de la propiedad: Para todo A, B ⊆ Rn

∂ (A ∪ B) ⊆ ∂A ∪ ∂B

Control 5

1. Considere la función f : {(x, y) ∈ R2 : x 6= 0} → R, definida por


y
2 2

f (x, y) = x + y arctan
x

¿es posible definir f en (0, 1) de forma que f sea continua en tal punto?

475
Apuntes Mat023 (versión preliminar actualizada 23-05-2014)

2. Sea f : R2 → R definida por



xy


 x2 +y
si x2 6= −y
f (x, y) =


0 si x2 = −y

∂f ∂f
Hallar ∂x
(x, y) y ∂x
(x, y) en todos los puntos que estas existan y determinar si son
continuas en (0, 0).

Control 6

1. Considere el sistema

x = u+v+w
y = u2 + v 2 + w 2
z = u3 + v 3 + w 3

∂v ∂w
Calcule ∂y
(2, 6, 8) y ∂x
(2, 6, 8) donde (x, y, z) = (2, 6, 8) es la imagen de (u, v, w) =
(1, 2, −1).

2. Considere el sistema

u3 + xv 2 + y = 0
v 3 + yv + u2 = 0

a) Pruebe que es posible despejar x = x (u, v), y = y (u, v) en vecindades de los


puntos (x, y) = (1, −1) y (u, v) = (0, 1).
∂u ∂v
b) Hallar ∂x
(1, −1) y ∂x
(1, −1).

Control 7

1. Un servicio de entrega de paquetes requiere que las dimensiones de una caja rectan-
gular sean tales que la longitud más el doble del ancho más el doble de la altura
no rebase 108 cms. ¿Cuál es el volumen de la caja más grande que podrá enviar la
compañı́a?

476
Apuntes Mat023 (versión preliminar actualizada 23-05-2014)

2. Considerar el sistema

xy 2 + xzu + yv 2 = 3
u3 yz + 2xv − u2 v 2 = 2

¿Es posible despejar u = u (x, y, z) y v = v (x, y, z) en vecindades de U de (x, y, z) y


∂v
V de (u, v) = (1, 1). Calcular ∂y
(1, 1, 1).

3. Resuelva la ecuación
dy 1 − xy 2
=
dx 2x2 y
haciendo la sustitución v = y/xn para algún n.

4. Muestre que la ecuación


dy y y
= + xm y n f
dx x x
se transforma en una ecuación de variables separables usando el cambio de variables
y = vx donde v = v (x). Use lo anterior para resolver la ecuación
y

dy y sec2 x
= +
dx x y2

Control 8

1. Encuentre la solución general de la ecuación

d4 y d3 y d2 y dy
4
− 3
+ 2 −3 − 6y = 0
dx dx dx dx

2. Un estanque contiene 50 litros de agua pura. Al estanque entra salmuera, que contiene
C gramos de sal por litro a razón de 1,5 litros por minuto. La mezcla bien revuelta,
sale a razón de 1 litro por minuto. Si después de 30 minutos la concentración de sal
en el estanque es de 30 gramos por litro. Hallar el valor de C.

477
Apuntes Mat023 (versión preliminar actualizada 23-05-2014)

Control 9
dz
1. Hallar dx
si se cumple x3 + y 3 + z 3 = 0 y x2 + y 2 + z 2 = 1

2. La matriz  
1 −1
A= 
1 2

representa el jacobiano de una función f de clase C 2 de R2 en R2 en el punto (1, 1).


Suponga que f (1, 1) = (−1, 0) y que f (x, y) = (u, v). Pruebe que f es localmente
∂y
invertible en (1, 1) y encuentre ∂u
(−1, 0).

Control 10

1. Muestre que la ecuación diferencial

2x4 yy 0 + y 4 = 4x6

se reduce a una ecuación homogénea mediante el cambio de variables y = z n para


cierto n ∈ R. Determine el valor de n y resuelva la ecuación.

2. Resuelva la ecuación x3 yy 0 + 2x2 y 2 − 1 = 0 usando el cambio de variables u = x2 y.

Control 11

1. Usando la transformada de Laplace resuelva el problema de valor inicial

ty 00 − ty 0 + y = 2 et − 1


con y (0) = 0, y 0 (0) = −1.

2. Obtenga la transformada de Laplace inversa de

s2 − s
s3 + s2 + 9s + 9

478
Apuntes Mat023 (versión preliminar actualizada 23-05-2014)

Control 12

1. Resuelva el problema de valores iniciales


dy
− ex−y + ex = 0
dx
y (0) = 1

Ayuda. Use el cambio de variables u = ey .

2. Resuelva la ecuación y 2 dx = (x3 − xy) dy usando un factor integrante de la forma


xn y m .

Control 13

Sea α ∈ R − Z.

1. Encontrar el desarrollo en serie de Fourier de la función f : ]−π, π[ → R

f (x) = cos (αx)

2. Verificar que " ∞


#
1 1 X 2α
cot (απ) = −
π α n=1 n2 − α2

Control 14

1. La recta normal, en cada punto (x, y) de la curva dada, pasa por el punto (2, 0). Si
la curva pasa por (2, 3) encuentre su ecuación. Justificar.

2. Resolver la ecuación

(2x + 1)2 y 00 − 2 (2x + 1) y 0 − 12y = 6x

Control 15

1. Obtenga la solución de las ecuaciones

a) (6xy 2 − 3x2 ) dx = − (6x2 y + 3y 2 − 7) dy

479
Apuntes Mat023 (versión preliminar actualizada 23-05-2014)

y
b) xy 0 − y = ln y−ln x

2. En la schoperı́a de “Don Ramón” se vende cerveza artesanal a 4 o C. Dado que se


está en época de verano la temperatura interior de la schoperı́a es de 30 o C. El señor
Ramón es muy exigente con la calidad de su producto y pide a los meseros no servir
Schops si es que éstos están a más de 7 o C. Sabiendo que a los 60 segundos la cerveza
llega a los 5 o C y que además un mesero demora 2 minutos en transportar la cerveza
desde su fuente hasta la mesa ¿podrán los meseros cumplir con las exigencias de Don
Ramón? (justifique matemáticamente)

Control 16

1. Calcular  
sin t
L (s)
t
2. Resolver
ty 00 + 2y 0 + ty = 0

con y (0) = 1 e y (π) = 0.

3. Encontrar el desarrollo en serie de Fourier para



 0 −π < x < 0
f (x) =
 π−x 0<x<π

y usando su resultado calcular



X 1
n=1
(2n − 1)2

Control 17

1. Si ϕ es una función de una variable y z = yϕ (x2 − y 2 ) probar que

1 ∂z 1 ∂z z
+ = 2
x ∂x y ∂y y

480
Apuntes Mat023 (versión preliminar actualizada 23-05-2014)

Control 18

1. Sea T : R3 [x] → R3 [x] definida por

T a0 + a1 x + a2 x2 + a3 x3 = a0 + a1 (x + 1) + a2 (x + 1)2 + a3 (x + 1)3


a) Demuestre que T es una transformación lineal.

b) Si B = {1, x, x2 , x3 } encuentre [T ]BB

c) Determine dim (T ). ¿Es T inyectiva? Justifique.

Control 19

1. Resolver la ecuación
x2 y 00 − 3xy 0 + 3y = 2x4 ex

Control 20

1. Resuelva la ecuación integro diferencial


Z t
0
y (t) + y (u) du = f (t)
0

donde 
 1 0≤t<1
f (t) =
 0 t>1
con la condición inicial y (0) = 0.

Control 21

1. Para x > 0 resuelva el P.V.I.

x2 y 00 − 2xy 0 + 2y = x4

con y (1) = 0, y 0 (0) = 1.

481
Apuntes Mat023 (versión preliminar actualizada 23-05-2014)

Control 22

1. Considere las superficies en R3 dadas por las ecuaciones

y = f (x)

y
z 2 + 2xz + y = 0

Determine la función f si se sabe que ambas superficies tienen el mismo plano


tangente en todo punto donde intersectan.

2. Dado el sistema

u = f (x)
v = g (x, y)
w = h (x, y, z)
∂(f,g,h)
para el caul se cumple ∂(x,y,z)
6= 0:

a) Demuestre que localmente se pueden despejar las variables y, u, z como funciones


de (x, v, w) y que
∂u ∂y
= =0
∂w ∂w
b) Pruebe que  
∂z 1 ∂ (h, g)
=−
∂x gy hz ∂ (x, y)

Control 23

1. Calcule, si existe, el siguiente lı́mite



y x sin (y 3 )
lı́m
(x,y)→(0,0) x4 + y 4

2. Sea 
x−y


 x3 −y
si y − x3 6= 0
f (x, y) =


1 si y − x3 = 0

Encuentre el dominio de continuidad de f .

482
Apuntes Mat023 (versión preliminar actualizada 23-05-2014)

Control 24
2 /(4y)
1. Sea f (x, y) = y n e−x . Hallar un valor de n tal que f cumpla la ecuación
 
2 ∂f ∂ 2 ∂f
x = x
∂y ∂x ∂x

2. Demostrar que el tetraedro acotado por los planos coordenados y cada plano tangente
a la superficie xyz = a3 es de volumen constante.

Control 25

1. Encuentre todas las soluciones de la ecuación


dy 3
= x2 (y − 1)3
dx 2

2. Resuelva el problema de valor inicial


5
y0 + y = 3x3 + x
9x
con y (−1) = 4.

Control 26

1. Se está celebrando una fiesta en una habitación que contiene 1800 pies cúbicos de
aire libre de monóxido de carbono. En el instante t = 0 varias personas comienzan
a fumar. El humo que contiene 6 por ciento de monóxido de carbono, se introduce
en la habitación a razón de 0.15 pies cúbicos por minuto, y la mezcla, removida por
ventilación, sale a ese mismo ritmo por una ventana entreabierta. ¿Cuándo deberı́a
abandonar una persona prudente esa fiesta, si el nivel de monóxido de carbono
comienza a ser peligroso a partir de una concentración de 0.00018?

Control 27

1. Un edificio tiene dos pisos. El primer piso está sujeto al suelo rı́gidamente y el segundo
es una masa m que pesa 16 toneladas. La estructura elástica del edificio se comporta
como un resorte que resiste a los desplazamientos horizontales del segundo piso;

483
Apuntes Mat023 (versión preliminar actualizada 23-05-2014)

requiere una fuerza horizontal de 5 toneladas para que el segundo piso se desplace
una distancia de 1 pie. Suponga que un temblor de tierra hace que el piso oscile
horizontalmente con una amplitud A0 y con una frecuencia ω, resultando una fuerza
externa F (t) = mA0 ω 2 sin (ωt) sobre el segundo piso.

a) ¿Cuál es la frecuencia de las oscilaciones del segundo piso?

b) Si el suelo sufre una oscilación cada 2.25 segundos con una amplitud de 3 pulgadas
¿cuál es la amplitud de las oscilaciones forzadas resultantes del segundo piso?

Control 28

1. Resuelva el problema de valores iniciales

ty 00 − ty − y = 0

con y (0) = 0, y 0 (0) = 3.

2. Resuelva el problema

 t si 0 ≤ t < 3
y 00 − 4y 0 + 4y =
 t + 2 si t≥3

con y (0) = y 0 (0) = 0.

Control 29

1. Resolver las ecuaciones

a) xy 2 y 0 + y 3 = x cos x
dy 2
3x y+y2
b) dx
= − 2x 3 +3xy con y (1) = −2

Control 30

1. Sea T : R2 [x] → R2 definida por T (p (x)) = (p (0) , p (1))

a) Pruebe que T es una transformación lineal.

484
Apuntes Mat023 (versión preliminar actualizada 23-05-2014)

b) Determine una base para Ker(T ) e Im(T ). ¿Cuáles son sus dimensiones?

c) Sea B = {1 + x, 2 + x2 , 4 + x + x2 } base de R2 [x] y sea C la base canónica de


R2 . encuentre [T ]CB .

Control 31

1. Resolver la ecuación
1 2
y0 = y − y + 4x (x + 4)
16x2
Sabiendo que una solución particular es de la forma axb .

2. Determine la función M (x, y) más general de modo que la ecuación diferencial


 
xy 1
M (x, y) dx + xe + 2xy + dy = 0
x

sea exacta y resuelva la ecuación para una de tales M .

Control 32

1. Determine los máximos y mı́nimos globales de

f (x, y, z) = 2x2 + y 2 + z 2 − xy

x2 y2 z2
sujeto a 2
+ 4
+ 8
≤ 1.

Control 33

1. Use la transformada de Laplace para resolver el sistema

x0 = 4x − 2y
y 0 = 5x + 2y

con x (0) = 2 y y (0) = −2.

485
Apuntes Mat023 (versión preliminar actualizada 23-05-2014)

Control 34
1
1. Suponga que un tanque cilı́ndrico recto con radio de la base 2
metro y altura 4 metros
tiene inicialmente 2 litros de agua pura. Una solución de salmuera se bombea hacia
1

el tanque a una rapidez de 1 + 1+t litros por minuto, la concentración de sal en el
1
flujo de entrada es de 2
kilogramo por litro. La solución en el tanque es homogénea y
1
se extrae a 1+t
litros por minuto. Determinar la cantidad de sal en el tanque cuando
este se llena.

2. Sean u = (1, 0, 1), v = (1, 0, −1) y w = u × v vectores en R3 y C la base canónica de


R3 :

a) Si T : R3 → R3 es una transformación lineal definida por T (u) = w, T (v) = v


y T (w) = u, determine [T ]CC y [T ◦ T ]CC

b) Determine explı́citamente T −1 o argumente que no esta definida.

c) Si B = {u, v, w} determine [T ]BB

d ) Encontrar una matriz A tal que A−1 [T ]BB A = [T ]CC

Control 35

1. ean a, b ∈ R, a 6= 0. Considere la ecuación diferencial autónoma

dx
= a ((x − 1) (x − 4) − b)
dt

I) Determine los valores y/o condiciones sobre a y b de modo que la función


x (t) ≡ 5 sea una solución de equilibrio y además un atractor.

II) Para los valores y/o condiciones obtenidos en la parte anterior, bosquejar el
diagrama de fases.

III) Para los valores y/o condiciones obtenidos en la parte I), analizar el comporta-
miento de la solución definida por el P.V.I.

dx
= a ((x − 1) (x − 4) − b)
dt
x (0) = 1

486
Apuntes Mat023 (versión preliminar actualizada 23-05-2014)

esto es: Intervalos de crecimiento, decrecimiento y concavidad, limites a ±∞ si


estos tienen sentido (examinar el intervalo maximal de definición).
Obs.: No se entregarán puntos por resolver explı́citamente y luego realizar el
análisis.

Control 36

1. Considere el sistema de ecuaciones

d
X = AX + B
dt
     
t
x (t) 2 4 e
donde X =  , A =   y B = 
−2t
y (t) −1 −3 e

d
a) Resolver el sistema homogéneo X=
dt 
AX, 
bosquejar
 el diagrama de fases y
x (0) 3
clasificar la solución de equilibrio. Si  =  determine la ecuación
y (0) 2
de la recta a la cual tiende la curva solución cuando t → +∞.
d
b) Resolver el sistema no homogéneo dt
X = AX + B

2. Resolver el problema
t
d2 y
Z
= t+2 e−(t−u) y (u) du
dt2 0
y (0) = 0
y 0 (0) = 0

Control 37

1. Considere la función
xα y


 si (x, y) 6= (0, 0)
 x2 + y 2

f (x, y) =



 0 si (x, y) = (0, 0)

a) Determine los valores de α ∈ R+ para los cuales f es diferenciable en (0, 0)

487
Apuntes Mat023 (versión preliminar actualizada 23-05-2014)

b) Determine (para todo α ∈ R+ ) el plano tangente a la gráfica en el punto 1, 1, 12 .




2. Sea f : D ⊆ R2 → R definida por


p
x2 + y 2 − 1
f (x, y) = p
4 − x2 − y 2

a) Determine el dominio D de f .
 ◦ √ 
b) Demostrar que 32 , 0 ∈ D, 1, 3 ∈ ∂D y (0, 0) 6∈ D.

c) Describir los conjuntos de nivel de f y graficar la función.

488
Capı́tulo 13 : Certámenes

Certamen 1

1. Sea f : R2 → R diferenciable tal que f (1, 1) = 1 y ∇f (1, 1) = (−2, 3). Sea


z = g (x, y) = f (x2 y, y 2 f (x, y)). Encuentre la ecuación del plano tangente a la
superficie z = g (x, y) en el punto (1, 1, 1).

2. Considere la función

1


 x
sin (x2 + x2 y 2 ) si x 6= 0
f (x, y) =


0 si x = 0

a) Pruebe que f es continua en R2

b) Encuentre fx (0, 0) y fy (0, 0)

c) Determine si f es diferenciable en (0, 0).

3. El sistema

x3 + uy 2 + v = 0
y 3 + yv + x2 = 0

define a u y v como funciones de x e y, en vecindades de los puntos (x, y) = (0, 1) y


(u, v) = (1, −1). Sea w = (1, 1), determine la derivada direccional de v = v (x, y) en
el punto (1, 0) en la dirección w.

4. El material para el fondo de una caja rectangular cuesta el triple por metro cuadrado
que el material para los lados y la tapa. Determine la máxima capacidad que tal caja
puede tener si la cantidad total de dinero disponible para el material es $12000 y el
material para el fondo cuesta $600 el metro cuadrado.

489
Apuntes Mat023 (versión preliminar actualizada 23-05-2014)

Certamen 2

1. Considere la aplicación lineal T : R3 → R3 tal que T (1, 1, 0) = (2, 4, −2), T (1, 0, 1) =


(0, 2, −2) y T (0, 1, 1) = (2, 2, 2):

a) ¿Es T invertible?. Justifique.

b) Pruebe que existe una base B de R3 tal que [T ]BB es una matriz diagonal.

2. Sea g : R → R una función continua y defina f : R2 → R por


Z y
f (x, y) = g (t) dt
x

a) Pruebe que f es diferenciable

b) Pruebe que el plano tangente a la superficie z = f (x, y) en el punto (a, b) pasa


por el origen si y solo si
Z b
bg (b) − ag (a) = g (t) dt
a

3. Encuentre el máximo de ln x + ln y + 3 ln z sobre la porción de esfera x2 + y 2 + z 2 = 5r2


donde x > 0, y > 0, z > 0. Use el resultado para probar que para números reales
positivos a, b, c tenemos  5
3 a+b+c
abc ≤ 27
5
4. Sea U = {(x, y, z) ∈ R3 : xy 6= 0}. Definamos f : U ⊆ R3 → R por
 
z z
f (x, y, z) = g x + , y +
y x
donde g : R2 → R es una función de clase C 1 . Suponga que para cada
∂f
(x, y, z) ∈ U , (x, y, z) 6= 0
∂z
a) Considere un punto (x0 , y0 , z0 ) ∈ U tal que f (x0 , y0 , z0 ) = 0. Argumente que
f (x, y, z) = 0 define implı́citamente a z como función de las variables x e y, es
decir, z = z (x, y) en una vecindad de (x0 , y0 ) tal que z0 = z (x0 , y0 ).

b) Demuestre que la función z de la parte anterior satisface la ecuación


∂z ∂z
x +y = z − xy
∂x ∂y

490
Apuntes Mat023 (versión preliminar actualizada 23-05-2014)

Certamen 3

1. Resuelva la siguiente ecuación

y 0 = e2x y 2 − 2y − 9e−2x

con y (0) = 4 sabiendo que tiene una solución particular de la forma y (x) = aekx .

2. Un tanque contiene inicialmente 60 gal. de agua pura. Entra al tanque, a una tasa de
2 gal./min., salmuera que contiene 1 lb. de sal por galón, y la solución (perfectamente
mezclada) sale de él a razón de 3 gal. por minuto; el tanque se vacı́a después de una
hora exactamente.

a) Encuentre la cantidad de sal que hay en el tanque después de t minutos.

b) ¿Cuál es la cantidad máxima de sal que llega a tener el tanque?.

3. Resuelva la ecuación
2 2
x 1 − x2 y 00 − 1 − x2 y 0 + x3 y = 0

usando el cambio de variables t = − 12 ln (1 − x2 )

4. Encuentre la solución general de la ecuación


2
x2 − 2x y 00 + 2 (1 − x) y 0 + 2y = 6 x2 − 2x


sabiendo que y = x − 1 es una solución de la ecuación homogénea asociada.

491
Apuntes Mat023 (versión preliminar actualizada 23-05-2014)

Certamen 4

1. Resuelva las ecuaciones

dy
a) x3 dx + x2 y = 2y −4/3

b) (x2 + y 2 + 1) dx − (xy + y) dy = 0

2. Suponga que un individuo infectado se introduce en una población de tamaño N,


todos los individuos de la cual son susceptibles a la enfermedad . Si suponemos que
la tasa de infección es proporcional al producto del número de infectados y el de
susceptibles presentes, ¿cuál será el número de infecciones en el tiempo t?.

3. Para x > 0, considere la ecuación

2
xy 00 + x2 − 1 y 0 + x3 y = e−x /4


a) Use el cambio de variables t = x2 /2 para encontrar la solución general de la


homogénea.

b) Resuelva la no homogénea usando variación de parámetros.

492
Apuntes Mat023 (versión preliminar actualizada 23-05-2014)

Certamen 5

1. a) Pruebe que el cambio de variables u = ax + by + c transforma la ecuación


diferencial y 0 = f (ax + by + c) en una ecuación diferencial autónoma, donde
a, b y c son constantes reales.

b) Use el resultado anterior para transformar la ecuación


dy
= (y − 2x) − (y − 2x)2 + 2 (ec-1)
dx
en una ecuación autónoma.

c) Haga la linea de fase de la ecuación autónoma obtenida en el item b).

d ) En el plano xy haga un esbozo de las soluciones de la ecuación (ec-1).

e) Obtenga la solución general de (ec-1).

2. Determine una solución de la forma y (x) = xn para la ecuación

x2 − 1 y 00 (x) − 2xy 0 (x) + 2y (x) = 0, x > 1




donde n es algún entero positivo.

Usando el resultado anterior, hallar la solución del problema de valores iniciales



 (x2 − 1) y 00 (x) − 2xy 0 (x) + 2y (x) = (x2 − 1)2 x > 1
 y (2) = 2, y 0 (2) = 10

3. Considere el sistema lineal de ecuaciones diferenciales


dx
= y
dt
dy
= −4x − 2ay
dt
con a ∈ R:

a) Para los diferentes valores del parámetro real a, haga un retrato de fase del
sistema de ecuaciones indicando las direcciones en las cuales las curvas son
recorridas.

b) Obtenga la solución general del sistema.

493
Apuntes Mat023 (versión preliminar actualizada 23-05-2014)

4. Supongamos que la tasa de cambio del precio x de un bien, crece en el tiempo a una
razón constante c como resultado de la inflación constante, al mismo tiempo cae en
forma proporcional a la diferencia entre la oferta y en el tiempo t y alguna oferta de
dx
equilibrio y0 , es decir, dt
= c − α (y − y0 ). También asumimos que la tasa de cambio
de la oferta es proporcional a la diferencia entre el precio y algún precio de equilibrio
dy
x0 , es decir, dt
= β (x − x0 ). (α y β)son constantes de proporcionalidad positivas.
Asumiendo que en t = 0,x = x0 y y = y0

a) Muestre que el precio y la oferta oscilan alrededor de x0 y y0 + c/α respectiva-


mente.

b) Si en el tiempo t = T0 el precio es máximo ¿en qué tiempo la oferta es máxima?

494
Apuntes Mat023 (versión preliminar actualizada 23-05-2014)

Certamen 6

1. Se ha determinado experimentalmente que la variación de peso de un tipo de pez


sigue la ley
dp β
= αe− 3 t p2/3 − βp
dt
donde p = p (t) representa el peso, α y β son constantes positivas. Si p (0) = p0 > 0
determine el peso máximo del pez.

a) Hallar una ecuación diferencial lineal con solución general

c1 et cos (2t) + c2 et sin (2t) + t5

b) Considere la ecuación y 00 + q (x) y = 0 donde q es continua en todo R y dos


soluciones y1 , y2 las cuales satisfacen y1 (0) = 1, y10 (0) = 0 y y2 (0) = 3, y20 (0) = 1.
Si W (x) es el Wronskiano de y1 e y2 en x demuestre que W (x) = 1 para todo
x donde las soluciones estén definidas.

d y 2 dy
2. Muestre que ϕ (x) = ex es una solución de la ecuación x dx 2 − (x + 2) dx + 2y = 0 y

determinar la solución general de


d2 y dy
x 2
− (x + 2) + 2y = x3
dx dx

3. Sea C la base canónica de R3 . Sean T, L : R3 → R3 transformaciones lineales tales


que T (1, 1, 1) = (1, −3, 3), T (1, 1, 0) = (2, −3, 2), T (1, 0, 0) = (−1, −1, 2) y
 
−2 −3 −2
[L]CC = 
 
 1 1 1  
4 6 5

a) Determine T explı́citamente.

b) Determine [T ◦ L]CC

c) ¿Qué relación existe entre T y L?

d ) Determinar Ker(T ) e Im(L).

495
Apuntes Mat023 (versión preliminar actualizada 23-05-2014)

Certamen 7

1. a) Calcular la serie de Fourier de la función



 1 si 0 < x < π


f (x) =


−1 si −π < x < 0

π 1 1 1
y usando la serie muestre que = 1 − + − + ···
4 3 5 7
b) Usando la serie de Fourier anterior, determine la serie de Fourier de

 a si 0 < x < π


g (x) =


b si −π < x < 0

para a, b ∈ R (No se entregarán puntos por calcular esta serie con las fórmulas
de los coeficientes de Fourier).

2. Resolver el P.V.I. 
2  t si t < 1
dy
+ 9y =
dt2  0 si t > 1

con y (0) = y 0 (0) = 0.

3. Resolver el sistema
Z t Z t
t−u
e x (u) du + (t − u) y (u) du = t2
0
Z t Z 0t
x (u) du + (t − u)2 y (u) du = t3
0 0

4. Sea α ∈ R− {1, 2}. Considere el sistema de ecuaciones


    
d  x   α α−2 x
=  
dt y 1−α α y

a) Clasificar la solución de equilibrio (silla, atractor, repulsor, etc.) para los distintos
valores de α.
3
b) Para α = 2
determine la solución general del sistema y bosquejar el diagrama
de fases.

496
Apuntes Mat023 (versión preliminar actualizada 23-05-2014)

Certamen 8

1. Hallar y clasificar los puntos crı́ticos de f : R2 → R definida por

f (x, y) = x3 + y 3 + 9x2 − 3y 2 + 15x − 9y

¿Posee f extremos globales?.

2. Considere la función
x2 y


 si (x, y) 6= 0
 |x| + y 2

f (x, y) =



0 si (x, y) = 0

a) Determine los puntos del dominio de f en los cuales la función es continua.


b) ¿Es f diferenciable en (0, 0)?
∂f
c) Si a es un vector unitario de R2 y (1, 1) es la derivada direccional de f en
∂a
(1, 1) en la dirección a, determine el valor de
∂f
máx (1, 1)
2
a∈R , kak=1 ∂a

3. Determine las funciones f : R → R de clase C 1 (R) tales que las superficies

S1 = (x, y, z) ∈ R3 : y = f (x)


S2 = (x, y, z) ∈ R3 : z 2 + 2xz + y = 0


son tangentes en los puntos de intersección.

4. Suponga que la función de clase C 2 (R), f : R2 → R, (x, y) → f (x, y) satisface la


ecuación en derivadas parciales
∂ 2f ∂ 2f ∂ 2f
−2 −3 2 =0
∂x2 ∂x∂y ∂y
y defina g : R2 → R por

g (u, v) = f (αu + βv, δu + γv)

donde α, β, δ y γ son constantes. Determine valores enteros no nulos de α, β, δ y γ


para los cuales g cumple
∂ 2g
=0
∂u∂v

497
Apuntes Mat023 (versión preliminar actualizada 23-05-2014)

Certamen 9

1. Resolver la ecuación diferencial

d2 y dy
t 2
+ (1 − 2t) − 2y = 0
dt dt

con las condiciones iniciales y (0) = 1, y 0 (0) = 2.

2. En el paraboloide 4x2 + y 2 + 4z 2 − 16x − 6y − 8z + 25 = 0 hallar el punto más cercano


y mas alejado del plano 2x + 2y + z = 0.

3. Muestre que existen funciones u = u (x, y), v = v (x, y) definidas en un abierto U que
contiene a (1, 1) tales que u (1, 1) = 0, v (1, 1) = 1,

ux3 + xy 2 + u sin u = 1

y
v 3 − 2xv + y 3 = 0

además estudiar si la función F : U ⊆ R2 → R2 , (x, y) → F (x, y) = (u (x, y) , v (x, y))


es localmente invertible cerca de (1, 1) y en caso afirmativo calcular DF −1 (0, 1).

4. Sea f : R2 → R, (x, y) → f (x, y) una función de clase C 2 .

a) Muestre que si g (r, θ) = f (r cos θ, r sin θ) entonces

∂ 2f ∂ 2f ∂ 2g 1 ∂ 2 g 1 ∂g
+ = + +
∂x2 ∂y 2 ∂r2 r2 ∂θ2 r ∂r

b) Determine una función f : {(x, y) ∈ R2 : 1 ≤ x2 + y 2 ≤ 9} → R tal que

∂ 2f ∂ 2f
+ = 0 para 1 < x2 + y 2 < 9
∂x2 ∂y 2
f (x, y) = 1 si x2 + y 2 = 1
f (x, y) = 3 si x2 + y 2 = 9

suponiendo que g (r, θ) = f (r cos θ, r sin θ) no depende del ángulo, es decir,


∂g
∂θ
= 0.

498
Apuntes Mat023 (versión preliminar actualizada 23-05-2014)

Certamen 10

1. Determine todas la funciones de clase C 1 y positivas f : R → R+ , que cumplan


f (0) = 1 y tales que en todo intervalo [a, b] el área bajo la gráfica de la función y
sobre el eje x sea igual a la longitud de arco de la gráfica.

2. Determine la solución general de la ecuación

d3 y d2 y dy
3
− 6 2
+ 11 − 6y = et + t
dt dt dt

3. Si se sabe que una solución de

d2 y dy
2
+ f (x) + g (x) y = 0
dx dx

es ϕ (x) = x y f (x) = − x1 (x + 2), determine g (x) y la solución general de

d2 y dy
2
+ f (x) + g (x) y = xex
dx dx

4. Construir una transformación lineal T : R4 → R4 tal que

ker (T ) = {(x, y, z, w) : x + y + z = 0 ∧ x − 2y + w = 0}

e
Im (T ) = {(x, y, z, w) : x − y − z = 0 ∧ x + 2w + y − z = 0}

y determine [T ]BC donde

B = {(1, 0, 0, 0) ; (1, 1, 0, 0) ; (1, 1, 1, 0) ; (1, 1, 1, 1)}

y C es la base canónica de R4 .

499
Apuntes Mat023 (versión preliminar actualizada 23-05-2014)

Certamen 11

1. Determine La serie de Fourier de ϕ : [−2, 2] → R



 0 si x ∈ [−2, 0]


x → ϕ (x) =


x si x ∈ [0, 2]

¿a qué converge la serie de Fourier de ϕ al evaluarla en x = 6?

2. Resolver el P.V.I. 
 0 si t < 4

d2 y

−y =
dt2 

t2 si t > 4

donde y (0) = y (0) = 0.

3. Determine las funciones de orden exponencial ϕ, φ : R → R que cumplen ϕ (0) = 0 y


Z t
ϕ (t − u) φ (u) du = t2
0
Z t
et−u ϕ0 (u) du = t
0

a) Analizar para los distintos valores de µ en R la naturaleza de las soluciones de


equilibrio del sistema
    
d  x   µ 1  x 
=
dt y −1 µ y

y bosquejar en cada caso el diagrama de fases.

b) Resolver el P.V.I.
    
d  x  2 1 x
=   
dt y −1 2 y
x (0) = y (0) = 1

500
Apuntes Mat023 (versión preliminar actualizada 23-05-2014)

Certamen 12

1. Considere el siguiente modelo de dinámica poblacional:

dy  y
=r 1− y − Ey donde 0 < E < r, k 6= 0
dt k

a) Hallar y clasificar las soluciones de equilibrio.

b) Si y (0) = k2 1 − Er bosquejar la solución y determinar lı́mt→+∞ y (t).




c) ¿Existen valores de los parámetros E, k, r tal que dos puntos crı́ticos consecutivos
sean atractores?

2. Resolver el problema de valores iniciales

xy 00 (x) + (2x − 1) y 0 (x) − 2y (x) = x2 e−3x , x > 0

donde y (1/2) = y 0 (1/2) = 0, si se sabe que y (x) = eαx es una solución de la


homogénea asociada para un α adecuado.

3. Considere el problema

00 2
 π
u (t) + a u (t) = 2 sin ωt +
4
0
u (0) = u (0) = 0

donde a es una constante conocida.

a) Si a 6= ω resolver la ecuación diferencial con las condiciones dadas.

b) ¿Para qué valores de ω ocurre el fenómeno de la resonancia? Justifique su


respuesta.

4. Considere el sistema de ecuaciones

dx
= −5x + 4y
dt
dy
= −4x + 5y
dt

a) Elaborar el retrato de fases del sistema y clasificar los puntos de equilibrio.

501
Apuntes Mat023 (versión preliminar actualizada 23-05-2014)

b) Bosquejar en el plano de fase la curva solución con condiciones iniciales x (0) = 2,


y (0) = 3 ¿Existe algún instante t en el cual x (t) = y (t)?

c) Bosquejar en el plano de fase la curva solución con condiciones iniciales x (0) = 4,


y (0) = 2 ¿Existe algún instante t en el cual x (t) = y (t)?

502
Apuntes Mat023 (versión preliminar actualizada 23-05-2014)

Certamen 13

1. Sea f : R3 → R diferenciable y tal que

∇f (0, 2, 1) = (1, −1, −2) y f (0, 2, 1) = 4

Considerar además la función g(u, v) = f (u − v 2 , 3u − v , 3u2 − 2v) . Encuentre la


ecuación del plano tangente a la superficie z = g(u, v) en el punto (1, 1, 4) .

2. La ecuación x2 + y 3 + z 4 + u5 = 2 define a u como función de x , y y z y la


ecuación x + y 2 + z 3 = 1 define a z como función de x e y . Entonces, se puede
considerar u como función de x e y . Entonces:
du
Hallar → (1 , −1) , donde →

v está en la dirección (1 , 1).
d−v
3. Sean F : R3 → R3 y G : R2 → R3 dadas por F (x , y , z) = (u , v , w) determinado
por el sistema

u = x−y
v = x2 − y 2 + z
w = x−z

y G(x , y) = (x − 2y + 1 , 2x − y , x2 − y 2 )

a) Determinar los puntos de R3 en los que F es localmente invertible.


dy
b) Determinar (0, 1, 0)
dw
c) Calcular el diferencial de F −1 ◦ G en (1, 1)

4. Considerar la elipse que se obtiene al interceptar el cilindro x2 + y 2 = 1 y el plano


x + y + z = 0 . Encontrar la longitud del semieje mayor y del semieje menor.

503
Apuntes Mat023 (versión preliminar actualizada 23-05-2014)

Certamen 14

1. Resolver Z t
00 0
y − 2y + y − 2 y(u)du = 5 ; t≥0
0
con y(0) = 0 ; y 0 (0) = 0 .

2. Resolver el sistema

     
x0 (t) 1 2 3 x(t)
     
 y 0 (t)  =  0 2 3  ·  y(t) 
     
0
z (t) 0 0 2 z(t)

3. Considerar la función
f (x) = eλx ; −π < x < π

y f (x + 2π) = f (x) . Pruebe o refute que:


π 1 X (−1)n λ
= +2
sinh(λπ) λ n=1
λ2 + n2

504
Apuntes Mat023 (versión preliminar actualizada 23-05-2014)

Certamen 15

1. Encuentre la solución general de la ecuación de Euler no homogénea:

x2 y 00 + 2xy 0 − 2y = 10 cos (ln x)

2. Una masa que pesa 8 lbs. estira 4 pies un resorte. Al principio esta masa parte
del reposo a 2 pies abajo de la posición de equilibrio y el movimiento ocurre en un
medio que presenta una fuerza de amortiguamiento igual a la mitad de la velocidad
instantánea. Deducir la ecuación del movimiento si se aplica una fuerza externa igual
a f (t) = t cos(2t)

1
3. Para x > 0 y haciendo el cambio x = encuentre la solución general de la ecuación
t
 
4 00 3 0 1
4x y + 8x y + y = tg
2x

4. Considere la ecuación diferencial

2
y 00 + 4xy + 6 + 4x2 y = x2 e−x .


2
a) Pruebe que si y(x) = u(x)e−x es solución, entonces u satisface una ecuación
diferencial lineal no homogénea con coeficientes constantes.

b) Use el resultado de la parte anterior para determinar la solución y(x) con

y(0) = 1 , y 0 (0) = 0

505
Apuntes Mat023 (versión preliminar actualizada 23-05-2014)

certamen 16

1. La Ley de Malthus supone que la tasa de crecimiento de una población p , es


directamente proporcional al tamaño de la población en cada instante.

a) Escribir la ecuación diferencial que representa esta relación y verificar que


p(t0 ) = p0 , entonces p(t) = peα(t−t0 ) , para alguna constante α > 0
b) Sabiendo que la población de la tierra aumentó, en promedio, el 2 % anual desde
1960 (α = 0,02) y que en 1965 se estimaba en 3.340 millones de personas.
Calcular mediante este modelo en cuanto tiempo la población se duplico (respecto
de 1960).
c) Este modelo tiene una corrección propuesta por Verhulst en 1837, que asume
que al crecer mucho la población y tener que competir por el alimento y espacio,
el crecimiento se ve afectado por la falta de recursos. Este modelo está dado por
la ecuación logı́stica
p0 = αp − βp2

con α, β constantes reales positivas. Resuelva esta ecuación.


d ) Calcular para ambos modelos el lı́mite cuando t tiene a infinito y explicar su
resultado.

2. Determine la distancia máxima y mı́nima del origen (0, 0, 0) a los puntos de la curva
definida por la intersección de las superficies:

z = x2 + y 2 ; x + 2y + z = 10

3. Considere la transformación F : R3 → R3 dada por

F (x, y, z) = (u, v, w), donde u = xy 2 v = x + 3y w = z − x.

Se verifica que F (4, 1, 2) = F (1, 2, −1) = (4, 7, −2).

a) Pruebe que en torno a los puntos (4, 1, 2) y (1, 2, −1) existen inversas locales

(x, y, z) = G(u, v, w) ; (x, y, z) = H(u, v, w)

que satisfacen G(4, 7, −2) = (4, 1, 2) y H(4, 7, −2) = (1, 2, −1) respectivamente.

506
Apuntes Mat023 (versión preliminar actualizada 23-05-2014)

∂x
b) Calcular en el punto (4, 7, −2) para G ó H (ESCOGER SÓLO UNA DE
∂v
ELLAS).

507
Apuntes Mat023 (versión preliminar actualizada 23-05-2014)

Certamen 17

1. Resuelva la ecuación diferencial

x3 y 000 + 4x2 y 00 + xy 0 − y = x2 ln x

para x > 0.

2. Considerar la función 
 0 si 0 ≤ t < 1
f (t) =
 1 si t≥1
Resuelva el sistema

9x0 − 32y 0 − 32y = f (t)


Z t
0
−2x + x (u) du + 8y 0 + 8y = 0
0

con x (0) = 32 e y (0) = 9.

z
3. Use el cambio y = 1+x2
para resolver la ecuación

2 2
1 + x2 y 00 + 4xy 0 + 2 3 + 2x2 y =
 
+ + 2x
cos (2x) sin (2x)

508
Apuntes Mat023 (versión preliminar actualizada 23-05-2014)

Certamen 18

1. Sea T : R3 → R2 una transformación lineal. Sean B = {(1, 1, 1) , (1, 1, 0) , (1, 0, 0)} y


U = {(1, 1) , (1, −1)} bases de R3 y R2 respectivamente.

Considere  
1 0 −1
A = [T ]UB =  
−1 1 2
Determine:

a) [T (1, 1, 0)]U

b) T (3, 2, 1)

c) ¿T es inyectiva?
 
y−x z−y
a) Si w = f xy
, yz . Probar que

∂w ∂w ∂w
x2 + y2 + z2 =0
∂x ∂y ∂z

b) Sea f : R2 → R definida por



 x2 si x2 ≥ y 2
f (x, y) =
 y si x2 < y 2

determine el máximo dominio de continuidad de f .


 3
2 2 2 2 2 2 2 R2
2. Pruebe que el máximo valor de x y z bajo la condición x + y + z = R es 3
.
Deduzca de esto que
p
3 x2 + y 2 + z 2
x2 y 2 z 2 ≤
3
3. Un canaleta cuya sección transversal tiene forma de trapecio, con ángulos en la base
iguales, se va a construir doblando bandas iguales a lo largo de ambos lados de una
larga pieza de metal, de 12 pulgadas de ancho. Encuentre los ángulos de la base y las
dimensiones de los lados que producen la máxima capacidad de acarreo.

509
Apuntes Mat023 (versión preliminar actualizada 23-05-2014)

Certamen 19

1. Dada la función definida por



(y−2)2 sin(xy)

x2 +y 2 −4y+4
si (x, y) 6= (0, 2)
f (x, y) =
 0 si (x, y) = (0, 2)

¿Es f una función continua en R2 ?


2. El volumen de un elipsoide de semiejes a, b, c es 3
abc. Hallar el elipsoide con centro
(0, 0, 0) de volumen mı́nimo que pasa por (2, −3, 5).

3. Considere las ecuaciones

uv − 3x + 2y = 0
u4 − v 4 = x2 − y 2

Verifique que ellas definen funciones u = u (x, y), v = v (x, y) en torno al punto
(u, v, x, y) = (1, 1, 1, 1), además, determine la ecuación del plano tangente a la
superficie u = u (x, y) en (1, 1, 1).

4. Sea f : R2 → R de clase C 2

a) Pruebe que si f es homogénea de grado p, es decir, f (tx, ty) = tp f (x, y) entonces

∂ 2f ∂ 2f 2
2∂ f
x2 + 2xy + y = p (p − 1) f
∂x2 ∂x∂y ∂y 2

Ind.: g (t) = f (tx, ty) derivar dos veces respecto a t.

b) Probar que si (x, y) · ∇f (x, y) = pf (x, y) para todo (x, y) entonces p es ho-
mogénea de grado p.
Ind.: Defina g (t) = f (tx, ty) − tp f (x, y) y calcule la derivada.

510
Apuntes Mat023 (versión preliminar actualizada 23-05-2014)

Certamen 20

1. Sea 
xy 3

x2 +y 2
si (x, y) 6= (0, 0)
f (x, y) =
 0 si (x, y) = (0, 0)

a) ¿Es f continua en (0, 0)?

b) Hallar fx (x, y) y fy (x, y).

2. (Plano tangente y regla de la cadena)

a) Dada la superficie S : x2 + 2y 2 + 3z 2 = 21 hallar la(s) ecuación(es) del (de los)


plano(s) tangente(s) a S que es (son) paralelos(s) al plano x + 4y + 6z = 0.

b) Si z = f (x, y) es de clase C 1 y x = r cos θ e y = r sin θ, probar que

∂z cos θ ∂z ∂z
sin θ + =
∂r r ∂θ ∂y

3. (Máximos y mı́nimos)

a) Encuentre los máximos y/o mı́nimos de la función

f (x, y, z) = x2 + y 2 + z 2 + xy + yz + xz + x − 2y

b) Determine los ángulos α, β, γ de un triángulo de modo que el producto de sus


senos sea máximo.

511
Apuntes Mat023 (versión preliminar actualizada 23-05-2014)

Certamen 21

1. Considere la función f definida como sigue:



 2yx3 si (x, y) 6= (0, 0)
x2 +y 2
f (x, y) =
 0 si (x, y) = (0, 0)

a) Determine si la función f (x, y) es continua en todo R2 .

b) Determine si la función f (x, y) es diferenciable en todo R2 .

c) Determine el valor de fxy (0, 0) y fyx (0, 0).

2. Sean x la cantidad de sillas e y la cantidad de mesas producidas por un fabricante.


Si las funciones f (x, y) = 256 − 3x − y y g (x, y) = 222 + x − 5y corresponden a los
precios unitarios de venta de los productos respectivamente, hallar las cantidades
de sillas y mesas de modo de obtener máximas utilidades sabiendo que el costo de
producción total es C (x, y) = x2 + xy + y 2 .

3. Dada la ecuación
sin (yz) + sin (xz) + sin (xy) = 0

a) Encuentre las condiciones para que z este definida implı́citamente como función
de las variables x e y cerca de (x, y, z) = (1, 0, π).

b) Encontrar el plano tangente a la gráfica de z = g (x, y) en (x, y, z) = (1, 0, π).

4. Dada la función z = f (2x + 3g (y))

a) Encuentre las condiciones para que la función z = f (2x + 3g (y)) sea dos veces
diferenciable en R2 .

b) Bajo los supuestos encontrados en la parte anterior, determine el valor de k de


modo que
∂z ∂ 2 z ∂z ∂ 2 z
 
k= −
∂x ∂x∂y ∂y ∂x2

512
Apuntes Mat023 (versión preliminar actualizada 23-05-2014)

Certamen 22

1. Sea T : R3 → R3 la transformación lineal definida por

T (x, y, z) = (x + z, y + 3z, x + y + αz)

con α ∈ R:

a) Determine el valor de la constante α para que dim Ker(T ) = 1 y en este caso


Calcule Ker (T ).

b) Para el valor anterior de α calcule Im (T ).

2. Sean p (x) y q (x) dos funciones continuas. Verificar que la sustitución y = ez con
z = z (x) transforma la ecuación diferencial

y 0 + p (x) y = q (x) y ln y

en una ecuación lineal de primer orden. Usando lo anterior, resolver la ecuación

xy 0 = 2x2 y + (x + 1) y ln y

3. Hallar la solución general de una ecuación diferencial lineal a coeficientes constantes


homogénea, cuya ecuación caracterı́stica es:

λ5 − 2λ4 + 6λ3 − 9λ2 + 8λ − 4 = 0


√ 
3
sabiendo que y = ex/2 cos 2
x es una solución de dicha ecuación.

4. Hallar la solución general de la ecuación diferencial

d2 y dy
2
+ tan (x) + cos2 (x) y = 0
dx dx

utilizando para ello el cambio de variables t = sin x.

513
Apuntes Mat023 (versión preliminar actualizada 23-05-2014)

Certamen 23

1. (40 pts.) Sea f : R2 → R la función definida por:





 −2y 2 , (x, y) ∈ A


f (x, y) = 0 , (x, y) = (0, 0)
2 2


 x +y
sin (x + y) , (x, y) ∈ AC ∧ (x, y) 6= (0, 0)


 2
x + |y|
en donde:
(x, y) ∈ R2 : y > 0 ∧ −x ≤ y ≤ x

A=

∂f ∂f
a) Calcule (0, 0) y (0, 0).
∂x ∂y
b) ¿Es diferenciable f en (0, 0)?

2. Sean f : R3 → R una función tal que ∇f (1, 1, −2) = (−1, 1, 0) y h : R2 → R otra


función definida por h(s, t) = 2s2 + st:

(a) Encuentre un vector unitario normal a la curva de nivel (s, t) : h(s, t) = 2 en
el punto (1, 0).

(b) Considere ω : R2 → R definida por

ω(s, t) = f 3x + 2xy + z 2 , x + y 2 , x − 2z 2


en donde: x(s, t) = 2st, y(s, t) = s2 − 2t y z(s, t) = s + t. Calcule


∇ω(1, 0).

(c) Encuentre la derivada direccional de ω en el punto (1, 0) en la dirección dada por


el vector calculado en la parte (a).

3. Sean f : R2 → R una función definida por:

f (x, y) = x − x2 − y 2

y el conjunto U = {(x, y) : x2 + y 2 ≤ 1}:

a) Mediante el criterio del hessiano, determine los extremos de f en Ů = {(x, y) : x2 + y 2 < 1},
en caso de existencia.

514
Apuntes Mat023 (versión preliminar actualizada 23-05-2014)

b) Mediante el método de los multiplicadores de Lagrange, calcule los extremos de f


en ∂U = {(x, y) : x2 + y 2 = 1}, si acaso existen.

515
Apuntes Mat023 (versión preliminar actualizada 23-05-2014)

Certamen 24

1. Sea x = x (t) una función que satisface el sistema de ecuaciones

x00 = y − z
y 00 = x0 + z 0
z 00 = − (1 + x + y)

donde x (0) = x0 (0) = 0, y (0) = −1, y 0 (0) = 1 y z (0) = 0, z 0 (0) = 1. Calcule, de ser
posible, el valor de x (π).

2. Sea f : R → R la función periódica definida por

f (x) = e−x para − π < x < π

y f (x + 2π) = f (x). Calcule la serie de Fourier de f .

3. Sea f : R2 → R una función de clase C 2 (R2 ). Considere el cambio de variables

x = u+v
y = uv 2

y la función g : R2 → R definida por g (u, v) = f (x (u, v) , y (u, v)). Calcule el valor


de
∂ 2g ∂ 2g
(1, 1) + (1, 1)
∂u2 ∂v 2
∂2f ∂2f 2
sabiendo que ∂x2
(2, 1) = ∂y∂x (2, 1) = ∂∂yf2 (2, 1) = 1 y ∂f
∂y
(2, 1) = 2.

4. Se desea construir un tanque que consistir a de un cilindro circular recto de altura h


y radio r, una tapa superior semiesférica de radio r y finalmente una tapa inferior
plana del mismo radio. Suponga que los costos de construcción de la tapa semiesférica
son de $20 por [m2 ]; que los costos de construcción de la cara lateral cilı́ndrica son
de $8 por [m2 ] y de $5 por [m2 ] para la base circular plana.

a) Hallar el valor de h y r de modo que el costo de construcción sea mı́nimo


asumiendo que el volumen debe ser de 200π [m3 ].

b) ¿A cuanto es igual la relación h : r?.

516
Apuntes Mat023 (versión preliminar actualizada 23-05-2014)

Certamen 25

1. Resuelva usando la transformada de Laplace el siguiente problema de valores iniciales

ty 00 − 2y 0 + ty = 0 con y (0) = 1, y 0 (0) = 0

2. Sea f : [0, π[ → R la función definida por:



 2t si 0 ≤ t < π2
π
f (t) =
 2(π−t) si π ≤ t < π
π 2

Desarrollar f (t) en una serie de Fourier en términos del seno.

3. Considere la ecuación diferencial de primer orden

dx √
= (x − 1) (x − a) x x2 + x + 1
dt

en donde a es un parámetro real. Determine condiciones sobre a de modo que la


solución de equilibrio x = 0 sea un punto atractor.

517
Apuntes Mat023 (versión preliminar actualizada 23-05-2014)

Certamen 26

1. Sea T : R2 [x] → R2 la función definida por


 Z 1 
0
T (p (x)) = p (1) , 6 p (x) dx
0

a) Pruebe que T es una transformación lineal.

b) Hallar el núcleo de T y una base para la imagen de T .

c) Sean B = {1, 1 + x, x + x2 } y D = {(1, −1) , (0, 1)} bases ordenadas de R2 [x] y


R2 respectivamente. Calcular [T ]D
B.

2. Hallar la solución general de la ecuación

x2 (1 − x) y 00 + 2x (2 − x) y 0 + 2 (1 + x) = x2

sabiendo que la homogénea asociada tiene una solución de la forma y = x−2 .

3. La ecuación homogénea

y (4) − 4y 000 + 11y 00 + 8y 0 − 26y = 0

tiene una solución de la forma y = e2x cos 3x. Resolver la ecuación

y (4) − 4y 000 + 11y 00 + 8y 0 − 26y = x

4. Se suministra bacterias como alimento a una población de protozoos a una tasa


 gr 
constante de 1 min . Se ha observado que las bacterias son consumidas a una tasa
de cuatro veces el cuadrado de su cantidad c (t). Hallar c (t) en función de c (0) = c0 .

518
Apuntes Mat023 (versión preliminar actualizada 23-05-2014)

Certamen 27

1. Sea x > 1. Resuelva la ecuación diferencial:

(1 − x) y 00 + xy 0 − y = (1 − x)2 cosh x

sabiendo que una solución de la ecuación homogénea asociada es y = ex .

2. Laplace:

a) Calcule L t sen t .

b) Si L f (t) = X (s), calcule f (t), sabiendo para ello que X (s) satisface la
ecuación:
2s (1 − e−2πs )
s2 + 1 X (s) =

s2 + 1

3. Considere el siguiente sistema de ecuaciones:


 Z t
 x0 + 2x + 6
 y (u) du = −2
0

x0 + y 0 + y =

0

en donde x (0) = −5 e y (0) = 6. Calcule el valor de x (ln 2).

4. Sea f : R2 → R la función definida por:

xy 2

, si y > x2




 x 2 + y2


f (x, y) = x2 y
2 + y2
, si y ≤ x2 ∧ (x, y) 6= (0, 0)
x






 0 , si (x, y) = (0, 0)

Determine todos los puntos de R2 para los cuales la función f es continua.

519
Apuntes Mat023 (versión preliminar actualizada 23-05-2014)

Certamen 28

1. Considere la función f : R2 → R definida por:



 xy sin x , (x, y) 6= (0, 0)

f (x, y) = x2 + y 2
0 , (x, y) = (0, 0)

a) Determine si f es continua en (0, 0).

b) Determine todas las derivadas direccionales de f en (0, 0).

c) Determine si f es diferenciable en (0, 0).

2. Sea z = z (x, y) una función de clase C 2 . Escriba la ecuación:

∂ 2z ∂ 2z ∂ 2z
+ 2 − 3 =0
∂x2 ∂x∂y ∂y 2

en las variables u y v definidas por las ecuaciones:

u = 3x − y ∧ v =x+y

3. Determine los valores de las constantes a, b y c en R de modo que la derivada


direccional de:
f (x, y, z) = acy 2 + byz + cz 2 x3

en el punto (1, 2, −1) tenga un valor máximo de 64 en la dirección del eje X positivo.

4. Hallar las dimensiones del paralelepı́pedo rectangular de mayor volumen con aristas
paralelas a los ejes coordenados que puede ser inscrito en el elipsoide de ecuación:
 x 2  y 2  z 2
+ + =1
3 4 5

520
Apuntes Mat023 (versión preliminar actualizada 23-05-2014)

Certamen 29

1. Resuelva la ecuación diferencial:

(1 + x)2 y 00 − 3 (1 + x) y 0 + 4y = (1 + x)3

utilizando para ello el cambio de variables et = 1 + x.

2. Resuelva la ecuación diferencial:

x00 + tx0 − x = 0, x (0) = 0, x0 (0) = 1

3. Sea z ∈ C 2 . Simplificar al máximo la ecuación:

∂ 2z ∂ 2z ∂ 2z
y + (x + y) + x =0
∂x2 ∂x∂y ∂y 2

considerando para ello el cambio de variables dado por:

u = y 2 − x2 ∧ v =y−x

4. Sean f : R3 → R una función diferenciable tal que:

∇f (0, 2, 1) = (1, −1, −2) ∧ f (0, 2, 1) = 4

y g la función definida por g (u, v) = f (u − v 2 , 3u − v, 3u2 − 2v). Hallar la ecuación


del plano tangente a la superficie z = g (u, v) en el punto (1, 1, 4) .

5. Determine el máximo y el mı́nimo absolutos de la función:

z = sin x + sin y + sin (x + y)

en la región:
π π
0≤x< ∧ 0≤y≤
2 2

521
Apuntes Mat023 (versión preliminar actualizada 23-05-2014)

Certamen 30
x
et
Z
1. Sea g (x) = dt. Hallar todos los valores de la constante a tales que la función
1 t
f definida por:
1 a g(x)
f (x) = e
x
satisfaga la ecuación diferencial lineal:

x2 y 00 + 3x − x2 y 0 + 1 − x − e2x y = 0
 

Utilizar la información anterior para determinar la solución general de la ecuación en


el intervalo (0, +∞).

2. Utilizando la Transformada de Laplace, resuelva la ecuación diferencial:

xy 00 − 2y 0 + xy = 0

sabiendo que y (0) = 1 e y 0 (0) = 0.

3. Considere la función f : R2 → R definida por:



 x|y|3/2 , (x, y) 6= (0, 0)
x2 +y 2
f (x, y) =
 0 , (x, y) = (0, 0)

a) Determine si f es continua en (x, y) = (0, 0)

b) Determine si f es diferenciable en (x, y) = (0, 0)


Z x
2
4. Sea f (x, y) = ln (x + 2xy + 1) + sin2 t dt
0

a) Calcule ∇f (π, 1)
∂f
b) Calcular →

v ∈ R2 tal que k→

v k = 1 y que → (π, 1) sea máxima.
∂−v

5. Determine el máximo y el mı́nimo absoluto de la función:

z = x3 + y 3 − 3xy

en la región:
0≤x≤2 ∧ −1 ≤ y ≤ 2

522
Apuntes Mat023 (versión preliminar actualizada 23-05-2014)

Certamen 31

1. Hallar la solución general de la ecuación:

xy 00 − 2 (x + 1) y 0 + (x + 2) y = x3 e2x

para x > 0, bajo el supuesto que la ecuación homogénea tiene una solución de la
forma y = emx

2. Resuelva la ecuación diferencial:

2 (1 + x)2 y 00 − 6 (1 + x) y 0 + 8y = (x + 1)3

3. Considere la función f : R2 → R definida por:



 x|y|3/2 , (x, y) 6= (0, 0)
x2 +y 2
f (x, y) =
 0 , (x, y) = (0, 0)

a) Determine si f es continua en (x, y) = (0, 0)

b) Determine si f es diferenciable en (x, y) = (0, 0)

4. Sean f, g : R → R funciones de clase C 2 . Considere z : U ⊆ R2 → R definida por:


y y
z (x, y) = x f +g
x x
∂ 2z ∂ 2z 2
2∂ z
Calcule el valor de: x2 + 2xy + y
∂x2 ∂x∂y ∂y 2
5. Determine el máximo y el mı́nimo absoluto de la función:

z = x3 + y 3 − 3xy

en la región:
0≤x≤2 ∧ −1 ≤ y ≤ 2

523
Capı́tulo : Bibliografı́a

[1] Kreyszig, E. Advanced Engineering Mathematics, 9th Ed., John Wiley & Sons Inc.,
Singapore, 2006.

[2] Piskunov, N. Cálculo Diferencial e Integral, Editorial Limusa S.A. de C.V., México,
2007.

[3] Osses, A. Ecuaciones Diferenciales Ordinarias, CMM, Departamento de Ingenierı́a


Matemática, U. de Chile, Santiago, 2010.

[4] Derrick, W & Grossman, S Ecuaciones diferenciales con aplicaciones, Fondo Educativo
Interamericano, México, 1984.

[5] Apostol, T. Calculus: volumen 1, Editorial Reverté, Barcelona, 1967.

[6] Fernández, C. & Rebolledo, R. Ecuaciones diferenciales ordinarias, Ediciones Univer-


sidad Católica de Chile, Santiago, 1995.

[7] Hsu, H. Análisis de Fourier, Addison-Wesley Iberoamericana, EE.UU., 1987.

[8] Rocha, J.M. & Villa, G. Cálculo infinitesimal de varias variables: vol. 1, I.P.N, México,
2003.

[9] Taylor,H & Wade, T. Cálculo diferencial e integral, Limusa–Wiley, México, 1972.

[10] Gavilán, E. Dossier de problemas resueltos, Departamento de Matemáticas, U. de


Concepción, Concepción.

[11] Martı́nez, C. Cálculo real y vectorial en varias variables, Instituto de Matemáticas, P.


U. Católica de Valparaı́so, Valparaı́so, 2000.

[12] Jerrold E. Marsden & Anthony J. Tromba. Cálculo vectorial, 5th Ed. Pearson. Estados
Unidos 2010.

524
Anexo 1 : Modificaciones del apunte

En el capı́tulo de transformada de Laplace se modificó la respuesta de t ∗ sin t (en


esta versión p. 250)

En el capı́tulo de transformada de Laplace, se reparó error en el ejemplo de la


transformada inversa (en esta versión p. 251)

Nuevo ejemplo de convolución p. 251.

También podría gustarte